You are on page 1of 315

Gauge theory of

elementary particle
physics

Problems and solutions

TA-PEI CHENG
University of Missouri - St. Louis
and

LING-FONG LI
Carnegie Mellon University

CLARENDONPRESS . OXFORD
2000
OXFORD
UNIVERSITY PRESS

Great Clarendon Street, Oxford OX2 6DP

Oxford University Press is a department of the University of Oxford.


It furthers the University's objectives ofexcellence in research, scholarship,
and education by publishing worldwide in

Oxford New York

Athens Auckland Bangkok Bogot6 BuenosAires Calcutta


CapeTown Chennai DaresSalaam Delhi Florence HongKong Istanbul
Karachi KualaLumpur Madrid Melbourne MexicoCity Mumbai
Nairobi Paris 56o Paulo Singapore Taipei Tokyo Toronto Warsaw
with associated companies in Berlin Ibadan

Oxford is a registered trade mark of Oxford University Press


in the UK and in certain other countries

Published in the United States


by Oxford University Press Inc., New York

@ Ta-Pei Cheng and Ling-Fong Li, 2000

The moral rights of the authors have been asserted

Database right Oxford University Press (maker)

First published 2000

All rights reserved. No part of this publication may be reproduced,


stored in a retrieval system, or transmitted, in any form or by any means,
without the prior permission in writing of Oxford University Press,
or as expressly permitted by law, or under terms agreed with the appropriate
reprographics rights organization. Enquiries concerning reproduction
outside the scope of the above should be sent to the Rights Department,
Oxford University Press, at the address above

You must not circulate this book in any other binding or cover
and you must impose this same condition on any acquirer

A catalogue record for this book is available from the British Library

Library of Congress Cataloging in Publication Data

(Data available)

rsBN 0 19 850621 X

Typeset by
Newgen Imaging Systems (P) Ltd., Chennai, India

Printed in Great Britain by


Biddles Ltd, Guildford & King's Lynn
Preface

Students of particle physics often find it difficult to locate resources to learn


calculational techniques. Intermediate steps are not usually given in the research
literature. To a certain extent, this is also the case even in some of the textbooks.
In this book of worked problems we have made an effort to provide enough details
so that a student starting in the field will understand the solution in each case.
Our hope is that with this step-by-step guidance, students (after first attempting
the solution themselves) can develop their skill, and confidence in their ability, to
work out particle theory problems.
This collection of problems has evolved from the supplemental material devel-
oped for a graduate course that one of us ( L.F.L.) has taught over the years at
Carnegie Mellon University, and is meant to be a companion volume to our text-
book Gauge Theory of Elementary Particle Physics (referred to as CL throughout
this book) rather than a complete assemblage of gauge theory problems. Neverthe-
less, it has a self-contained format so that even a reader not familiar with CL can
use it effectively. All the problems (usually with several parts) have been given a
descriptive title. By simply inspecting the table of contents readers should be able
to pick out the areas they wish to tackle.
Several new subjects have entered in the field in the fifteen years since the
original writing of CL. Although we have not revised the book to incorporate them
because we would not be able to do them justice, we hope this set of problem/
solution presentations is the first step towards remedying the situation. We have
incorporated a number of new topics and developed further those that were only
introduced briefly in the original text. Listed below are some of these areas:
o Relations among different renormalization schemes
r Further applications of the path-integral formalism
o General relativity as a gauge theory
r Superconductivity as a Higgs phenomenon
o Non-linear sigma model and chiral symmetry
o Path integral derivation of the axial anomaiy
o Infrared and collinear divergence in QCD
o Further examples of the parton model phenomenology
o QCD and A1 j rule in the non-leptonic weak decays
-
o More on gauge theories of lepton number violation
r Group theory of grand unification
o Further examples of solitons
Many people have helped us in preparing this book. Our thanks go particularly
to all the students who have taken the course and have worked through a good part
of these problems. One of us (T.P.C.) also wishes to acknowledge the enjoyable
vi Prefoce

hospitality of the Santa Cruz Institute of Particle Physics when finishing up this
project . The original literature has only been referenced casually, and we apologize
to the authors whose work we may have neglected to cite.
This book and CL share a page on the World Wide Web at the URL
ht.tp z / /vrvw. umsl . edu/-tpcheng/gaugebooks . html. Misprints or
other corrections brought to our attention will be posted on this page. We would
be grateful for any comments about these books.

St. Louis T.P.C.


Pittsburgh L.F.L.
Jantary 1999
Contents

1 Field quantization
1.1 Simple exercises in ),Qa theory 1
1.2 Auxiliary field 6
1.3 Disconnected diagrams 8
I.4 Simple external field problem 9
1.5 Path integral for a free particle 1l
1.6 Path integral for a general quadratic action 13
1.7 Spreading of a wave packet t6
1.8 Path integral for a harmonic oscillator 11
1.9 Path integral for a partition function 2t
1.10 Partition function for an SHO system 23
1.11 Non-standardpath-integralrepresentation 25
Llz Weyl ordering of operators 26
l.13 Generating functional for a scalar field 32
l.I4 Poles in Green's function 35

2 Renormalization
2.1 Counterterms in )"Qa theory and in QED 37
2.2 Divergences in non-linear chiral theory 39
2.3 Divergences in lower-dimensional field theories 4l
2.4 n-Dimensional 'spherical' coordinates 43
2.5 Some integrals in dimensional regularization 46
2.6 Vacuum polarization and subtraction schemes 49
2.1 Renormalization of ),Q3 theory in n dimensions 53
2.8 Renormalization of composite operators 57
2.9 Cutkosky rules 59

3 Renormalization group
3.1 Homogeneousrenormalization-groupequation 63
3.2 Renormalizatron constants 64
3.3 B-function for QED 67
3.4 Behaviour of g near a simple fixed point 69
3.5 Running coupling near a general fixed point 70
3.6 One-loop renormalization-group equation in massless ,1,@a theory 7t
3.7 B-function for the Yukawa coupling 72
3.8 Solving the renormarization-group equation by coleman's method 75
3.9 Anomalous dimensions for composite operators 77
Contents

4 :;"", theory and the quark model


4.1 Unitary and hermitian matrices 78
4.2 SU(n) matrices 19
4.3 Reality of SU(2) representations 79
4.4 An identity for unitary matrices 81
4.5 An identity for SU(2) matrices 82
4.6 SU(3) algebra in terms of quark fields 83
4.7 Combining two spin-] states 85
4.8 The SU(2) adjoint representation 87
4.9 Couplings of SU(2) vector representations 89
4.r0 Isospin breaking effects 90
4.rl Spin wave function of three quarks 93
4.r2 Permutation symmetry in the spin-isospin space 96
4.13 Combining two fundamental representations 97
4.t4 SU(3) invariant octet baryon-meson couplings 100
4.15 Isospin wave functions of two pions 105
4.16 Isospins in non-leptonic weak processes r07

5 Chiral symmetry
5.1 Another derivation of Noether's current 1r0
5.2 Lagrangran with second derivatives 111
5.3 Conservation laws in a non-relativistic theory 113
5.4 Symmetries of the linear o-model 115
5.5 Spontaneous symmetry breaking in the o-model r22
5.6 PCAC in the o-model r23
5.7 Non-linear o-model l 126
5.8 Non-linear o-model II t28
5.9 Non-linear o-model III 130
5.10 SSB by two scalars in the vector representation 133

6 Renormalization and symmetry


6.1 Path-integral derivation of axial anomaly r36
6.2 Axial anomaly and 11 --> yy 140
6.3 Soft symmetry breaking and renormalizabllity 142
6.4 Calculation of the one-loop effective potential 143

7 The Parton model and scaling


7.1 The Gottfried sum rule t46
7.2 Calculation of OPE Wilson coefficients r47
7.3 o,o,(e+e- --> hadrons) and short-distance physics 151
7.4 OPE of two charged weak currents 155
7.5 The total decay rate of the W-boson 156

8 Gauge symmetries
8.1 The gauge field in tensor notation 158
8.2 Gauge field and geometry t6r
8.3 General relativity as a gauge theory r63
Contents IX

8.-1 O(n ) gauge theory 165


8.5 Broken generators and Goldstone bosons r67
8.6 Symmetry breaking by an adjoint scalar r69
8.7 Symmetry breaking and the coset space 17t
s.8 Scalar potential and first-order phase transition t72
s.9 Superconductivity as a Higgs phenomenon t73

9 Ouantum gauge theories


9.1 Propagator in the covariant R6 gauge 175
9.1 The propagator for a massive vector field 176
9.-1 Gauge boson propagator in the axial gauge 177
9.-1 Gauge boson propagator in the Coulomb gauge 178
9.-s Gauge invariance of a scattering amplitude 180
e.6 Ward identities in QED 180
9.t Nilpotent BRST charges 184
e.8 BRST charges and physical states 186

10 Ouantum chromodynamics
r0.I Colour factors in QCD loops 188
I (1.2 Running gauge coupling in two-loop t9l
r 0.-3 Cross-section for three-jet events 193
n 0.+ Operator-product expansion of two currents 198
I (1.5 Calculating Wilson coefficients 20r

11 Electroweak theory
I1.1 Chiral spinors and helicity states 205
l l.l The polartzation vector for a fermion 206
i r.3 The pion decay rate and f, 208
r l.-l Uniqueness of the standard model scalar potential 212
I 1.5 Electromagnetic and gauge couplings 213
l 1.6 Fermion mass-matrix diagonalization 214
I 1.7 An example of calculable mixing angles 2r5
r 1.8 Conservation of the B - L quantum number 2t6

12 Electroweak phenomenology
1t.l Atomic parity violation 218
11.2 Polarization asymmetry of Z -- f f 221
1-.J Simple z-lepton decays 222
n.-t Electron neutrino scatterings 223
11.5 CP properties of kaon non-leptonic decays 225
1t.6 Z --> H H is forbidden 226
t2.7 A,I j enhancement by short-distance QCD 227
-
n.8 Scalar interactions and the equivalence theorem 230
l].9 Two-body decays of a heavy Higgs boson 234
Contents

13 -0,"" in flavourdynamics
13. l Anomaly-free condition in a technicolour theory 238
13.2 Pseudo-Goldstone bosons in a technicolour model 239
13.3 Properties of Majorana fermions 239
13.4 LL --> ey and heavy neutrinos 244
13.5 Leptonic mixings in a vector-like theory 250
13.6 Muonium-antimuonium transition 252

14 Grand unification
t4.t Content of SU(5) representations 255
14.2 Higgs potential for SU(5) adjoint scalars 256
14.3 Massive gauge bosons in SU(5) 258
14.4 Baryon number non-conserving operators 260
14.5 SO(n) group algebra 260
14.6 Spinor representations of SO(n ) 263
14.7 Relation between SO(2n) and SU(n ) groups 267
14.8 Construction of SO(2n) spinors 269

15 Magnetic monopoles
15.1 The Sine-Gordon equation 275
15.2 Planar vortex field 280
15.3 Stability of soliton 282
15.4 Monopole and angular momentum 283

16 lnstantons
t6.t The saddle-point method 289
16.2 An application of the saddle-point method 292
t6.3 A Euclidean double-well problem 295

References 301

lndex 303
1 Field quantization

1 .1 Simple exercise s in ),Qa theory


In )"Qa theory, the interaction is given by

H1 - fio^ra. (1.1)

(a) Show that, to the lowest order in .1., the differeitial cross-section for two-
particle elastic scattering in the centre-of-mass frame is given by

do 72
(r.2)
de: 128"4
where s
- (pr * pz)' ,with p1 and p2being the momenta of the incoming particles.
(b) Use Wick's theorem to show that the graphs in Fig. 1.1 have the symmetry
factors as given. Also, check that these results agree with a compact expression
for the symmetry factor

s:B n 2fl(n0"" (1.3)


n:2,3,...

where g is the number of possible permutations of vertices which leave unchanged


the diagram with fixed external lines, a, is the number of vertex pairs connected
by n identical lines, and p is the number of lines connecting a vertex with itself.
(c) Show that the two-point Green's function satisfies the relation

(n, + p\(0lr @@)Q0))10) : QV(03(x)d0))10) - i6a(x- y).


*
Also verify this relation diagrammatically to first order in.l..

Ftc. l.l.
ry s:4
Symmetry factors.
2 Field quantization 1.1

(d) A Green's function involving the composite operator Q(.r) _ LrO'@) it


defined as

G3'Q; xt, . . . , xn)


- (0lrg|@)Q(xr)' . .d(x,))10). (1.4)

Write down, to the first order in ),, the various contributions to G$) (x; x1, x2).

Solution to Problem 1.1


(a) The tree diagram for a two-particle elastic scattering is shown in Fig. 1.2. Thus
to this order the scattering amplitude is simply T -i,1. giving rise to a differential
-
cross-section: (see Cl-Appendix A for rules of cross-section calculation):

do: I 1 1 l-i).e
---'
otlt o'!o'
lvr - v2l2E12E2' (2n)3282 (2tr)32Eq
x12n1434(h * pz - pt - nDL. (1.5)

The last I factor is inserted to account for the presence of two identical particles
in the final state.
We then have the phase space factor of

e- Ieto4a4@- Pz- rD#h#h (1.6)

where f - h * pz. In the centre-of-mass frame, the four momenta can be


parametrized as pt : (E,p), pz : (E, -p), pt : (E' ,p'), and pa : (E' , -p').
After performing the d3pa integration, the phase factor becomes

e --
IQn)-23(2E -28')#
-| eo>-2t(zr, -2E')41# o"
- 32nzl'lE an (r.7)

and thus the differential cross-section

do
: tllpl L2
(1.8)
dQ yt
f'r - 4E' 32"'E 2
1.1 Simple exercises in ),Qa theory 3

After substituting the flux of lvr


: - vzl : l@r/Er) - @z/E)l :2lpl/E and the
invariant variable s (pr -f p)z - 4E2 into the above expression we obtain
do L2
(1.e)
dQ: nao\'
(b) (i) The diagram in Fig. 1.3 corresponds to a second-order term in the pertur-
bation expansion

* (+)' I oo r, do v, (otrIQ',)Q@z):Qor)Q(vr)Q(yt)Q(yt):
:Q 0)Q 0)Q 0)Q(y2) :l l0) . (1.10)

The amplitude like the one above but with the interchange 1l1 + yz has the same
contribution. This doubling cancels the first factor of fi in the above expression,
which comes from the Thylor expansion.
Wick's expansion leads to the following contractions. There are four ways to
contract Q @ ) with any one of the d (yr ) s and similarly four ways to contrac t Q @z)
with any one of the @(y2)s; then there are 3! ways to contract the remaining pairs
of QOt) and Q0). The (inverse) symmetry factor is

s-r - (; ,) (i)' o o,, - il (1 1,)

This checks with the result obtained by using eqn (1.3) directly, because g : l,
a3 : 1, and fr :0.
(ii) The diagram in Fig. 1.4 is first order in the coupling

+ I dov \0lrIQr,)Q@):QQ)Q0)Q(y)d(y):ll0). (1.t2)


There are four ways to contractQ@r) with any one of the d(y)s and three ways
to contract Q@) with any one of the remaining three d0)s. Hence

c'-r 4'3
- u -z'
-l ( 1.13)

This checks with the result obtained from eqn (1.3), since g : l, d, : 0, and
fr:L

Frc. 1.3.
4 Field quantization 1.1

(iii) The diagram in Fig. 1.5, like the one in Fig. 1.2, corresponds to the second-
order term as given in (i).
The multiplicity is determined by noting that there are four ways to contract
Q@) with any one of the QOr)s and three ways to contract Q@) with any one
of the remaining three d(yr)s.And there *" (:) - 4.3 ways to contract the
remaining Q0) pair to all the possible pairs out of the four @(yz)s.

s-r- (+,)# 434 3:I (1.14)

Equation (1.3) also yields S 4 because in this case I : 1, a2 : 1, and F : l.


-
(iv) Figure 1.6(a) is a fourth-order diagram. There are t such diagrams corres-
ponding to 4! ways to permute the y1,23,a positions for a fixed x1,2,3,4, and the two
categories of diagrams as illustrated in Fig. 1.6(b) are actually identical. Thus the
Taylor series factor of fr is only partially compensated.
For brevity, for the remaining part of the amplitude we will only display the
position factors of the fields

x1x2x3x4 lrlrjrlr jzlzlzjz lzlzlzlz !+lql+lq (1.15)

!a

>o< y3
b
1.1 Simple exercises in ),Qa theory 5

and examine its combinatorics. There arc 4. 3 ways to contract Q@r) and Q@z)
with the four @(yr)s, and the same number of ways between Q(x) and Q@a)
and the Q0)s. For the remaining two @(y1)s to contract into the respective four
d(y:)s and Q(y+)s there are2.4 . 4 ways. Similarly, for the remaining two @(yr)s
to contract into the respective remaining three d(y:)s and Q(ya)s there are2.3 .3
ways. Finally, there are two ways for the remaining two @(y:)s and d(y+)s to
contract into each other.

/l 4!\ I
ro_r: (i ' (4''2 ' Q'4'4) ' (2'3 '3) '2
; ) .*
:!(o'.t').!','" 1'
2 @04 - 4' (1'16)

This again checks with eqn (1.3), since g :2, a2 : l, and fr :0.
(c) First we show that the differentiation of the two-point function with respect to
.r yields

al Qlr (Q @)Q0)) l0) - (01 r (3P Q @)d (y)) l0)


+(0ltd(x) , Q0)ll0)6(xo - )o) (1.17)

where the equal-time commutator actually vanishes. Differentiating for the second
time we have

n, (01 r Q@)Q0)10) : (0lr (.Q(x)d0))10)


+(01 taod(x), 00)l 10)6(.ro - )o). (1.18)

From the equation of motion zQ@) _ -p2O - 0./3!),@3 and the canonical
commutation relation [Ood @), Q(y)] 6(xo - ]o)
- -i6a(x - y), we then obtain
the result stated in the problem.

(tr, + p\\olr(Q@)Q0))t0) : -* pv(Q3Q)Q0))10) - i64(x- y).


(1.1e)

To verify this relation diagrammatically we note that the first order in ,1, diagrams
for the two-point function are given in Fig. 1.7(a).
The Feynman diagrams lead us to the relation

(0lr(Q(x)d0))10) : ia,p(x- y) + (+)


*
|
aor[i Lrrx - z)lli Lek - D]iAr(0) (t.20)

Using the relation (tr, + 1"2)Lr(x - y) - -64(x - y), we obtain the left-hand
side of eqn ( 1 . 19) to first order:

-i34 (x - y) + (+) I o^ rti64 (x - z)tti Lrk - Dtiar(0)


),
- -;Lr@ - y)ar(o) - i64(x - y). (r.2r)
Field quantization I.2

yx z
(a)

Frc. 1.7.

Xlzx2
(a)
*o _0* (b)

Frc. 1.8.

Writing out the Green's function on the right-hand side, we have


).L
-t(0lr@@)Q@)Q@)0(v))10) - -ri Lr@ - v)i ar(0). (r.22)

Equations (1.21) and (1 .22) clearly show that the relation (1.19) is satisfied. The
Feynman diagram for eqn (1.22) is shown in Fig. 1.7(b).
(d) There are three first-order diagrams for the two-point function

c$) {x; xr, xz)


- (01 r (trQ'@)Q@)0("r2))10). (t.23)

We shall explicitly work out the case of diagram (a) in Fig. 1.8.

p1r)62@)Q@t)Q@) (#) | ooroory)to)

: (+) oorUAr(xr - v)ltt Lr(xz- v)ltt Lr@ - Dtz ' (1.24)


I
The symmetry factor of S
- 2 canbe understood by noting that there arc 4.3 ways
to contract between Q@)Q@) andtwo @s n 04 (y), and2 ways to contract 02 @)
with the remaining two @(y)s. Thus the 4! factor in the coupling is cancelled,
and we are left with the I factor from the composite operator. The diagrams in
Fig. 1.8(b) can be worked out in the same way. Their symmetry factors are also 2.

1.2 Auxiliary field


The Lagrangian density for a set of real scalar field Qo , a : 1,2, . .. , N, is given as

L - ;@,Q\(at'o,
,
- *r'r' - * (o"o\2. (1.2s)

(a) Work out the basic vertices in this theory by calculating the four-point ampu-
tated Green's function to the first order in )..
1.2 Auxiliary field
(b) Consider the Lagrangian density

L' -:@t o')Gt'Q") - tr"r' * *r' - I"r'r" {r.261

where o (x) is another scalar field.


(i) Show that if we eliminate o (x) by using the equation of motion, we end up
with the Lagrangian in eqn (1.25).
(ii) If we do not eliminate o (x),and take the propagator for o (x) in the momen-
tum space to be -i ), (which can be justified by adding a term (e l2)(3*o)(0po)
and then the limit of e -+ 0 after the propagator has been worked out), show that
.C' gives the same basic vertex for Q(x) as that given in part (a).

Solution to Problem 1,.2

(a) To the first order in ),, the four-point Green's function with the four external
lines carrying the internal indices a, b, c, d is given as

\ov Q' ob Q' r' (+) o' o' oi oi tot (r.27)

where we have grouped the four fields in the interaction term into two pairs labelled
i and 7, respectively-repeated indices are always summed over.
As displayed in Fig. 1.9, there are two ways @is can be contracted with QoQb,
and two ways between @/s and Q,Qd; these four ways are to be multiplied by 2
corresponding to the interchange i <> j. Thus the factor of 8 is cancelled and the
vertex is given by -i),3"b6'd. There are of course other ways we can pair off
the four external lines. Removing the propagators for the external lines, we have
the basic vertex for this theory:
gbd + 6"d |b,y. (1.28)
-i;(6ob Ard 1 5ac

(b) (i) Since the L' does not contain the 0ro field, the equation of motion for the
o field 0L'l3o - 0 is simply a constraint equation: of ). - lQ"Q". Substituting
this condition into the L' Lagrangian density, the o-dependent part becomes

*"'- )"r"r'- t
(o'o\2 -X6'r')': --f,ro"o">' (t'2s)

and thus L' - L.

\./ \,/

,/\
(a) (b) (c)

Frc. 1.9.
Field quantization 1.3

oo
/
,/
\ \
to'
Ftc. 1.10. Feynman rules for the Lagrangian L' .

ab ad
\,/
V l"
ab
\/ \,/
\,/\,/
\,/
Y
i"
,^' /\ I

,/\ ,/ \,
c
,/\ 'd c
/\ 'b

(a) (b) (c)

Frc. 1.11.

(ii)
The Feynman rule from the Lttheory is shown in Fig. 1.10.
From this we can construct the Feynman diagrams for the four-point function
in Fig. 1.11.
Diagram (a) yields (-i3"\(+i)(-i6", - -i6ob6'd. Similarly, diagrams (b)
and (c) give -i 6o'6bd and -i6od 5b", respectively.

Remark. Very often this kind of auxiliary field is introduced to make the calcu-
lation more tractable. For the case here, the use of the o-field makes the flow of
the internal symmetry indices easier to monitor.

1 .3 Disconnected diagrams
Consider the unperturbed and perturbative parts of the scalar field theory

Lo : f,.ruur"r' - *r', Lr : -!r'. (r.30)

In the perturbation theory, the two-point Green's function is given by

6@ (xr, x2) : plT (Q@)Q(r2)) l0)


(0lT(Q0@)00@)expl-i I Ht(x)dxl)10) ,,1 ?1\
(0lZ(expl-tIn'6)dx])10) ' \r'Jr''
Use Wick's theorem to demonstrate explicitly that the respective disconnected
graphs in the numerator and denominator cancel.
I.4 Simple external field problem

r+Lr+\-Jr
D:'+r-\+^.1

N: + \-/ O O+
+ \-----

+
Frc. 1.12.

+ i
N: + .
(r ) X
('1+O+O+ )

Solution to Problem 1.3


The two-point Green's function

(0lr(do(x )Qo@) exp[-i .[ n'@) dx])10) N


G@ (*r, x2) :
\ t' ^z) - E
- D_ (1.32)
(Olz(expt -t I n'6) dx])10)
has the following Wick's (diagrammatic) expansions, shown in Fig. 1.12, for the
denominator D and the numerator N, respectively, where the dot represents the
'vertex' of L1
- -1m2 /2):Q2(x):. This is equivalent to the expansion as shown
in Fig. 1.13.
We see that the disconnected contribution has been cancelled.

1.4 Simple external field problem


Suppose the Lagrangian for a scalar field is given by

l'^ P2
,:t- 'n''t
,(ouQor' - 70''' - J(x)Qk) (1.33)

where J(x) is a real c-number function.


10 Field quantization 1.4

(a) Calculate (01@(x)10) andthetwo-pointfunction (0lT(:Q(x)d(0))10) exactly.


(b) Treat the term J(x)Q(x) as a perturbation and calculate (0ld(-r)10) and
(0lf @(x)d(0))10) to the lowest order in -/(x).

Solution to Problem 1.4


(a) The Lagrangian yields the equation of motion
(r+ p\O(x): -J(x). (r.34)
If we define the usual Green's (propagator) function,
(r + p.2)Lr@ - )): -60(* - y), (1.35)

the field operator can then be written as @(x) : Qo@) + Q@), where ds(x) is a
c-number function:

: oo, Lr'x- y)/(y). (r.36)


Qo@)
I
6(x) satisfies the homogeneous Klein-Gordon equation, (l + 1"2)ir@): 0, and
can be expanded in terms of the usual creation and annihilation operators, satisfying
the commutation relation lo(k), or (k)f - 63(k - k'):

(r.37)

Because dt"llO)
- (0lA(r) - 0 the vacuum expectation value of the unshifted
field operator is non-vanishing:

(0ld(.r)10) : Qo@): oo, Lr/u - y)J(y), (1.38)


I
and the two-point Green's function is also shifted as

Qlr O(;)d(0)10) : (0lzdo("r)do(0)10) + (0lr O@O(0)10)


- Qo(x)Qo(0) + iA'r@). (1.3e)

(b) The 'interaction vertex' in the Feynman's diagrams for this theory is given in
Fig. l.I4(a).
(i) The perturbative expansion for the vacuum expectation value can be repre-
sented by a diagram similar to Fig. 1.14(a).

(old(x)10) : QVQrt")I +
* oor,irrrf ,(yr).. .doynJ(y)Qr(y,)10),
I
- -i' Jf 0",
,q-. (0lrQr@)QrQ)10)/(y)
,n

: oo, L|u - y)J(y) : Qo@).


I
1.5 Path integral for a free particle l1

J(v) J(v) J(vz)

x Y x Yt lz 0

(a) (b)

Frc. 1.14.

(ii) The perturbative expansion for the two-point Green's function is given by
Fig. 1.14(b).

\0lr Q(x)d (0) l0) : (0lr Q r @)Q r (0)

e:t I dor, dav., J (y1)J (y)Qro)Qr(y2)10).


" 2l Jo''a)'2"
:+ I oor, doyrliAr(x - )r)i Le?y)J (y)J (yz)

* (yr + J)l
- @6(x)@e(0). (1.40)

1.5 Path integral for a free particle


Show that the transition amplitude for a free panicle (mass la) moving in one
dimension has the expression

(q',t'tq,t) -lfrG--o]'".*o l+#l (r 4r)

You should check that this result can be obtained by starting either from the
Hamiltonian or the path integral (Lagrangian) representations of the transition
amplitude:

I @'l H (t' - tlllq)


(q',t'lq,t)--l "^pl-i (r.42)
,- +, r
N |ldql.^n li [,'' ar" tf

where H - p2l2m and L - (ml2)q2 and the integration measure in the path
integral representation is given by

NIdq)-J1i (#A)"''Eon, (r43)

with (r'- r) being divided into n equal segments of A: t,t1,t2,..-,tn-r,tt = t,,


having the corresponding positions Q,Ql, Q2, ...,Qn-t,Q' = Qr.
12 Field quantization 1.5

Solution to Problem 1.5


(a) The Hamiltonian method

(q' , t'lq , t) : (q'l exp[-i H (t' - t)llq)

- (q't^rl#rt'-'tftq)' (1.44)

Inserting a complete set of momentum states:

(q' , t'tq , t) : * (q'texpl# (t' - ilftp) @tq)


I
: *"^'l#(t'-t)-tip(q'-"f (r4s)
I
which can be integrated by using the Gaussian integral formula:

exp(-ax2 -t bx) : ,[: *, (#). ( 1.46)


I-**o*
In our case, we have a
- (i /2m)(t' - t) and b - i(q' - q). Thus,

(q',t'rq,,) :l# -,))''' "*rlry#l (r.47)

(b) The path integral method


The action can be written in terms of the space-time intervals as

[" ,or" - [' Yo, dt" -r t (q' -


q'*'\'o
s - J, J, 2- 2?\ A /
- #lrq - q)2 + (qt - qz)2 + "'+ (q,-r - q)'1. (1.48)

Using this and the given integration measure, the transition amplitude can be
expressed as

(q' . t'tq, tt - (#)''' fidqi


expl#u, - qt)2

* (qr - q)2+... * (q,-r-- qr'ltl


)'I l.
l.
(t.4e)

The successive integrals can be calculated by using the formulae for Gaussian
integrals of the form

f@

J_*o*exp[a1x - x)2-tb(x - *)'l:l;h]"'"*o lhtr -.,r')


1.6 Path integral for a general quadratic action T3

so that one has

I or,*o { #rt - q)2 * (qt - n,r'll


:l^'l
l2niL llt/' - q)2f
'*oL^ ')
f im (q

I on,*o{ #lt;t *(az-n"']}


:l*l2riL l)
21'/' f i* - q)21
'*oL^ 3 I
(q

I on'*o{ #ls+ *(ez-"'fl


:l'# *'l#t;t] tuf'''

and so on. In this way, one obtains

(q',t'tq,t) - )*(#A)* (4#)@-'), (;? +)'''


xexpl*(o-o'ff
-,,ry} GA)\*lWl
:G#'-o)'"'*oW+l (1.s0)

where one has used n A


- (t' - r). This result agrees with that obtained in (i) by
using the Hamiltonian as the generator of time evolution.

1.6 Path integral for a general quadratic action


We will study the case of the action containing at most quadratic terms

slql - | o,lo@4'+b(t)Q*c(t)qq+d(t)q*e(t)qz+ f@| (1.s1)

(a) Show that


(qr, trlqi, ti)
- F(tf ,/;) exp li s,(qr, ty; qi, t)] (r.s2)
where Sr(qt,tyi ei, /;) istheactionfortheclassicaltrajectory, and F(ty, /;),being
independent of q; and q y, can be written as

F(tr,t): -
I:,,:,"
tdq(r)1.^o o,lo,t'+ cnrt * rr1l. (1.53)
{, 1,,"
Namely, we have the boundary condition r7(r) - nG') - 0. Thus ry(r) can be
thought of as the difference between a given q(t) and its classical trajectory.
t4 Field quantization I.6

(b) Show that the prefactor F (t f , ti) can be expressed in the compact form of

F (t f , t) : N' /[det A)t t', (1.54)

with the differential operator - -a(d2ldt\ + c(dldt) * e and, N,being a


constant.

Suggestion. Expand n@ as a series in terms of some orthonormal basis functions


X,G) (with n - I,2,3, .. .):
nG) -Dr,x,G) (l.ss)

where
- 6n- and xnQ) : x,Ui :0. The inregration measure
Il,t x,G)x*(r)
NldnU)l - Nfl,dnj) can be taken as NlTndrn Thus we can obtain an
alternative definition of the path integral as

(ey,tylqi,ti) :N (1.s6)
lrydcnexpislql.

Solution to Problem 1.6


(a) The path integral representation of the transition amplitude has the form

fQt-tt) 7
(Qy,tylqi,ti)
- :N I Vqlexpi latSr(q,q,t)1. (1.57)
J @'''') J

Let qr(t) be the solution to the equation of motion

ff:o.r *(#)-ff:o (1.s8)

with the boundary condition q,(t) : Qi and q,(t) : qf . An arbitrary path


q(t) can always be written as q(t) : e,Q) + nQ). Namely, rl(r) is defined to
be the deviation of q(t) from the classical trajectory with the boundary condition
0. In terms of the unique classical trajectory andrl (r), we can
n(ti) - n(tf) -
express the transition amplitude as

\Q7,t]lqi,ti) : N srar'tt\ vtt(t)lexp{i s[4. + ry]]. (1.s9)


J Q,.r,)

The action S can be expanded in powers of 4 (r) : S[q, ry] I


S[4.] * Sr * Sz, where
-
51 is linear in r7Q) and 52 is quadratic. Since the classical trajectory, according to
the variational principle, corresponds to the path with respect to which the action
is stationary, we have Sr 0. Thus
-
slq,+ql- slq,l+ [ ' dtla(t)r:2 +c(t)rtit+e(Dqz). (1.60)
J,,
1.6 Path integral for a general quadratic action 15

Slq,l is independent of a(r). Evidently 52 is independent of q,, hence also of q;


andq y . Ghis is only true for a quadratic action.) One then has

(q
r, t rlqi, ti) - exp(i s [q.])N Ii'.,',t' ldrt(t)l
x exp
I, 1,,"
dt laif * cnn * ,r'll
- F (t r,/;) exp li S,(q r, ty; ei, t)f (1.6r)

which is just the claimed result.

Remark. In many physical applications of the path integral formalism it is not


necessary to know the prefactor F (t y , /;), which does not depend on the coordinates
(q
t , qi).
(b) Start from the expression

F (t r,t) : N
I wna)l exp
f, 1,,"
dt lart2 * crtn * ,r'll
:N i dtr'1t)l-"# +,*+ "] ,r'r}
Itdrt(t)t*o { 1,"
(1.62)

where to reach the second line we have performed an integration by parts.


Now expand rt!) in terms of a complete set of orthonormal functions: q(t) -
Lnrnx,(r) with the condition of X,Qi) - X,Gf) - 0. We then have

F(tr,t): -
| rydc,expl, 1,," at a(DArt(Dl (1.63)

where I
is the differential operator given in the problem. For convenience, we can
choose the orthornormal functions to be the eigenfunctions of ,4:

Ax,(t)
-l-"#.,ft *,fr,(r) - knx,(t). (1.64)

Then

[,"
Jti
o, rt(t)Art(t) -n I dt c,c*k*xnQ)x*(r) - Dr'; ,, (1.65)

and the prefactor for the path integral becomes

F(tf ,ti):Nf] I or,*o(,+r?0,) (1.66)


16 Field quantization l.j
For each term with n : I we can use the Gaussian integral I dx exp(-ax2) -
(n /a)r/z to obtain the promised result:

F(tr,t) :N f] (*)''' : N' det A-rl2 (t.67)

We can check this result by working out explicitly the simple case of a free particle
S : [l,t dt (m /2)q2. Thus A -(m /D@2 / @tz) andthe eigenvalue equation has
-
the form for a simple harmonic oscillator equatio n, - (m 12) (d2 p / dt2) : kn X, ,
which has the solution of Xn - a,nsincr-r,(t - t),witho.,
- (zkrl*)r/2.The
boundary condition ron?f - ti) : nr,with n being an integer, implies thatthe
eigenvalues kn
- @/2)(nr/tf - r;)2. Thus, the determinant has the value of
det A-r/z : f]n k;t/' .Include the multiplicative factor from the Gaussian integral
of (m /2)(iflt/z and choose the normalization factor N' : [2n I im(t y - t)f"/z so
that we obtain the expected value (as determined in Problem 1. 1) for the prefactor:

F (t r,ri) _ N, ff (g',. (,,;_,,) :lr" (r 68)


^ti),,,
1 .7 Spreading of a wave packet
The time-dependent Schrcidinger wave function is defined by ,1, (q, t) - (q, tlV) .

(a) Show that

rL@r, t y)
-I rn r, tylQt, t)t(qi, t) dqi. (1.6e)

(b) For the free particle, suppose t\(q, t : 0) is a Gaussian wave packet:

: (#)''0.*o
t(q, o)
l-t#l (1.70)

Show that it will spread as time evolves:

:
t{r (q, t)t2
l* "t^]
"'
"^o I #l (1.7 r)

where

o'(t)-o2(t* #) (1.72)

Remark. One may recall the physical interpretation for this spreading Gaussian
wave packet. The initial Gaussian wave function can be thought of as a super-
position of plane waves ei p'*. As they evolve with time, such plane waves acquire a
momentum and time-dependent phase e-ip" /2- ,whichwill make the superposition
go out ofphase for t t'0.
1.8 Path integral for a harmonic oscillator 17

Solution to Problem 1.7


(a) This connection between the initial and final wave functions by the transition
amplitude can be obtained simply by inserting a complete set of states in the
expression for the wave function,

$(qr,t) - \qr,trlrh: I on,(qr,trlqi,ti)@i,til,D. Q.73)

(b) Substituting into the above equation the expression for transition amplitude
for the free particle case as derived in Problem 1.1 and the wavefunction ,lr (q , 0),

(q,ttq,o) : (#)"'"*ol+9+.1 t'tor


we end up with a Gaussian integral of the form shown in Problem 1.5

t,(q, t) :I dq, (#)'".*o lryt+] (#)''0."0 e#l


: (#)"' (#)' l##l' ^ " .*o
| #;*l (1.7s)

The lt* lr has a simpler expression; it is straightforward to show it checks with the
result given in the problem.

1.8 Path integral for a harmonic oscillator


The Lagrangian is given by

, -Td'- j, ( t76)

(a) Show that the transition amplitude has the form:

: f mo lttz .*o I imu;


r .t rlqi , til
@
l*, ,i" rr, - al l,,r",N,, _ a
,l(q? + q) cosat(t y - t) - zq ru,ll
I
. Q.77)

(b) Show that for an initial wave packet of the Gaussian form

t,(q, o) : (?''o "*olryrq - o)rf. (1.7s)

we have

: (T)''t ,t)'f . (1.7g)


lr1"(q,t)12
"*rl-*r(q- acos
Namely, there is no spreading of the wave packet.
18 Field quantization 1.8

(c) In general, the transition amplitude, as a Green's function, can be expressed


in terms of the energy eigenfunctions as

(q' , t'lq, t) E,(t'-t)


-T o, @)Qi,(q) ,i (1.80)

where Q"@)
- \qln) and Hln) - E"ln). Show this and then work out the ground
state energy and wave function by taking the limit of t 0 and t' --> -ioo in the
transition amplitude (q' , t'lq, t).
-

Solution to Problem 1.8


(a) The action being a quadratic function, the transition amplitude, according to
Problem 1.6, has the form of

lqf ,tflqi,til : F(tf ,/;)exp lis,(qt,tyi ei,t)1. (1.81)

Thus we need first to calculate the classical action S,, then the prefactor F.
Given the Lagrangian we can immediately write down the equation of motion:
q + r2q _ 0. Its solution corresponds to the classical trajectory: q,(t) _
A sin at + B cos rttt wrth its coefficients A and B to be fixed by the boundary
conditions of q, (t : ti =0) : e; and Q, (t
A (Qf - {; cos roty)lsinoiy. Thus,
- t) - qf.Wefind B - Q; and
-
sin a(tr - t) * (1.82)
Q,(t) - =+
SrntDt s [q;
- arsina,lr]

with the velocity

Q,Q) - -= [-q,.o, o(tr -


Sttt {Dt.y -
t) * qrcosal/] (1.83)

The classical action is

s,(qr,trt qi.o)
- T Ir" a,la?(r) - ,rqltttl
: #=uT Ir" at tq? cos2a(tv - t)'
+ q? cos2oi - 2qiq r cos a,l (t y - 2t)1. (l .84)

lt is straightforward to do the r-integrals

mco2 f ''sin2otl - sinatff


s,(qf,t7:qi,o):
2rir] ,r,lot
+ qi''sin2otr
2. - 2qiq t , )
^
: ma
ltq? + a2) cos cot r - zq,q rf. 1.8s)
ffi (
1.8 Path integral for a harmonic oscillator 19

This is the expression for the classical action that appears in the transition
amplitude:

lqt, t flqi, ti) - F(tf ,/;) exp lis,(qr, ty, Qi, t)1. (1.86)

Now we are ready to calculate the prefactor F (t f , /r). It can be determined from
the condition of

r
(q r, t rlqi, ri) -
J
I dq(qr, tyle, t) (q, tlqi, ti) (1.87)

where we have inserted a complete set of state {14, t)} for a fixed time t. Explicitly
writing this out, we have

F (t f , /;) exp li S,(q r, t y; ei, t)l

- F(tf,t)F(t, D I dqexpli S,(qr,tyi e,t) + iS,(q, ti 4i,t)f.' (1.88)


J

The integral on the right-hand side has the form

t/- dqexp(-Ao'+ na>)'c - (i)''' *,(#* t) (1.8e)

where

r' -
A
ma fcosa,,ftr - t) |
cosr,,l(r - li)l (1.e0)
2i lsina(ts - t) '
-t
-------)-i----------- sin a(t - ri) I'

For our pu{pose of calculating the prefactor, there is no need to work out the B q * C
term as it only contributes to the exponent, which must match the i S,(q f , t y , Qi, ti)
-l
factor on the left-hand side of the equation. Thus with the presumed cancellation
(check this) of the resultant exponential factors on both sides of the equation, the
only relevant Gaussian integral is
r/2
- ti) .
IorexpAq2:(;)'/2-1ffi (1.e1)

This means that the prefactors have the following relation:

lffi'";rtn a(tr -,)f''' F(ty,t;)

- [sin co(tt - t)ft2 FUf ,r) ' [sin @(t - t,)11/'F(t,t). (1.92)

From the above equation we can deduce the desired result:

F(tr,ti):1ffi)"' (1.e3)
20 Field quantization 1.8

(b) From Problems 6 and '7, we have the relation

th@, t1
- J[ on'@, tlq' ,ol4(q',0) (t.e4)

where

(q, q: + q'2v cosat


- ,nn,ll
trq' ,
l#:ir,;f''' "*rl* l(q, .

(1.es)
and, as is given,

Vo(q',0) - (T)"0 "*rlry(q' - ',f . (r.e6)

Putting them together,

,Lo(q, il : (T)''^ {;r)''' I on'


x exp + q'21 cosat - zqq'l - ry(q' -.'fl.
{ffiI@'
(r.97)
The exponential integrand having a quadratic function of q': {...} : -Aq'z +
Bq' + C as its exponent
@/ ma -ima
-ima cos
A-.......................................- J__-_^tal
2sinat ' 2 2sinat
B - -imat
. @ -f ia sina.rt) (1.98)
sln adr
imtocos o,tt *,
, _ o, _ nr.
2sinat 2
the integral is of the Gaussian type discussed in part (a) and yields the result of
(n /A)rl2 exp((B'lqil + c). we obtain the final wave function

{"(q, t) : (T)''^
l## *ry,-,,,f"' ""0r.
.. I (l.ee)

where the exponent has real and imaginary parts:

-ima r-it''tt 1 imcocoso;t -> rlle)


2ri"rt @+iasinr,;t)'+ Zr.rnr, q'-;o'
.
1"'l: a

: -ry@ - acosrot)2 * rrysinarl (2aq * a2 cos,,t). (1.100)

In this way we obtain the wave function

V,(q , t) : (?t/4 ,-r., p "*p [- T rn - d cos .tfl


x exp ,lrysina;r (2aq -to2 cosat\), (I.I0t)
and an expression for l{"(q,t)12 just as that quoted in the problem. There is
no spread of the wave packet because the original Gaussian wave function is an
1.9 Path integralfor a partitionfunction 2l

eigenfunction of the SHO Hamiltonian, and the time evolution comes in only as
an overall phase.

(c) We will first express the transition amplitude in terms of the energy
eigenfunctions:

(q',t'lq,t) : (q'lexp[iH(t' - t)llql -l\a' lexp[i H(t' - t)lln](nlq)


E,(/ -t) E'(t' -t)
(nlql O"@)Qi,(q1 ,i
- I ei @' ln) - \, .

: -i -+
Setting t
- 0 for convenience, it is clear that in the limit of t' Z with Z oo
this sum is dominated by the ground state l0) contribution:

]i1y@'
, -irlq,0) - 1im Qo@)Q6(q) e-Eor . (1.102)

This should be compared to the limiting expression for the transition amplitude
obtained in part (a)

: (2rffi/)"' + co,,t' - ,n'dl


(q, , t,lq,0) .*n
lffi l(q'' q'y
(1.103)
,

Noting that both cos ol and i sin ar' inuease as )e'r in this limit:

, -ir tq , 0) : (#)''' - ,n' nll


(q'
"*o l+ l(q'' + e\ !e'r

: (T)''' "*nl+r(q,, * rrrl,-a,rtz. (r.104)

Compare the expressions in eqns (1.102) and (1.104) and we obtain the ground
state energy and wave function as

I
Eo
-;r, Qo(il: (T)''o "*o(+n) (1.10s)

1.9 Path integral for a partition function


Show that the partition function of canonical ensemble Z - Tr@-fln1with B -
(kD-r and H the Hamiltonian, for a simple case of one degree of freedom system,
can be written as a path integral representation as

t: I dq" Itdqtexp(, I,-'u at xq,q>) (1.106)

where eo : e(r - 0) - q(-ip) and A(q, ri) is the Lagrangian in Euclidean time
r -it.
Field quantization 1.10

Solution to Problem 1.9


The trace in the partition function can be written as the sum of the matrix elements
of the e-flH operator between the eigenstates of the q operator,
z :D@,le-flH lq,). (1.107)
Q,

Compare this with the path integral representation of the transition amplitude, cf.
in particular Cl-eqn (I.47),
(q r, t flqi, til - \q r1e-inGr-ttl 1n,

Ir^."0 (, 1,," o,ron - H(q,e)r) .


: (r.r08)
lron,
We see that the partition function in eqn (1.107) can be viewed as the special case
of the path integral representation in eqn (1.108) with the restriction
tf - ti - -iP :0
or ti and tf : -ifl (1.109)

and the initial and final position identified qf : ei : Qo. Namely, the path q(t) is
periodical
q(tr) - q(ti) : Qo. (l.l l0)
For convenience, we can introduce the new variable z _ i r. The integral in
eqn (1.108) becomes

,
Io-'u
d.t[pq - H(q,D]: Iou
orl,r# - u<n, ilf (1.111)

and therefore

z - lroa IrdptexpU,u o,l,r# - H(q,r,]l (t.tt2)


For most cases, n- (p212*) + V (q), the momentum integral is Gaussian

I oo*o o,l-*p2 +,fr-


{ lou "]}
: N,*r{ louo,l_:(#)' _"] (1.113)
}
And we have the partition function

Z - N I,on,*, t - I,' o,lT (#)'* u]l,1 114)

where the combination -t@/2)(dqldr)2 + Vl is just the usual Euclidean


Lagrangian |y(e, q). The constant N is independent of temperature (hence has
no physical significance). The integration is over all the periodic paths with the
boundary path qo
- qQ) - q(p).
From this problem we see that the partition function can be obtained from the
usual path integral method through the steps (i) set ti : 0, and ty : -ifl, and
q(tr) q(ti) : Qoi and (ii) integrate over qo.
-
1.10 Partitionfunctionfor an SHO system 23

1 .10 Partition function for an SHO system


The partition function for the case of the simple harmonic oscillator V (q) :
(ma/2)q2 can be obtained as follows

Z - tr(e-fln ) - lr-fr"*(tt2)tu) : (rrt"n T) ( 1.1 15)

Now use the path integral method, as outlined in Problem 1.9, to recover this
result in two ways:

(a) by making use of the SHO result of Problem 1.8, then performing the inte-
gration over boundary values of qo _ qGt) _ q(t;) as a simple Gaussian
integral, or
(b) by using the approach of calculating the path integral as indicated in
Problem 1.6: Z o< det A-112 with A being the appropriate operator for ttie SHO
case.

Solution to Problem 1.10


(a) From Problem l 8, we have obtained the SHO transition amplitude

r, t rlqi, t, :
@
lffi]''= .,,0
l#;6=6
,lra? + all cosa(t y - t) - ,n,n,ll. (1.1 16)

To get the partition function by following the method given in Problem 1.9, we set
ti:0 and qy :
Qi :
qo;the exponent in the above equation becomes

lm(t)
{...} : ;ll=Zqor(cos aty - l) - -2imasinz(aty
12)
q3
I Stn tDt S
sinruot y

_q; 1_ _unaq;
-imosin(aIS2) . atf
).tan _.
_______________
ru (1.1 l7)
cos(utt1l2) 'u 2

By integrating over qo and setting tf : -ifl, we then obtain

t : (*#,,,)''' I dqo expl- ('*"""7) n:]


: (^#r, :(" ry)-'
G#a)''' *)"' "n
: (r,t"n T) (r r18)
24 Field quantization 1.11

(b) We start with the path integral representation of the partition function as given
in eqn (1.114):

Iu,, l: (#)' . + nll


Z_ N raot*, -
I {

*' lrdqtexpl-: I,' 0,, (-#. *)rl


N"[det A1-r1z (1.1 1e)
-
where A
- -(d2 ldt') * of and det A : lln a, with an beingthe eigenvalues of
A on the space of periodic functions f (r) : f (t + P). We expand this periodic
function as /(r) : D, crsin2tttB-', then the eigenvalues are an : o)2 *4n2n2 p-z
for the eigenfunction ein2o'F-t .Inorder to evaluate such a determinant of a series,
we first take the logarithm of the determinant

ln der A :Lln(a2 + 4n2n2 B-21. . (1 .r20)

To evaluate this series, we note that

rndet o :
Io'
dr4!# :r lo' r orW (r.r2r)
In the integrand we have substituted otby X.
*oo
d
].'
d X'
t"detA D <^, +4n2n2B-\-t
- n:-@ (r.r22)

: x-2 +zi{x2 +4tt2n2p_,)-'


n:l

:x-2.;+E(,'.m)
an) *& / n) 1r -l

: 'ty-2jrTL ,or,n("U\ - P L
2n \ 2n/ 2X"or1-,f 2
where we have used the identity

ny cothTTy : |* 2y2
D(n' + y')-t (t.123)
n:l
In this way

lndetA- ['pax"otn
Jo ry:zh(sinh +) 0n4)
or det { - (sinh (flx l2))2. Thus

z - N (rinr'
\
ry)-'
2/
(t.t2s)
which is in agreement with the result obtained in (a) when we recall that the
temperature independent constant in front has no physical significance.
1.11 Non-standard path-integral representation 25

1 .11 Non-standard path-integral representation


Consider the Lagrangian with a position-dependent 'mass'

(q)q'. L : (r.126)
:f
Thus the canonical momentum is p : f (q)q and the Hamiltonian H _
(p212f (q)). Show that the path-integral representation of the transition ampli-
tude has the form

:
(q',t'tq./) N'
I fdqlexpl, I o,ltr,d) - Lurort' /(n)]] e.nr)
Suggestion. One can start with the expression of the transition amplitude

(q',t'tq,t): rv
I taat I ,^*, {
i
l,'
attpq - H(q, p)tl
I

-.u:" I* *dq' dq'exp{'tu'1,'(+)


-,(,,,u#))l (, 128)

and then explicitly perform the momentum integrations.

Remark. This is the counter-example, first given by Lee and Yang (1962), show-
ing that path-integral representation is not always of the form

(q',t'lq,t) : N'
I @qlexpl, I or trn,ill. O.tzs)

Solution to Problem 1.11


For the given Lagrangian, we have

Iro^*r{ i l,' atlon-


(q',t'tq,t): w
lraat hl
-Jls *onr dq'-r
l*
xexp{'i u,lo,(r+-)
#l (, 130)

The momentum integrals are of the Gaussian type IjS a- exp(-Axz + B) -


(tr lA)1/2 exP(B2l4A):

,, :
Io* dp ; exp
f,
u,
lo,
(r;+) #;l
:l2of (n,)f"'
'J ^-^ l-
2f (q) , - - ,rl
- ai-r)')' (l'l3l)
I iar '*P L ffi@i
26 Field quantization I.I2
Since we are eventually interested in expressing the [dq] integrand in the form of
an exponential, we will now write the position-dependent part of the prefactor in
the exponential form lf @)lt/2 - exp { it" f (q) }. In this way the above integral
becomes

,, : (#)'" "*o
[,Iry (+.)' -'t#f,, . (1.132)

This means that the path-integral representation can be written

(q', t'lq, t) : 0n,. . . dqn-r


)yy{zoidt)-n1z I
x exp
{, I lry H-)' -u#),,1
itn
-*| rdqtexpl'
Ilrynz - [(q)6(0)] n'l (r r33)

This is the claimed result. To get to the last line we have used the expression for
Dirac's delta function as

E(0) : rim
dr-*O
l.6/ (1.134)

1 .12 Weyl ordering of operators


Notes on operator ordering
For the simple system of which the Hamiltonian in the form

H,(p,q):#*rrn, (1.13s)

has no terms that depend on both p and q, the quantization is straightforward:


just replace the classical variables (p, q) by operators (F,,i). Thus the quantum
Hamiltonian operator is unique:

^.,
nrp.A) - !lm *t<i,t. (1.136)

and we have the path-integral representation of the matrix element

(qi+rtn@, illq) : *r, (0,.,*+t) eipi(Q +,-q). o.r3i)


I
For the more general case of which the Hamiltonian function Hr(p,4) can have
mixed terms of p and, q, for example p2 q2, the quantized theory is not unique.
Each of the choices: P'A'. Ap'A, PA'P, and Q2P2 will have the same classical
l.I2 Weyl ordering of operators 27

limit. If these choices are non-equivalent, experimental measurements will, in


principle, pick out the correct choice. Problem 12 will illustrate the point that for
a Hamiltonian containing terms of the type qn p^, the path integral quantization
conesponds to, in some sense, the most symmetric ordering, called Weyl order-
ing. It is defined to be the average of all possible orderings of ps and qs. For
instance,

I ,-, I
(pqn)w =
n+l Ita'ra
l:0
)
n+l

I
n
2
(p2qn)* = q^ pqtpq"-t-m. (1.139)
tu -t' -/ ,L,
\' -r 2\, l,m:O
lttu

An instructive discussion of Weyl ordering can be found in the book by


Lee (1990).

Remark. To do Problem 1.12, you may find the following identities useful:

n1
D,-;n(n+t)
t:l
(r-1)

na
rAl
> l' : -tt(n -l 1)(2n * l) (r-2)
E6
1,' :l:,t, + rrl' (r-3)
ELzl
and

2" :r(;) (r-4)

n2'-1 :D'(;) (r-s)

n(n - I)2'-2 :D-,t(t-t'(;) (r-6)

where (;) - @t/tt(n - /)!) are the binomial coefficients. Can you prove these
identities?

Suggestion. One approach to the first three relations will be to use the equalities
Li:o(t - l)* - Li:rl^ - (n + l)* for m :2,3,4. we note that the left-hand
sides are just different combinations of Di:o/ft with k
- 1,2,3. The last three
identities (I-4,I-5, and I-6) simply follow from the successive differentiations
(d" /dx')(l * x)" at x : I for m : 0, 1,2.
28 Field quantization I.I2

Problems on Weyl ordering

(a) Show that the Weyl ordering of operators p and q can be written as

(pq,)w = L- @, p -f qn-t pq+ ... * pq,)


n+r

- :1n i I'r . "' ,, , q' pqn-'.


z"Et!|n-t)!
/-t
( 1.139)

with the matrix element of

\qi+rl(pq\wlqi) - ('+)' rn,*,lplq). (1.140)

(b) Show the Weyl ordering of the operator product with two powers of p:

(p2q')w = (n +#+r) ,t^:rn*ont,rn-t-m


n' (l.l4l)
- :on,Lt
/"
f ,,. ,., nt ,2nn_.t.
llln - l)l
-l_t
which leads to the matrix element of

(qi+rl(p'q')*lqi) - (**)' {n,*,lp'lq,}. (t.142)

Remark. According to eqns (1.140) and (1 .142), the matrix elements of q in the
Weyl ordering are just of the form (4;11 + qi) /2 as prescribed in the path-integral
formalism.

Solution to Problem l.l2


(a) Before working out the situation for general n,let us first consider the simplest
non-trivial case of n 2'.
-
(pq')* = \{a2 n -t qpq + pq') - pqz + iq (1.143)

where we have used the commutation relations Iq* , pl :


imq^-r to move 4s to
the right of ps. The right-hand side is shown to be just the claimed result (as given
in the problem) by further application of these commutation relations:

l."l -t 2qpq * pq') : + ziq1 + 2(pq2 + iq) + pq2l


)to'n olrna'
: pqz * iq. (1.144)
I.I2 Weyl ordering of operators 29

The matrix element then has the simple structure:

\q+rl(pq')*lq) - )<n,*rlqz p -r 2qpq + pq'lq,)


I
-*\
:(!*,+qi\2,- @i+tlPlqil.
t. (1.14s)
\ '; )
Now let us repeat the above steps for the general rz situation,

I .n
(pq")*=-,-Ltr'
n _f | .1:tJ
pq'-t)

| ."
l{nu" * ilq"-t!
n*I',_u

-- pqn + lnq'-t
2'' , (t.146)

where to reach the last line we have used the identity (I-1). This result is the same
as given in the problem, eqn (1 .139), because,

+E#"q' pqn-' :* +i tq"-'\ )


D"#"(pq"
- +r
' 1:o
(1) (Pq' + itq'-')
\'/

: pq' * Lnn"-t (1.147)

where we have used (I-4) and (I-5). The combination of eqns (1.146) and (t.t4t)
yields the claimed result:

(pq")*
lJ- nl
: 2nf;;q' pq"-'. (1.148)

The general matrix element can now be written as

(qi+t\(pq")y|qi|-+n"*r!-^.(q,*,Iqtpq"-tIqi)

:; (qi+rt
f 0'" *'qi-' Ptqi)
(Qi*t--r qi)' (n,*, lplq). 1.r4s)
- \ 2 /\
(
30 Field quantization l.l2
(b) Before working out the general n situation, let us first consider the simplest
non-trivial case of n 2:
-
(p'q')* : f{a'n' * qpqp + qp'q + pq'p * pqpq + p'q'). (1.150)

The right-hand side can then be rearranged by using the commutators

lq*, p'f - im(q*-t p + pq*-t), (1.151)

so that

(p'q')* : * { lp'q'
+ 2i(qp + pil) + t@q + i)qpl
+ t@q + i)pql + ln'o' + zipqf + ln'a' + ipq) * o'n'l
- Ircp2q'+ tzipq - 3) : p'q' +2ipq - i. 0:52)
This last expression can be shown to be just (q2 p2 I2qp2q + p'q\ 14 which is
eqn(I.l4l)withn -2:
j{a'n'*2qp2q + p'q') tol{n2a'+2ipq *2iqp)
-
+2(p2q *2ip)q + p'q'l
- Il+p'q'+*ipq -2]: p'q'+zipq - L.
(1.1s3)

Thus

. 1 ., I . )
(p'q'), -;(q'p'+2qp'q + p'q'), (r.1s4)

and

@i+tt@'q')*lqt) - (*+)' ,r,*,1 p'lq,). (l.lss)

Now let us repeat the above steps for the general n situation.

(p,qn )* = -(n +, :,_


lt(n f z) f,
,6 a, na* unn-t-m
1.-^:O

2
(n D <on' I
* l)(n * 2) r.*:o
ilqt-t)q* pq"-'-*

2n
{na'*m onn-t-m * ilnt+--t
(n -f l)(n l2) |
onn-t-m 1

t.*:o
r.t2 Weyl ordering of operators 3l

2
(n*l)(n*2) lpq'** + i (t + m)qt+*_,) nn-r-^ .

,|lp
+ rt lnat+*-t + i(l + *- l)qt**-z1n,-_.t_.-l

(n+t)(n+2)
n

x Dlo'n" +i(2t +m)pq"-t -l(l *m-t)q"-'1. (1.156)


l,m:0

We will discuss separately the three terms on the right-hand side of this equation.
In the first term we have the sum
nnn-lnnn
I
l,m:o
- II - D@
l:0 m:0
l:0
-t +1) - I(n*t)-
l:0
II:0 I
- (n -r r)' - ln@ + r) : r t)(n * 2) Lz@ (1.157)

where the identity (I-l) has been used. We then evaluate the sum in the second
term of eqn (1.156):

nnn-mnn-m
D,ru*m)
l,m:0
-zDI,*D,*D,
m:0 l:0 m:0 l:0

: zf;@ - m)(n - m *l) + I m(n - m * t)


m:0 m:0
n

ltn@ i
- m:0 r) - nml - n(n + D2 - ln21n + t1
- )n@ * r)(n + 2), (1.158)

where (I-1) has been used. To evaluate the sum in the third term of eqn (1.156),
we will need to use all three identities (I-l,I-2, and I-3):

D,'u-tm-r)
l.m:0
n n-l n n-l n n-l
-ItD*+It'D-ItI
/:0 m:0 /:0 m:0 l:0 m:0
n

- It:0 lrltn - t)(n - t + r) + Q2 - D@ - / + 1)]


lg
- .',)u
t
)'[-3l3 i3@ + DP -
!:o
@ +2)tl
32 Field quantization 1.13

I l- + n(n*l)(2n+1)
::l-3
21
n2(n
4
.
l\2
*3(n*l) - (n *"rJt+Il
: 1
-f l)(n + 2)(n - l)' (l.1se)
8'@
Substituting the results from eqns (1 .151) to (1.159) into eqn (1.156), we have
(p2q")* :* inpq"-' - i"(n - l)q"-z.
p2qn (1.160)

we next show rhat rhe right-hand side is equal to (l/2')Li:o(1)q'p'q'-',

r2' i (,\o, ,,r,-,


k\t )'
: ;E (;) lP'q' -t it(qt-'\P * Pqt-')f q'-'
: +f (;) lP'q" -t itlqt-t + Pq"-')l
'nn-t
: ;f (;) lp'q' *2itpq'-' - t(t - t)q'-'l' (1'161)

Using the identities (I-4, I-5, and I-6), we have

;k}n n'n-t - p2q' + inpq'-' - fu@ - r)qn-'' (t'162)

Comparing eqns (1.160) and (l .162) we see the relation of eqn (1.141) is satisfied.

1 .1 3 Generating functional for a scalar field


The generating functional for the free scalar field @(x) is of the form

w,Ut : a^. r,l


I ror,..n { i I
( 1.163)

where the Lagrangian density with an external c-number source "I("r) is given by

Lt :)rrrf -).r'r' + lo. (1.164)

(a) Show that such an Ll leads to the equation of motion (n + p\O - ,/ with a
classical solution that can be obtained by the usual Green's function method:

Q,@) :- doy Lo(x - y)/(y). ( 1.16s)


I
The Green's function Ai''(x - )) is the Feynman propagator for the scalar field in
position space:
f d4k uik(x-t)
Ar(x-rr:Je"f p_uz*t, ( 1.166)
1.13 Generating functional for a scalar field 33

(b) Show that, by a change of variable Q@) : Q,@) * ry@) in the Lagrangian
density, the generating functional can be expressed in the form

w"Ul- Nexp daxday r(x)Lr@ - y)/0,)} (r.167)


{ tI
where N is a constant, independent of ./.

Remark. One way to understand the i e prescription in the expression (1.166) for
the propagator Ap(x - y) is to note that the path integral expression in (1.163)
is not well defined because of the oscillatory behaviour of exp {i I aax Lrl for a
real L1.In principle, we have to go over to the Euclidean space-time t : ir in
order to convert this oscillatory behaviour into a damping one. We then return to the
Minkowski space by the method of analytic continuation. However, a much simpler
approach that will accomplish the same task is to add a terrn exp{i I d4x(ie)E2]
with a > 0 in the generating functional. This will provide a strong damping to
the Gaussian integral. The generating functional will then be well defined. The
Green's function for the corresponding equation of motion (l + p2 - i e)Q
- 1
is of the form

(n" + p2 - ie)Le@ - y) - -64(x - y) (1.168)

with the solution as given in eqn (1.166).

Solution to Problem 1.13


(a) The minimization condition of the action (modified according to the ie pre-
scription as discussed in the Remark):

Jol
Ioo*Lt: Io^.llraOf - ie"' -
srrdl - ie)Qz + (1.16e)

is simply the Euler-Lagrangian equation

0L' r"a,,( aLt \--(pr'-ie)Q+J-nd:0.


6os, - aO -
- (.rCr"Ol/ (1.170)

Thus the equation of motion is

(r,+p2-ie)Q@)-J(x). (1.171)

This equation can be solved by the usual Green's function method


r
Q,@):- JI Lr1u.-y)J(y)day (t.172)

with the Green's function (the propagator) being defined by the equation

(n, + p2 - ie)a^p(x - y) - -64(x - y). (r.173)

Equation (1.I73) can be solved by Fourier transform with the solution being given
by eqn (1.166).
34 Field quantization 1.13

(b) The change of variable Q@) : Q,@) * n@) in the Lagrangian density of
eqn (1 .169)

:
s.,tdl -I oo* Lt
I o^.lLor-" - p\o * ta) (1.17 4)

leads to an expression of the action in the form of

I o^.{l+r,r
2)Q,
silQ, * ql - +t a rQ,)

-ryt(n + p\0, - rl - i,rtfa + u'>nll. (1.17s)

Because @, satisfies the equation of motion, the second term on the right-hand side
vanishes and the first term can be simplified, .

I o^.I-io,ro + p\Q, a lQ,) - I oo*l-.Lo,t a rQ,)


: dax J(x)Q,e)
+
|
- -+ I o^, l(x)Lp(x - y)l(y)day.
(1.176)

The generating functional being

:
w,IJl:
| ,or,exp{i s"rtdl} I ,or,exp{i silQ, * nll,
(r.r77)

we can then factor out the part of action which is independent of the 4(x) field:

w'tr)- l (x)a,rQ - Dr (v) o^rl


exp
t ;I dax

* + u'tnatl
| wru)l exp I ; I do* n(x)(n

- y)r(t)o^rl
Nexp
t ; I dax l(x)Ar(x -
(1.178)

where

*: -, I do,,t (x)(r + ttz)rl@) (1.r79)


lvn@)t*o {

is independent of J (x).This is the desired result.


l.I4 Poles in Green's function 35

1.14 Poles in Green's function


Consider the following generalization of the LSZ reductionformula: Iet T (q, . . .)
be a Fourier transform of the vacuum expectation value of a time-ordered product

r(q, ...) -I o^, eiq' (llr.zre)B(y)...)10) , (1. r 80)

where the operators A(x)B(y) ... can be either elementary field operators like
Q@) or composite operators llke Q2(x). Suppose the operator A(x) has a non-zero
matrix element between the vacuum and some one-particle state carrying quantum
number a,

(0lA(0)lp,a)#0, with (p'f mllrtz-Eo. (1.181)

Show thatintheupperhalf of the energy plane Qo > 0 the functionT(q, ...) has
apole structure of

tim T(q, ...) - (0lA(0)lp' a-)(p' a-lz(B(v) "')10)


- for qo > 0. f r.rszl
q2-^i q2 - mj + ie

Solution to Problem 1.14


We only need to consider in detail the simplest non-trivial case of two operators.
Generalization to cases involving more than two operators will be straightforward.
f
(q)
T
- JI do * ein'' (0lT (A(x) B(0)) l0)

: I aor ria * l0(x)(olA(,r)B(0)10) + 0(-x)(0lB(0)A(x)10)]


J

_ [ ao * ,,n - le(*) f (0lA (x )ln) (nlB(0) l0)


tl?
* o(-x) f fotB(o)tfr)(tte(x)to)l. (1.183)
kt
For the one-particle state lp, al in {ln )}

we can use the


P-l#
translational invariance to write (0lA(x) lp, a)
( 1.184)

_
(0lA(0) lp, a) ,-in'' , and

o^- rita-D'xg7x) : (2n)363(p - q) lo* orexp{i (qo - E,) .t}


I
- - q) --+
12n)3 63(p
Qo- La*te-
t-(qo +-n)
: lztr)363(p - q)
''qt-mf,+ie'. (1.185)
36 Field quantization I.I4

where we have used p - q so that

eo_ Eo _ _13 t=3, _ qT_-p' _yZ _


-@o+E,) (eo+n; / -*',.
@o+n;'
(1.186)

Inserting this into eqn (1.183), we can evaluate the function T (q) in the limit of
q2 - mf,wrtheo > 0

r (q) - I #(2tt13631p - offia


x (0lA(0) lp, a) (p, alB(0) l0) + .. .

. (0lA(0)lq, d) (q. al B(0)10)


-iffi+..' (l.l87t

where the ellipsis stands for the remaining terms, which are clearly free of poles
and hence can be dropped in this limit. This is the desired result.
We also note that for the case of (q, a lA(0) l0) + 0 there will be a pole term in
the qs < 0 region of the form

(0lB(0)l-q' a)(q' alA(0)10)


tim T(q) --- for 4s < 0. (1.1 88)
qi-l*Z^ "' ie 1
q2 - m] +
2 Renormalization

2.1 Counterterms in )"Qa theory and in OED


(a) Use the power-counting argument to construct counterterms and draw all the
one-loop divergent 1PI graphs for the real scalar field theory with an interaction of

Lin,: -\o'-fr0^ (2.r)

(b) Use the power-counting argument to construct counterterms for the.QED


Lagrangian

L : ,lr(iy*0r" - m)rl, - erlryPrh Atr - Ir*"r" (2.2)

where Fp, :3*A, - 0rAp.

Remark. One of the key features of the QED theory is that it is invariant under
the gauge transformation (see Cl-Section 8.1 for details)

th@) --> vb'@) : s-iu(x) lr@)


--> lb'@) - ,ia(x) ,hf*l
^b@>
Au@) * A'u(x)
- A*(x) + ! auo61. (2.3)

The desired counterterms must also be gauge invariant.

Solution to Problem2.l
(a) The superficial degree of divergence D is related to the number of external
boson lines B and the number of 03 vertices nrby Cl-eqn (2.133):

D-4-B-nr. (2.4)

(i) B - 2 (the self-energy diagram): see Fig. 2.1(a) for the one-loop divergent
lPI graph. Thus D
- 2-nr. Since the number of external lines is even, ,? I must also
be even: nt :0 and 2,Ieading to quadratically divergent Q2 and logarithmically
divergent }rQ \rQ counterterms.
(ii) B - 3 (the Q3-vertex diagram): see Fig. 2.1(b) for the one-loop divergent
lPIgraph. Thus D
-I -nt:0,asn 1 mustbeodd(hence nt:1), leadingtoa
logarithmically divergent @3 counterterm.
(iii) B - 4 (the Qa-vertex diagram): see Fig. 2.2for the one-loop divergent lPI
graph. Thus D 0leading to a logarithmically divergent Qa counterterm.
-
0+ <
38 Renormalization 2.1

(a) (b)

O
Ftc. 2.2.

Y OO8
YYY
(a) (b)

Frc. 2.3.

Remark. We may also consider the 'tadpole diagrams' of Fig. 2.3 with B - |
and D - 3 - n1 with odd nr : 1 and 3. The quadratically divergent ones are
shown in Fig. 2.3(b), while the logarithmic divergent one is shown in Fig. 2.3(a).
(b) Here both the external boson and fermion line numbers enter into Cl-eqn
(2.r33)
a

D:4-B--F.J2 (2.s)

Let us enumerate all possible terms starting with the lowest possible external
fermion (electron) and boson (photon) lines:
(i) f : 0, B- 2 (the vacuum polarization diagram, Fig. 2.4(a)): thus the
degree of divergence D
- 2 (i.e. quadratically divergent). In order to have a finite
term we need to expand this contribution r ur(k) beyond the second order in photon
momentum ft:

nu,(k) : ftp,(0) + k2gptrr(0) + krk,r2(o) +7p,(k). (2.6)

Thus the required counterterms are (A)2 and G il2 . But there is no gauge-invariant
counterterm of the non-derivative form (ilz .However, there is one gauge-invariant
term of @ilz: (}uA, - A"A)(AI'A' - A'Ap), which is the same form as the
photon kinetic energy term FurFP' .
2.2 Divergences in non-linear chiral theory 39

.affiiF4,
\,/^\,/-\-4.#i.4i$fifl#ril-r\-/\-/^\-,
IL \gW V

(a) (b)

F,?.n+^ *& /
vvvlf!#4!{fiw
A\FffiFN\

YiidHE#.y \
/

\:&:!v \i!.i:ijz \

(c) (d)

Frc. 2.4.

(ii) F :0, B : 4 (the photon-photon scattering diagram,Fig.2.a$)): here


we have D - 0 (i.e. logarithmically divergent).
I pvt p(ki) : I pvt p(0) +I prxp(ki) (2.7)

where Tprxp(ki) is convergent. The required countertenn is of the form (D4.


However, it is not possible to construct such a term which is gauge invariant. Thus
we would expect I t rxp(k;) itself to be convergent.
(iii) f : 2, B - 0 (the electron self-energy diagram, Fig.2.4(c): the degree of
divergence is one; hence it is linearly divergent.

t(p) - t(o) + ilE'(o) + t(P) (2.8)

where we expect a(0) to be linearly (or logarithmically) and I'(0) to be logarirh-


mically divergent, t (p) being convergent. The required counterterms are I (0){r{/
and E'(0) rh y * E rrlr, respectively.
(iv) F : 2, B _ I (the electron-photon vertex diagram, Fig. 2.a@): it is
logarithmically divergent, because D 0.
-
I p(p,Q) : fp(0) * fu @, q) (2.e)

with a counterterm of the form $ yP {t .

2.2 Divergences in non-linear chiral theory


The non-linear SU(2) x SU(2) chiral Lagrangian is of the form
t2
L-+TrePu'atrt) (2.t0)

where
/ it. 6\
u_exp('
r /
and t- (rr, rz, 13) are the Pauli matrices.
40 Renormalization 2.2

Also / : 94 MeV is the pion decay constant. (For more discussion of non-linear
chiral theories, see Problems 5.7, 5.8, and 5.9.) Use power counting to enumerate
all the superficially divergent Green's functions at the one-loop level and construct
the appropriate counterterms.

Solution to Problem2.2
The superficial degree of divergence is given by

D-4-B+L",(bi*dt-4). (2.rr)
i

For the chiral Lagrangian, we have the number of derivatives in the i th vertex being
di :2, for all vertices. Substituting the topological relation

2(rB) * B :Dn,U, (2.12)


i

into

7-(IB)-Ini]-r (2.r3)
i

we get

l-
t: tLr,(bi B
-2) -t + t. (2.r4)

The superficial degree of divergence is then

D
-4- B +D",(bi -2):2+2L. (2.ts)

This gives the result that at the one-loop level (L : 1) the degree of divergence
D : 4, independent of the external lines. It implies that the number of derivatives
in the counterterms should be four or less. The term with two derivatives is just the
term in the original Lagrangian. The four derivative terms should have the form

@pilQt"Q)(a,d)(A'0)Q", neven. (2.16)

Similarly for the other counterterms. Taking into account the SU(2) x SU(2)
symmetry, the counterterms are of the form,

lrr(Tpur a*U112
Tr(|pUl 3*U A'Ut A,U)
Tr(|pUUt A,UUr ApU[Jt A'UUt)
T r (0p uI au u)T r (a,ur a*u1

T r (Ep u Ut 3,U Urlr rlApu Ur A' U UI). (2.17)


2.3 Divergences in lower-dimensional field theories 4I

2.3 Divergences in lower-dimensional field theories


Consider theories with scalar field $, fermion fieId rlr, and massless vector gauge
field A, in a d-dimensional space-time (d - 1 space coordinates and 1 time
coordinate).
Express the superficial degree of divergence of a diagram D in terms of the
number of external fermion F, boson B lines, and the number of fermion fr,
boson b; lines and the number of derivatives d; in the ith type vertex. Namely,
deduce the generalization of Cl-eqn (2.133) to a d-dimensional field theory. Keep
in mind that the propagator in d-dimensional field theory has exactly the same
form as those in our physical four-dimensional momentum space. For example,
the propagator for the gauge field Au in the : I Feynman gauge is

i Lrr,(k)
'
: ::+ (2.18)
k' * tt
Use the formula deduced in (a) to write down all possible renoffnalizable and
super-renormalizable interactions for dimensions: (i) d 2 and (ii) d 3.
- -
Solution to Problem2.3
From the structure of the graph we have the relations

B + 2(lB) :Dr,b,, F + 2Qn :Dn, f,, (2.re)


ii
as well as

(IB)+(In-D",rI-L (2.20)

where -L is the number of loops in the graph. The superficial degree of divergence
can be calculated by using the relation (2.20)

D - dL -z(tB) - Qn * Fn;d;

: a (ua> + en- ,,* r) - 2(tB) - (In *lnidi- Q.21)


P i

Eliminatin g IB and IF by eqn (2.19), we get

,:(=) hnb-,]
.G) [P.,,, -,)
+Ini(di-il+a
i

-d-(=)'-(+),
a \-,,1a, . (T) u, * (? r, - o)
42 Renormalization 2.3

Or

D_d_(=)u_f+)
\ 2 )- \ 2 )
F*lni6i e22)
i

where d; is the index of divergence,

u,:(T)u,*(+)r *di-d (223)

Clearly, these results check with Cl-eqn (2.133) for the case of dimension d - 4.

(i) The d
- 2 case: Equations (2.22) and (2.23) are reduced to

D-2-{*)-, ,-,1i6i' : 'fi


- 2'
T*
6' di (2'24)
2
. super-renormalizabt" in,".u.,ion,6; < 0

O', n:3,4,... 6(0")--2,


,hrh[', n:1,2,... S(rlrr[O\- -1,
,ltyuLeu, S(lryur\Au):-1. (2.25)

Note the interaction of the type ArAp is super-renormalizable but not gauge
invariance.
. renormalizable interaction,6i : 0

di :0, (rlrrD', (r/rrlD'Q'


di : l, .
,lryrLapOO',LYr\urh Fu' ,

dr :2, (\qQOPQ)Q'. Q.26)

(ii) The d
- 3 case: Equations (2.22) and (2.23) are reduced to

t=
D -3 - -F * f/r n;6;,
' 6i: di +?I +' firt - 1. (2.21)
2

. super-renormalizaUt. int.ru.tion, 6; < 0

di : o, O' , Oo, Ot , ,lryrtl, Au ,


di : l, SuQApf. Q.28)

. renormalizableinteraction, 6; : 0

di : 0, 06, 'l"ltO',
di : l, orQN'L2. Q.29)
2.4 n-Dimensional'spherical' coordinates 43

2.4 n-Dimensional'spherical' coordinates


In an n-dimensional space, the Cartesian coordinates can be parumeftized in terms
of the 'spherical' coordinates as

xt : rn sin 9n-1 sin 0r-2" 'sin 02sin0y,


x2 : r, sin 0r- 1 sin 0n-2 " ' sin 02 cos 01,

x3 : r, sin 0r-1 sin 0n-2" ' sin 03 cos02,

Xn: rnCOS0r-1 (2.30)

where

0< il 12tr, 0 < 02,02,...,1n-r 17T, (2.3t)

and

1-r?+xl+...+*3 (2.32)

Show that the n-dimensional infinitesimal volume is given by

dx1 dx2 dxt . . . d.x,


-rf*l 0n-)'-2 (sin0,_2)'-3 . . .
lsin
x (sin e)@Ot d?z. . .d1n-) dr, (2.33)

as used in CL-p. 53.

Solution to Problem2.4
We will solve this problem by finding the relation between the volume factors in
nand n - | dimensions. Namely, we will proceed from the simplest n 2 to
higher-dimensional
-
cases :

(n
-2) ----+ (n:3) ----+ (n
- 4) -+ (generaln). (2.34)

(a) n
-2
Here the two Cartesian coordinates (xt,xz) are related to the familiar polar
coordinates (rz,9r ) by

xt:r2sin91 , x2-_ r2cos?t, 0<gr <2T, rl-xl+*?. Q.35)


The distance ds2 between neighbouring points is given by

(dt)2 -(dx)2 *(dx)z -(dr)2 +r?@e)2. (2.36)

The 'volume' elernent is then the product of segments in orthogonal directions,


i.e. the product of the coordinate differential with the appropriate coefficients as
indicated by the (quadratic) distance relation:

dV2: dxt dxz: (dr)(rzd?) - rzdrzd9t. (2.37)


44 Renormalization 2.4

(b) n
-3
Consider a sphere in three dimensions. If we cut this sphere by a plane perpendic-
ular to the -r3-axis, we get a series of circles in the planes spanned by Cartesian
coordinates (rr, x2) which are related to the polar coordinates (rz, gr) by

xt:r2sin91 , x2:r2cosel, 0<il<2ft, rl-xl+*3. (2.38)

The infinitesimal distance on this plane can be expressed in these two coordinate
systems as

(d*)2 + (d.x)2 - (dr)2 + rl{de)2 (2.3e)

We can also cut the sphere by a plane containing the x3-axis, resulting in a series of
'vertical circles'. On these two-dimensional subspaces, the Cartesian coordinates
arc (r2, x3) and the corresponding polar coordinates are (ry,02).We recognize that
02 is the usual polar angle.

12: f3Sin02, X3: \CoS02, 0< 02 < n. (2.40)

rl - r]-t xl - *? + *l + *1. Q.4t)


As in eqn (2.39), the infinitesimal distance can be expressed in two equivalent
ways:

(dr)z + (dx)2 : (dry)2 + r?@e)z. (2.42)

Combining the two sets of coordinates in eqns (2.38) and (2.40), we get the usual
spherical coordinate relations,

xt : 13 sin 02sin01,

x2 : 13 sin 02cos01,

X3: \COS02. (2.43)

We can turn the distance formula

(dt.)z - (dxt)2 + (dx)2 + (d.x)2


into

(dt)' - 7dr)2 + rlgetlz + (d.x)z (2.44)

by using eqn(2.39). This can be furtherreduced, by using eqns (2.40) and(2.42),to

(ds)2 (dx)2 + (d.x)2 + (d.x)2


-
(dry)2 + rlget)2 + ,?@e)2
-
- 1dry)2 + rr2 sin e|@et)z + ,lge;z. (2.4s)

The volume element can be obtained from the product of these three terms

dV3
- @r)(r3 sin 0z d7)(rz de) - r.2 sin 7z(drz dil d0). (2.46)
2.4 n-Dimensional'spherical' coordinates 45

(c') n-4
We can also imagine cutting the sphere in four dimensions by the manifold (xq
-
constant) to get a three-sphere with radius 13 : 14 sin 03, where we can introduce
three-dimensional spherical coordinates (r:, 0r,02) as in eqn (2.43)

l':,=;l :l
,;
- 1 # ,.-,;J'; (2.41)

with the distance formula (2.45) "':"1';:'


(dx)2 + (d.x)2 + (d.x)2
- (dry)2 + ,l{ae)' + ,? sin2 e21de;2. (2.48)

Now we introduce two-dimensional polar coordinates (rq, gl) in the (r3, xa) plane

13 : 14 sin 03, x4 : 14 cos 03, (2.49)

with the distance formula

(dr)2 + (d.xq)2
- (drl2 + rlgeS2. (2.50)

In this way the infinitesimal distance in this four-dimensional space can be rewritten
by using eqns (2.48) and (2.50)

(dtq)2
- [(dx)2 + (dx)2 + (d.x)2] + (dxq)2
- [(dry)2 + rl@e)' + r? sin2 ez(det)'] + (d.*02

- 1(dry)2+ (dx:+)21+ rlge2l' + r? sin2 e21d0;2


(dr)2 + r?@e)' + rl6e)' + rl sinz e2@e)2. Q.5I)
-
The infinitesimal volume element is then

d u+ - (drq) (rq d0) (rt d0) (rz sin 02 d0)


: ri sin2 03 sin 02(dra d7t d7z d0) (2.52)

where we have used eqn (2.49) to reach the last expression.


(d) General n

xt : r, sin 0n-1 sin 1n-z '' ' sin 02 sin01,


x2 : rn singn-1 sin 0r-2"'sin 02cos01,
x3 : rn sin gn-1 sin 0r-2" ' sin 03 cos02,

::
xn : r, cos9n-1 Q.53)
where

1-r? +xl+...+*?, 0 <h <2n, 0 <02,0t,...,7n-t 1rT.


(2.s4)
46 Renormaliz.ation 2.5

The infinitesimal distance is

(dt,)2 - (dxr)2 + (d.x)' + . . . + (d.x,)


(dr,)2 + 4@e"-r)' + rj sin2 0,4(do,-2)2
-
* rl sin2 0,4 sinz o,-2(do,-)2 + . . .

* rf, sin2 0n-1sin2 0,-2.. . sin2 0z@0)2 (2.55)

and the volume element is

- dxrdxz...dxn
'
dVn
:':'i,ii;!:;;,r,;,'.:';;];
es6)

2.5 Some integrals in dimensional regularization


Use the dimensional regularizationto derive the following results for the Feynman
integrals with denominator power u in n dimensions:

(a) Is(a.rt)-
I#
.(-n)n/2 f(a - nl2) I (1 <'7\
(2n)" f (o) (M2 - P2 + ie)u-(n121'

(b)
. fdnk
Iu\a,rrl:Je"y@ kp

: -pplo(a,n). (2.58)

(c) Ir,(a,"r-Je"yffi
f d'k kuk,

: Io(cv, * )3r,ffi) e.sg)


^lrrp,

(d) Irur(a,n)-l#ffi
: 1o(cv.
t-,
n\lnup,pp + i(su,pp I Bupp, I S,ppp)

M2-p2 I
"1"-42-nl'
2.5 Some integrals in dimensional regularization 47

Solution to Problem2.S
(a) In the Feynman integral

Io@,n):l# , (2'60)

the denominator can be written as

D - *2p. k + M2 * ie - (p -f k)2 + (M2 - p2) + ie


k2
:
k'2 + (M2 - p2) + ie (2.61)

where k'
- k * p. For the case p2 , M2, we can perform the Wick rotation to get
D - -82 + M2 - p2 + it - -(82 + o2) (2.62)

where a2 : p2 - M2 -ie and

fd"k I I
(2'63)
Is(u' tt'
-' J Q"y FD" G'z + a'zY'
As usual [see Cl-eqn (2.112)1, the n-dimensional angular integration gives

f
on' :
2nnl2
(2'64)
J ro'lD'
Then

rg(a.n):,8;#1,-##
: i (-1;-" 2nn/2 , f* 7nl2-ttry
(2ry r or lDl Jo e + ary
t(-t)-" rn/2 I t@12)l(a - nlz)
(2n)' l(n12) (q2\a-tn1zl f(cY)
: (-n)"t2 l(u - n/2) 1i
\ 2"a5 t
Qn, l(" ) @'z - p'? + D"-";'?'
One of the most common convergent Feynman integrals has cv 3,
-
1o(3,rl):i#
_ (:),,1_'r(3 - (nl2)) e.66)
(2n), f (3) (M2 - pz + ie)3-n1z
which gives. for n 4,
-
fdlk I
1ur3,-lr:;/
e"y@
il (2.67)
32n2 (M2 - p2 + ie)'
48 Renormalization 2.5

(b) I,(a,d:1ffiffi. (2.68)

As before, we set k'


- k* p. Then

I,(a,,,:l#ffi (26s)

and the term linear in k'u gives zero because of the symmetric integration. The
result is

I r(u, n) : - ppls(u, n). (2.70)

(c)lu,(a,n):l#ffi
f d'k -
=JQ"Y@
(k'u pil(k', - p,)

: I d'k kuk., + PuP' f dnk I Q'71)


J (2r)n (k2 + ',* J QTry- &' +ary'
In the first term we can replace kuk, by (I ln)kz g, to get

f dnk krk, dnk


: ;gu, f e"y 1rz

J e"y &, + ary J &, + try

--+t#+#
! l,or" - r, n) - a2Is(u, df . el2)
Using the identity f (x + 1)
- xf (x), we get
Io@-r,n)-;=%ts(u,n) (2.13)

and

Iu,(a,n) :lorr, * lsu"lM2 - r\, -fi - gf,oro.nr. (2.74)


For the case cv : 4, we have

I,u(4.n)-l#ffi
-lr,p, t )B,,tM2 - p\ c+,)f ,oro, nt. e.ls)
2.6 Vacuum polarization and subtraction schemes 49

which gives for n :4,

It,(4,+1
-fnup,t|B*"1M2 - n2l]-L--f
96n2 (M2 - p2 ..-) .
+ ie
(2.16)

(d) I,o(u,n):l#ffi
- P')(k' - P)(k'' - Po) e..t\
- Jt !L(k''
(2r), (k2 + a2)"

Dropping terms with odd powers of k', we get

I*o(u,n) puk,kp) - pt'p,ppl


- I #@+r1[-(k*k,Pr*k*pukp*
: -ppp,pplo(a, n) - (pplp, + pplrp I prlpp)
l- n..n^ r ltn,,.,n^ * s,,^n,, * s.,^nu\ M' - P'
: Io(q,r)l+nrp,pp+ f
)Gu,ppl Eppp,* B,pptt) _ @ n12_ l)l
.

2.6 Vacuum polarization and subtraction schemes


Use the dimensional regularization to compute the one-loop vacuum polarization
in QED.

Solution to Problem2.6
The usual vacuum polarization in QED is given by

io"fl(q): (q"qfl - q2g"fl)itr(q2)

:-(-iesr
I #r,lr"T_;Trr, t _u _^*uf
(2.78)

In the dimensional regularization, we can replace

I#-->p'I ,*e (2is)

where e -- 4 - d and trr is some arbitrary mass scale that one can introduce in the
dimensional regularizatron scheme. The integrand can be simplified as

-"lr i
-rrl"*il-m+br,uil-d-m+ie) 1

r
- - tp, - m, + b ll-
:_[ )lA - -' q)2 m2 + ie
I
)
xT rlyo (il + *)yF (il - d + Dl. (2.80)
50 Renormalization 2.6

The numerator is of the form

N"P - Trly" il yP @ - ill + m2Tr1y" yf ). (2.81)

Dirac algebra in d dimensions gives

{y" , yfll - 2g"fl Ia Q.82)

where fu is the identity matrix in d-dimensional Dirac algebra space, with the trace

rrl4-f@). (2.83)

Here f (d) can be any function so long as it has the property f (4) - 4. It is
straightforward to see that

rr(dbd d) : f (d)[(a.b)(c.d) - (a.c)(b.d) + (a-d)(b-c)]. (2.84)

Using these we get for the numerator

NaP - f @l[p"(p -ilP + pP(p -q)" - g"0p'(p -q)]+*'g"fl| .

(2.8s)

The denominator is calculated in the usual way,

o--.-J ['4 (2.86)


(p2 m2 + ie) l,@
- - il2 - m2* iel lo 4z
where

A : (t - a)(pz - m, + oI@ - ilz - m2l


:p2-2up.q-*2+oq'
- (p - aQ)2 - a2 (2.87)

with

o2 : m2 - u(l - a)qz. (2.s8)

The vacuum polarization is then of the form

Trop(q)
-i,z I o" I #r'G# -A (2Bs)

To simplify the integration we shift the variable, p --> p luq. Then the numerator
(2.85) becomes

N"fl --> f @ll@ +aq)"(p - (r - + (p +aq)p@ - 0 - a)q)"


a)q)P

- s"P[*2 - (p * uq) .(p - (l - o)q)]]


: f (d){zp" pp - 2a(I - u)q" qfl
+8"0f*' - (p' - a(r - oq\ll t2.90)
2.6 Vacuum polarization and subtraction schemes 51

where we have dropped terms linear in p which will vanish under the symmetric
integration in p. Then we have

ir"P (q)
-i e2ort' f ,, Ir'
o,
| # @+
, lrr" ,u - 2u(t - a)q" qfl

+ g"Fl*2 - (p' - a(l - ">q\ll. (2.et)

The term proportional to g"F can be written as

-[p'-*2 *cv(l _ u)qzl*2u(r_ a)q2


- -(p'-ari2a(l -o)q'
so we get

2u(l - u)q" qF
ron(q) o,
-ieitt'rat fr' I errr!:Ll#-Y. (p2 - a2)2

I-
. 2a(l - u)q2g"fl g"P I
' (p' - a2\2 p' - o'l

- iettr r @\ Jo[^' o, I #{l#, #]


f_r
. 2u(l - q)(q2gofl
- q"qP)l (2.e2)
' (p2_a2y2 I

The relevant formulae for the dimensional integration are eqns (2.57) and (2.59).

- f ddp
tu: I : i(-n)d/'l(u-dl2) I
+2'931
J (p'? -;ry ,2n',a l(") et'?Y-dt'z'
";F
: I ao u pp p'
1r"..
gu' (
- !2)
' - I Q")^ Q'z -r'zY - 1o - 412- ,, 10. (2.g4)
r"'t'
Using these we get

I a'p p" pf _ g"P i?ildtz f Q - d/2) I /? qs)


\L''J'I
I 12n)d (p2 - a2)2 (l - d 12) (2n)a f (2) (-a2'1t-dt2'

f !L- g:-
: soni( o)0,/'r(1 - dl2) .
I Qild (p, - a2) u (2n)d l(t ) --l-
(-a2)t-d/2' e.s6)

Using the identity

. ('- :): (' - i), ('-:) (2.e7)


52 Renormalization 2.6

we can show that

f do p f 2p" pP s"P 'l _ o. (2.s8)


J enlTV_pg- p,_t,
Now the vacuum polarization is of the gauge-invariant form,

,'F (q) - r\p' f (ilG4m@oqo - q'sof)


| ,"ffi
Expanding in the power of e,

r(el2) 2
*tn(4n)+ o(e) (z'ee)
@";rt:;-Y
where T :0.5772. . . is the Euler constant,

ue 62;e /2 : I - |U o2 - tn p2) (2. 100)

we get

r (q) -
+# Io'
ooa( r - ", [: - y -r tn(4n) - rn
lt;":4ll
Write

f (d): f (4) + (d -4)f"(+) +...- 4(t +ae *...) (2.10t)

where a : -Lf '<q. Then we have


r(q) - *l|t3 - v-rtn(4n)+2af
ft fm2-a(l_ a\a211
_ daa(t _ u)'"
J, L___=_l l
Different subtraction schemes
From this we see that different choice of f (d) conesponds to different constants in
the finite part of n(q), which is arbitrary anyway. For convenience, we can choose
a:0,or f (d) -4foralld.
Minimal subtraction scheme (MS). Here we subtract out the pole in e to get

nr,as(q) -- l9r-y
zTt' I 3
*rn4n)

_1,'daqr_a)"lryll
This corresponds to choosing the renormalization constant,

1Zr1)vs-t*6*. Q.to2)
2.1 Renormalization of )"03 theory in n dimensions 53

Modified minimal subtraction scheme ftvtsl. The renormalization constant is


chosen so that the term (-y +ln4tt) is also removed from the finite part,
zna5 (c) :# Io'
ood(l -, "l#l (2.103)

From this result we can study the low- and high-energy behaviour of z(4). For
lql' < *2,
f m2 u(l ulo21 m2
lnf - .,-
a2
l-ln )-cv(l-c-):= (2.104)
Lp'Jp'
and

oo u(r - ultnl
fm2-a(l- v u)
lr' ,t
fm2
(l-a)lln
- fo'ooo L lt'" -, a ( I-a ,#l
1m2
:-ln-
6p2 -301 (#)+
(2.10s)

Then we have

n6(il:-#mt,.#(#)+ (2106)

In the other limit lql' > *',

and
^1#] ='"(="ff) ['- m2
q2a(l _ a)
I

l
m2-u(7-u)q2 -I

Irt
oocu(l - ")t" I p2

:"(#) I,'duu(t-u ,
* Ir'
do o1t - a) ln[a(1 - cr)]

:*,n (#)-*+ (2.107)

Thus the limit is

z1a5(a) : -#"(#). #+ (2.108)

2.7 Renormalization of ),Q3 theory in n dimensions


Consider the ),Q3 theory where the Lagrangian is of the form,

L::(o,Q)2 -!r'- j.o' (2.109)


54 Renormaliz.ation 2.7

(a) Show that ).s has dimension (6 - n) 12 in n-dimensional space-time. To have


l.s with fixed dimension for arbitrary n, we can define

lo: )'s(p7@-n)12, lo: Lo&', with ,-o (2.110)


^'
2

where trr, is an arbitrary mass scale.


(b) Show that the one-loop divergent graphs for the case,? : 4 are those given in
Frg.2.5.
(c) Cany out the renormali zationprogram for this theory by using the MS scheme.

Solution to Problem2.T
(a) Since the action S : .[ d" x L is dimensionless, Lhas the dimension n. From
the mass term or the kinetic etergy term we see that @ has dimension (n - 2) 12,
which gives the dimension of i.e as (6 - n) 12.
(b) From eqn (2.23) we know that the index of divergence for the Q3 interaction is

6-3(+)-n-(+) (2.ttt)

which, as expected, is just the negative of the dimension of ).e. The superficial
degree of divergence is then

D:n-(+) u*,, (2.112)

where u is the number of 03 interactions in the graph. The number of loops L is


given by eqn (2.14)

7-|{v -B)+t. (2.113)

Thus in one-loop we have v : B, and

D:n - 28, (2.r14)

and for four-dimensional theories n - 4, only B -2self-energyandB-1


tadpole graphs are divergent.

Rc. 2.5. Self-energy and tadpole diagrams in the ).@3 theory.


2.1 Renormalization of )'Q3 theory in n dimensions 55

(c-I) The self-energy graph

o'o
-iE(p)-(iLott')2 [ (2n)" --:- ' . (2.115)
2 I &2 - mz) I& - p)2 - m2l'

It is straightforward to evaluate this integral to get

L'01"" t(t)'= oo
E(p) - ^[' (2.116)
2 (4T),/2 Jo lm2 - d(l - a)pz - iele'

Using

f(e)- !-y*O(e), a' :l*elna*O(e2) (2.111)


t
we can expand t (p) around r: 0, to get

'(p) --f.f*t:- y*tn(#)


;'r
_ [' dalnlm2 - u(l -_a)p2 -
I

Jo | *'-ll
We now rewrite the Lagrangian as

- I - . 4a,-Lr'6,-!u*rO,
L:t@uf)'-ia-- ya"-z (2.118)

with 6m2 : m2 - *2*. This amounts to the replacement m2 - m2* and add the
term l6m'O'ut a new vertex. The new self-energy tR(p) is then

ER (p, mn) : E(p, mn) * 3m2. (2.119)

Now if we choose

I r I* I
d*'-{^2 (41rY (2't2o)
L-; '.1
where c. is finite for e -+ 0, but is otherwise arbitrary (different renormalizatron
schemes correspond to different choices of cr), then the pole at e
- 0 cancels
out and

ER @, m n) : + #1,^ - y t tn (H)
-1,'d^nlryll
is finite.
We now study this choice of c^ in various renormalization schemes.
56 Renormaliz.ation 2.7

(i) Momentum subtraction Suppose we choose


ER (p,mn)lpr:-uz : 0 (2.r21)

for some M2. (This corresponds to normalizing the propagator such that it is the
same as the free propagator at p2 - -M2 ) Then the constant c^ is given by

c* : v - tn (ffi *
I,'
dutnlryf <r rrr>

and the self-energy is

:La | ro^l*?, "l,t - u)pz - iel (2'123)


-; @"Y J,[' dc
>*(p,mn)
Lm'*,,(r - afu' -
")
(ii) Minimum subtraction (MS scheme) This corresponds to the choice
c^ :0, and it means that we subtract only the pole at e
- 0 and the self-energy is

-1,'d^nlry- rs]
l
I

(iii) Modified minimum substraction (MS scheme) It turns out that the
combination

(I/e) - y -lIn4n (2.r24)

always appears in the dimensional regularization. Thus it is convenient to choose

cm : y -ln4n (2.12s)

and we get

,R (p,rn n) :-- rA I ft
2 (4n)2= lo
dr"l*'o - u(l fi u)p2 - ie ] erz6)
(c-II) The tadpole diagram
We have
j'k_ ,
2 If Q"Y k? - *'*=_LLot'f(e_r)(mn)t-,.
ir_QLo_t"') (z.tzj)
2 (4n)2-e ^ "

This will contribute to the vacuum expectation value

(Old(0)10) : T
., (2.r28)
m'R

and will give an infinite constant vacuum expectation value for the field. To elim-
inate this infinity, we can add another counterterm to the Lagrangian of the form

Ltad: -tQ. (2.r29)

This counterterm will have the effect of cancelling all the tadpole terms, without
interfering with any other consideration.
2.8 Renormalization of composite operators 57

Remark. The parameter of renorrnalized mass, ntp, c&rtbe related to measurable


quantities as follows. The physical mass of the Q-particle,ffip, is defined to be
the position of the single-particle pole in the two-point function. This gives m p as
a function

ffi p : m ,(m n, )", l,L, c^) . (2.130)

We can solve for m p in terms of other parameters,

trt R : m p(m p, )", Lt, c*) . (2.13I)

It is important to note that the parameter mp has implicit dependence on the


arbitrary mass scale p.

2.8 Renormalization of composite operators


Consider a theory with a fermion and a complex scalar field. The Lagrangian is
given by

L : (Lr"Q\@pQ) - p2Q2 - !tO. O>'


z
+rblypap - m)V + g{tt\Q i h.c. (2.132)

Show that the composite operators

Ol : rlry*V, ol:i(Q.opQ-Qo*Q.) (2.133)

mix under the renormalization.

Solution to Problem 2.8


Vertices for the composite operators are displayed in Fig 2.6,where solid lines are
fermions and dashed lines are bosons. Add two terms to the Lagrangian for these
operators

L --+ L + i Jrp@)Ol (x) * i J2r@)Ol (x). (2.134)

The one-loop divergent diagrams for these composite operators are shown in
Fig.2.7.

oOf
: yt'
/\Az *t :(n'+o;v
/
/\,/\
Pt P2

Ftc. 2.6.
58 Renormalization 2.8

aoi'
/\
/.\.
4/'
"ro,
(a) (b)

,'q'| l1noy
tt
.r'
4/
i-----\\p' pr '/
-!.'-- \' p2

(c) (d)

Ftc. 2.7.

Figure 23(a) is a logarithmically divergent Green's function for the operator Ol


with two fermion external lines. We can separate out the divergent part as
lf"(nr, pz) : yqlro(0, o) + prpl + ' " (2.13s)

where the first term is logarithmically divergent and all other terms are finite. Thus
we need a counterterm of the form

-iluQ,O)rlry'rl, Jrp(x) - -iln(0, 0) Jtuol . (2.136)

Figure 2.7 (b) gives a logarithmically divergent Green's function for the operator
Ol withtwo scalar external lines. Again, separate out the divergent part as

lf6(kr, kz) : (kr * k)Pls(o, o) +" ' (2.r31)


It shows the necessity of a counterterm of the form

-i f 16(0, 0)JtpOtr. (2.138)

Figure 2.7(c) shows the necessity of a counterterm of the form

-if2.(0, 0)J2pOf . (2.t3e)


Figure 2.7(d) shows the necessity of a counterterm of the form

-ilza(0, 0) Jr*Ol. (2.t40)


Thus the effective Lagrangian which contains the composite operators and their
one-loop counterterms is of the form

L, : iJtpof(1 fr,(0, 0)) + iJ2potr0- fz,'(0, 0))


-
-i JruOll16(0,0) - t J2uOlly (0, 0)
- i hpof ztr * i J2*ol Zr, + i Jtpol zt, + i J2pol Zrl

-i(rr,tn(t:" t::)(3)
- i 4'zij or QJ4t)
2.9 Cutkoslcy rules 59

where

Zn :1 - f ro(0,0), Zn: -f 16(0,0),


Zzz : 1 - f27(0, 0), Zzr : -f2.(0, 0). (2.142)

The renormalization constants are now in the matrix form, explicitly display the
mixing of these operators,

t- : Zrt Zn\
(
(2'143)
\tr', z;; )
which is neither symmetric nor real. Nevertheless we can diagonalize this
by biunitary transformation (see Cl-Section 11.3 for the details of biunitary
transformation),

Z -
(r ZaVI . (2.144)
where

ry (2, o \
'r:(o zr) Q't4s)

is diagonal. V and U are unitary matrices. Then we can write

L, : i Jl zij ol : i i( zr)i + i il zzol e.t46)


where

0l : v,rio!, i! - uliJf
This means that neither Of nor Ol are multiplicatively renormalizable. But the
combinations

0l : vl,of + vlrol, 0l : v],of + vLotr e.147)


are multiplicatively renormalized.

2.9 Cutkosky rules


In the )'Q3 theory, the one-loop diagram in Fig. 2.5 gives the contribution

r(s) - t(p') : + I ##@=hd Q t48)

Show that in the complex s plane, the imaginary part for s > 4p2 is of the form
I
Imf(s) :
rlf(s * -
ie) f(s - te)l

: 72 If d4k G2tti)26((p + D2 - p2\6(k2 - p\. (2.t4s)


, J C"f
60 Renormalization 2.9

Solution to Problem2.9
From the structure of the two propagators, the poles are located at the following
two locations in the complex ks plane. From the second propagator, we have

ko
- * (t'+ u')t'' + ie - L(Er, - ie) (2.150)

where Er, : (k2 + p,2)r12, and from the first propagator, we have

ko: -po+ ((p+k)2 + p.2)r/2 Tie - -poL(Ep+r-ie). (2.151)

The integrand is of the form

(ko - Er, * is) (ko + Ek - i) (h + p0 + ++k - is)


1

(ko*Po-Ep+r*ie)
We can close the contour in the upper half plane and get the contribution from the
residues atks - -Er*ie andko: -po- Ep+k* ie.
(i) Residueatks: -Er I ie

t^
rr - -Lr" QEk-is) CEr+ Er.r+ pr- lt) (-Er- Er.r+ pr+lt)
(2.ts2)

The last two terms in the denominator can be put into the form

(Er, - po)2 - E?*o- [tt< + p\t/2 - po]' - (p + k)' - t"'


: p2 -2poEr,-2p.k+ie.
For convenience we can take pp : (p0,0), then Po : and
"f
f _ l1 __- _ _(_2ni). (2.153)
"- zEo5-2159*+ie' '
Since Ep 2 &,wa see that for S , 4p' the factor (S - 2/S E* * i e)-l has
singularity along the path of integration (from E* : tr"r. to oo) and will contribute
to the discontinuity for S , 4p2.
(ii) Residue at ko: -po - Ep+r * ie

, -LrL'
12: ^_: _
QEp+k- is) (pr+ Er.r - Er+lt) (no+ Er*r,* 81, _ ie)'
(2.rs4)

In the denominator, we have

(po+ Ep+r)z - E7-ie: p3+ (p+k)2 + t"2 l2psEr*o-k'- p'2

- Pt * 2PoE p+k' (2'155)


2.9 Cutkoslcy rules 6l
and

/l\t
Iz:-2ir,\n*)W. (2.156't

see that this denominator never vanishes for s > 4p.2 andwill not give
It is easy to
discontinuity in the physical region. Thus if we close the integration contour in
the upper half fts plane, only 11 will contribute to the discontinuity in the physical
region.
For the calculation of the discontinuity, we write

/ l\ I
Iy
- \-2it,, \m ) p@ __ zEk + b,
t2.t57 t

where p - \/i. Using the formula


11
x-attt, -P-ain6(x-a)
(2.158)
x-a
we can obtain the discontinuity across the cut for the case r , 4p',

disc 11
- I{p * ie) - IrQ - ie)
- (-hri)'d(p - rtr, (+) (z.tss)

To get a more systematic rule for calculating the discontinuity, we write 11 as

f"l
tr: dkoG2ni)6&2 - p2) (2.160)
J
We see that this corresponds to replacing the propagator (k2 - p.2 + i e)-l by
(-2n i)S (k' - p'). Similarly,

disc11 'J- [ oor(-2ni)6&2 - p2)(-2nr)6t(p +D2 - p2l e.r6t)


or

7z f d4k
disc r(s) -; J r"y?2tri)6(kz -
p211-2trt)dt(p + D2 - p2l Q.r62)
which is the requested result.

Discussion

As indicated by this calculation, we can obtain the discontinuity of f (s) by putting


each particle in the loop on the mass shell with the replacement of

,
" "-
K'-p'+tt
1
-----, 1-2n i)s&2 - p\e (kd . e.t63)
62 Renormalization 2.9

In fact, this discontinuity can be written in terms of physical matrix elements, as


we now illustrate:

L2 fdak, dak, aa I I
f(s)- T p)&r,
_u?+is\E_u
J e"ye;F(2ir)*6"(kr*k2-
Then using the replacement given above we get the discontinuity as

discr(s) :+ I #(2ni)d(fr? - p2)o(krc)

x [4, a

I fzrl,G2ni)6(k3 - P2)(2n)4

x 6a (fu * kz - D0 (kzi. (2.164)

The integration over fts is of the form

I #(2ni)s(k' - ,\e&o): # (2.16s)

where E* : (k2 + Lhr/z, and

disc r(s) : ;I Jf d3kt f d3kz . .^.)


Qil4 t4 (rcr * k2 - D.
dffi J r-.r+(-iD2
The factor (-rf.)just the scattering amplitude in first order of )", T1
is
- -i).. Thus
to order ),2,the discontinuity of the scattering amplitude in the variable s : p2
can be written as the integral over the phase space of lzl12. This is the essence
of the unitarity of the ,S-matrix, SSt - StS I which implies for the Z-matrix
-
(s-1+ir)
T _TT _TTt (2.166)

Tir - T,? :Dr,,r;, (2.167)

The prescription of replacing (k2 - p2 * ie)-t by (-2ni)S(k2 - p\ is a simple


example of the Cutkoslq rule whtch gives a general method for computing the
discontinuity for an arbitrary Feynman diagram and is summarized below:
o Cut through the diagram in all possible ways such that the cut propagator can be
put simultaneously on the mass shell for the kinematic region of interest (e.g.
only for s > 4p2 can both propagators be put on the mass shell).
r For each cut, replace the propag ator (k2 - p2 +ie)-t by (2r i)3(k2 - p.\0 (k0)
and perform the loop integration.
o Sum the contributions of all possible cuts.
3 Renormalization group

3.1 Homogeneous renormalization-group equation


Consider the ),64 theory in d-dimensional space-time, where the Lagrangian for
e :2 - (d 12) is given by

L-;@,0)2-!r'-fi0^
I ^ 2 Ln
^'^,2
- ){a,Qo)'-;O'^- (counterterms) (3.1)
iu'0X+
where Ln, Qn, and mp are renonnalized quantities, and p is the arbitrary mass
scale one needs to introduce in the dimensional regularization.
Use the fact that the unrenormalized n-point Green's functions l@) (pi, Lo, mo)
depend on the bare parameters (ms, )"0) and are independent of the arbitrary mass
scale, trr,, present in any scheme of dimensional regularization,

rhl@) (p,, )"s, ms) :0, with ms,)'e held fixed (3.2)

to derive the renormalization group (RG) equation for this theory.

Solution to Problem 3.1


Recall that the relation, Cl-eqn (3.50), between unrenoffnalized and renormalized
Green's functions is given by

l(") (p,, Lo,


- zi"/zt,' @t, Ln, ntn, &)
mo) (3.3)

Thus the statement of p.E l3 pl@) - g means that

-*lt;'prg' (p,, LR,mR,r)] - o. (3.4)

Note that both ).p and mp depend implicitly on p. Thus we have

f, n A A 0)"" 0 Emo E I
ztt^dplnZa*lt^dtt +p-*dlt d),.p*u-=:
* ap a^ * l
I )
fg) (pi, Ln, //tR, lD :0.
"
Defining the quantities,

.laDlnDmn
YQn): tu fl{x*1 - P
A;, Y.(Ln)mn: F r,
*lnzo.
(3.s)
64 Renormalization group 3.2

we can write the renormalization group equation as

(rn) - ny (xil tmnv*6il .g' (pi,Ln,tIR, tD:0.


lrh+f # h]
(3.6)

Remark 1. Implicit in such calculation is the fact that the bare quantities ,1.6 and
rfls zta held fixed.

Remark 2. This equation is a homogeneous equation which is more convenient


to work with than the original Callan-Symanzik equation.

3.2 Renormalization constants


In ),Qa theory, the renormalized and unrenormalized coupling constants in the
dimensional regularization scheme are related by

LnQ-t)
- p-uz-t(p)lo Q.7)

where Z is the coupling constant renormalization constant of the form

Z-t - z;t zi (3.8)

where Z7 and Zq are defined in Cl-eqns (2.23), (2.36), and (2.40).


(a) Show that the B-function can be written as

udZ
fl (Lil - -sln - Z du^o (3.9)

(b) In the one-loop approximation, we have

z^t:t-#' zo-1+o(x')' (3'lo)

Show that
y,2
fl(L): . (3.11)

""r+o(,t,3)
Solution to Problem3.2
(a) By explicit differentiation of eqn (3.7), we have

0).o_ pi:
Qil _ p=
A
F ztDxo)
dp dp'@_,

- -elt-'Z(lr)fo - Lr.-'l-t+;,
d 1t'

pdz"
--strn- 7 ou^- Q't2)
3.2 Renormalization constants 65

(b) Substituting the one-loop result (3.10) into eqn (3.8), we get

Z-t-z^tzto=z^t-r- # (3.r3)

or 2 - t + (3xRl t6tr2e). Thus

dZ: 3 dLn 3 (Ln)


* G.t4)
d, t6n4&7; - ;**f
and, ignoring higher-order terms, we obtain

tdz 3
(rn) . (3.1s)
Zu du = r6jr%P

In this way the relation (3.9) becomes

fl(;il--srn -(;fr) rR:-srn -ffir(rn) (3.16)

Solving for B (ln), we get

:"*")
, freil--srn(t* #)'--,)".(t- (3r7)

Taking the limit e -+ 0, we obtain the stated result:

fl gil : ro7r'
::i"+ o ('r'|) ' (3'18)

Remark. More general analysis of e dependence of d(,1,) can be carried out as


follows. We first write eqn (3.7) as

lo : L(tDp'Z. (3.19)

From p.d)"gldp 0, we get


-
l- - d -l - d
u'le(),Zl * u OuQ,Z))-O or - eQ"Z)
- tr (3.20)
dpQ,Z).
In the MS schem e , 2 can be written as power series in (l I e),

_ br(l),J9 + ...
Z-l* , ++ U r.. (3.2r)

then

).2:^*ar(l)
t'
+29 +... (3.22)
66 Renormalization group 3.2

where ai(L) - Lbi(),). Differentiating both sides of this equation, we get

,*.^2)-['.I *;]-h: ['+r ,^*;]pe) (323)


L n:l

Equation (3 .20) becomes

_r[^+ate)*"rg*...-1= f da,t da, l r


I e ez )=P()")Lt*^i+ ax- '* I (3'24)

Assume that the B(,t,) is a finite series in e,

fr(\):l1o+frf*fl2e2 +...+ fl*t'\. (3.2s)

By identifying powers of e on both sides eqn (3.24), we see that the B(),) series
must terminate after the first power of e (fl* :0 for ft > 1):

F0) : flo * flp. (3.26)

From eqn (3.24) we then get

flr: -)" (3.27)

and

flo * B1# : -o, or Fo : -ar * ( 3.28)


^*.
Thus the B-function is given by

F(L) : -)'e - ar I )"


' --ar * as e -+ 0. (3.2g)
o^4 ^#
Using at : Lbr, we get

F(L):-^'\i).
^'(U\ (3.30)

In fact, _eqn (3.24) also relates different powers of (l/e) in the expansion of
Z(or ),2). From the coefficient of (l/e)'we have

-anr, : uo* * p,+#: (-', . * - ^+#


^*)
or

.
-an+t*n^dan+t._(_n,*^do,\+.
aX
:\-o' *^^)^ (3.31)

Thus all the coefficients cn with n>I can be determined from ar, by repeated
use of eqn (3.31).
3.3 p-functionfor QED 67

3.3 B-function for OED


The photon propagator is given by

'ef
iezD*u(q) - -Vffitr, (3.32)

where

,' : ;3--*o101 - Zte\, Zz :# ry I


-n(o), (3.33)

and

n(q) (sp" - eue,) - nu,(Q) Q.34)

is the vacuum polarization tensor. Define the running electric charge as

e2(il= ,' ,., (3'35)


1 + Reu (q'z)
=

where fr (q2)
- n(q2) - z(0) is finite. Show that in one-loop, we have
,'errf) - e2(-p2R, * #r^4 (3.36)

for the case of Fzo, &R > m2.If we define

F@) - p^9,
dFn
(3.37)

then we will have

fl(e) - #*o@s). (3.38)

Solution to Problem 3.3


Vacuum polarization in QED is calculated in Problem 2.6, and is given by

n(qz):*##t1,'oofta_n,a;4,,
#{ (: *h4n-,)
pt
- a) r"l---i-)*
fm2-o2u(l-a)l I
-u
J,
dau(r o,',1 (3.3e)
68 enorrnalization group 3.3

Thking LL : m, the subtracted quantity becomes


fr (q') - r(q2) - z(0)

: -#
Io'
oou(t - o"1#l (3 40)

Write

:
ffi5 - Re n(q\l' ( 3'4t)
"(q') "lr-
For the case of lq'l > m2, wehave

fr(q,) = '3 "'\ *'


- l2jr2 G.42)
^(-4\)
,'(q')=ul,*#^(#)l (343)

Thus, at (-q2) increases, ,'(q') also increases.

Different subtraction schemes


From the vacuum polarization ir@\ given in eqn (3.39) we can illustrate the
difference of different renormalization schemes.

Momentum subtraction scheme. In this scheme, we make a subtraction at q2 -


- M2 , then
o*(q',M): # I,'doo1r -cv) ^lml (3.44)

Suppose m2 > lazl anA M2,then


^2 t -2 tt2'
,*(q'.M)- #"(#,#)--+o (34s)

This means that a heavy fermion will decouple in the vacuum polarization at
energies much smaller than the heavy fermion mass. This property will enable
us to ignore all the unknown particles which are much heavier than the present
energies.

MS scheme. Here we subtract out the pole at t -- 0 and some constants,

,y,(q') :# ooa(t e
In the limttm2 > lq'l we get
Io'
-
^^l#l 46)

' -'3=
o1r(qr) --, h(4\)
r2rz"'\p,
(3.41)

which is non-zero. Thus in this scheme, the heavy particles do not decouple at low
energies. One way to remove the effect of the heavy particles is to integrate out
3.4 Behaviour of g near a simple fixed point 69

the heavy fields in the Lagrangian and work with the effective Lagrangian without
the heavy particles.

3.4 Behaviour of g near a simple fixed point


Derive the ultraviolet behaviour of g(r) in the case that the B-function is given by

fG) _ s@, _ s2) (3.48)

with a being a known constant. This example illustrates the typical behaviour of
the running coupling near a simple fixed point.

Solution to Problem 3.4


To analyse the asymptotic behaviour, we plot B(g) vs. g,

Frc. 3.1.

-
The initial condition for the running coupling constant is

E(t) -+ So at t - 0.

Then it is clear from this plot that

g(t)+a if Bo>0
g(t) --> -a if 8o < 0.
This can be verified by more explicit calculation as given below.

#:E(o'-E') + Id%!r: lo, e4s)

Carrying out the integration and using the initial condition, we get

#1"(h)_h(_+)l :, (3.s0)
70 Renormalization group 3.5

or

r3 *), _
\E:- ,-2a2t (3.sr)
E'(s3 - o')
so that

E':=#* with o:#. (3.s2)

Taking the square root, we get

-ta
E: (3.53)
" (l - Ae-2oz'1r/z'

To choose the sign, we need to go back to the initial condition that g gs &t / 0.
- -
For the case ge > 0, we take the positive sign

_a
o
(t - lt-2"")'l
so that at t :0,
g----o-:,?, a:go.
ov
Q - 41112 @lsd
In this case, g --s. at as / -+ oo. For the case go < 0, we need to choose the other
sign

_-a
: (3.5'+.1
E
"-(t- 1.s-zazt)t/z'

Then we have E
- -a as / -> oo.
3.5 - fixed point
Running coupling near a general
At the stable critical point g : go, show that

(a) if B(g) has a simple zero: B@) - -b(S - a) with b > 0, then the approach
of S(t) to gs as t --> @ is exponential in r;
(b) if B(g) has a double or higher zero: B(g) - -b(S - a)" with b > 0 and
n > l, then the approach of g(t) to ge as t + oo is some inverse power in r.

Solution to Problem 3.5


(a) Simple zeroz fl(S) -b(s a) - -
From the renormalization group equation

ds
--o-p(il, f dE f ..
l=--:---lbdt, (3.55)
ctt J 8-a J
3.6 One-loop renormalization-group equation in massless )"Sa theory 7I

then with the initial condition of g -+ gs at t : 0, we have

(3.s6)
^(ffi)--u,
with b > 0,

E:o*(go -a)e-b'. (3.s7)

Then, E * a exponentially in the asymptotic t -+ oo limit.


(b) Double zero or higher: fl(S) - -b(S - o)n , n > l.
The same calculation yields

I I
r= I l- 1 ' (so-a)'.t)-u'
[J G-a)n
-oE --[uo,
I tu- DLA-q)n-t
or

'w'
E:a*[ l-' aio(t-r/t,.-rt1
- \- . (3.s8)
lll3o_ a)n-t +@ - t1U lr-oo--'

3.6 One-loop renormalization-group equation in


massles s theory
^.04
In the renormalization of the massless )"Qa theory, we can momentum subtract at
p2 : -142 to avoid infrared singularities. In this case the renormalization-group
equation takes the form

p(r)
*^ -
,r(D)r$) tp, , p2, . . . , p,) :0. (3.5e)
l, #+
Verify explicitly, the one-loop result for the four-point function f [o) {f t , pz, pt)
satisfi es this renormalization-group equation.

Solution to Problem 3.6


From CL-eqn.(2.31), the four-point function in one-loop is of the form

f[o){r, t,u) - -ilo * f (s) + f (/) + f (rz) (3.60)

where l(p2) in the dimensional regularization scheme is given by [Cf. Cl-eqn


(2.12r))

r@\ : #th - Ir' dartnd(r - ,,)l - rnr-p')l (3.61)

Suppose we make a subtraction at some space-like momentum p2 : - M2 . Then


we have

i(p') -r(pz) -reMz): #"(#) (3.62)


72 Renormalization group 3.7

where the dependence on M is rather simple (compared with the p2 dependence


in the massive theory). The renormalized Green's function being
t!a){s, t, u) - -i)" + i(r) + i(r) + i(r) (3.63)

we get

a ,A\ 3iL7 3i),2


* t,u) - + O()"'). (3.64)
*fi-'(s, 16r,
Also we have "F:
3 ,n, /T"2\
P(L)- r[*'(s, t, u) - + o(r)] (3.6s)
\TrF )[-t
where we have used Cl-eqn (3.47)

rhererore, rrom cl-eqn ::,::ff^] :;],,


satisfies the renormalization-group equation to order i,2.
*" see rhar .*,,i ;-]

3.7 B-function for the Yukawa coupling


The Lagrangian for the Yukawa interaction is given by

L -,lr(iy*a,-m)(t + f rtrrtrO*),ru-fr'- {f'. Q.67)


Compute the Callan-Symanzik B-function for the coupling constant /.

Solution to Problem3.T

ftc. 3.2.

It is convenient to set all the masses to zero.


(i) Vertex correction

t_(-ir, Ids(h)';
: r' - --a(t -u)P2 (3'68)
I # l fd witha2
3.7 B-functionfor the Yukawa coupling 73

where we have combined the denominators by using the Feynman parameter and
have shifted the integration variable k --> k I ap. The divergent part is then

f d4k I i / 1v2 \
J'
(3'6e)
(2r)a (kz - szl v:t6o'('t"7+ )
so that

if3 / 1v2 \
r:*n(.t"o+ (3.70)
).
The vertex renormalization constant is then
s2
zr: |+
#tGnA2 +...). (3.71)

Recall that the B-function is given by

fr:-r#rrr, where 2r:z7tzrz2r e.i2)


Here Zq and Zrp are the wavefunction renormalization constants for scalar and
fermion fields. Thus the contribution coming from Z y is

z7' :
frr --laoh fi<'> (3'73)

(ii) Fermionself-energy

, f d4k i i(il-r)
>v@) - (-if)' J e"y pffi' (3'74)

Combine the denominators in the usual way


I I f do
P1r-oy:1 E Q'ls)

with A : (k - ap)2 - a2 and a2 : -a(l - o) p' .Shift the integration variable


k --> k I op; the numerator becomes (1 - u)l - # .Then we have
E,r,@)_f2
l#lo'oog##
-- f2 [^' oo(r -a')il G.76)
Jo t6jrr(^5. )
and the wave function renormalization constant is
r2
Z,b:l-
njtrlnA2 +...
(3.77)

and its contribution to the p-function is

af3 : (3.78)
flz - -/ a(lr, ,,\)
Z,L
*;2.
Since there are two such diagrams in the vertex, this contribution should be
multiplied by two.
74 Renormalization group 3.7

(iiD Scalar self-energy

Eo@) _ (-if)2 n,
I #r, (; 6+)
_,\-/r
tr2 t d4k rr1#@ -f)l (3.7e)
- J e"y kre _ b,
The numerator is, in the dimensional regularization,

N - rr[tl @ - #)] : d (p . k - k') . (3.80)

The denominator is

lfr : I
:
k\p - ky
o" with o2 -a(r - a)pz (3'81)
J0 @ -4
where the shift k --> k * aphas been made. With the shift the numerator is then

N -d[(k*op)z - p-(k+ap)]--> d[k2 -cv(l -a)p2]-d.(k2 +o2).


(3.82)

The self-energy is

>6(il--r2
I# Io'oo*!*&2+a\. (3.83)

Using the formulae

f ddk t i(-)" t(n - d/2)


J (2n)d (kz - szln (4nf 12 l(n) (#)"-''' '3
84)

f k 1rz i(y-t r(n-d/2-t)


t-

I
dd
(2tt)d (kz - 6z7n (4rf 1z r(n ) #)"-',-' (x) (3 85)

we get

The self-energy is then

Eo@) : - r,
Io'
oo
#; (Ih) . (' - ) (#)'-''' (3.87)
3.8 Solving the renormalization-group equation by Coleman's method 15

The divergent part, which is relevant to the wavefunction renormalization, is then,


with d --+ 4 andr(2 - d 12) + (21 @
- d)),

-in2f2 / 2 \
Eq(P)ai,: -*fr\o -io2f2 L2.,
u ) - -Gf2tn (3.88)

where we have used the correspondence (21@ - d)) -+ ln A2 (see CL-p. 56). The
wave function renormalization constant is

sz Lz
Zo:l+G*zt",, (3.8e)

and its contribution to the B-function is

flt:- -L'
16n2''
(3.e0)

The total contribution to the B-function is then

fl:(flrt2flz+Ft):#(r*r* ,l *, \) - i,r.
5f3
(3.er )

This is the result given in Cl-eqn (10.16).

3.8 Solving the renormalization-group equation by


Coleman's method
Consider a one-dimensional fluid with velocity u(x) and in the fluid there are
bacteria (see Coleman 1985). Letp(t,.r) and g(x) be the density and the growth
rate of the bacteria, respectively.

(a) Show that the density of the bacteria p(t, x) satisfies the differential equation

s(")] p(t,x) -o' (3.e2)


[*. 'ri{-
(b) The position of a fluid element is described by I : I(t, x) with the initial
condition x(0, x)
- x. Namely, the fluid element which was at x at t - 0 is now
at 1 at time t. Clearly I(t, x) satisfies the differential equation

d
x) : u(t). (3.e3)
ati(t,
Show that if p(0, x) - po(x), then at later time p(t, x) is given by

p(t, x)
- ps(r(t,x)) exp
Ur'
or'rrt(t' , i)f . (3.e4)

Solution to Problem 3.8


(a) The term due to the growth rate g(x) is self-evident. We will concentrate on
the second term which is due to the motion of the fluid.
76 Renormalization group 3.8

Consider a fluid element / with length dx locatedat x. The bacteria in this fluid
element is just p(t , x) dx. At a later time, t + Lt , this fluid element is replaced by
those which were located at (x - uLt) at time t. Thus the rate of change in the
bacteria density in / is

lp(t,x)-p(t,x-uA,t)l (3.95)
A,t =ry0x
where we have made the approximation p(t, x - uA,t)
= p(t, x) - u\p/3xA,t.
This gives the second term in the differential equation.
(b) Integrating eqn (3.93) for I, we get

J- + [' 0,,.
[' u(y)_ Jo"'' (3.s6)

We can differentiate this equation with respect to x to get


tar
- o'
1
(3'97)
,@ a- ,G)
Then for any function /("r) we can show that
Adx(i(t,.x)): f'(x)
*f dt: f'(i)u(x),
,@*fG(t,x)): u(x)f'(I)#:f'(I)u(i). (3.e8)

Combining these we get

'0* l"f GG*)):0.


l:LAt-,(")3'l ' (3.ee)

Or changing t --> -/, we get


l-a aI
LA*u{x)*J
fEGt.x))-0. (3.loo)

Then it is straightforward to verify that the solution is of the form

dt'g(It-r',
"ll] .
p(t, x)
- po(i(-t, x)) (3.101)
^rUr'
Remark. The generalization to a higher dimension is simply
l-a* ui(xr,...,x,) a I :0.
- 8(xr,. '. ' r,,l)0U,x1,....f,,,) (3.102)
L* axt
Define
d
xt, ..., x,) - ui(Ir, ..., i,) with I;(0, x1 xn) : xi.
Eit(t,
(3.103)
The solution is then

p(t, xr x,) : po(ii(t, xr, . . . , x)) dt'g(Ii(t' , xr .


^oUr' ",))]
(3.104)
3.9 Anomalous dimensions for composite operators 7l

3.9 Anomalous dimensions for composite operators


In the ),Qa theory, compute the anomalous dimensions for the composite operators,
Q2 and 06 , in the one-loop approximation.

Solution to Problem 3.9


(i) Anomalous dimension of 02 As was described in Cl--Section2.4, the only one-
loop divergent graph involving Q' it in the two-point function ff2r), anA is of the
form

r'rz,trn:pr.pz):(+)l#l"Hl1t,-r,,-o)(3r's)
This has exactly the same structure as the function l(p2),given in Cl-eqn (2.10),
which appears in the four-point function. Taking over the result, we have

rft)rn; pr,pz) : *[^# - Ir'


dutnfpcz- cy(1 - a)p21+ ' 1
(3.106)

and

Zo, - | +ffr)(0, o, o) - I - #rr*. (3.107)

The anomalous dimension is then


a),
YQ2: -arrA ln26': ,6nr' (3.108)

(ii) Anomalous dimension of Q6 The one-loop divergent graphs are all of the type
shown in Fig. 3.3 with altogether (!) Oistinctive diagrams.

Frc. 3.3.

This again can be expressed in terms of f (p2). Taking into account the combina-
torics we get

zou - r + r5ro9(0,o,o) - 1 - #t \. (3.10e)

The anomalous dimension is then


15),
YQU
: (3.110)
16rrr.
4 Group theory and the quark
model

4.1 Unitary and hermitian matrices


Show the following relationships between the unitary and hermitian matrices:
(a) Any n x n unitary matrix UtU :1 can be written as

U - exp(i H) (4.1)

where 11 is hermitian, HI - H.
(b) det U : |implies that H is traceless.

Remark. This result means that n x n unitary matrices with unit determinant can
be generated by n x n traceless hermitian matrices.

Solution to Problem 4.1


(a) The matrix U can always be diagonalized by some unitary matrix V

vuf - ua (4.2)

where (la is a diagonal matrix satisfying the unitarity condition UaU)


- 1. This
implies that the each of the diagonal elements can be expressed as a complex
number with unit magnitude el".

,, :[
/ eio, \
'l ,0,,
I
\ ,'o')

where o;s ttre real. It is then straightforward to see the equality tla : siHo, where
Ha LS areal diagonal matrix: Ha - diag(u1, d2, . . . , d,). We then have
(J : Vt (laV : yl ,i Ha V : eiH (4.4)

with F1 Vt HaV. Because Ha is real and diagonal, the matrix is hermitian:


- F1

st - (vt nov)' : v, ulv - H. (4.s)

(b) From the matrix identity ,rrA - det(eA), we have for U : eiH

- det(eiH) : detu.
,irrH (4.6)

Thus detU : 1 implies that TrH :0.


4.3 Reality of SU(2) representations 79

4.2 SU(n) matrices


The n x n unitary matrices with unit determinant form the SU(n ) group.
(a) Show that it has n2 - I independent group parameters.
(b) Show that the maximum number of mutually commuting matrices in an SU(n)
group is (n - 1). (This is the rank of the group.)

Solution to Problem 4.2


(a) To count the number of independent group parameters, it is easier to do so
through the generator matrix. From the previous problem, we have (J : eiH,
where F1 is an n x n traceless hermitian matrix. For a general hermitian matrix,
the diagonal elements must be real, Hti : F/,]. Because of the traceless condition,
this corresponds to (n - 1) independent parameters. There are altogether (n2 - n)
off-diagonal elements and thus (n2 - n) independent parameters because each
complex element corresponds to two real parameters, yet this factor of two is
cancelled by the hermitian conditions Hij : F$.. Consequently, we have a total
of (n - |* n2 - n) : (nz - l) independent parameters.
(b) From the discussion in Part (a) we already know that there are n- 1 independent
diagonal SU(n ) matrices, which obviously must be mutually commutative. On the
otherhand, if there were more thann - I mutually commuting matrices, they could
all be diagonalized simultaneously, thus yielding more than n - 1 independent
diagonal matrices. This is impossible for n x n traceless hermitian generating
matrices.

4.3 Reality of SU(2) representations


This problem illustrates the special property of the SU(2) representations, their
being equivalent to their complex conjugate representations.

(a) For every 2 x 2 unitary matrix U with unit determinant, show that there exists
a matrix S which connects U to its complex conjugate matrix U* through the
similarity transformation

s-r us : (l* (4.7)

(b) Supposefr and,[2are the bases forthe spin-] representation of SU(2) having
eigenvalues of *] for the diagonal generator 73,

: lln and Tzrlrz: _|r!tr,


ftt1t1 (4.8)

calculate the eigenvalues of Z3 operating on fi and rlri, respectively.

Solution to Problem 4.3


(a) We will prove this by explicit construction. Problem 4.1 taught us that the
unitary matrix U can be expressed in terms of its generating matrix U - exp i H .
Group theory and the quark model 4.3

Thus the matrix S, if it exists, must have the property of


s-lHs : -H* (4.e)

sothat S-rUS: S-1(exprFl)S : (l* - exp(-iH*). Thegenerating maftix H,


being a 2 x 2 traceless hermitian matrix, can be expanded in terms of the Pauli
matrices

H:atot*a2o2*a3o3 (4.10)

with real coefficients of expansion cs. Since cr1 and 03 ata real, o2 imaginary, we
have

H*:atot-azoz*aloz. (4.rt)
Equation (4.9) can be translated into relations between S and Pauli matrices:
S-torS : -dr, S-lo2S : oz, and S-1o3S -o3. Namely, the matrix S must
-
commute wrth o2, and anticommute with o1 and o3. This can be satisfied with

S:c6z (4.t2)

where c is some arbitrary constant. If we choose c : I,the matrix S is unitary and


hermitian; for c i, S is real.
-
(b) The statement 'lr1and rlrz arethe bases for the spin- j representation of SU(2)'
means that under an SU(2) transformation (i I,2) -
: Uiirlti with U - exp(ia. o).
tbi --> ,lri (4.t3)

In matrix notation, this is ,lr' : U r[.The complex conjugate equation is then

,lr'* : u* rh*
- (s-1 u $(t. or (sU*) - u (st*). (4.14)

This means that Sry'* has the same transformation properties as vb. Explicitly, with
S : ioz, we have

'u.: (j, ;) (rt)


:(i,) (4.15)

To say that it has the same transformation properties as

r: ([':) (4.t6)

means that, for example,

,'(i):('(' -?,,) ($,) (4.17)

Namely, the eigenvalues of the T3 flenerators are

tz@rD-tz((rr):i
tz@r)-tz(L)--t. (4.18)
4.4 An identity for unitary matrices 8l

Remark. This shows that the T - | representation is equivalent to its complex


conjugate representation. We say that it is a real representation. This property
can be extended to all other representations of the SU(2) group, because all other
representations can be obtained from the T
- j representation by tensor product.
Paft (b) shows that the matrix ^l transforms any real diagonal matrix, a.g. 03,
into the negative of itself. In other words, S will transform any eigenvalue to its
negative. Thus the existence of such a matrix ,S requires that the eigenvalues of the
hermitian-generating matrix occur in pairs of the form La 1 , lu2, . . . (or are zerc) .
It is then clear that for groups of SU(n) with n > 3, such a matrix S cannot exist
as eigenvalues of higher-rank special unitary groups do not have such a special
pairwise structure.

4.4 An identity for unitary matrices


(a) Show that if A and B are two n x n matices, we have the Baker-Hausdorff
relation

eiA Be-iA : B + ilA, Bl + Bll+ ..'


;e,lA,
i' .fA, B). . .ll + .'
* ,lA,fA, .. . (4.1e)

(b) Show that the matrix B is invariant (up to a phase) under the transformations
generated by the matrix A, if these two matrices satisfy the commutation relation
of[A,B]:8.

Solution to Problem 4.4


(a) The matrix ./, defined as ,/(i,) - eiLA gt-i)'A, being a function of some real
parameter 1., can be differentiated to yield:
dJl
: eiLAipA, Blr-'^o + -l
dL lx:o -
ilA.Bl:iCr
H
d2J azll .
N - ei^AizIA, IA, Bffe-iLA + -ldL'lx:o - i2IA,[A, B]l: i2cz

::
{: L- - r cn-ie-ixA
,i)"AyfA, + {:l i'fA, c,-rf : cn.
dLn l^:, -
in
d)rn - - rt- t J-

Expand /().) in a Taylor series:

r(\\ -i7:o '''dL' |lx:o {n!- i,,


?-o
c'-, )"
(4'20)

where Co : B, Cr : lA, Bf, and Cn : LA,Cn-r]. Setting J. : 1, we have the


desired result
.)
eiABe-iA - B I ilA, Bl+';e, [A, B]l +... (4.2t)
82 Group theory and the quark model 4.5

(b) To show that 'the matrix B is invariant (up to a phase) under transformations
generated by matrix A' means to show that

,iuA gr-iuA : B (4.22)

for an arbitrary real parameter a. But from Part (a) we have already shown that

,iaA6r-ia^: f rr! (4.23)


E
where Co : B, Cr - [A,Bl, and Cn -- IA,Cn-r]. For the case at hand of
[4, B]: B wehaveC,
- B for alln - 0, 1,...
,iaAgr-iaA: Bj,,{ grio (4.24)
-
This is the claimed result.

4.5 An identity for SU(2) matrices


Prove the identity for 2 x 2 unitary matrices generated by Pauli matrices o
(o1, o2, o3):
-

exp(ir .o): cosr * (i.o)sinr (4.2s)

where r - lrl is the magnitude of the vector r and ?


- r /r is the unit vector.

Solution to Problem 4.5


We will first derive a useful identity for Pauli matrices. Consider the multiplication
of two matrices

(A . o)(B . o) : (o; oi)AiBi

- |l@r oi * oi o) * (oi o j - oi o;))A;Bi


- |{{ot, oiI tloi, oil)AiB1
- l1Z6i1 * 2ie41,o1,)AiBi @.26)

where we have used the basic commutation relations satisfied by the Pauli matrices:

foi, oiJ - 2ieijror, and {oi, o jl : 26ii . (4.27)

Thus we have the identity

(A. o)(B . o) : A .B
* io. (A x B). (4.28)

SetA: B: r,weget(r-o)2:r2+io.(rxr) - r2 and(r.o)3 -r2(r.o):


13 (? .o). It is then straightforward to see that

(r . o)2' : r2n and (r . 612'+r - ,Zn*t (i . o) (4.2e)


4.6 SU(3) algebra in terms of quarkfields 83

with n : I,2,. . . . The desired identity for the unitary matrix then follows as

exp(i r .o):+;$.o)n
_ IL n:
'!r, +(i.o) f4.nt'1r'
':ooo
: ;r;"* (i . o) ,,n,. (4.30)

Remark. This relation holds only for 2 x 2 unitary matrices and does not hold
for higher-dimensional cases, where anticommutation relations are much more
complicated than just the Kronecker delta.

4.6 SU(3) algebra in terms of quark fields


(a) Given a set of composite quark field operators

Fi : (4.3r)
I ,,urtn@)d3x
where the quark field operators
{

tr;:i
i.*
q(x) (4.32)
tit
ll t' -
irr.
flt;
f;i,i
G.i:)
[:l

frI satisfy the anticommutation relations


fi,1
!i*
hr
il'$
iiT,

,i,;i
fa"(r),
qlDll',:r, - 6ob63(x - v), (4.33)

lli,
fi:i
llt; and where ).t, with i- 1,2, ... , 8, are the Gell-Mann matrices
fi
th,

(4.34)
ii.,
li
;ll
qi
It'+l-;vii't'
fi'
show that { Ft }, if assumed to be time-independent, generate the Lie algebra SU(3):

lFi , Fil - ifiik Fk. (4.35)

(b) Calculate the commutators [W!, W!1for the non-hermitian generators

w! : ,;@)q"(x) d3 x. (4.36)
|
Show that W) is just the isospin raising operator. Similarly, Wl and W] ate,
respectively, the U-spin and V-spin raising operators.
84 Group theory and the quark model 4.6

Solution to Problem 4.6


(a) The proof can be obtained by applying to the commutator [Ft , Fi] the iden-
tity of

lAB,CDl _ -AC{D, B} + A{B,ClD - C{A, DIB + {C, AIDB, (4.37)


which for the present case has {A, Cl
- {8, D} : 0,
tFi , Ft1:
I f x d3vlrlr.t (:) ,reue), q:(, (+) ,,or{ttf
I o'.d,y6,(x - y)lnl<.>(:) ",u,"(+),oaao)
- qtl) (+) ,,u., (:)
",au@]
-| a" qt (x)l:' - ir'jo Fo ' (4'38)
+lq'l
where, because Fts are assumed to be time-independent, we have chosn -rs - )s
for convenience, and applied the equal-time anticommutator of the quark field
operators.
(b) Again from the identity eqn (4.37) and the quark field anticommutation rela-
tions, we have

lwX,wll: I f xd3ylol<*ta"@), q)g)a,@f


- I o'* dry 6'(x - y) lalraaia,(y) - q)ry>d!q"@f
-- a|w! - au,w!. g.3s)
If we write (qt, qz, Q) = (u, d, s), the non-hermitian operator

w): lnl<.>nz@)d3x- li@)d(x)d3x (4.40)

is shown to be an operator which transforms a d-quark to a u. Clearly l4lrl is the


isospin raising operator. Similarly, we have , lL d and, lL u.

Remark. In this notation the third component of the isospin generator Z3 takes
the form of

rz: qJq)a'*
L
lrr,, - d.ra)d3x: L l<n[n, -
: trwl - wh @.41)
and the hypercharge in the form of
y : ! rur, + dId- 2sts) d3x: +wi +w: -zwil. @.42)
|
4.7 Combining two spin-) states 85

4.7 Combining two spin-j states


Consider a doublet ,L : (lrr,tb) of the SU(2) group. Show that the composite
operators S
- lrIV and V - ltltlr, t : (rr,rz,z3) being the usual Pauli
matrices, transform as a scalar and a vector respectively. Also, demonstrate the
vectorial transformation property of V in several ways:
(a) for a general infinitesimal rotation,
(b) for a finite rotation around the 3-axis,
(c) for a general finite rotation.

Solution to Problem4.T
Under the SU(2) transformation, we have
'9.43)
,lt V' : r-ia't/z{r, VI ,lr'I
- ',yIria'r/z
- -
where a
- (ar, az, a3) are the three arbitrary real parameters. It is clear that tlrl tlr
is an invariant under SU(2) transformation.

S'
- ,h't ,lrt - tlrl ,+i u'r /2 ,-i u't /' ,1, - ,lrI ,1,
- S. (4.44)

(a) To demonstrate the vectorial transformation property of V under a general


infinitesimal rotation

tb -----> rb' = 0 - iu. t/2){, (4.45)


,lrl ,lr't -,/t(t + iu. tl2),
-
we note that the transformed composite operator can be written

n' n r t2,v
: I::,i: :,w' i i?J,t-r' (4.46)

But the commutation relation for Pauli matrices yields

iaieirtrt - i(t x a)t .


lT, ,o)--
"'17, ,07: @.47)

Thus we have demonstrated the vectorial nature of V under the infinitesimal SU(2)
transformation

V' : rltI(t - r x a)lr :V -V x a. (4.48)

(b) To demonstrate the vectorial transformation property of V under a finite rota-


tion around the 3-axis:

,[' : Rlt, ,lr'l - {t nI with p _ ,_iuztt/2 (4.4e)


86 Group theory and the quark model 4.7

the transformed V operator can be written

V' : ,lr'I trlr'


- t1i pt T RV - ,,yt ,iatt/2 , ,-iazrt/2p. (4.50)

Applying the formula of eqn (4.19)

RtzR -r*,1ry,'] *'Ofry,1ry,,]] + (451)

we clearly see that, because lGt/2),ttf :0, and thus Rtz3 R : 13, the third
component of V is unchanged under a rotation around the 3-axis. For the other two
components we need to calculate [(q 12) , z1 ,2 ] . This can be considerably simplified
if we work with the combinatiors z1 : rr *. ir2, which obey the commutation
relation of

llL2' '*l
"-J - tr+.
- 4L-'
(4.s2)

In particular, we have

17,17, l7 '*l ll -r,, (4.s3)

and thus

RtzlR -r1 (t*(ia:) +).tior>'+.. r+eio', (4.s4)


):
Rtr- R:r-(t*(-ia:) +).r--i*)z+ r-e_.io' (4.ss)
):
It then follows that

RtrlR - I (r* eio' + r-r_.'o')_ cos r,zrr - sinu3q @56)


2',
and similarly

RI t2R
- e-io' + ,- r*'o'): sin azrr * cosa3r2. @.57)
){r*
Consequently, the three components of V have the following transformation
property under a finite transformation around the 3-axis:

Vi : coscv3Vr - sin atVz,


vJ : sin a3 % + cos dzVz,
VJ: Vt. (4.58)

(c) For the case of arbitrary SU(2) transformation, we have

V' - ,lr't trlr'


- ,lrt Ut rU V where g - ,-iu't/2 . 959)
urtu - t*,1(T), '] *'^X+),1(T) ,']] +
4.8 The SU(2) adjoint representation 87

The basic commutator can be calculated as follows:

t(T) , ,o): "'l+, ,o): uiiei*rrt- (- u.t)p1r1 (4.60)

where we have recognized the spin-l representation of the rotation operator

(ti)u: -ie 1u. (4.61)

For the double commutator,

t(?, l(T), "ll - (-(-a.t)u?u't)tiri


u t),,1(T)'',1
-
.t)21,1t1
- (- a G.62)

and so on. Thus

-t-
UttpU : II
L
+ (-, a .t) ' -, u .D2 + . . .fJnr,, -
* ,.( (e_.io*)p1t1, @.63)

or

Vj + Vj : (r-'"\ joVo. (4.64)

4.8 The SU(2) adjoint representation


(a) Suppose @ transforms as a vector under SU(2) as discussed in eqn (4.64):

Qi-Q'j:(r-'"'\ioQo. (4.6s)

Show that the transformation law for the 2 x 2 matix defined by A - z. d is


given by

.t, ---- 6' : ut ou with g - ,-ia't/2. @.66)

(b) Suppor. i ir a2 x 2hermitian traceless matrix which transforms as

i ---- i' : uliu with g - ,-iu't12. (4.67)

Show that i' is also hermitian traceless, and with det i' - det i. Since i and
i' *. hermitian and traceless, they can be expanded in terms of Pauli matrices

i-t.6 and i'-r.O'. (4.68)

Show that S and Q' are related by a rotation.


(c) Suppose we have the nucleon in the isodoublet representation N - (p, n),
and the pion in the isotriplet representation r - (nr, Tz,llt) with z3 : v0,
(nr - in) /rfZ - ni , and (21 + in) /Ji : 7r- , construct the SU(2) invariant
pion-nucleon n NN coupling.
88 Group theory and the quark model 4.8

Solution to Problem 4.8


(a) We will show that 6'
- UI6U follows from the transformation eqn (4.65):
6': Q'iti - @-i"'t) jtfir,tj (4.69)

: Qolll-t"l+ (-, a.t) +


r.(-id.t)'+... )0,r,
which can be written, according to eqn (4.63), as

6' - Qr,Ur r*u - Ur 6u. @30)

Remark. This problem shows that there are two alternative ways to describe the
transformation of the vector representation (more generally, the adjoint represen-
tation): as in eqn (4.64) or as in eqn (4.66).
(b) To show that i'is hermitian if i is hermitian:
i't - (Uti (l)t : UriIU - (JtiU - i'. (4.71)

To show that i' is traceless if X is traceless:


tri' - trutiu - truuti - tri - 0. (4.72)

To show that det i' - det i:


det i'
- det uliu - det uuri - det i. (4.73)

Expanding i' and i in t"r-r of Pauli matrices:

i-r'o-(r,l ,r,r'-;,r'), (4-74)

it is easy to calculate their determinants:


deti _ -(o?+o'r+a?). @.7s)
Thus the above result of det i'
- det i implies that the transformation 0 * 0'
leaves the length l@l unchanged. This must be a rotation.
(c) From (a) and (b) we see that the2 x 2hermitiantracelessmatrix fi: t.n,
formed from an SU(2) vector z, transforms by the similarity transformation:
ft ---- fr' - utnu with g - ,-ia't/2. @.76)
In this form, it is easy to see that the product NtfrN, where N is an SU(2) doublet,
is invariant under SU(2) transformations. This suggests the invariant pion-nucleon
tr N N coupling to be

L,,NN-st t rN - s(F,
r_
^(;*-g:)(f)
- sltnn - nn)no + Jr(pn,T+ + nnr-tf , G.77)
and thus the relations among coupling constants are

: : 11 : : 8.
Sppro -*nnno @.78)
780"ftt 68nn"-
4.9 Couplings of SU(2) vector representations 89

Remark. The relation gorr* _ gnpo- follows from the hermiticity of the
Lagrangian density (or charge conjugation).

4.9 Couplings of SU(2) vector representations


(a) The p vector meson has isospin I (it has three charge states: p+, p0, p-).
Construct the SU(2) invariant prr coupling.
(b) The a,l vector meson has isospin 0. Construct the SU(2) invaiant apz coupling.

Solution to Problem 4.9


(a) Since p has spin 1 and isospin 1, we can represent the p-fields as pr("r) and
define a2 x 2 matix F, : 7 . pu.As one has seen in Problems 4.8 and 4.9, it
transforms under SU(2) as

F*- FL-urFru (4.7e)

justas the pion matrix ti ---- li' Urfi,U,where U is an arbitrary 2 x 2 SU(2)


-
matrix. This suggests the invariant coupling to be

Lpo, - gff(Ptlar'fln)
- s,r(r'; , '::J:)(t# tll;:J)(;)" ti:)
- gl2p0' (arn+r- - orn-r*) + 2p*u (arr-rU - 0rn0n-)
* 2p-u (arn+no - outtor+)]. (4.80)

This implies, for example, the equality of decay rates

f (p0 --> n+n-) - f (p+ -- n+n}) - f (p -. n-tr}). (4.81)

Remark 1. The decay p0 --> n0n0 is forbidden because the (r0n0) system can
only have even orbital angular momentum because of the Bose statistics. Hence
angular momentum conservation will forbid this decay. Note: the same argument
can be applied to the vector gauge boson Z to forbid the decay into two identical
Higgs (scalar or pseudoscalar) bosons.

Remark 2. The other possible coupling tr(Funarn) is not independent of the


one just considered. This can be seen by applying the Pauli matrix identity

(2. A)(2. B) : (A. B) +iz. (A x B) (4.82)

which implies that

,, (F*arfrn) -ipp(}Ptrxn), (4.83)

,, (Frnarn) - -i pp(At'r x r). (4.84)


90 Group theory and the quark model 4.10

(b) The SU(2) invariant Lorentz scalar combination out of the p, T , and a meson
fields can be constructed as

Lpn. - gtr(Fpfi)r*
- 29 (ptr"- * oun+ * oorno) coP. (4.8s)

4.1O lsospin breaking effects


Exact SU(2) symmetry implies the degeneracy for particles in the same irreducible
representation of SU(2). But the SU(2) isospin symmetry is broken in nature by
electromagnetism as well as by up-and-down quark mass difference. The first-order
electromagnetic breaking, involving the emission and absorption of a photon (and
thus the electromagnetic charge operator Q acting twice), contains an isospin-
changing A,I 1 piece, as well as a A1 2 piece. On the other hand, the quark
-
-
mass-difference:

muuu + mddd
- ^u !^*o (uu * aa'l +
22 ^' :*o @u - dd) (4.86)

contributes only a A,I _ 1 breaking, as the last term transforms as the third
component of the isospin generator 13. Thus the strong interaction Hamiltonian
can be written as

H -- Jlrct * H'rry * T{'p1 (4.87)

where ft10; is SU(2) invariant, H'r, is the A/ - 2 electromagnetic breaking


term, and the A1 contains both the electromagnetic and up-down
- 1 piece Ht6l problem you are asked to calculate the first-
mass-difference breakings. In this
order mass shifts due to 17' by using the Wigner-Eckart theorem for the following
isomultiplets:

(a) 1:i.,(p, n),


(b) 1- 1: (x+ t0, t-),
(c) 1- f : (A++ , a+, ao, a-).

Solution to Problem 4.10


According to the Wigner-Eckart theorem, the matrix elements of a tensor operator
O{ ,having isospin 7 and third component value M,have the simple structure of

\t' , r:lo{1,, tr) - (I' , Illr, M; I, (4.88)


4(t' llo'lll)
where the first factor on the right-hand side is the Clebsch-Gordon coefficient and
the second factor is the reduced matrix element, which is independent of Iz, I!,
and M . For this problem, the operator O : 11'61 andH'py which transforms as the
(T : l, M :0) and (T : 2, M : 0), respectively.
4.I0 Isospin breaking fficts 9I

(a) 1 | multiplet (p, n). Thefirst-order mass shift due toH'111can be evaluated
-
by the Wigner-Eckart theorem as

6(t)mp
- (pluiul p) - (i,Ll71',rl+,+)
- (+,| | r, o; L,Ll d,',** : -J!30)mn (4.8e)

and

a(t) m, - (nl'rl'sl "l - (+, - L luiul +, - +l

- (+,-ll r,o; L,-+)6o)mr,r - ifidom* (4.e0)

where 6(r)mu is the /3-independent reduced matrix element.


Exactlythesamecalculationshowsthatthe L,l g
- 2shifts 6Q)mp - 5Q)*n -
coffesponding Clebsch-Gordon coefficients vanish (because an I
as the
- 2 oper
ator cannot connect two 1 I /2 states).
-
In this way we find that the proton and neutron mass shifts \mp,n - 6(r)mp,n
have the same magnitude but are opposite in sign:

6*, - -3m,. (4.91)

Remark 1. We can apply this result to any other I: i multiplets. For example,

6mso - -6ms-, 6mso - -6mx-, etc. (4.92)

Remark 2. Altematively, we can write down an effective mass term in the


Lagrangian, which contains an operator having an isospin value of (I : l, M :
0). For the isodoublets this can be represented by a 2 x 2 matrix, 23. The effective
mass term for the nucleon can then be written as

L6** : N3m7,rr3N - 6mN(t, r) (; jr ) (;)


- 6mN Qp - nn), (4.93)

which yields

3*o
- -6mn - 6mx. (4.94)

(b) 1
- 1 multiplet (E+, t0, t-) The Wigner-Eckart theorem yields

3(t)m>*
- (t* lu'rulE*) : (t, +t lu'r) 1, +l)

- (1, +111,0; 1, *l75i]Jm2 - ,f !6{t)m' g.g5)

6(t) mzo
- (ro luiul xo) : (t,0171'sy I t, o)

: (1, 0l 1, 0; 1,013(r) mr : 0 (4.96)


92 Group theory and the quark model 4.10

3o)mz-
- (t- lH',rl r-) - (1, -rl'tl'o>l t,-t)
: (1, -111,0; 1, - I)60m2 - -if )8{Dm" (4.e7)

and

6Q)m>*
- (t* |\l'rrrlE*) : (1, +1 12,0; r, +rllQ)m' - ,fru Q)*> (4.9s)

6Q)m>o
- (to lU|r)Eo) : 0,012,0; 1, 0)6(2)*" : -rl|u"'*, (4.g9)

6Q)m>-
- (t- lH'rrrl>-) : (1, - 112,0;1, -175Q)m2
- ,f
L*a{2)mr. (4.100)

Combining these results and using the notatiorr t/t1 - Jl$rlm> and m2:
-L*a<ztm,, we have
,f
t7t2+:mO*mt+m2,
tTt2o-_mO*0-2m2,
mt-:ttxg-mtlmZ. (4.101)

The /- 1 and I- 2 mass splittings can then be isolated:

tltt:l@r.-mE), (4.r02)
t7t2 : j@r. * m>- - 2m2o). (4.103)

While ml contains both the electromagnetic and up-down mass difference effects,
m2 is purely electromagnetic in origin.

Remark. The same analysis holds for the isotriplet pions. In particular, we have,
besides 60)moo 0, the result
-
60) mr* - -3(r) mo_ . (4.t04)

But z+ is the antiparticle of n- and should have the same mass. Hence

6mn* - 6mo-. (4.10s)

The only way to reconcile these two eqns (4.104) and (4.105) is to have the reduced
matrix element 6o)m, :
0. (The Wigner-Eckart theorem does not by itself give
any information about the reduced matrix element.) The same result can be seen
from writing out the mass term in the Lagrangian:

L-(n) - L0)mo" (nrrn)

- L(,,m,,,(;*_ ti:) (; j,)( i: t:::)


- Lo)mn(no' -2n+r- - n02 l2n+r-) :0. (4.106)

Thus the pion mass differences (and for p mesons also) are entirely due to the
I- 2 electromagnetic corrections.
4.lI Spin wave function of three quarks 93

(c) 1 :3/2multiplet (A++, A+, Ao, A-;


$ The Wigner-Eckart theorem yields
jfi
,!

6o) m t
sa)*^**
6++ - 11
6** -r-11 tl, 0; 12,
(1r, +?rl +)l
+)l 6(t) J l6(t) m t
n - nfatrl
3G) m
tnt 12,

6o)mn*
- (1,+Llr,0;1,++l6o)mt - if ft6{Dmo
6o)mno : (1,-ilt,o; ].,-+l6Q)mt: -u[a
o)mt

g.t07)
6o)mr - (1,-,11r,0;1,-'116o)mt - - iflr{l)mo
and

6Q) m
- (1, +112, 0; 1, +zrl m n - ,f m o
6** atz) Lratz)

6a)mr - (1,++12,0;),+Lr)6Q)mt - -[ru'2)mt

6Q)mto : (2, -Llz,ot ].,-'Lr) sr'r*o: -,4u,2)md,

3Q)m n- - (1, -,1lZ,Ot 1., -tr) Sr" * o - if Lr$2) m


o. (4.108)

Combining these results we have

tlt6++ - t/t11* 3m1 * mz,


tlt!+ -mO*ftlt-tTlZ,
lfl 60 : tftg - llll - lll7,

tTtL-:rng-3mt*mZ. (4.10e)

Besides allowing us to isolate the I - | and 1 - 2 mass splittings, these equations


also imply a mass relation of

- tlt6- :3(mt
tTt6++ - m*). (4.110)

This simply reflects the absence of an I : 3 piece in the symmetry-breaking


Hamiltonian.

4.11 Spin wave function of three quarks


As an exercise in Clebsch-Gordon coefficient calculation, construct the spin states
of baryons, which are composed of three spin-] quarks.

Solution to Problem 4.11


The possible spin states for two quarks are S12 : 0, l, where Srz : Sr * Sz. As
discussed in the text, the,Srz : 1 states lSrz, Srz. ,.1 are

11, 1) : d,tdz

I 1, o) - h(q flz * frpz)


11, -1) : hflz (4.111)
94 Group theory and the quark model 4.Tl

where d
- l,l)
|
and F :
l+,-+l are the spin-up and spin-down states, respec-
tively. Also we have the spin-zero combination:

10,0) - h@rflz - Fp). (4.1t2)

CombiningtheSrz: I stateswithS: - i,*"obtainS: I andI states,where


S:Srz*S:-Sr*Sz*S:.
ls - i, & : t) - tl, 1) I Lr,+)
- or1d2d3. (4.113)

To reach the other S : I states, we use the lowering operators


s-ls, s.) : fts + s.)(s - sz + Dftt ls, s. - 1) (4.114)

to obtain

s-l;,7):Jil|,+l . (4.11s)

On the otherhand, S- - (Srz)- + (S:)-. Thus

s_ll, 1) l;,i) : (s,r)_11, 1l li,+i) + 11, 1)(s:)- l+,+l

-Jzll,0)o3+11, l)Fz. (4.116)

Combining eqns (4.115), (4.116), and (4.111), we get

1,, +l - +lAv o)a: + lr, rlP,]

- hl@rflz * fuu)q * uP2B3l


- h[atflzaz * fezaz *
ap2B3]. (4.117)

Similar to eqn (4.113), we have

ls - ?,s. - -|lt - tl, -1) l+,-Lt - p'Fzflz, (4.118)

and using 51 we can obtain

l),-+l - h[arfrzfz * Fpzflz * fufzui' ( 'tte)

The state lS
- i, S. : |)mustbe orthogonal to lS - i, S. : j) in eqn (4.117):

l+,+), - h[-,t,0)o, + Ov rlr,]


- +l2ap2B3 - (flpz * a1B2)ql,
(4.120)

where the subscript S signifies the symmetry property of the state under the per-
mutation of quarks 1 <> 2. Similarly, we have

l+,-i), : rtrzT'flzaz - @r7z * flp)fli. @.12r)


4.11 Spin wave function of three quarks 95

nut lf , ++) can also be obtained from combining the S12 - 0 and S: : 1 states.
Such combinations are antisymmetric under the permutation of quarks I <+ 2:
l+,+l^- h@rfrz- flp)uz
l+,-Llo: fi@fi2 - flp)fl2' (4.122)

To summarize,wehave four S : lstates, which are completely symmetric under


any permutation of all three quarks (1,2,3):

: ,tot2o3
llr'1rl
l1'+) - hlarflzq* BP2u3*uP2B3l
l'r'-+l - hIarflzflz* flpzflz* hflzazl
11,-1) : fl'flrfl' (4.r23)

and two S : I states ll, +l), = XM,s,which have mixed symmetry with respect
to the permutation of ( I , 2, 3) but are symmetric under the permutation of | <+ 2:
l;' +)r - kl2ap2B3 - (fpz -t arfl)azl
l+, -l), : ftlzfrtflr"z - (atflz * P1u)Bt1
(4.124)

and two S :
I shtes li, +ilo = XM,A,which have mixed symmetry with respect
to the permutation of (1 ,2,3) but are antisymmetric under the permutation of
1<>2:

li,+)o-#@rfz-fp)az
l+,-Lrlo: $@,fl2 - fp)fr2. (4.12s)

Remark. If we are interested in the isospin of three non-strange light quarks u and
d (as for the nucleons and the A resonances), we can work out the corresponding
isospin wave functions by the simple substitution of cv --> u and B --> d:
(a) Symmetricisospinl
- lstates
A++ - 11,t): utuztt3
A* : 11, +l - *luizuz * dp24 t up2d3)
Ao : l?r, -+) :
+ludzdt * dp2d3 i d1d24l
a- - ||,-zri - ata2dt (4.126)

(b) Mixed-symmetric (but symmetric with respect to the interchange 1 <+ 2)


I:istates Qu,s:

I +' -;)r *[2upzdt - (dpz * ud)ui


l+, -l), : ftQataruz -
(udz -t dp)di. (4.r27)
96 Group theory and the quark model 4.I2

(c) Mixed-symmetric (but antisymmetric with respect to the interchange 7 <> 2)


I:LstatesQu,e:

l+'+)o - h(udz - d1u2)u3


l+, -Lr; o : fr{rra, - d1u)fu . (4.128)

4.12 Permutation symmetry in the spin-isospin space


Show that the spin and isospin combination of the mixed symmetry states dis-
cussed in Problem l4.ll, Xu,sQu,s * Xu,eQu,t rs invariant under the general
permutations of particles indices. Hint: Such permutation operations are repre-
sented by orthogonal matrices in the 2D spaces spanned by mixed-symmetric spin
wave functions, Xl lya,s and Xz XM,A, or isospin wave functions, Qt : Qu,s
- -
and Q2 : Qu,e.

Solution to Problem 4.12


The general permutation of three indices can be denoted as

.:(,1 ?, ;) (4.rze)

where (I,2,3) are replaced by (ir,i2,fu)-thus a permutation of the particle


indices (1,2,3). There are six elements in this permutation group 53:

Pp: (: 1), Pt:: (l i ?), ,":(l:"),


Pn3 : (;??)
" Pnz : (l

"'r),':(ii;)
It is clear that under any of the permutation operations, the mixed-symmetric spin
wave functions /1,a,s and XM,A transform into linear combinations of /y,5 and
Xu,e. The trivial examples are 1 and Pp. By construction, we have

PpXu,s: XM,s' PpXu,,q,: -XM,A, (4.130)

i.e.X*,s and Xy,a are eigenstates of Ppand 1. The operator 1 and Ppcanbe
written as an orthogonal matrices:

ir = (; ?) (l) , F,,i = (l j') (l;) (4.131)

The more general cases can be exemplified by P13 acting on

xr - - JCl2ap2B3 - (fpz * u1B)a31,


ll, +l),
xz -- lLr,+Lrlo - #@rflz - flp)az, (4.t32)
4.13 Combining two fundamental representations 97

to yield

PnXr - - (fltaz * uzfl)qi


*12a3a2fu

- -+x, - fxz. (4.t33)

and

Pnxz

::;ru,'* ru)'"' (4.134)

Thus Pp c"n also be represented by an orthogonal matrtx

F,,i:(_+ ;?(::) (4.13s)

It is not difficult to convince oneself that, because the permuted states P X - X'
must remain orthogonal to each other, all the six permutation elements can be
represented by orthogonal matrices:

FrF-FFr-f. (4.136)

From this property it follows that the combinations such as

i., *" : 'r)


xi + X;, *.r Q : xrQr * xzLz (4.131)

are invariant under all the permutations of the particle indices.

Remark. Since the combination Xu,sQy,s * Xu,eQu,a is invariant under any


permutation of quark indices, it is totally symmetric in the spin and isospin space.
From this we can conclude that the nucleon wave function, which is a product
of spin, isospin, and the totally antisymmetric colour wave functions, is totally
antisymmetric with respect to interchange of any of its three quarks. This is com-
patible with the requirement of the (generalized) Pauli principle as the nucleon is
a system of fermions (quarks).

/3.13 Gombining two fundamental representations


Work out the tensor products of the defining representations of SU(2) and SU(3):

(a) Let ,1,


- $:,) - U) be an isospin doublet with its hermitian conjugate being
{t : (rlri lr;) - (uI at).nin0theisospinof theproduct rlrirlri (where i - 1,2).

(b) Let r : ([:) : (?) be an su(3) tripler Decompose rhe product ,t f {i


(where i: 1,2,3) into irreducible representations of SU(3).
Group theory and the quark model 4.13

Solution to Problem 4.13


(a) It is useful to denote the complex conjugate lri by t'. Namely, ,b' = ,lri.
From the fact that under an SU(2) transformation,

$i-rbi :Ui;r\i-Ulrlri (4.138)

where Ul = Ui1 and U is unitary, we have

,lri rb!*:Uiitj -(/;(ul). (4.r3e)


-
,lr' rlr":rlriuj (4.r40)
-
where U j = (Ul)..It is clear that the combination Vi ry'; is an SU(2) invariant:

,lr"rbi
- lli Ujulrlro - Vi UiiU,orlro - tyi aj(rt - ,lri rlri. e.t4t)
Thus ,h' ,lr, has isospin I : O.It is easy to see that the remaining three combinations
in the product ,lr'Vi transform as an I : I triplet. We can remove the 1
combination by the following subtraction:
- 0
Tj : rlr' rlr
i- LS',(rlrk rlrr,) (4.142)

which has the property of Ti - 0. The { components can be explicitly written


out to be:

T]:rlrtrhr:utd-ir
T? : V'rlrt dtu - T*
-
and

Tl : rlrtq,- L @t h t rlr'rlrr) - i @t rlr, - rb'ryr)


- l{uru - dril - hro @.143)

T] : - L Urt V, + rlr'rlrr) -
V'rlrr. -+ (rlr'rlr, - rb'r[r)

- -+(rI u - dI d1 - i"o. (4.144)

Sometimes it is convenient to write fj as a traceless matrix

,:(7 fr):+(;" ':) (4.r4s)

We can summarize the result in the form of a direct sum:

2* x2 -L+3, (4.146)

where the representations are denoted by their respective dimensions. The triplet
is called the adjoint representation of SU(2).
4.I3 Combining nuo fundamental representations 99

(b) Again the SU(3) invariant trace

t' ,h, : ttl tt + dI d * sts (4.147)

is an SU(3) singlet. The remaining eight components transform as the octet repre-
sentation under SU(3),

3x3*-1+8. (4.148)

Following the same procedure as in (a) we can display the adjoint representation
of SU(3) as
A,j : rlr'rbi - \s',(rlrkrbr). (4.149)

To display the quantum numbers of various components:

A'r: tid - T-, A?: dIu - ttt,


Atr:Ltls - K-, A1 :srtt - K*,

A'r:rld-Ko, A3:dls-Ko, (4.150)

for the diagonal elements

Al : uI u - !o'u + dr d* sts) - (4.1s1)


#. +,
where
o
,T0 : I , I rtr,
- dId.),
n
,o -
I + -+
* drd - 2sts). (4.152)
ir"'" frfutu
Similarly, we can work out

.) no tf and Atr^- -e. 2ry0


AZ - -n+ e (4.rs3)

These octet components can be organized as a traceless hermitian matrix:

lei A1 A?\ (;:*


A-lei A3 All- o l -tz:a |,)
_,!*_u ,.^t.
^i) \ K-
\^i Ai "fu"" -,+,)
Because of the transformation properties of the defining representation and its
conjugate:

Vt {i:ulti, ----'1"'-'l'j(lj, (4'154)


-
the adjoint representation transforms as
'b'

Atj A"i : ui,uttef - (ui1)* A!(u), (4'155)


'
or, in terms of matrix multiplication, it has the simple form of
A ----> A' - 0t 40. (4.156)

Let us recall that here the U matrix is the defining representation of the SU(3)
group.
100 Group theory and the quark model 4.14

4.14 SU(3) invariant octet baryon-meson couplings


The baryon octet can be represented by a 3 x 3 matrix

++ 2+ (#*h
I F _ E=\
h+)
V6

": (* E-
s'-
-Eo
!z
-
l^
B_ -a** Edl
=; u', vo
go
\ p #)
and the pseudoscalar meson octet by

7t'
a
.4)
M_
(rlr n0,48
Jz
-..-:+....-:

fr
Je

(a) Construct the SU(3) invariant EnU couplings.


(b) Express all the above meson-baryon couplings in terms of two SU(3) sym-
metric couplings. This implies numerous coupling relations. But most of them
actually follow from SU(2) isospin symmetry. Thus it is useful to express these
SU(2) invariant couplings directly in terms of the two SU(3) couplings.

Solution to Problem 4.14

(a) In terms of these baryon and meson matrices, it is clear that there are only two
invariant E n ru couplings as there are only two independent traces of multiplying
these matrices together:

LBu, - J1[s, tr(Enw + szft(Eun)l


: "/1[r' (ul B: MX) + s, (ni u;4)] (4.ts7)

A common way to express these couplings is to write

D+F D_F
8r: '62
O^: , (4.159)

so that

F D
LEu* - +tr(BlB, M *
, Ml) ,= tr(B{M,, B)).
B (4.1s9)
J2 ,{2

Namely, the F coupling is proportional to the commutator, and D to the anti-


commutator. We now work out these couplings in terms of individual baryon and
4.14 SU(3) invariant octet baryon-meson couplings 101

meson fields:

(i) The z+ couplings

Lo* - Jfurr? (s,e4nj + sznln;)

:./in+{,,[* (#-+) .(- -.+) ,-+"'-,-]


*"[('-+) '-+'r(-#.*) ."] ]
:,D.n+1ry(*"' - tu"-) .
V(*n - lr-)
+ sl EoE- + srp"f. (4.160)

Or

Lo* : J2r+{#(-ro - rr-) . t"-)


+r(*n -
+ D+F __ O-r _ |
2 Eu E- + z p"
l.
(4.16t)

(ii) The rro coupling

Loo : olrl (s'einj + s,E|ri) * 4 (r,ElB? + na? ut)l


--7ro{''[(=.*) (#-+) +r->-+ e-e-
-'+E+ ( #.+) c#-+) -t"]
*,,[ (i-+) (#-*) +>+>+ +Fp

->'- ( #.+) c#-*) -,"11 .


: ir'[,r' - sz) (>->- - t+E).
ry (run * n"o)

* ,, (*E- - tut') + s2(Fp - r^f. ( 4.162)


t02 Group theory and the quark model 4.I4

Or

Lno :,'[o (-r- - >+r.) + (t^ * n"o)


#
.'+(* r- - eor') *'+(F p - rrf G 163)

(iii) The K+ couplings

'..
:::.lrr,'{r{l B + nE- - fro*l
*,,
[
(+. *) E- + >+ eo -
+r^]l
- riK+lr, (# + nE-) . r, (+* r'r')
r-+tzre-.+u^] (4164)

Or

Ly* : rzx*Iry (# + nr-) .'+(+ + re')


D+3F-AE- _--_-
D_3FDAIl.
zJo - zJo I (4.16s)

(iv) The Ko couplings

Lrco - Jzu] (srEln? + srE? B\)

-,DKo{,, [u". *o (-
#.+) - +o"]
*,,[-E-+ (rr.*) Eo- +,"]l

-riKo[', (-#.u".) ." (-#*r='-)


.-#uneo+ +on] @.rc6)
4.14 SU(3) invariant octet baryon-meson couplings 103

Or

Lyo : .h*lry
e+ rr.) .'+( + + t=E-)
D + 1F
zr eo - '
- 2o onl.
- zJ6 2J6 I
@.167)

(v) The 48 coupling


Lq, : "[zlui(s'ri n] + s,E] n7) + M] (s'Eln? + s,Eln;)

+ M3 (s,ri n] + srE?4) ]
: (ul n] + EIB? -zelal)
*[''
+sz (n| Bi + n7 ni - 2E?ul) ] +'''
so that

#[r't (= . *) (:"- +)
Lr': + >-r- + e-e- + >+r+

. (-4r*+) C#* *) + e0e0 -2pp -Znn- io^]


*,,[(i* ;;) (t* +) + >+>+ + pp+ >->-
.(-4r,*+) (-t"-+)
*in - zl--s- - zeoeo - io^]l .
Or
\
+ >o>o + rr*)
n8[ /-- :7
Lr, - U*lrr,-r g) (r="- - (gr * sz)AA
_I
+ (gr -2sr) (*t- +Eoeo) - Qst - sz)(pp *nn)l't
: (r=t- + >o>o + E+r*) - Pnn
#[,
.t+(*r-+Feo) -'+@p+r^f (4 168)

The other couplings are related by hermitian conjugation:

- Lr-, LI*o: (4.r6e)


- Ln-,
Lto* Lly* Lxo'
lO4 Group theory and the quark model 4.I4
(b) The baryon octet contains the following isospin multiplets:

singlet: A

doubrets: N : (r"), e: (;:) @.r70)

tripler, i - ( --": /-t"".)


W"- -Eo /
while the meson octet contains

singlet : q8

doublers : K: (il) , K": (f,) @ t7t)

tripret, fi_ tl:)


G
From this it is straightforward to construct all the SU(2) invariant couplings,
cf. Problem 4.8(b):
(i) Three singlets

,tLlt (4.172)

(ii) One singlet O two doublets

nN N , T E E, KA E, K'AN (4.173)

(iii) One singlet O two triplets

4 tri'i, A trin @Ji4)


(iv) One triplet O two doublets

,rrnlr, eng, NiK, eir' (4.175)

(v) Three triplets

tri'in @.n6)
By working out the components such as

l,tnlf _ (Fp - nn)no +.n@nft+ + npr-y


trt'ifr - z+ (ftro - >or ) - "- (*"0 - rox*)
- /_ \
i tru (>-r-
\/ - t+>+ ) @.177)
4.1,5 Isospin wave functions of two pions 105

and comparing them with the couplings shown above, we can easily express the
SU(2) invariant couplings in terms of the SU(3) D and F couplings.

D-F
S(7rNN)- Z , s(nIE)-P, (4.178)

and similarly

D
-g(ryAA) - s(nEX) : sjr /YE) : ------=

Jt
D +3F
s(ryNN) - s(KAE)
2'E
D -3F
s(ryEE) : s(KAN) - 2"8
s@aE) : -s(KNI) - D+F
2
D-F (4.r7e)
s(KE>) - -g(ttNN):

Remark. Just as we have seen in Problem 8 that, in terms of the Cartesian com-
n :(ttr n2 n3),^the coupling l,tnl,t can be written as N zN ' n,thethree
ponents
triplet coupling trE'EfI can be written as i E' x ) . r, where we have used the
identity

tr(qt-Tn) : it*n. (4.180)

4.1 5 lsospin wave functions of two pions


Two pions can have total isospin 1 2,I,0. Use the relations
-
I-lI, :t(I + I)(I -
Iz) Iz I I)lt/2 ll, Iz - l) (4.r81)
I+lI, It) : t(I - I)(I * Iz * Dl/z lI, Iz * l) (4.182)

to construct the total isospin wave functions of two pions.

Solution to Problem 4.L5


(a) The I
-2states
Starting from

- 1n[n]l12,21 (4.183)

we can use 1- 12,2) :212,1) and I-1,ll


- J211, 0) or l-ln+) - Olno) to get

t2.t) : + lzfzf)) : (4.r84)


+(1"1"9)
r06 Group theory and the quark model 4.15

anduse I-l2,ll - J61\0) and l-lnl) - ",f21n-)


toget

: 1.,
12,01 + zltrlrl) + ln; $l) (4.18s)
fr(|"{ ";l
.

Similarly,

12, -t) : O(1"?";) + ln; fi))


I

12, -2) - ln, rl) . (4.186)

(b) The I states


-l
The 11, 1) state is a linear combination of l"l";) andln, nf) whictr is orthogonal
to the 12, +ll state. Thus if we write

I 1, 1)
- a lr[ nll + b l"l"]l (4.r87)

with la 12 + lbl2
solution of a :
- l, the orthogonality condition becomes I lJl@ + b) - 0. The
-b - I lO can be chosen:
:
t1, r)
il"["8] - l"?"])) . (4.188)

This choice (as opposed to a : -b : -l /"n) corresponds to a particular con-


vention for the Clebsch-Gordon coefficients. It is easy to see that using the isospin
lowering operator 1- we can get the other I : I states:

:
tl, o)
iM[";l - 1";"{))
t1, -r ) : - l";"il) . (4.18e)
+1"?";l
(c) Thel-Ostate
The 10, 0) state must be orthogonal to both 12, 0) and 11, 0). This fixes it to be

: - l"i"il + ltr;n{l)
ro. o)
+|td (4.1e0)

Remark. We note that the I - 2 and / - 0 states are symmetric under the
interchange of particles 1 <> 2, while the 1 1 states are antisymmetric. (This
-
is why the combination l"?"ll is absent in the ll,0) state.) In general, for two
particles with the same isospin, the largest total isospin states are symmetric under
the interchange of particles I <> 2. Then the next isospin states are antisymmetric,
followed by symmetric states, etc. For example, in a system with two 1 :312
particles, the state with 1 : 3,I is symmetric, while the one with 1 2, 0 is
antisymmetric.
-
4.16 Isospins in non-leptonic weak processes 101

4.16 lsospins in non-leptonic weak processes


The low-energy AS : 1 non-leptonic weak Hamiltonian is given by

't1* : %Vr, e - ys)d.lliyr(r - ys)ul * h.c. (4.r9r)


'/2
The first term (ud) is an isospin state I 1, 1) and the second term (sa) +). From
^l+,
isospin addition of

t1, 1) li, i) : J"l+,+) + Jtll,+) , (4.r92)

we see that this weak Hamiltonian can be decomposed into two pieces with definite
isospins:

7{*:lltp*Hz/2. .(4.re3)

(a) Use this isospin decomposition of '11, and Clebsch-Gordon coefficients to


evaluate the decay amplitudes for

K+ --> lTtlTo, Ko -+ tt*x-, Ko -- noo', (4.te4)

in terms of two reduced matrix elements.


(b) Repeat the same calculation for the decays A -+ pn- and lv --> nno

Solution to Problem 4.16


(a) Here we need to evaluate the matrix elements of (tnlll.lKl _
(nnl'llrplrK) + (nnl'l13plK). Since (K+,K0; is an isospin doublet, lK+) :
-
lL,+L) and lKo) l+, -|), we have
HtptK*) - l+,+)li,il - t1, 1)

tlzptK*) - .1,+)l+,i) -+12,r) - |lt, t), (4.1es)

HrptKo) - :
l+, i)l+, -ilr h(l l, o) + lo, o))
HzplKo) - l+,+)l+,-;) -- t-rr(12,0) + 11,0)). (4.1e6)

From the result obtained in Problem 4.15,

t2, t) - h (lt "il + lnlnjl)


| 1, 1)
- i (1"[ "il - l"?"{)) ,
(4.r97)

K+ --> nr n0 must be either of the isospin


we see that the final state of the decay
states 12,l) or 11, 1). But from angular momentum conservation, n*to must be
in a relative orbital angular momentum L - 0 state, which is symmetric under
108 Group theory and the quark model 4.16

the interchange of ft* <> 20. Thus we must use the symmetric combination as in
12, +I) and only the 1 3/2 of the Hamiltonian can contribute:
-
qr+ n01H,lK+) r01r
- qr+
- 2)ytltplK+) - (2, t1Tfipl+, +). (4.19s)

The Wigner-Eckart theorem can then be used to relate it to a reduced matrix


element A3p

(2, t lr6pl +, +) _ 12, I l), +; +, +) Arp : * Arp (4.ree)


Namely, the decay amplitude is evaluated in terms of the reduced matrix element
AS

T(K+ -- r+no) : Arp. (4.200)

As for the decays K0 -+ T*n- and K0 --, ,r0n0, we have from Problem 4.15

12,
- )G (lt ";l + zlnlrvl) + ln; j))
o) n

ll,0) - h(lt";l - 1",


"j))
10,0) -
* ";) - |"|"il + ln, n]))
(1"[ (4.201)

which shows thatthe I


- lstateis antisymmetric in r* <> z-, whichis forbidden
by angular momentum conservation and Bose statistics. we then have

h (lt"l) + fu;r,*)) : ,l-+,r,01 + n(10, oy

l"l"l) - fip,oy - uf1o, oy. (4.202)

Thus

(n* o- l'tl*lK}) - h(Q, 0l + thg, 0D11dK0)


: # (lz, oluyzlKol + Jie, ol'tlr pl 1d))
- hl{z,ol},lt +,-+) Ar/,.
+ O(o,o I 1,1, 1,-+l Arpl

- +, (|a'n - A,n)
: *Ar,r. - frArn. (4.203)

Or

T(Ko + r+r-) : #uArp - *At,r, (4.204)

and similarly,

T(Ko --, nonol : #At,, - ftArn. (4.205)


4.16 Isospins in non-leptonic weak processes 109

Remark. Experimentally, it has been observed that

T(K+ -- n+no) <T(Ko --> r+n-) (4.206)

and

r(Ko ---!.:) o.
r1<o.--'r1T\ --YL' @.207)

This can then be translated through eqns (??), (??), and (??) into the amplitude
relation of

Azp K AtP. (4'208)

A/ - | rule of non-leptonic weak decay.


This is the celebrated
(b) For the A --+ pir- and A -> nvo decays, we have the isospin structure of A
being an isosinglet, and

tpn-t :,1!l;. - il - lz,l+, -;)


t ,') - Jurll,-+) + rltl+,-+l- @.zos)

Thus

(pn- t77*tN -,lI G, -.lll1znt^) -,1 z t+, -+l'.ttpllt)


: fi^r,, _ JtrAr,r, (4.2t0)

(nn, 1't1,t r\) : 14 tl, - +lHz pt +,lt \t, - |lllt pt rr)


rv)

: {4^''' + J!A'P' (4.2t1)

where Ar/, :
(1r,-llllzplA) and Ar/z : l+,-llltplA). Again the exper-
imental data are in agreement with the A1 - | rule Azp K A1p prediction
of
I(A -+ pir-) : _Ji. (4.2r2)
Z(A + nno)
5 Chiral symmetry

5.1 Another derivation of Noether's current


Consider a system of scalar fields {@;}, i - 1,2,...,fl, with a Lagrangian
depending on Qi and 0uQi : L
- L(Qi, 0rQ).Under an infinitesimal space-time-
dependent (local) transformation on Qi:

Qi@) ---- Q',: Qr ]-6Qi : Qt + e(x)f,1E1 (5.1)

where e (x) is some infinitesimal space-time-dependent parameter and fi(Q) is a


function of the scalar fields {dr}.
(a) Showthatthecoefficientof}uein6LisjusttheNoether'scurrentasdisplayed
in Cl-eqn (5.15):

AL
jp'(x) : (s.2)
u@*Qrf,'
(b) Show that the coefficient of e in 6L is then the current divergence 0P juQ).

Solution to Problem 5.1


(a) Consider the variation of the Lagrangian:

6L: AL AL
(5'3)
uhud'* uUo6(oPQi)'
Because variation and differentiation commute, and because d@; : fi:
e

6(0*Qi) : Ep(6Qi)
- (oue) fi * e\u f; (s.4)

we have

6 L -, l# f, *
#h,u, r,) * E,e
l, *-^ tl (s.5)

Noether's current (for 3re:0) in Cl-eqn (5.15) is just the coefficient of the 0re
term in eqn (5.5).
(b) The divergence of Noether's current can be evaluated directly:

Dp j*(x) : op
#^r, -la' #")r, * hou r, (s 6)

Using the Euler-Lagrange equation of motion,

__a
AL
*
_-0.
AL
(5.7)
\rQ, 0(0uQi)
5.2 l-agrangian with second derivatives 111

we have

0P jr(xr: AL + ur'^\u.f,,
AL
(s.8)
Wft
which is just the coefficient of the e term of eqn (5.5).

5.2 Lagrangian with second derivatives


Considerthe case where aLagrangian depends on @; and}pfli as well as the second
derivatives:

L -- L(Qi, 3uQi, 0pE,Qi). (s.e)

(a) Derive the Euler-Lagrange equation of motion for this case.


(b) Derive Noether's current for a global transformation:

Qi@) --- Q', : Qi * 6Q,


- Qi + e fi141 (s.10)

where e is some infinitesimal space-time-independent parameter.


(c) Show that the current derived in (b) is the same as the coefficient of 8re in
dS : S f Lda x for a localtransformation. Hint: The higher orderderivative terms
0*3re can be reduced to \re upon integration-by-parts.

Solution to Problem 5.2


(a) The variation of the action

s - J[ rto,, orbi, oro,Q)dlx


being

:
I l#tr, * #^ou(sQ) . #^a,a,(60)) ",
6s oo (s.1 1)

where we have used the property that differentiation and variation commute,

6(0rQ) : 0p(3Q) and 6Qpa,Q) :0p3,(6flt), (5'12)

we can obtain the Euler-Lagrange equation of motion through integration-by-pats:

AL AL AL
(s.13)
w - a' a(ar"il + apa' uuuo) - o'

(b) For the global transformation, 6Qi : efi(Q), the variation of the Lagrangian
becomes

*
6L -,1#,r,* #ru,"o,>lurt #^ara,r,f. (s.r4)
112 Chiral symmetry 5.2

After using the equation of motion, and combining terms with the same number
of derivatives:

3L: rrr#^fi - elro,ffif,


AL AL
+' a4ro)oPJ'i *' uarao'13'3'f'
l-aL I I aL aL I
* uuruua,f,
- e\, Lr,r"o, ft l+ e\ul-3, urrilo)f, l' (s'ls)
If 4 is invariant under the global transformation, the conserved Noether current
can then be identified through 6L : eSrjP - 0:
AL
: a@Mfi AL AL
iu - o' urrrra)fi + (s.16)

Remark. After applying the equation of motion 6L


^uu*ri,fi'
can always be written as the
divergence of some 4-vector (which is the essence of Noether's theorem) because
the equation of motion follows from 65
- [ 6Ld4.r - 0 which comes about
because terms having the same number of derivatives combine into a total diver-
gence so that it vanishes upon integration-by-parts.
(c) Here we consider the local transformation
\Qi : e(x)ft(Q) and 0*(tQ) - (Ore)fi * e\rfi. (s.17)
In the variation of the action,

:
d's
I l#tr' * #^0,(6Qi' * #ffiaua,Gq;] dax

flaL aL
*
- J lra'f'+ uUu?Pefi
eouf''1

-u,#^(opefi + eauf)]dax, (s.18)

the very last term can be rewritten upon integration-by-parts


faL^faL : -J
-J a'
,arrn'ouf' dax au dax

: "ilr,are\'fi
I #^(D'eL'fi t eluouf;) da*

(5'19)
so that all terms are either proportional to e or ;re:
:
65
I {l#r' * #aro'ri -r #^aua'ri]el)
.l#^ri - o,#^r, * d#h,u. r,fau,lao, (s zo7

From this, we see that the coefficient of Ore is precisely Noether's current.
5.3 Conservation laws in a non-relativistic theory lI3
5.3 Conservation laws in a non-relativistic theory
Consider a non-relativistic system described by a Lagrangian L - L(qi, 4;) with
ei, i - l,2,...,n,being the generalized coordinates. Suppose L is invariant
under the infinitesimal transformation

ei ---> q'i : qr * ietilqt. (5.2I)

(a) Show that the quantity (Noether's charge) given by

Q - *tiiqi 6.22)
dqi

is conserved, dQldt :0.


(b) If the Lagrangian is given as L : Lr*o'- V(r), where v - drldr with
r _ (xt, xz,4) and r2 - xl + *7 + *?, show that L is invariant under the
infinitesimal rotation

xi -----> x'i: xi * iixi 6.23)


where ij : -e ji. Explicitly construct the conserved charges.
(c) For the case where L - l.*r', show that L is invariant under the spatial
translations

r---+r/:r*a 6.24)
where a is an arbitrary constant vector. Find the conserved charges.
(d) Consider a system of two particles interacting with each other through a poten-
tial which depends only on the relative coordinates V (r1 - r).Show that the total
momenta P : mtvt * m2v2 are conserved.

Solution to Problem 5.3


(a) The variation of the Lagrangian is given by
aLaq,
ut:
u'' +*dq,.
oQi
(s.zs)

Using 6q, (dldt)(Sq;) and the equation of motion


-
- ;fr - o'
AL dAL
6z6)
aq,

we get
daL
6L - at a4'6a'
* aLd
,,av.Jai)
: *,(#'r,) -*L*(ft,,'n,) (s27)

Thus the invariance of the Lagrangian 6L : 0 implies the charge conservation


dQldt 0, with the charge being g (0Ll1qi)tiiQi.
- -
114 Chiral symmetry 5.3

(b) We can see that

6L
-!*a"'-u'^"'ur:o
20r
(5.28)

follows from e;; - -e ji because

^ 0r ^ x; ^ I
dr
- *6xi : -6xi: -xiijxj -0 (5.29)

and

6v2 :2ii6ii - 2iieijii : 0. (5.30)

We can use the expression in eqn (5.27)

6L:."d (aL".\- !r,,!(aL, 3L \


-eija(rl'j)-rijn (ri'r- un,-, )
(5'31)

'
to find the conserved charge for this rotational symmetry,
AL AL
Qti :
Ai,*i U4*, - mi;xi - miix; - Pixi - P jxi (5.32)

which are just the familiar angular momenta.


(c) Because a is a constant, we have v/ v and thus 3L : lm6v2 0. To obtain
- -
the charge of this spatial translational symmetry, we note that, for an infinitesimally
small a, one has 6x; dj and thus
-
6L
- *(#',,) ",*(#) :",frr*nl (s33)

Consequently 6L mii, which are just the


- 0leads to the conserved charges
familiar linear momenta.
(d) The Lagrangian for this case is given by
l.l"
t: V(rr - rz).
,*rvi * Z^rri -
(5.34)

Clearly, L is invariant under the spatial translation of the form

ft ----+ f't : ft + a, lZ -----> f'Z: fZ + a. (5.35)

For infinitesimal translations, we have dr1 - 6rz


- a ?nd
d |/aL aL \
3L - * drri -f *.. Srzi I
-dr \ 3r1; \izi /
d
- ai dt(miu * m2i2i). (5.36)

Thus the total momentum

P : mrvr * ntzYz 637)


is conserved.
5.4 Symmetries of the linear o -model 1 15

5.4 Symmetries of the linear o-model


The Lagrangian for the linear o-model is given by

t.: Ll{aro' + @*o)2)+ NtyuarN


+ sN(o * iyrt.r)N . +(o' + o') - Xt* + o')' (s.38)

- (;) is an isospin-| nucleon field, r-(trt, rT2, n3) an isospin one


where N pion
field, and o an isospin zero scalar field. It is convenient to use a 2 x 2 matrix to
represent the spin 0 fields collectively:

E:o*ir.tr. (5.39)

(a) Show that the Lagrangian is invariant under the isospin transformations:

N ----+ N' : (JN, X ----+ I' : UEU|, (5.40)

where U exp( is an arbitrary 2x 2 unitary matrix with a : (ur, dz, o"3)


L" . t)
-
being a set of real constants. Find the corresponding conserved isospin vector
currents V;, i :7,2,3.
(b) Show that the Lagrangran is invariant under the axial isospin transformations

N ----+ N' :..0 (, +yr) lr, E ----+ E' : vrEvl, (s.41)

where V exp( iF . t)is an arbitrury 2x 2 unitary matrix with B (flr, fz, : fl)
- Find conserved axial-vector cur-
being a set of real constants. the corresponding
rents A;.
(c) Calculate the charge commutators

rei , ejl, le' , Qtj), fQ, , Qtjl, 6.42)

where

: o'.vd@) and Q'' : a3xeoa.


Q'
I I
(d) Calculate the commutators of particle fields with the vector charges:

LQi , N"f, LQ' ,,il, lQi ,o) (5.43)

and with the axial-vector charges:

jf ol.
Iesi , Nol, lQtt , o , [Qs' , 6.44)
116 Chiral symmetry 5.4

Solution to Problem 5.4


(a) It is useful to define the left-handed and right-handed chiral nucleon fields:

- )O - ys)N and : y)N.


)U *
N7 NR (5.4s)

Thus N - Nr * Nn, ysNr - -Nr, andy5Np- *Nn. Also,


xxt - (o ]-ir.tr)(o -it.n):02 +G.n)2 - (oz + tt21l, (5.46)

where we have used the Pauli matrix identity

(z'A)(z.B) : (A.B) -f it.(A x B). (5.47)

We can then write the Lagrangian as

t - ltr (ar>ap>t) + Nrtyp\,Nr * NpiyrarNp


+s (NrENn * NnEtN. ) * t,rrErt) - 161
ltr(>,lr)lr. (s.48)

For the isospin rotations, we have

Nr ----+ NL: (J NL, Nn ----+ Ni : U Nn, and X ----+ E' : gEUf .

(s.4e)

Thus,

tr(Et2/11
- tr((uEUr)(u>tut)) - rr(EEt) (5.50)

and, in the same way, tr(O*E'Apt't)


- tr(SutapXl).Also,
wLiyuIrNL NrUIiyuar(J NL
- - NriyrSrNr (5.51)

and, similarly, for the NpiyuSrN* term. Furthermore,

NLE'Nh
- Nruru>uruNn - NLxNn. 6.52)
Thus ,C is invariant under the isospin rotation. To get the conserved current, we
need to work out the infinitesimal transformations:

N ---+ N' -.*p (;


T)t - (t + t\) x (s.53)

or

6N _ ,yi*. (s.s4)

Also,

E----+ (r+;T)@*it.r) (r-,7)


E'-
- o*it-tttllT,r.of. (s.s5)
5.4 Symmetries of the linear o-model It7
From ft;, ri) - 2ieij&t we can work out the last commutator to be

lT' t' o): iij*arr. jrk: i(u x lt)' r' (s.s6)

Thus,

E' : o' +iT.it' - o +it .r - it.(u x r) (5.57)

6t:6, nt':\t-axfi, (s.s8)

Namely,

6o :0, 6n
- -q, x r. (5.59)

The conserved isospin vector current V, is simply the Noether current foq this
symmetry transformation :

aL dN + ut uo + 3L 6,
-a.Yp - A(asN) 3(0ro\ 0(3utt)
- -NvuTN - At'r'(ax r). (5.60)

Vp - tt1ulN _ At" r-xn. (s.61)


2

(b) The axial transformation of the nucleon field

N -+ N' :.^o (t
Tr,) t (s.62)

takes on a simple form when expressed in terms of its chiral components

NL:VIN, and Ni:VNn (5.63)

where v _ .*n (; q. It is then easy to see that both Nriyu\*Ny and,

NpiyplrNp ?re invariants. Similarly, both

trlE'E't1 - tr((Vt tyt)(ytty)) - rr(EET) (s.64)

and tr(}rDaptt) are also invariants. For the Yukawa couplings,

r,tL>'wh - Nrv(vt tvl)vNn - &>lr^. (s.6s)

Hence ,C is invariant under the axial isospin rotations. To get the conserved current,
we need to work out the infinitesimal transformations

N----+ N': *o (, T^) t = (t *t+n,) t (s 66)


118 Chiral symmetry 5.4

or the infinitesimally small change of the nucleon field under axial transformation
being

d.N
"2 : i'8.t vrN (s.67)

and

r ----+ E'- (t- ,+)@*ir n) (t- ,+)


- o * ir. tr-i(f . t)o .{+,, . rl+... . (s.68)

From {z;, t1l :26i7 we c&ri work out the last commutator to be

{+,' ol- p r (s.6e)

Thus,

E' : o' + ir. tt' = o i (F - r)+i(t. r)-io(F. i (s.70)

ot:o*0.r, rtl:rt-oB. (s.71)

Namely,

dsd - B. r, 6stt - -o B. (s.72)

The conserved isospin axial-vector current A, is simply the Noether current for
this symmetry transformation:

: *
-p. At,
h6sN +
#t" #t,
- '2 *vrN
-Nyu * [Po(F . w) - olp tr . B. (5.73)

Or

AtL - Nyryri|
, t-2 - (r|po - olpn). (5.74)

Remark. We can combine the transformations in (a) and (b) as follows:

Nn ----+ Ni : RNn (5.7s)


Nr ----+ NL: LNr (s.76)
E ----+ E' : LXR1 (s.77)
5 .4 Symmetries of the linear o -model 1 19

where we have introduced the right-handed and left-hand transformations

R
- exp (,T) and L - exp(r?) (5 78)

with y : d : a for the vector transformation, and y - -d - B for the axial


transformation. Under the infinitesimal transformation, R
= (1+ iff):
v.t
dnNn : iLt:N* and dnNr - 0 (5.79)

and

E'-(o*ir.it)(t-t ,+)
2/
- o +it.tt-iT"*T+iry.' (s.80)

or
v.fi ,Exy y
6no-;, 64o-;-;o. (5.811

From these field variations, we can immediately work out the corresponding con-
served current

Rr' - Nnvplx^ - !a'o -)ru'n x lt-oYptt)' (5'82)

Similarly for the left-handed transformation, L - (l + i6 . t):

61Na - Q and 6rNr : ,T *, (5.83)

and

E'-(r*tlj) 2) @iir'it)
\
=o+ ir . ,ti+"-ry + i" lr6 ., (s.84)

5ro - -+, 6rr: + * )o, (s.8s)

leading to the conserved current

LP' - Nly"rNr*, z.
rhevectorand
;T;;-'T,:-1, :" . -Lt" (s87)
120 Chiral symmetry 5.4

(c) Let us first consider the vector charges

Q':lot-Vo@). (s.88)

Because of current conservation, \PVl" : 0, the charges Q' must be time-


independent. We can then choose to work with equal-time commutators:

[Qi, Qi]*o:]o : at*at tl*'7* - it*llntn^, *'7* - ein*lon'rkf


I (s.8e)

which can be evaluated, see Problem 4.6, as

["t,", Z* r*r,nt(y) | x ot],0:y0 nr(") lZ,7lN(y)83(x -


y)

: iij* (*rZ") ,'(x - y) . (5.e0)

and

e i1a i,r,f}o nt 1*1n* (x) , 3o n" (y)no (y)1,0 : )0

: i1sink(-tt'o 30n'(y)r*(x) + i6^"00trt(*)oo0))63(x - y)

- -i (ei1r*inrr00n'r* - i1,nimkEUntrk)63(x - y)
- -i(00ni ni - 5ii 60onn" + 6ii a0Tt nrt - 30ni ni )63(x - y)

- -ierjr,1,1*(00rt)n-63(* - y). (5.91)

After substituting the results of eqns (5.90) and (5.91) into (5.89):

IQ', Qjl*':)0 : ir,1o a3* (wtlN - ep1*30rtn*) : ie,ioQk ' (5'g2)


I
In a similar manner, we can verify the other commutation relations of

[Q', Qti] - ir,joQto and LQt', Qti) - ir,ioQk. (5.93)

Remark. If we define the right-handed and left-handed charges as

Q'n: Q' + Qti , Q'r: Q' - Qti , (5.94)

the algebra becomes

ln'r, oLf --
i',ir,Qkr, lQ'*, OLf - i',it Qk^ (s'es)

and

lQ',, oL] - o. (s.e6)

Namely, each set of {Q'rl and {Qi} separately form an SU(2) algebra. This is why
it is referred to as the SU (2)L x SU(2)n algebra.
5.4 Symmetries of the linear o-model t2l
(d) Here we calculate the commutator of the various fields with the isospin charge

:
e'
I
ot- (,,,rtf"l
,*U, - iikaliri fxlnkfx>) . (s.e7)

For the isodoublet nucleon field,

tei, No(y)r
- I or.[r,,r,r (i)" N,(.r), N,o)] , (s.e8)

we can use the identity

lAB,Cl: A{B,Cl - {A,ClB (s.ee)

to get

lQ' .No (y)l : - (t)"' N'(y). (s.100)

For the isotriplet pion field,

Ie' , o' (y)l - -eiir, I a3*faoni @)rk(*),ot(y)l

: -eiit, ot.(- rdit)ro(")dr(x - y ) (s.101)


I
ieitknk 7y1. (s.102)

For the singlet o-field,

[Q',o(y)]-0. (s.103)

By comparing these results with the eqns (5.54) and (5.58) obtained in Part (a), we
note that these charge-field commutators just yield the variation of the field under
the isospin transformation with some parameter ai:

[cY . Q, Q@)l - i6Q@), (5.104)

where Q(x) - N(x), iti(x), or o(x). Similarly, for the axial charge, we have
lP'Q, Q@)l - i6sQ@):

tes, ,N,(y)r : - (;) ysN, (y), (5.10s)

lQt' , nt (y)l
- -i 6ii o (y\, (s.106)

IQt',o(y)l -ini(y). (s.107)


122 Chiral symmetry 5.5

5.5 Spontaneous symmetry breaking in the o-model


In Problem 5.4 the effective potential for the scalar fields has the form of

tlY --{r",
- -4ro'
2r"
+t totrz) *Xr", + or)r.
ttz\I + I (s.ro8)

For the case of p2 > 0, the minimum of this potential is at [see Cl-eqn (5.168)],

o2 + ttz : u2, ,: (+)''' (s.roe)

In the text, we chose the vacuum configuration to be

(otl - (o') - (o3)


- o, (o)
- u. (5.llo)
Now consider the alternative configuration of

(ot) - (o') -(o) - o, (o') - r. (5.111)

(a) Show that the charges which do not annihilate the vacuum are Qt , Q2, and
Qs3, and the Goldstone bosons are rr , fr2, and o fields.
(b) Show that the remaining charges, Q5t , Qt', and Q3, form an SU(2) algebra.
(c) Show that the fermion bilinear L^ :
guV tysr3 N generated by (o3) u can :
be transformed into the standard fermion mass term of L* :
m N N'N' by some
chiral rotation. Find this transformation.

Solution to Problem 5.5


(a) Given that (zr3) t' 0, we seek charge-field commutators eqns (5.100)-(5.107)
which are proportional to the zr3 field:

[Qr , o2l : -fQz,nrl - IQs', of : ir3 . (5.112)

We see that the charges Qt , Q2, and Qs3 do not annihilate the vacuum [otherwise
the above equation would imply that (n3) - 01, and nr , T2, and o are Goldstone
boson fields.
(b) From the charge commutators calculated in Problem 4 we have

[est, et'] - iet, -re3, ettl - iet', let', e'l - iett. (5.113)

This means that the charges Qsr , Q52, and Q3 form an SU(2) algebra.
(c) To find the chiral rotation it is useful to decompose the fermion field into its
chiral components, N - Ny * Nn. In this way we have

L*: guNiysr3N : gulNrir3No - Notr3Wr]. (5.114)

Consider the chiral transformation

Nr : LNL, NR : RNi, (5.115)


5.6 PCAC in the o-model 123

where L and R are unitary transformations. To turn

L^ : gulN'LrI it3 RN'^ - N; nt it3 tl'r] (s.116)

into the standard fermion mass term of - (wLU| + NiNi), we require

Ltit3R--1, Rtit3L:1, (5.117)

which is actually one condition as (fIitznlI


- -Rtir3L : -1.
solution to this condition, hence the required chiral rotation, is
One simple

L : --ir3-exp ( -,rl), R : 1, (5.1 18)


\ 2 /'
where we have used the identity of

""0 (,, +) : "o,l* ir,i'- (5.11e)

Remark. In spontaneous symmetry breaking, the choice of the vacuum expecta-


tion value (VEV) direction is a matter of convention. All different choices yield-
ing the same symmetry-breaking pattern are physically equivalent. In the example
under discussion, both choices of (n3) I 0 and (o) * 0 give the same symme-
try breaking, SU(2) x SU(2) -+ SU(2) or equivalently SO(4) --+ SO(3), and have
exactly the same physical content.

5.6 PCAC in the o-model


Suppose we introduce a symmetry-breaking term into the o-model Lagrangian

Lsn : -co(x) (s.120)

wherecisaconstant.

(a) Find the new minimum for the effective potential,

tr2 1 . ), a
v - -7f"'+ n2) +
)toz +
1.
r2)2 t co. (s.t2t)

(b) Show that in this case, pions are no longer massless and, in the tree level, their
masses are proportional to the constant c.
(c) Show that the axial-vector current A, derived in Problem 5.4 is no longer
conserved. Calculate the divergence 8pA, and show that it is proportional to the
pion field rr.
124 Chiral symmetry 5.6

Solution to Problem 5.6


(a) We have the minimization conditions

av.: [-p' * )"(o2 + tr2)]o * c o (s.t22)


; -
and

av.: l-t' * ).(o2 + n2y1ni - o. (5.t23)


#
Since it is not possible to have l- p2 * \.(o2 + r2)l
- 0 for a non-vanishing c,
we must have ni 0 and the o-field satisfying the cubic equation:
-
-ptzo*).o3*c-0. (s.r24)

Remark. In this case the vacuum configuration is unique because the symmetry-
breaking term of eqn (5.120) has singled out a direction.
(b) To discover the physical content of the model, we shift the fields

vt:v, ot:o-u, (s.12s)

where u is the solution to the cubic equation -Llu * )"u3 * c :0. The terms in
the effective potential become

o2 + tt2 : o'2 + tt'2 +2uo' + u2 (5.126)


(o' + ,t2)2
- 4u2o'2 +2u21o'2 + r'2) * non-quadratic terms.

We then have the mass terms in the effective potential

v2 - -{ t"" + o'') +}l+r'"'2 +2u2(o'2 + n'\f

(+" - +)o'2 + (I* - +)


" (s.rzi)
- (u' - X) o'2 - *,''
where to reach the last line we have used the cubic equation for u. Thus

:zPt' - 3c
mto (s.128)
i
)c
m; : -;. (s.rze)

If we pick c < 0, then both m2" and m2, are positive. That all three components
of n have equal mass means that the explicit breaking Lsn :
-co (x) still leaves
isospin SU(2) symmetry unbroken.
5.6 PCAC in the o-model t25

(c) The divergence of the axial current is related to the variation of the Lagrangian
as [seeCl-eqn (5.14) or Problem 5.1]

F.3'Ar:ysL. (s.130)

The right-hand side would vanish were it not for the presence of the symmetry-
breaking term Lsp : -co("r). Thus

65L:35LsB - ffi&" : --cF. r, (s.131)

where we have used a result obtained in Problem 5.4, eqn (5.72).In this way we
find

0'A, : -cIt. (s.r32)

Remark 1. The constant c can be related to the pion mass and the pion decay
constant, m, and fo.For the r -+ &updecay, the amplitude is proportional to the
axial current matrix element, which defines f" by

(olAi,e)lnb (p)l fo pp. (5.133)


- i6ob

Thus the matrix element of the divergence is given by

Plap ALQ)Vrb (p)) - 6"b fo*? : -c(01n" (O)lttb (p)). (s.134)

Or

(s.13s)

In this way, the divergence has ,^::;^:*^


a* Ai, (s.136)
- fom2on" .

The specific value of the pion decay constant is fixed as follows. The amplitude
for the nr --> l.t+vp decay can be written as
Go
r- (D)tty*(r - y)u (s.137)
72(olA;(o)lz+
with
(olA;(o)lz+(p)) - iJ2f, p*. (s.138)

With this definition, one finds from the decay rate that [see Problem 11.3(c) for
the calculationl
0.93
f, - -mn:92MeV.
(s.13e)
\/2
Remark 2. Comparing eqns (5.I29) and (5. I 35) we see that the VEV of the o -fi eld
is simply the pion decay constant:

u: fo. (s.140)
r26 Chiral symmetry 5.7

5.7 Non-linear o -model I

In the o-model of Problem S.4,thecombination E Et : 02 + n2 isinvariant under


the SU(2) x SU(2) transformation. The non-linear o-model is obtained from the
linear o-model by imposing the constraint of

o2 + ,' : f' ,f : constant. (s.141)

We can solve for o as in

(f' - n2)t/2 (s.142)

which is interpreted as a power series

/ ltr2 ltra
o:f(t-rr-87+ )
. (s.143)

(a) Show that the linear o-model Lagrangian eqn ( s.38 ), after eliminating the
o-field through eqn (5.I42), is of the form

,._t I
: l- " + p:AQr ' 0p")' ^l -
NiyP 0,N
,l(aun)' )+
+ sN
I,n' -e * iyrr."f N. F.144)

(b) Calculate the scattering amplitudes, in the tree approximation, for the
reactions:
(i) N'(pr) + Nu(pr) + N'(pz) + Nd(p+),
(ii) zt(ftr) + No (p) --, tti (k) + Nb(pr).

Solution to Problem 5.7


(a) We have the basic relation

(s.145)

To obtain an expression for u ro,


*" rlr**"^'rlJ*"ntiating o2 + tr2 - fzto obtain
o\uo : -n .EpT, which can be written as

11
duo--lrrjrtt--
n * (f2-fi2)tl2
n.'drtt. (s.146)
o
Substituting eqns (5.145) and (5.146) into the Lagrangian for the linear o-model

L -:l{arr^' + (sro)'l+ Niywa*N


2t

+ sN@ * iyrt.n)N . +@2


+ r\ - jo' + n2)2 (s.t47)
5.7 Non-linear o-model I 127

we get

' (n'0 "r\2-l


L-!f
,lru,r)z + ffi.1 + rvi Y' 3u N

- +sNI(f2-r2)t/2*iytt.nlN+... (5.148)
1
+ gY* a* - mt)N
- 1@ur,)2
w

+sN
C:+.ivst ") " .o+!+ (s r4e)

(b) (i) N'(p) * Nb(p) --> N'(pz) + Nd1po\.

Pt P3
ca

b#d b
p2 p4 p2

Ftc. 5.1. NN scatterings with pion exchanges in the t- and u-channels, respectively.

In the tree diagrams for these two processes, the basic pion nucleon vertex is
the same as in the linear o-model. For the first diagram with a pion exchanged in
the r-channel:

I
T1
- g2fu@)iysGk),ou(pr)lt -mi"lu(pq)iysQk)auu(pz)l (5.150)

where t - (pr - p)2. Use the identity of Cl-eqn (4.134),

l{ro),"(ro)ou -z(a,ud,o -)u,"uuo), (s.1s1)

to reduce the above amplitude to

Tr
- 2s2 (u,uu,, - )u,"uuo) oror,rru(p) t _ mr,u(p)iy5u(p).
(s.rs2)

For the second diagram with the exchanged pion in the u channel, we have

Tz - -2s' (uoou,, - |uo"uu,) rruortysu(pt) , _ rrr,,@)iy5u(p2)


(5.1s3)

where u - (pr - pq)2 and the extra minus sign in front is required by Fermi
statistics.
t28 Chiral symmetry 5.8

tJ ij
\- --.--
/' \/'\.
-a' t\,'
-)'-.-. \/,/

(a) (b) (c)

Flc. 5.2. Tree diagrams for zN scatterings.

(ii) z'(,tr) + N"(p) --> ri (k) * Nu(pr).


Here we have three tree diagrams

Mr - Doror>,sys(rb) ir,Vrrt _ __
(r")rii gysu(p)

I
- u(p)igysPt+# i gysu(p)(tb ro) j,, (5.1s4)
- *#r-mn
Mz-u(p)iyys, igysu(p)(r'rb)j,, (s.1ss)
" -
Pt-lfz-mt't
Mt - u(pz)u1p;6"b61i. (s.1s6)

5.8 Non-linear o -model ll


The constraint (5.141) can also be satisfied by parametrizations other than
eqn (5.142) resulting in different versions of the non-linear o-model, to be studied
in this and the next problem.
(a) Show that the constraint o2 + ,r2 : f' can also be satisfied by the
parametrization

E-o*it.tt:fexp(r+) (s.1s7)

where 0 : (Qr, Qz, Q) are arbitrary functions.


(b) Show that the Lagrangian in this representation is of the form
f)
L - !:rr
4
(o*ta'it) + NiyrapN + sf (Nrilro * h.c.) (5.158)

with

i_exp (,+) (5.15e)

being the same as E of eqn (5.157) exceptforthe overall factor of f , and 4 is


invariant under the transformations
i -+ i'- LiRt, Nr -> NL: LNr, Nn -+ Ni: RNn. (5.160)

(c) Calculate the scattering amplitudes for the same reactions as those in
Problem 5.7(b).
5.8 Non-linear o-model II r29

Solution to Problem 5.8


(a) From

E:o tit.tt:f..0(,+) (s. r61)

2t- f exp(-,+) :" -it.n, (s.162)

we get

EtE - o2 +,t2: f'"*p(-,+) *n (,+) - f2 (s.163)

(b) Since

(o,o)2 * (o*tt)2 : )r, OrEaPtt) (s.164)

and

lVto + iyst.nlN - NrtN* * NnXNr (s.165)

the Lagrangian can be written, using E - /i, as

f)
Lz: Tr,
+
(auDaui') + Niyrat'N + sf (Nrllro * h.c.). (5.166)

If we write Niyr}uN - Nilyr\PNr + Npiyu}PNn,it is easy to see that L2is


invariant under the transformations

i -- LiRt, Nr --> LNr, Nn -+ RNn. (s.167)

(c) Expanding i in powers of @

f2 l"
47,
(Artast')- iQub)' +...
(s.168)

l-- / it'6
sf(Ntilro *h.t'.): sf * -f #+ *r,)
Lt, \, )"^
- mNNN + SN(iysr. 0N
--g-N@')n +... . (s.16e)

Comparing with the interaction given in Problem 5.7, rt is clear that the @ fields
play the same role as the r fields as far as their couplings to N("r) are concerned.
Hence we recover the same scattering amplitudes as calculated in Problem 5.7.
130 Chiral symmetry 5.9

Remark. Problems 5.7 and 5.8 are equivalent ways to realize the chiral SU(2) x
SU(2) symmetry without the scalar field o(x). For example, in Problem 5.8 we
have, under the axial transformations, L - Rt - exp ("!)

i-+i,- *r(":) *o (-f)*r(":) :.*o (+)


For the case I
pl < 1, we can write

(' *'+) *, (tf) (' .'+) - exp (+)


_1+++
We can write the left-hand side as

*,(lf).i1, o,^,(tf) l: '*'+ *. it'zF'+"' .

We then see that

o' : o* tP+... . (s. r70)

Clearly, the relation between $ and @' is quite complicated and is, in general,
non-linbar. Thus the theories discussed in Problems 5.7 and 5.8 are referred to
as non-linear rcalizations of the chiral symmetry. In Problem 5.9 we will study
another non-linear representation.

5.9 Non-linear o-model lll


Suppose we redefine the fermion field of Problems 5.7 and 5.8 by a local axial
transformation

Nr --+ NL : LNr, Nn --+ Ni : RNn, (s.171)

with

L - Rt - exp (-,#) = Ut sothar ' :i. (s.r72)

(a) Show that the Lagrangian can be written as

1,3
"^-,f)
: Tr, (arfaui') + NLiyu [a, + rErlN',
r lri; yrll, + arrflri + sf (Nr'N'*+ Filri; . 6.173)
(b) Calculate the scattering amplitudes, in the tree approximation, for the
reactions:
(i) N'(pr) + Nu(pr) + N'(pt) + Nd1oo7,
(ii) zt(ftr) + N'(p) - ni (k) + Nb(p).
5.9 Non-linear o-model III 131

Solution to Problem 5.9


(a) From

NR : RtNi - 6tN; , Nr - Lr NL
- NL, (5.174)

we get

NriN^ * h.c.
- NLr iftlri * h.c.
- NLN'I * h.c. (5.175)

where we have used

qtirt-exp (-'+) *o (,+).^n(-,#):' et76)

From

o*N^- (ar6t) Ni + I E*Nh, orN, - @t"ilNL + a*NL, 6.17i)


the Lagrangian (5.158) then becomes

f)
f - r -Tr (lutar,t') lmuN'N'
"r-n
+ tttriy*lo, + rarf NL + Nhiy, [4, + 6ar6r] N;. (s.178)
Remark. In this Lagrangian, the coupling of the Goldstone boson @ to the N
fermion always contains a derivative,

Io,: (' -,++ ),t:f Gns)

and

Lt : !r, @riartt) + N'(iyt"at, - mu)N'


+, ,,
N'u,,u. ( 'uu0\ ,,*'
rtJ,) ' 2f
\ )
aN'yug#F@+... (s.r8o)

(b) (i) N'(pr) + N/ (p) + No (pt) + Nr(pa). The matrix element for the first
diagram in Fig. 5.1 is given by

7i - lu (p)y,ys Go) *i u (p tU'S;#


x lu ( p q) y, y s e
o
) r i u (n r>l'9i! (:
)
132 Chiral symmetry 5.9

Using the Dirac equation we have

[u (pz) y r"ysu (p t)]'9;/ : tu @)i ysu(pr )l


+
: -glu(p)iysu(pr)1.
Similarly

Iu(pq)y,ysu(p)l'95/ - -s[u(p+)iysu(p)]. (s.181)

Thus we see that this is the same scattering amplitude as obtained in Problem 5.7
(hence also Problem 5.8). Clearly, this is also true for the other diagram for the
NN scattering.
(ii) n"(kr) + N' (pt) -+ rb (k)+ N/ (p2). The matrix element for the diagram
in Fig. 5.2(a) is

: u(p) (n ii.
-_ (n
M'1 u(p)(rbro) (s.182)
il*-; _

Write

#zys:t-ys@z-f#z - mri - @z- mN)ys -2myysl. (5.183)

Then

M'1 : ut#rrn,>l-vrtt r - NvsTrtt ,,fvr,rn,t. (5.r84)


2m
- -rt
Also using

# rys:lUr +#t - mx)ys + y5Ur - mu) *2m1sy5l (5.185)

for the second term, we get

M'1 : t#lorr,r(# r - 2m y)u(p)

- u(p)(zmxy)V,Tt (s.186)
- --(2mtyys>r@)|.
The second term is seen to be the same as the amplitude Ml obtained in Problem 5.7
after using the relatiort rlt x gf . Similarly, for the diagram in Fig. 5.2(b)
-
M'' : tr#f - ao'>Ql z + 2m
^)u(P)

- u(p)(zm r,r y),--:- (2m x Y)u1prSl . (5.187)


Fz-$r-mx J
5.10 SSB by two scalars in the vector representation r33

For the seagull graph in Fig. 5.2(c) we have

M\: r) + (torb)ii#z]u@t). (s.188)


fioroz)l{rbt")iiGtl
The first terms in M\ and M',r, combine with M\ to give

Mr: ;rtlrorrrrl{rur'),,(tl r -2mx) - (torb)iiQlz*2m1,1)


* (rb r") 1i?k ) + G" rb)i,tl ,)"<pSl
I
: *f tb)4u(p)
QJuo(p)(-2mN)(rbf
-jn(pr)6ou6,iu(pt). (5.189)
- J

This is precisely the amplitude Mz given in Problem 5.7.


We have demonstrated in Problems 5.7, 5.8, and 5.9 that these different ways to
define the pion field all give the same on-shell S-matrix elements. The differences
are in the off-shell behaviour. For example, in the realization of Problem 5.9,
the pions couple to N fields through derivative coupling and will vanish in the
soft pion limit (ki, -+ 0). Since off-shell matrix elements are not measurable
quantities, all these different realizations are physically equivalent. However, if
one approximates some measurable quantities by some off-shell matrix element,
then the difference in these realizations become significant. Which of these is the
best approximation can only be decided by experiment and clearly will depend on
the physical quantities of interest. For example, therealization given in Problem 5.9
seems to work quite well in the low-energy processes involving slow pions.

5.10 SSB by two scalars in the vector representation


(a) Show that a set of scalar fields rf which transform as a vector representation
in an O(n ) group can break the symmetry from O(n ) -+ O(n - 1).
(b) Show that for the case with two vectors in an O(n) model, the spontaneous
symmetry breaking (SSB) is at most

O(n) + O(n -2). (s.190)

Solution to Problem 5.10


(a) The @(x) fields belonging to a vector representation in O(n ) means that under
O(n ) rotations we have

Qi --> Q'i : Riitr' with RRr - RrR : 1. (5.191)


134 Chiral symmetry 5.10

Ri1 ure matrix elements that are real. The scalar product 0 . 0 is invariant under
o(n),
Q'iQ'i: RiiRirQiQo - 61*QjQr,: QiQi- 6-192)
The effective potential V (0) which is O(n) invariant can depend only on 0 . 0.
For example,
u2x
v(il - -;@.il ;@. 0)2.
+ (s.1e3)

In other words, V depends only on the magnitude @ : l0l of the O(n ) vector,
v (0
- v (0). (s.te4)
This means that the minimum of V (0) depends only on @,

0, : 0, : O? + 03 +... + Ql : rr. (s.re5)


We can then choose the vector @ to be in an arbitrary direction. For example, the
choice

d : (0,0, ..., u) (5.1e6)

will have the property that it is invariant under the rotation among then-l
coordinates Qr, Qz, . . ., Qn-1,

R',iQi : Qi, 4:(f ?), (s.t97)

with R, (n - l) x (n - 1) orthogonal matrix. Thus the symmetry breaking is of


the form

O(n)+O(n-1). (5.1e8)

Remark. In this case, the pattern of the symmetry breaking does not depend on
the fact that V (0) is a fourth-order polynomial in Q.
(b) It is easy to see that the O(n) invariant effective potential V can depend only
on the magnitudes of the vectors Q?
- h. :
0r, Q?. 0z- Qz,and the scalarproduct
:
of the two vectors 0r . 52, which can also be written as dr . 0z Qy$2 cos 0. The
effective potential V can then depend on three variables, Qr, Qz, and cos 0,
V V (Qr, Qz, cos 0). (s.1ee)
-
The minimization of V determines the values of these three variables, rp1 _
ut,Qz : u2, cos0 _ coscv. Clearly, these three variables define a plane, which
can be taken to be the (Q"_1, Q) plane. Two vectors Qt and Q2 can have non-zero
entries in the last two components. For example, one simple choice is
(s.200)
These configurations have the property that they are invariant under the rotations
of the first (n - 2) components. The pattern of the symmetry breakings is then
O(n ) -+ O(n - 2). (s.201)
Note that it is possible that as a result of minimization, we have a : 0 as the
solution. (This can happen if V depends on the even powers of cos 0 and the
5.10 SSB by two scalars in the vector representation r35

coefficient of the cos2 g term is negative.) This means that two vectors are parallel
and the plane degenerates into a line. The symmetry breaking is then O(n) -->
o(n - 1).
Remark. For the case of ft vectors in O(n ), the symmetry breaking is

O(n)+O(n-k). (s.202)

The generalization to unitary groups is straightforward and the result is that for
the case of k complex vectors in SU(n ) the symmetry breaking is

SU(n) + SU(n -k), k <n. (s.203)


6 Renormali zation and symmetry

6.1 Path-integral derivation of axial anomaly


For the fermions, the generating functional can be written as a path integral of the
form (see Fujikawa 1979)

Z[n,it:
Iro,nw,h]expl, I W+it *o,ff (6.1)

For simplicity, we will take the Lagrangian to have the form t


- ,[iprl with
Dr" : 3* - igA, being the covariant derivative and Arthe U(1) gauge field. One
way to define the integration measure of the path integral is to expand $ and $ in
terms of a complete set of orthonormal functions, @, (x),

,lr(x)
-I anQn, ,[f*> -I QI@)a,, (6.2)

where

I rr@)Q*@)dax :6,^ (6.3)

and define

lavllavl- fI da,flda*. (6.4)

(a) Compute the Jacobian for the u*iut ,iunrroniu,ron

{r --> ,lr' : ,io" rlr. (6.s)

Show that for an infinitesimal a,the Jacobian is of the form

J-I*iurr(D) where rr(D)-D Io^.(Oi,yrO,). (6.6)


(b) TrDis quite singular. If we take Qn(x) to * *. plane wave Q,(x):
u(p, s)e-iP' , we get

TrD
- J[ ao* eiP* ui (p, s)ysu(p, r\r-i'r
E4(O)ar (p, s)y5u(p, s) (6.7)
-
which is not well defined because A41O; --> oo, while uI (p, s)ysu(p, s) + 0. It
has been suggested by Fujikawa (1979) that we can regulate Tr(D) by Gaussian
6.1 Path-integral derivation of axial anomaly 137

cutoff

rr(D): ;TL
+I
ao* (orr,*o (- #) r.) (6 8)

where ,l.n is the eigenvalue of the operator i p,

iEx": LnX,. (6.9)

Calculate Tr(D) in the limit M -+ oo.


(c) Calculate the divergence of axial vector current A* as generated by the axial
transformation (i.e. the anomaly equation).

Solution to Problem 6.1


(a) Expandthetransformed fieldlr' -- siuvt ,[,inacompletesetofbasisfunptions,

,lr' :DU,O,@). (6.10)

The coefficients of expansion can be projected out by using the orthogonality


relation

b, : o^. Qi,@){'(x) : dax Qi@)rivsu {(x)


I I
: aaxQl(x)eiv'oDo^Q^(x): DC,*o* (6.11)
I
where

cn* : o^- Ql@)r'r,o Q^@). (6.12)


I
Similarly,

,h'
-Dl"f:,r*r, 6,:Dcn*d^. (6.13)

Thus the Jacobian of the transformation (an, a) + (br, En\ is

1 - (detc)2. (6.14)

For infinitesimal cv, we have

c,^x6,**tu (6.15)
IaaxQI@)vsQ*@)
or in matrix form

c x r -f iaD with D,* : o^, Qi,@)ysQ*@). (6.16)


I
138 Renormalization and symmetry 6.1

Thus we get for the determinant:

detC ry det(l -f iaD) = I * iaTrD x exp(iuTrD) (6.17)

where

rrD -+
I aaxQI@)ysQ,@), (6.18)

and we used the identity det (eA) - ,rrA. Thus we can write the Jacobian as an
exponential:

7 - (det c)2 * r\iurrD - exp lr,*|TJ/ dax Qlg)yr;,gll . (6.19)


I J

This means that the effect of an axial transformation can be included as an extra
term in the Lagrangian,

6Lo - 2"DQI@)ysQ,@). (6.20)

(b) Here we calculate the trace in eqn (6.18) with Gaussian regularization

rrD -+ | aaxQl@)ys,,.o ( #)Q,@)


(6.21)

where M is some regulator mass, and )., is the eigenvalue of the operator i p,

i Ex, : LrXn, Du - Ap - i8Ap. (6.22)

For the special case of 8 : 0, we have Ln : | , and

*o(-#)-exp (#) (6.23)

and the integral over ft is convergent. For the general case we choose Q"@) to be
the eigenfunctions of the operator ip and write TrD as

rr D : x *, (#) (6.24)
+I
aa QI@)ys Q,@)

Since the trace is invariant under the change of basis (unitary transformation), we
can now use the plane wave state

Q,@):s-ikx, and (t* I#) (6.2s)


6.1 Path-integral derivation of axial anomaly L39

to compute the trace. Simple algebra gives the result

D" \)
" -:?,i,?,.i,', ;;';:; l;;*:,':,: :,t.7'
: D'l qrou,Fu'
Lgr"{ot", - Tfy, y,f F*u -- D2 - (6.26)

where

Ft', - At, A, - A, Ap, ot", : llv*, r"1. G.27)

Also,

D2 : (au - isAr) @u - igAp) : D2 - 2igAp0u - igouAp - g2 At" Ap

p2 ,-ikx
- [- (k, + sAr)' - igl*Apfe-ir' (6 28)

Thus we have

/ o'\
.*p( + serfgauAuf
tut, 1 r-,0* - exp |
L -(k, W --trfe-'^'.-ikx (6.2e)

Putting all these together, we get

rrD-
l#lo^,,,(,,"-r(#)): Io^. I#
(r"+-'g!')'
'r,(r,*, [- - ]"-r"# - #u^'])
Changing the integration variable, k* - gA* : k'rM,

rrD -| ao*uo
I #,-k''r,(r, *n l-r",,r*' # - #r,^'])
It is clear that the last term in exponential, not containing any y-matrices, will not
contribute as Try5 0. We can expand the exponential
-

[- to,,r,, #f- #)
.-o exp
l-+y,yuFp,
- 1 - 'f rrr,r- J-
.:(T)' ,,r,rovpFu'r"u h+ "' . (6.30)
Only the first term and the M-a terms will survive as the M-2 term will vanish
after taking the trace, while the higher-order terms vanish in the limit M -+ oo.
I40 Renormalization and symmetry 6.2

Using the relation

Tr(YsYuY,y"yp) - 4itpuup (6.31)

we get

rrD- -+
I o^, I ffio,rruopFp'Fof
e-k'. (6.32)

From

f d4k t2 i
: t6r' 6'33)
J (2vYe-^
we get

82 o"* tt"oflPttv PUP


- ;n J[ao*,
TrD- 1fr.34)

(c) Thus the effective term in the Lagrangian is of the form

6L - r**r1,,oBFp'FoB. (6.35)

Since the divergence of the axial vector current is just the coefficient of cv(x) in
6C under the axial transformation, we see that the Jacobian here will contribute to
O*AP as

lt' ,Al'
A,, : f' =r,,,,n6FP'
Fvat FoP . (6.36)
1612"

Or, if we define
-I
Fr, : (6.37)
)trroBF"P,
this can be written as

02
orA* : (6.3g)
#Fp'Fpu,
which is just the axial anomaly equation.

6.2 Axial anomaly and q + yy


The decay q --> yy is very similar to n0 --+ yy. Suppose that the process also
proceeds, like the case for 20, through the axial anomaly. Parametnze the matrix
elements for the decays, as in Cl-eqns (6.61) and (6.63),

Alptq> + y (kr, er) y &2, e)] - el (k) ei (k2) iep,oBt<ikfr p (q'z) (6.39)

where P stands for either of the pseudoscalar mesons 4 or 20.


6.2 Axial anomaly and 11 --> yy I4l
(a) If we assume 4 is a pure octet, q 08, show that
-
&9
f4(o) -
Jj (6.40)

from the theory of anomaly.


(b) Show that the ratio of decay rates is given by

rJ"o --, vy,) :- (yr\'l\ \*?)l' 6 4t)


r@ -- yy\ \^, ) lrr@il )
Assume that

r" (*7) ro(o)


= t](o) '
,, (*r; ^,
(6'42)

compute the decay ratio and compare it with the experimental results.

Solution to Problem6.2
(a) From Cl-eqns (6.69) and (6.72), we see that

02 p2
f,(0) : (q2x) and f4(0) -- ;1rr (QzLt).
;1rr
Using

n::( -'1 ^':(' -1 ,) rs:+(' '


-,) -,)
we get

r" (o): rr (492)


.'t 7fr6: "/3'
E (6'43)

(b) The amplitude is proportional to f, t and the decay rate is proportional to


62.fnis means that we need m3, in the decay rates to get the right dimension,

t(P --> yy) o m3rt p (^?). (6.44)

Then we have

rJ:, _- yv,) :
r@ -+ yy) - (yr\' l\ \*?)l'
\^, ) lrr@'r) l
(6 45)

If we assume
r
"
(*,") ^- f,, (o) 6 (6'46)
T@ ffi:'/3'
142 Renormalization and symmetry 6.3

we get

t (no --> vv) _ (yr\'


3 _ 0.045. (6.47)
r@ --> yy) \m, / "
Experimentally, this ratio is about 0.0165. The discrepancy is probably due to the
assumption (6.42). As m2, x 0.02GeV2 which is quite close to 0, the approx-
imation f"(*7) - fo(O) should be fairly good, while m2, ry 0.3GeV2 and
r, (*'r) * ln(0) is probably not a reliable approximation. Another possibility is
that the r? meson does not transform as a pure member of the SU(3) octet.

6.3 Soft symmetry breaking and renormalizability


Consider the Lagrangian given by

L - :l{u,o,)' + (a,Q,)'f - t ft + 0,,) - Lr trl + o?)' (6.48)

(a) Show that L is invariant under the transformation


: cos?Q1 * sin?Q2
Qr --> 0\
Qz -+ 0L: - sin 0h -f cos0Q2. (6.4e)

Use this symmetry to construct all possible counterterms.


(b) Suppose we add a symmetry-breaking term of the form

Lsn:r(A?-A?) (6.s0)

Construct all possible counterterms and show that L * Lsa is still renormalizable.

Solution to Problem 6.3


(a) This transformation is simply a rotation in the (h, Q) plane, and it leaves the
combinationQ? + A7 invariant just like the ordinary rotation on the plane. The
superficial degree of divergence is given by

D-4-Bt-Bz (6.51)

with B1 and 82 the numbers of external Q1 and @2 lines. Note that owing to the
symmetry Qr
- -Qr, or Qz * -Qz, Br and 82 have to be even.
(i) Br --2, Bz : 0, or Br : 0, Bz : 2, implies that D- 2. We need the
symmetric counterterms of the form,

(auor)' + (auor)', (o? + 07) (6.s2)

(ii) Br-4, Bz:0,or81 :0, Bz:4,orBr:Bz:2 impliesthatD-0.


The counterterm which respects the symmetry is

(o? + o7)' (6.s3)


6.4 Calculation of the one-loop ffictive potential 143

(b) Lsr - , (Q? - 0i). The index of divergence is 6 : -2, and the superficial
degree of divergence is

Dsn:4-Br-82-2nsn (6.s4)

where n5s is the number of times L5p appears in the diagram. For diagrams with
ftsB :0, we need only the counterterms given in Part (a). For diagrams which
contain one symmetry-breaking vertex, the degree of divergence is improved by 2:

Dsn :2 - Br - 82. (6.s5)

Thus only the two point functions are divergent: \ _ 2, 82 - 0 or


Br : 0, Bz
- 2. The counterterms we need Ql and Ql. The combination
are
(O? + Q|) canbe absorbed in the mass term +p2 (0? + Ql) whllethe combination
(O? - Q|) can be absorbed in the symmetry-breaking term Lsu : , (A? - A3)
This implies that the theory with L * Lsn is still renormalizable.

Remark. This is an explicit example which illustrates the Symanzik theorem,


which states that if the symmetry breaking term has dimension d56 < 4, we only
need asymmetric counterterms with dimension < dsn.

6.4 Galculation of the one-loop effective potential


As given in Cl-eqn (6.121), the effective potential is of the form
1

v (O)- - t jrt,(o,
tt.
.. .,o)lQJ" (6.s6)
-

where ptn)(0, . . . , 0) is the lPI n-point Green's function in the momentum space
and Q, is the classical field. For simplicity of notation, replace Q,by O. At the tree
level we have

v - vo(Qr-tr,*hr^ (6.s7)

which gives

r[2)to) : -r12, r[o){0,..., ) - -),. (6.58)

Suppose we define a shifted field rf'by


6@) (6.se)
-0'G)+a
with a; an arbitrary constant, the Green's functions can then be expressed in terms
of Q' andfj')(0, . . ., 0).
(a) Show that the effective potential has the property
.av
=;6:, - -rS)(o)
v'(t';) , (6.60)

where fS)(O) is the one-point Green's function (the tadpole graph).


(b) Calculate f$) (0) at the tree level and integrate it to get V (a).
(c) Calculate ff; ) (0) in one-loop and integrate it to get V k'i.
I44 Renormalization and symmetry 6.4

Solution to Problem6.4
(a) The effective potential can be written in terms of the shifted field as

I
v (il - - ,.----\
I ;rr,r(0,
_ .. .,0)lol,
rl
.

: -;/-t 1.,,,(0,
,t
.. .,0)ro, +,f,
: -;Lnt 1rf'(0, .. .,0)lo')'
: -; 1rf,(0, ...,o)td - a)fn ( 6.61)
?nl
Thus it is clear that

(6.62)
#lr:,: -P(r)1s) -v'kD'
This means that we can calculate the tadpole diagram's one-point function ff )(O)
in the shifted field Q', and integrate f o)(0) over o to get the effective potential
v(il.
(b) Expanding the potential in terms of the shifted field

"--:,,:..[:.rrli.rI*h(Q'+'v.
(6.63)

we get

r#)(o)o
--(r,,*i,,) :-# (6.64)

Integrating this relation we can get

voki : # da : o,(u',* i,') - t,' * h,o


I I (6.6s)

or

u2^ L
vo@)
- T0'r (6.66)
^.Q".
This calculation, of course, is trivial. But it seryes to illustrate the relation between
p('r) un6 p(n).
6.4 Calculation of the one-loop ffictive potential 145

(c) From

vo@)
- vo(a) + (u', + f,r') O' + (+.+)Q'' +*r' *hao
(6.67)

we can calculate the tadpole graph

I
rf;r 1o; r - + (xa2l2)'
(6.68)

Note that ff)tOl is the 1PI one-point function and there is no propagator for the
external line. Integrating this, we get

vrkD : Irf;)(o),
dot: -+
|# I*
)"a da
(p' + (xa212)

--+l#mlr"-(u'.+). ir-lI + c (6.6e)

where C is independent oi ,.If we choose C such that Vk't) - 0 in the limit


), : 0, we have

if d4k.l-.
vr@)-tJ LQ2l2 I
e"ytn[t - F:An)
(6.70)

which agrees with the result given in Cl-eqn (6.139).

Remark. It is not hard to see that different choices of the ar-independent C coffes-
pond to different choices of counterterms and will not affect the properties of
V"5 (Q) once the parametersin V"y (Q) are fixed by the renormali zatronconditions.
7 The parton model and scaling

7 .1 The Gottfried sum rule


In the parton model, if we assume that the proton quark sea has the same number
of up and down quark pairs, i.e. in terms of the antiquark density u(x) a@).
show that
-

I,' +lr{r'> - Flt"ll - l (7.r)

Solution to ProblemT.l
From the parton model, we have for the proton structure function

rl@):*(t[u*ul+Ifa+d]+]ts+'1). e.z)
The neutron structure function can be obtained from the proton structure function
by the substitutions ,, <> d and u <> cl,

Fi@)-'(+[u+u]+]W+dl+]ts+'1) . e.3)
The proton and neutron difference is then

F{ (*) - F;(x) - ,lt@ * u) - \fa + all e.4)

Since the total isospin of proton is |, we have the sum rule

71

t - +
JoU, -u)+@ -illax. Q.s)

Combining eqns (7.4) and (7.5), we get

ft dx
F;(x)l : !
7t
J, TLF!@ - J,x, - + - htdx
d) (u

:++3Jrro-a)a*.
7l
e.6)

Thus if we assume u cl, the result is


-

I,' +l4ro - F;(')l - 1 (7'7)

which is known as the Gottfried sum rule.


7 .2 Calculation of OPE Wilson cofficients 147

Remark 1. This sum rule can also be obtained with a weaker assumption,

jro',.':,.:--';,:ii:1.,*^,*rromthesimpler,:::
Remark2.rheassumpriol
of the quark pairs in the sea being created by the flavour-independent gluons and
the up and down quarks having similar masses. However, for light mass quarks
in the long-distance range, perturbative quantum chromodynamics (QCD) is not
applicable. Since the proton is not an isotopic singlet, there is really no reason to
expect its quark sea to be symmetric with respect to the u andd quark distributions.

7 .2 Calculation of OPE Wilson coefficients


Consider the composite operato r J (x) - :Q2 (x): in )"Qa theory. Write the operator-
product expansion (OPE) as

rQ@) /(0)) - Cr@)or(0) + Cz(x)oz(O) + "' (7.9)

where Ci (x)s are c-number functions and O; (0)s are local operators.
(a) Write out the first three local operators, having the lowest dimensions, in terms
of @(0) and 3rQ(0) in this expansion.
(b) Define the Fourier transform by
f
I ao, eiq* T(J(x)/(0)) - ct@)or(o) + cz@)ozq)+ ... . (7.10)
J
Use the Feynman rule to calculate the matrix element

r (p, q) \r' Q @)J (o))lp)


- I[ ao* ,iq' (plr (7.11)

to order 1.0. Then take the limit q2 -> -oo to identify the coefficients
C r(q) , Cz(q) , Ct@).
(c) Draw Feynman diagrams which will contribute to Ci@) to order l'.

Solution to ProblemT.2
(a) Since /(x) is symmetric under Q --> -Q, we need to consider only operators
which are even in @:

dim operators
01 (7.r2)
2 :62101
4 :Q4(0):,:3,Q0uQ:, :Q0,3rQ:

T(J(x) /(0)) - Cr(x)l + CzQ):Q2(0): * Ct@):Q (0):


+cf" @):0,Q3rQ: * c{' (x):QE,luQ:. . . . (7.r3)
148 The parton model and scaling 7,2

From dimensional analysis, we see that

c{x) -
i, cz(i - ), ct,q@) - oQ). (7.r4)
(b) The first term in the OPE is a c-number and get its contribution from the
disconnected graph, as shown in Fig. 7.I(a), with contribution

r@(p,e):
I*r"+T.#_E (7.1s)

In coordinate space, this corresponds to

cr(x) - [i Ap(x)]2 where Ar(x) :


I #eft. n _[ a ie
.

For the connected graphs, there are two contributions to order ),0; they are displayed
in Fig. 7.1(b) and their matrix elements are

T0(p.q\:
\r'1' +'(p_q)2_ttz
(p*q)2-p2
i i
(7.16)
- q2 *2p.q
I

q'z -2p n
For q2large,

I I t [, 2p.q . /2p.q\2
: q'lt I
V+2p n: n442u 4n'1
' - n' * \ n' )
and

Tu(p,e):'-*Si(P'q)2 +.... (7.17)


q' q*
On the other hand, from the operator-product expansion,

I o^. eiq' T(J(x)/(0)) - cr(dr + cz(il:Qz(0): * e(e:Q4e):


+ cf' (q):0,90*Q: + c{' (fi:S\,\rQ:...
(7.18)

I ,&-*--8. t\
P.r' P+q P

aa,"-*-tt ./ la\
\.
,'t
tt-
a--.'
l+q ,&-*--8.
Pzq
P -r' '\ p
-------f-----
p ,/ ,1 \
\.

(a) (b)

FIc. 7.1. Feynman diagrams for the operator-product expansion of two currents.
7 .2 Calculation of OPE Wilson cofficients I49

The matrix elements between one-particle states are given by

I ao* eiq' (plr


\! (J (x)J (0))lp)
- cz(q)@l:Qz(o):lp)
J
+ C{q)@l:Q4 (O):lpl
+ cr" @)@l:a,QorQ:lp)
+ c{" (il@l:QouorQ:lp) . .. . (7.r9)
To order ).0, we have free field theory and Q@) canbe expanded as

Q@) - I @#fulo&),-ikx +atsl,^ire xf . (7.20)

Then the composite operator :d2(0): can be written as

:02(o):--
| @f*ff I @#;P
x la(k)a(k') + ot (/,ar (t ') + or 1k1a(k') + aI (t<')a(t<)). (7.21)
Using

lpl - l{zo)'zro11/2 ot 61p7 and [a(p), ar &)l - 331p - k)


we get

\pl:Q2(0):1il - 2. (7.22)
Similarly, a straightforward calculation gives

@l:Q4(o):lp) - 0 (7.23)
because each term in :@4(0): will have at least two destruction operators on the
right or two creation operators on the left. For the derivative of Q@), we have

Ir,Q@) : f d3k
s-ik ' + ar 1t<1rik x1 Q.24)
I p;;y(-ik*)ta(k)
and

e): :
:o uQ (o)o,Q
I @#ff I @"*!!;pGk pk,)
x la(k)a(k') + oI (k)oI (k') - aI (k)a(k') - aI 1k'1a1k11,
(1.2s)

:
:Qe)o,o,d(o):
I lr#pl@#*p?kuk.)
x [a(k)a(k') - or (k)ar (k') + ar (k)a(k') - aI (k')a(k)1.
(7.26)

From these we get


(pl:E,Q\rQ:lpl :2Ppp,, @l:Q0,0rQ:lp) - 0' (7 '27)
150 The parton model and scaling 7.2

Remark 1. The calculation of the matrix elements of the local operators is done in
free field theory for illustrative purpose (to show how this can be carried out). For
more general cases with interactions, these matrix elements are more complicated
than those in free field theory. However, from Lorentz invariance it is not hard to
see that the Lorentz structure of these matrix elements remains the same but the
coefficients will have more complicated dependence on the coupling constant ).,

ar, (7.28)
@l:Q2(0):lp) - - az,
@l:Q4(O):lpl

\pl:0"Q3 rf:l p) : a3 p p pv ; @l:Q0,0*Q:lpl : a4pppv (7.2e)

where at,az,a3,a4 are constants which depend on p2 : m2 and the coupling


constant ),. If the perturbation theory is applicable, we can expand these coefficients
in powers of coupling constants 1.,

oi : o[o) * ).alt) * A.2al2) + ... .


. (7.30)

Our simple calculation gives the first terms in this expansion,

o\o) :2, o5o) :0, o\o) :2, o[o) : o' (7.3r)


To this order, we can use these matrix elements to read out the c-number coefficients
from eqn (7.19),

Cz(q) : i..,L, Ct'@) : 4iqunv


(7.32)
q' q2

Note that because of) :0, we do not get any information on Cf'(4) from this
simple calculation. To get C{' (q) we need to use more complicated external states,
e.g. two particles in the initial and final states.

Remark 2. The basic idea of calculating the Wilson coefficients is to use the fact
that they are c-numbers and are process-independent. Thus we can choose some
simple external states to simplify the matrix elements of the local operators and
extract the Wilson coefficients.
(c)

/\/\/\
a-

/\/\/\
z\ /\

tlttlt
ll ll ll
tlllll
\/\/\/
tll, ll

-----&-----&'J-- ---&-'d--&---- -b'-&-----a----

/\ /\
6&
z^:

\/ tl
\/\/
tt
\/ /\.)o(.
-----)-{=j----.
------&----.&----
Ftc. 7.2. Feynman diagrams for one-loop contribution.
7.3 o,o,(e+e- --> hadrons) and short-distance physics 151

7.3 oy6(e+e- -> hadrons) and short-distance physics


Consider the process e* e- --> hadrons through one-photon annihilation.
(a) Show that the total hadronic cross-section (summing over hadronic final states)
can be written as

o1o1(e+e- -+ hadronl :-Y'& ao* e'q'' (}lUp(x), /r'(0)ll0)


[ O"* (7.33)
Z1nry, J
where Ju@) is the electromagnetic current and qp the four-momentum of the
intermediate photon.
(b) Suppose that Jr(x) is made of free quarks:

J*(x) -:Q@)YpQq@\ (7.34)


- I:4i@)eiY,Qi@):
i

where Q is the charge matrix and i is the flavour index, calculate the commutator
It*{x), /'(o)] and show that

4:'"'
o1o1(e+e- -+ hadrong - Tr(ez). ( 7.35)
Jq"

(c) Suppose that the current J *(x) is made out of free elementary scalar fields,

l*(x)-rtlOlO,,a*Qi- auO! e,i4if. e.36)

Calculate the commutator 7, -rU,,/r'(0)] ando1o1@+ e- -+ hadrons).

Solution to Problem 7.3


(a) The amplitude is given by

r : u(-k'11-ieyr)r6)9#(n l/,10) (7 .37)

and the cross-section is

do - + k, - pn)ry#"(; rzr')
N+),rroy*& ;
The spin sum of lZ12 is

Intr( : -!r,lrt'r,rtnl ltot l'tn)(nt*t') (;)' (7.38)

The leptonic tensor is

4lp, : rrl#'yr#y,l - 4(k'*k, + kpk!" - gp,k.k') (7.39)


152 The parton model and scaling 7.3

where we have used the approximation k2 : k'2 - m! - 0. The hadronic tensor


is given by

nru(e) : 44* (x), J,(o)llo)


| "ie'x(olt4,
f
: oo* eiq *l(ol/u lnl@lJ,loys-in,'
+ J
-(ol/,ln)(nlJplo)eip"''l (7.40)

As usual, the secondtermdoes notcontribute forthe case q0 > 0 (because pn > 0).
Then we can write

nr,(e): - p)(01/, ln)(nll,|O) (7.41)


T 4#Lrrro-12n1464(q
and the differential cross-section is

I 1r
I
do:
2E 2E'
Yrzr\a6a(o-
z-'' / \ ' e)
#"e4 t'u(ol /' ln) (nl /r' lo)
ry
|
: sllta) /:"
tp'rp,' (7.42)

From Lorentz invariance and current conservation, we can write the hadronic
tensor as

nrr(Q) - (q'8r, - qpq,)n(q2), which gives of : g&'Tp,


- 3qztr(qz).

Straightforward calculation yields

(q'gr, - epe)lpu q'(-2k'k') - (2k'qk' 'q - qzk'k') -qo.


- -
The total cross-section is then

lea
: gEE,F : 8n2u2
otot x q"n(q.) Tt (7.43)
,4

8n2a2 f
otot : oo* eiq'' (\lUu@), /r'(0)ll0). (7.44)
ffi J
(b) In Cl-eqn (7.146), the c-number singularity in T (Ju@)J"(0)) is given by

:
c,,(x)
ffi: {a cf:rF - iaur, [tr+] ]t' o'*
From the substitution rule for going from T-product to the commutator

->2nie(xs;5('-rr @\ (1.4s)
@+n @+.(-1)'
7.3 o,o,(e+e- --> hadrons) and short-distance physics 153

we get

C r,, : 2"if - \Bp,e(xil6" (x\ - iaua,[e(xs)E'(.r \]lr, O'# (7.46)

and

s"cr,, -2,,i{ - irnr)6"@\ - ia'[e(xs)d'(x'r]lrro'],. (7.47)

The total cross-section is then

otot:
# (ry) I o^.rio.* lere'"d6,,(*z) + tar[e(xs)d,(xrl]l .

Using the relation

o^. rio x5',)@\e(xo : .(7.48)


I #nri(qr)n-re(qr)r(qil
we get for the first term

o'- rio'x5tr7*2)e(xo) :T,n' (7.49)


I
and for the second term

o^- ,io'x g2 la'62ye1xo)] - -q'o'i.


I - -n' | 44, ,io'x 5'1xzSe1xsy

The total cross-section is finally

o,^,:-"1-1'(irroz\ ^ /2 r-1rr\
tot: rur, \; )n'\1a'- 6q- )
(z-so)

or

o6o,(e+e- --> hadronn:#Tr(ez). (7.51)

(c) Given the scalar field current operator

l,(x) - iDlo! o,,auqi - auQ! g,iaif (7.s2)

we have, after using Wick's theorem, for the c-number term in the operator product
expansion,
/++\
, (,0: @) e i i o rQ 1 e )::Qltyl e o, a,Qt 0),)
- i\i,Lr(x - fiialAi''(y - x)TrQ2
- -iai,Loio - y)il!, Le(:v - x)Tr Q2
a;Ap(.r)sj A,p(-x)Tr Q2 . (7.s3)
A
r54 The parton model and scaling 7.3

Similarly,

r (tq! 61Qti016,@)::0,Q1(0)Q*tQrQ),) IE,at Ar(x)lAr Gx)rr Q2

Using the relation

f
-gu' + I 4xux'
E,au L, p(x) -, l@:;* G#tr1 (7.s4)

it is straightforward to get the OPE for the whole current

r(r,,@)r"(o)): ,[#!;r. #* - #!;,1(#)' r,e2

_ _L (gp,x2 - 2xpx,) T"Y (7.s5)


- 4na (x2-ie)4 r oz .

Comparing this to the case with the spin-l/2 constituent, we see that the scalar
of Il4. Thus the total cross-section in this case is
case has an extra factor

o,o,(e+e- --> hadroni -:q,rr(Q2). (7.s6)


rq'
Remark. Usually, the e+ e- -+ hadrons cross-section is normalized to the
et-L_Ie- --+ l.t* l.t- cross-section which measures the cross-section for the point-
like particle and can be worked out for the tensor nru(e) as follows. For a final
state with pt+ (pt)l,t- (p), the tensor nu,(Q) is given by

irr,(Q)
r otlJ- ot
!, er)434(q - pt - pz)
- J effiett1tu,tz
" I u? pr)y,u(p)u(p)y,u(- p)
spin
(7.s7)

:# I+#$(q-P'l-p)(-4)
x (Ptpqz, I PzpPr, - Pr Pzqpr) (7.s8)

and

nt : p)30(q - pr - p).
3 I W(pt. (7.se)

Use the centre-of-mass frame where pr * p2


- 0, we get

n[(d:3lHgs(q-2E)
: fuo2Jf E2dE
aPz
6(o -2E)4n

q2
2n
7 .4 OPE of nuo charged weak curuents 155

Thus we get

o(e+e- --+ tt-) - V", : ( 7.60)


t-t+
#.
For the case where the hadrons are made out of spin-l/2 quarks, we get

o (e+ e- --> hadrons)

ffi-rr(Qz)-Do? Q'6r)

One simple way to interpret this formula is to treat o,",1r:* r- --> hadrons) as sum
over d (e+e- + Qi/) where QiQi are treated as point-like spin-l/2 fermions.

7 .4 OPE of two charged weak currents


The weak charged current at low energies is given by

ry@ -uypQ - y)(d.cosg *ssin 0) (7.62)


-:4@)y*(l -y)Csq(x):
where Cs, is the weak coupling matrix

/0 cosg sing\ /u\


'":(,3 3 3 QQ)-(1J (763)
)'
Use Wick's theorem to work out the operator expansion for the product

r (ry @;X*tot) e.64)


in the free field theory.

Solution to ProblemT.4
Using Wick's theorem, we get

r (ry 1x;.rf;t1oy)
- T(:Q@)yp(r - y)Cyyq(x)::Q(0)y,(r - y)ClrqQ):)

- Tr[iSrGx)yp(l - y5)i S e@)y,(t - y)c*clrl


* - y)CwiSe@)y"(I - y)CIrqQ),
:Q@)yuQ

*
:Qe)y,(l - y)Cf ;So1-"; y*(r - y)Cyyq(x):
:Q@)yu\ - y)Cwq@)4(0)y,(r - y)CIrqQ),
* Q.6s)
Assuming the fermions are massless, which is valid for lx I .
*, we have

sr(x)=iv'l#*+):#*+ (766)
156 The parton model and scaling 1.5

The first term is then

(#) r, (cwcl,) r,l-N vuo - v)/ y,(r - v)l (q,+

-2rr(c*ch)+H:# (767)

The second term is

:4Q)yr,0 - y)crci.,l--L
* lm, ---'!-1
er: o, )r,rt - vs)qQ):

_- nzlyzi*"ie)z L * ie*o'B):4(')yP (l - y)Cv'clrqloyl (7'68)


- l"rpcvup

where we have used the relation [see Cl-eqn (7 Jal]


ypN y,Q - y) - i (spo,p a ierouB) yP Q - ys)x" (7.69)

Similarly, the third term is

-ixo I . + I

7fu{{'u",oaie,o,p)..4Q)yp(1_y)C|,c*q{x):|.Q.70)
Thus we can write the operator product expansion in the free field theory as

r ef ril,wt(o))

- 2rr (c*cl,)+ft:# . -hft,u*,u + iepoup)

" l,U@)yP
(r - y)C*CIrq(0):: - 1q)ye (r - y)C[rc*q("),] ]
*:Q@)yr(r - y)Cwq@)49)y,(r - DC[rqQ): (7.71)

7 .5 The total decay rate of the W-boson


Use the operator-product expansion derived in the last problem to calculate the
total decay ratefor W+ --> hadrons.

Solution to Problem 7.5


The amplitude for Wt --> hadrons decay is given by

, : Lst" (k)(nlry l0) (1.72)

where $ is the weak chargbd current. The total decay rate is

1- pn)fr,#"(;
| - 2w
+I
1znra4@ -
; rzr') (i i3)
1 .5 The total decay rate of the W -boson 151

The spin sum is

lI trt2 : *(-r" .ffi)Ttot (It,)(ntryt0) (7 74)


sptn

Define the weak hadronic tensor by

o[,<tl -I oo* r'o * (oll4r @), ((o)] lo)

: f
oo, ,'o 'uolt[r l") @l( lo) e-iP"'"
+ J
-plJy ln)(n1t[I tol ,ip,'f

:
t U#"12n)a3a
(k - pr - P) Ptryttu)@l/Xl0)
(7.7s)

where we have used the fact that ko > 0 to eliminate the term proportional to eiPn'* .

The total decay rate can then be written as

-t-
'-48Mw\ (-.nr,
y_ 6 *' ry\ ,(y,&).
M'*)"'
(7.76)

From the Lorentz invariance, we can write

k'sp, * r{ krk, (7.7])


"[,ro - -oY
and the decay rate is then

,:fru'*nY. (7.78)

The weak charged current is generally of the form

4 : uy*(l - Y)(l,idi * Ey*(r - Y)U,idi Q.79)


where Uris ?re the Kobayashi-Maskawa (KM) matrix elements. It is easy to see
that we can simply replace

r, (cwcil) -* : I (lu,,l' + lu"it2) (7.80)


i:d,b,s

in the operator product expansion for the weak current worked out in the last
problem. The factor of 3 is due to the summation over colour. Then following the
same calculation as in the case for the electromagnetic current we get

l(w+ -+ hadrons)
f (W+ --> e+v) I (ru,,r' +ru,ir2).
-t i:d.b.s (7.81)
Gauge symmetries

8.1 The gauge field in tensor notation


(I :
The SU(n) groups consistofn x n unitary unimodularmatrices,(It [Jt - l.
(J
For infinitesimal group transformations, we can write

Uit - 6p * ielr (8.1)

where e is hermitian

ey: e[i. (8.2)

It is more convenient to use upper and lower indices so that


(8.31
e
1r = ,k e

and complex conjugation interchanges upper and lower indices

,
io
: G'-).
. (8.4)

The hermiticity relation (8.2) can then be written as

jo : j' (8'5)
' 'o
This means that the ordering of the upper and lower indices contains non-trivial
information.
The n-dimensional vector Q; and its complex conjugate @i have the following
infi nitesimal transformation law,

Qi --> Qi * ie,t4,,, Q' -. 6i - ieitQk, (8.6)

where

,,0 : (tto). and Q' : (Q)*. (8.7)

For the fields in the adjoint representation Q,/, we have

Q,i -Q,i+ie,t6ti _ iei*Qtk. (8.8)

(a) Construct the covariant derivatives for @; and Qi, respectively. Show that the
transformation law for the gauge bosons is

wuf -- w;! - wul + ie/wrl - iei ,8 -


owr! )uur,'. (8.e)

(b) Construct the field strength tensor F* for the gauge fields Wul
(c) Construct the covariant derivative for scalar fields in the adjoint representation.
8.1 The gauge field in tensor notation 159

Solution to Problem 8.1


(a) The covariant derivative acting on Qi should be of the form

(DrQ)i
- SpQi + ig(w*),khr, (8.10)

with (DrQ); having the same transformation as Qi,

(DrQ)'i: (DpQ)i * ie,t(D*Q)t. (8.11)

The left-hand side (LHS) and righrhand side (RHS) of this equation can be written
out as

LHS
- arQi + iswr:Qi - 0*(Q, -t ie,t6,) + igw't:i(Q, + istkOk)

- 0pQi * iSre,tqt + ieitaphr * igwil.(Q, + itkOk) (8.12)

RHS
- 0t Qi * isw'u,Q, * ie,' (ar[, + igw[,E*) . (8.13)

By equating these two expressions, we get

ilre,'0, + iswll (Q, + ietkQk)


- iswu:iQ1 I i2e 1tgwk1Qr,. (8.14)

Since Q1s are arbitrary, we can cancel them on both sides,

i}re,t * igw't tk (oo' + ieot)


- is (w'*, * ie,kw'-). (9.15)

Multiply both sides UV (Ai - ,r,t) and drop the terms of order e2, we get

w'r,i - wr! a ief wpti - iroi wu! - !*uur,t. (8.16)

(b) To find the field strength tensor Fr,l *, calculate the combination (DuD, -
D,Dp).
(D,D,Q)i : LD p(D,Q\i : \p(D,Q) i * i gW
*!@,il,
- ap (a,0, + igwf,Ao) + tgwr!, (a"ot + isw!,6*)
: 0p3,Qi * ig\rwf,Qo + ig (w,!\pQ* * wlia,Ok)
+ (iil2w'r,wX,Qo. (8.17)

Then we have for the antisymmetrized combination,

(DpDu - D,Dp)Oi : igla*wX, - E,w!,i + tg (w*!w,tk - w,/wrf)lQr,.


(8.18)

This means that the field strength tensor should be of the form

Fp,! : orwuik - a,wp: + ig (wrlw,rk - w,lwul). (8.19)


160 Gauge symmetries 8.1

(c) From the global transformation, it is easy to see that the combinati on W $ = Qw
which transforms as an adjoint representation is given by

(Qil ii - w ,,k Q,i - w ,i o,* . (8.20)

This can be seen as follows. Under the global transformation, we have

(Q'*) ,i - wl,o Q;i - w'uoi Q',0

: (wr: * - telwui\ (Qoi + iu{Q,l - irniQon)


ie,twu!
(w r/ * ient w r,i - i e,l w p{)(o,o + i t,^ Q,,l - it,o Q,")

- w r! Qoi - w rl Q,o + tw r! ?t,i oo") * i (e,tw un oki


w t l Qsi* o:) - (i kt w t"ti)oik

: (Qw),j I ie,t (Qiltj - it,l (Qw)i*. (8.21)

Therefore, the covariant derivative should be of the form

(DrQ),i
- yrQ,i + ig(wrfai - w-lo,*). (8.22)

Remark. If we expand Q,i andWr! interms of the hermitian traceless n x n


matrices i.",

Q,i : (L"),i Qo, wrl - lxby,iw!, (g.23)

we have

(DrQ),i : (\"),i DpQo - SpQo(L'),i

+ is (w"u(Lo)ik(xb)r,iQu - wart(xb)ik(L')ki Ob)

- o1"Qo(L'),i + ig (tx' ,ru),i w,r4r)

: a*Qo(L")ij + ig (i|"u',(L'),jW"Qu) (8.24)

and

DpQo - SpQo - gf"b'wurQ,. (8.25)

Or, if we write if"u' : (tb)or, the adjoint representation matrix, then

D1"Qo : \p[o * ig(tb1o,Wur6, $.26)

which is the standard form for the covariant derivative.


8.2 Gauge field and geometry 161

8.2 Gauge field and geometry


Under the local gauge transformations, fields at different points transform differ-
ently.

{(x) --> U(x)$(x), {(y) --> U(y)tb!) (8.27)

with U(x) * U (y). Thus the usual derivative, being proportional to the difference
of fields at different points,

arL@) q [rh@ * dx) - {r(x)1, (8.28)

does not have a simple transformation property because U(x -l dx) # U(x).
Suppose we introduce the gauge fields A, such that we can define

{L@ + dx) - v@ + d*) + Au@)th@) dx* (8.29)

so that {r@ + dx) transforms the same way as {(x), i.e.


{h @ + dx) -+ u (x)rl, (x -t dx) . (8.30)

(a) Show that if we define the covariant derivative by [see Cl-Section 8.2 for
discussion of the concept of a covariant derivative in connection with parallel
transport of a fieldl

i!r(* + dx) - rlt(x) : DptL 4xP (8.31)

then

Durb : (Ep + Ar)r/r. (8.32)

(b) Show that the gauge field has the following transformation property:

4'*:uApul -@p(DUt. (8.33)

Solution to Problem 8.2


(a) From the definition of

{r@ + d,x) - ,b@ + d*) + Ar(x)rL@) dxP


- t@) * 0*r!(x) dxp * Ar(x)t\@) dxp (8.34)

we see that the field difference is

&@ + dx) - ,1,@)


- Ept@) dxp * Ar(x)t@) dxp
(8.3s)

rhen the covariant deriva,,"", o.;r(';:,:i':::to


the rert-hand side or this
equation through the definition (8.31), D *lrdxq ,canbe expressed directly in terms
of the gauge field as given on the right-hand side:

Durl, : (3, * Ar)rlr. (8.36)


r62 Gauge symmetries 8.2

(b) By construction, the combination ry' of { andgauge field Au as given in (8.29)


has a simple local transformation:

dr'@+dx) -U@)dt@*dx). (8.37)

From this we can discover the required transformation property of the gauge field
by substituting in the relation (8.29) on both sides of the equation:

LHS
- lr'(x + dx) + A'pdxt"lt' -U(x + dx)r[(x * dx) * A'*dxpU(x)lt
- U(x)lr(x) + @rUrl, +UaptDdxp * A'udxt'U(x)rlr (8.38)

RHS
- U (x)lV<r> * 0*t dxP i A, dxv{f . (8.3e)

Equating these two expressions, we get

(ApU)rL * A'*U rh : U Aurlt. (8.40)

Eliminating r/ which is arbitrary, we have

A'u:uAput -@ru)ul . (8.4r)

Remark 1. Because the transformation is position dependent, we have the term


Gt U)Ur in the transformation of the gauge field. This means that Au does not
transform homogeneously as the ordinary field, being in some definite representa-
tion of the symmetry group. But this extra term is of the same form but of opposite
sign so as to cancel the corresponding term in the transformation of the ordinary
derivative. In this way the gauge field is just the compensating field needed to
enforce the invariance of the theory under transformations that differ from point
to point.

Remark 2. In this derivation of the transformation property of the gauge field we


have emphasized the geometric aspect of a local transformation as discussed in
Cl-Section8.2.In Section 8.1 we have already provided a derivation by explicitly
using the covariant nature of the covariant derivative: the covariant derivative is
defined to transform in the same way as r[(x),i.e.

D rrl, (x) + IDpr[(x)]' - U (x)lD *tL @)1. (8.42)

The gauge field Ar,having been introduced as the difference between the covariant
and ordinary derivatives

D *rb @) - }plt (x) + A u@)r[ (x), (8.43)

can easily be shown to have the transformation as given in (8.41).


8.3 General relativity as a gauge theory 163

8.3 General relativity as a gauge theory


In general relativity, one studies the general coordinate transformation

dxp --> dx'p - Uf(x) dx' , 3t" + 0'*: 1tl-11x11'ra, (8.44)

which can be viewed as a local transformation with


..
ul,(*) :;n3x'! , r
lu-t
t iu
6))'u: a*.
0x'
(8.4s)

Following the same procedure as suggested in Problem 8.2, we can choose to view
general relativity also as a gauge theory. Just as in eqn (8.44), a vector field f p(x)
and a (mixed) tensor fieldT((;r) have the transformation properties of

'r(*) - Ul(r)f '(x), (8.46)

T'{ (r) ul(x)tu-t (x)loxTuo@). (8.47)


-
Consider the differentiation of a field having a non-trivial transformation
property-the simplest would be the vector field f r'(x). Clearly, the ordinary
case
derivative 31f & does not transform homogeneously, as in eqn (8.47), because the
transformation is local, ExUp, 10. Or, stated more geometrically, this is because
P (x -fdx) transforms differently from * (x). As in Problem 8.2 we can introduce,
as in eqn (8.29), a modified vector field

E* @ + dx) - P (x r dx) - rtp" (x) dxB (8.48)

which transforms identically as f p(x):

E'u @ * dx) - (JP,@)E' @ * d.x). (8.4e)

A comparison of eqns (8.29) and (8.48) shows that the compensating field ff,
plays the same role as the gauge fie\d Ap . The llu field is called a connection in
geometry or the Christoffel symbol.
(a) Show that if we define the covariant derivative by

E" @ + dx) - " (x) = D*" (x) dxp -r o((dx)z), (8.50)

then

DxP (8.51)
-8aP -1,l.,'
(b) Show that the connection has the following transformation property:

l'f^ : u! (u-111,(u-t)'^l"py + (u-110,1u-t)'^ (aeuur). (9.52)

Namely, besides the usual homogeneous term (first one on the right-hand side),
there is an inhomogeneous term (the second one).
(c) Show that the Lagrangian for the vector field 6r'(x) given by
L : i (or") (o,e) Bp'BoB (8.s3)

is invariant under the general coordinate transformation. gaB@) is the position-


dependent metric tensor.
164 Gauge symmetries 8.3

Solution to Problem 8.3


(a) From the definition of

E" @ + d.x) - " (x + dx) - lf,o dx'lP


- o (r) + 0r" (x) dxp - lir@)o (x) dx" (8.54)

we see that the field difference is

E" e+ dx) - " (x) - E*o (x) dxp - l\r@)p (x) dx'
: (o*q" - tflrP) axP. (8.55)

Then the covariant derivative, being directly related to the left-hand side of this
equation through the definition (8.50) , D*o dxe , canbe expressed in terms of the
gauge field in the right-hand side:

Dr" - 3r" - f7,o0. (8.56)

(b) Under the general coordinate (gauge) transformation, we have

E''@ *dx) - (JP,@)E"@ * dx). (8.57)

We can extract the transformation property of the connection field by writing out
the components on both sides of this equation:

LHS
- * d.x) - f'#dx'Lq'o
'r(x
* (Jp,(x * dx)l' (x * dx) - f '#ULo dxo U'pp

- u*,(x)6'(x) + up,dxp0p" * Eouuud*"'


- f'#[JLodxouopP. (8.58)

RHS
- Up,(x) [6'(r) * d.xf 3p' - tiu dx" $pl. (8.59)

Equating these two expressions, we get

Souupd'x"qf - f'{e[ILodxou'pP : -(Jp,lIBd'x"qfl (8'60)

Since dx" and lF are arbitrary, we have

A"U\ - l'{ru^,Uop : -(l*,1'op (8.61)

or, written in the form as shown in (8.52):

l'{, : (u-t)p^(u-\ro(J*oT"p, + (u-t)0^(u-t)r, (aeu*r). (8.62)


8.4 O(n) gauge theory 165

Remark. We have derived the transformation rule of the Christoffel symbols by


starting with the combination (8.48) having a simple transformation property as in
eqn (8.49). This approach emphasizes the geometric aspects of gauge transform-
ation. Alternatively, we could have started with the definition a covariant deriva-
tle D1$P as in (8.51) which has the definite transformation property as a mixed
tensor [cf. in eqn (8.47)]:

(Dx\' - uf @)lu-t (x)tf (Do') (8.63)

The same transformations of the Christoffel symbol can be extracted.


(c) Asthecovariantderivative Do$P andthemetricgpu transfonnastensorsunder
the general coordinate transformation, it is easy to see that the Lagrangian given by

L - ;(o*") (o"e) lPu BoB (8.64)

is invariant because all the tensor indices are contracted.

Remark. Just as the field tensor Fop in gauge theory can be defined through the
commutator of covariant derivatives

(D"Dp-DpD"){t=Foprb, (8.6s)

the corresponding field tensor in general relativity can be defined in a similar way:

(D,Dp - DBD,)B' : RIp,Bu. (8.66)

The field tensor R[uu where

Rtp, : a[fp - oBtf" + fteu" - ft"fl,e (8.67)

is called the Riemann curvature tensor. From this, an invariant action of the gravi-
tational field can be constructed.

8.4 O(n) gauge theory


Considertwosetsofscalarfields, @, ,fuz,whrchtransformasvectorrepresentations
under the O(n ) group.

(a) Show that under infinitesimal O(n) transformations we have

(Q)i + (Q)i*eii(Q)i wherea:1,2 (8.68)

and eii - - e j i are the parameters which characteri ze the infinitesim al O(n) trans-
formations.
(b) Construct the covariant derivative for rf ,.
166 Gauge symmetries 8.4

Solution to Problem 8.4


(a) O(n ) transformations are charactenzedby n x n orthogonal matrices O;i,
OijOi* - 6jk. (8.69)

For infinitesimal transformations, we write

Oij 6ij I eij, with e;; (( 1 (8.70)

and the orthogonality relation implies

6;r * @p * eU) : j* -+ tij : -e ji.


6 (8.71)

Thus infinitesimal O(n ) transformations are characterized by Lr@ - I ) parameters.


In general , O(n) vectors transform by n x n orthogonal matrices,

Qi --+ Q'i : OiiQi- $.72)


For the infinitesimal transformations, we have

0'i : Qi -f eufi. (8.73)

(b) For the covariant derivative we need the adjoint representation of O (n). It is
not hard to see that they are just the second-rank antisymmetric tensors,

Qii --> Q"i: Qii * @irQ*i * eirhi) wrth Qi1 : -Qji G.74)
This gives the global transformation law for the gauge bosons Wpij. Write the
covariant derivative of @ as

DrQ, :\pfii * gWpt*Q,p with Wpik: -Wpri. (8.75)

Then by requiring the covariant derivative of @ to transform in the same way as @;

(DpQ)' - DpQi * eil DpQ; (8.76)

which can be written out as

LHS
- arh', + swL,oQL
- \p(Qt * eiiQ) * swl*(fu + e*ifi)
: ErQi * @pei)Q j * eij\rQi * gW'pit (fu + euQ) $.77)

RHS
- SpQi * gwp*Qt * e4(0rQ1 * gwplrQi. (8.78)

Combining these two expressions and cancelling out @; we get

lpsij * gW'pit (6O * geitWpkj.


t sU): BWptj (8.79)

Multiplying by (3;l I y) and using the property sij : -t ji, we get


e

\psit I gW'rit - gWp'il * gWptieir * $s*Wp'u (8'80)

or

W'*it: W1"it * Wpileit I eir,Wp*r - !@rr,,).


o
(8.81)

This is the transformation law for the gauge bosons.


8.5 Broken generators and Goldstone bosons 167

8.5 Broken generators and Goldstone bosons


Let Qi be the scalar fields in the vector representation of the SU(n) group.
(a) Write down the SU(n ) invariant scalar potential for Q;.
(b) Work out the possible pattern for the spontaneous symmetry breakingfor Qi.
How many Goldstone bosons are there in this case ?

(c) Discuss the possible spontaneous symmetry breaking pattern for the case where
there are two such scalar fields Q1; and Q21.

Solution to Problem 8.5


(a) As we have seen in Problem 8.1, the vector Qi and its complex conjugate 4|i
have the following transformation laws,

Qi --> Q',: Q, + ie,tQt, O' --, O'i - Oi - iei1,Qk. (8.82)

Thus the SU(n ) invariant combination is of the form

QiQi - Q',Q" - (ot + it/o') (ot - ittoQo)


: QiQi I ie,'O,O' - it'oQo4i : QiQi (8.83)

which is just the scalar product in the n-dimensional complex vector space. The
SU(n) invariant scalar potential can depend on this combination QiQi ,

v(il - -pzoio' +:(o,o')'. (8.84)

(b) Let QrQi : QiQi - p2.We can write the scalar potential as

v (Q)
-- t"2 p2 * Lo^ (8.85)

Then

3V . . ?.^ + rc
P-lT=u (8'86)
ap:(-tt'*LP4)2P-g
or

QiQi : o' - +. (8.87)

Without any loss of generality, we can choose

\Qilo:6inu. (8'88)

Clearly, the symmetry-breaking pattern is

SU(n) -+ SU(n - 1). (8.89)

To get the Goldstone bosons, we write the fields as

Qi:Q,i*6inu
168 Gauge symmetries 8.5

so that Q',have zero VEV. It is easy to see that

(0'0,) - (Q! -tsinr)(Q" +6i,r) -Q',oi +@"+a;)u*u2 (8.e0)

(o' Q,)'
- yo" oi + (0, + a;) u + u2f2
- u2 (Q, + oil' t 2u2 (o" o!) + . . . (8.e1)

and the quadratic terms in V(@) is

vz(il : -pz 1o,,oi) *:lf (0, + o:)' + 2,, (a,'a!)]

:t\Q"+Q;).
-
l"' ,^ , -r*\2
(8.92)

This means that Qr, Qz, . . . , Qn-r and Im Qn aremassless Goldstone bosons. Since
each of Qr, Qz, . . . , Qn-is a complex field and has two degrees of freedoms, the
total number of Goldstone bosons is (2n - 1). This is precisely the number of
broken generators, (r2 - l) - t@ D2 - 11, in the symmetry-breaking partern
-
SU(n) -+ SU(n - l).
(c) For the case of two vectors, it is easy to see that the SU(n) invariant combin-
ations are of the form

Q'Qrt, Q,Qz', Q,Qz', 0zi6ri (8.e3)

We can parametrize them as

Qrittt - p?, Qz,Q'' - p7, Qt,Q't : ptp2Z, QztQ" : prpzz*. (8.94)

Then the scalar potential V (0r, Q) candepend on pr , p2, &ndz. The minimization
of V (Q1, Q) will fix the value of these three variables. Without loss of generality,
we can choose the VEV to be

@'l)o ' (Qz)':[,,, (8es)

[| ),r,,,',,)
The symmetry-breaking pattern is

SU(n) -+ SU(n -2).

Note that just like vectors in SO(n ), we can generalize this to the k vectors to get
the symmetry breaking

SU(n) -+ SU(n - k).


8.6 Symmetry breaking by an adjoint scalar 169

8.6 Symmetry breaking by an adjoint scalar


Suppose Q,i *, scalar fields in the adjoint representation of an SU (n) group.
(a) Write down the scalar potential for Q,i .

(b) Work out the possible pattern for the spontaneous symmetry breakin gfor Q,i .

Solution to Problem 8.6


(a) From the SU(n) transformation properties for the adjoint representation,

Q,i * Q',i : Q,i + it/Q,i - itoi Q,k. (8.e6)

Note that Q,' :0 and (Q,i).


- Q j'. We have the following quadratic and quartic
invariants (obtained by contracting the tensor indices):

QIQ'1 - rr(Q2), (o!o',)' - lrr(02)12, O/OjOLOi - rr(Q4) ' (8.e7)

where we have written Q,i ut an n xn matrix. The scalar potential takes the form

v(il - -p2Tr(Q2) -t ),{Tr(Q')l' + )"zTr(Q4). (8.e8)

For simplicity, we have imposed a discrete symmetry Q --> -Q to remove the term
of the form T rQ3 .

(b) It is clear that Q is a traceless hermitian matrix, which can be diagonalized by


the SU(n ) transformation. Thus without any loss of generality, we can take @ to
be real and diagonal,

(8.ee)

The scalar potential is then of the form

v(Q) : -p24r?* ^, rr] + x,D,a! (s.loo)


[P
Since not all Qis areindependejr,, *, need to introduce the Lagrange multiplier to
take into account the constraint Q1 * Qz I. . . + Qn :0:

v(Q) - -tLz.r?.^,[P rr7 -,x,Da? -c (tr,) (8. r01)

and f is determined at the end by the condition

h-f0z+"'*Qn:0. (8.102)
r70 Gauge symmetries 8.6

For the minimum of V (Q), we have

-- -2t"'Q' + $''1 (ttt) * $'2E' - q - o (8.103)


# Qi

which means that each @; is a solution of the cubic equation

-2p'*+4).px*4)"2x3 -6:0, with a -DO?. (8.104)

Since the cubic equation can have at most three different roots, the most general
form for @; is

X1

X2
o- (8.r0s)

X3

with

nfir * nzxz * r\x3 : Q, nt * nz * n3 : n. (8.106)

Thus the general pattern for the symmetry breaking is

SU(n) -+ SU(n1) x SU(n)x SU(nr) with nq lnz+n3:n. (8.107)

In other words, the SU(n) group with scalars in the adjoint representation can
break at most into products of three SU(n;) groups. A more detailed calculation
(Li 1974) shows the following symmetry-breaking patterns:

SU(n) -+ SU(n r) x SU(n - n) x U(1) for ,1,2 > 0 (8.r08)

with

:1, n-fl
nr n even, nl: , n odd, (8.10e)
2
and for Lz < O,

SU(n ) -+ SU(n - 1) x U(1).

Remark. Forthe special case where lr - 0, and the constraint T rQ


- 0 is absent
(so that 6 : 0), we have

v(il - I (u'o? +xrQ!): I f @). (8.110)


i
8.7 Symmetry breaking and the coset space l7I
This means that r@;s decouple from each other. Thus each f (dt) is minimized
independently, and it is easy to see that all @;s should take the same value at the
minimum of f (il. Then the V E V is of the form

(d)o:,(t (8.1 1 1)
,)
This situation is realized in the spontaneous symmetry breaking of QCD in the
large N, approximation (see Coleman and Witten 1980).

8.7 Symmetry breaking and the coset space


Suppose the scalar potential V (il is invariant under the symmetry group G.

(a) Show that if Q : Qt I 0 is a minimum of V ($), then other @;s, which are
related to Qi by symmetry transformations of G, also minimize V (Q). (Vacuum
is necessarily degenerate.) Show that the symmetry operations that relate these
@;s form a subgroup, call it H, of G. The pattern of symmetry breaking is then
G --+ H.
(b) Because of the unbroken symmetry H , the minimum of V (Q) is a degenerate
one, i.e. there is more than one value of @ which minimizes V (il. Denote by M (Qo)
the set of rps which minimize V (il.Show that for a given pattern of symmetry
breaking, G --> H, M (Qd can be identified with the coset space G I H.
(c) For the case G - SO(n) and the scalar fields in the vector representation,
the coset space is SO(n)/SO(n - 1) : Sn-I, which is the surface of a sphere in
n-dimensional real space.

Solution to Problem 8.7


(a) V (il is invariant under G means that

v(il-v(sQ) YgeG
where gd is obtained from @ by the transformation Q --> g@ with I G.Then
if h * 0 is a minimum of V (Q),gdr is also a minimum of V (Q). It is clear that
those group elements which leave Ql invanant form a subgroup, call it F1. This can
be seen as follows .If h1Q1 : Qt, hzfir : @1, then hthzQr - Q1 and nrrh : Qr.
Thus if hr,h2 H,then(hth) e H andht',h;t e H.Inotherwords,Flisa
subgroup. The pattern for the symmetry breaking is then G + H.
(b) Recall from group theory that the coset space G / H is made up of (left) cosets
of the form g; H ,i.e. the collection of elements obtained by left-multiplying Si f H
with the whole subgroup F1. The cosets obtained this way have the property that
either they are completely different (no elements in common) or they are identical.
1 H,theng;H:8i^F1,i.e.two
Inparticular,if Si f H andgl f H,butg;g,
cosets are identical. But rf Sig jr / H, then g; H and BiH have no elements in
common.
ll2 Gauge symmetries 8.8

Now let us look at the set M(Qd. Clearly, for each @; M(Qd, we have
HQi : @;. Suppose we choose an arbitrary h e M(Qe). Then by the action of
the coset giH we have (g; f H)
SiHQr:qifrfh. (8. l 12)
This means that the action of the coset giH on rp1 will generate another element
Qz : giHQt, which is different from @1. Since the potentialV (d) is invariant
under the group G, the new element @2 must also be in M(Qd. Furthermore,

siHQr * siHQt if sis;t / H. (8.1 13)

Namely, different cosets generate different @; s in M (Qd. Thus if we identify @1


with F1and the image g;@1 with the coset gi H , we have a one-to-one mapping of
the @s in M (Qd with the coset G I H . This mapping is onto if M (Qd is transitive,
i.e. every element in M (QO can be obtained from a given h by the action of a
group element in G.
(c) For vector representation in SO(n ), the scalar potential V (il depends only on

0.0-o?+...+o: (8.l 14)


and it is minimized for a particular value of this combination

a?+...+Q?,:rr, (8.1 1s)

where u is related to the parameters in V(d). Thus

M(Qd-[010.0-r,l (8.116)

and it is just the surface of a sphere in an n -dimensional real space, Sn-I. Therefore,
we have the result

SO(n )/SO (n - l)_ $n -r (8.117)

Remark. It is easy to generalize this to the unitary group to get


SU(n)/SU (n - l) - 52n-t . (8.1 18)

8.8 Scalar potential and first-order phase transition


Consider the case of one hermitian scalar field Q with scalar potential

u2 .
- -+0'I
^1"
vo@) .Q". (8.11e)
z4
We have shown in the Cl-eqn (5.134) that Vs(Q) has a degenerate minimum at
Q : :lu, with u Q| /L) l/2. Suppose we add a cubic term to vo@)
-
v6<o> : -tr' *'*r' * Xr^. (8.120)

Show that the degeneracy in the minimum of Vo(Q) is now removed. Find the true
minimum of Vs($). Also, show that, as a function of the parameter f , the VEV
(@)s changes discontinuously from (@)e
- -u to (d)o - u &S f changes from
positive to negative values going through 0.
8.9 Superconductivity as a Higgs phenomenon I73

Solution to Problem 8.8


The minimizationcondition leads to the equation

Y_o
a0
+ QGt"r+2O+)"621 -s. (8.121)

The non-trivial solutions are

Q : Q+: i t-u + G2 * ),p.21/21. g.t2z)


The two minima no longer have the same value for the potential V6@. For small
f , we have

Q+:*u- l"*o(')-or-I (8.r23)

with @9 _ u or -u, and

v6(QD- vo(u) . (+ - ro) Qo : vo@) - *r,u. 6.t24)


Thenforf > 0,V6@) hasminimumat@-,whiletheminimumis atQ+forf < 0.
This means that as f varies from f < 0 to f
discontinuously. This is usually referred to as a first-order phase transition.

8.9 Superconductivity as a Higgs phenomenon


Consider the scalar QED with Higgs phenomena with the Lagrangian

2 - (D,,Qr)@uo) +
{0, o - }ro'o>' - IFp'
Fp, (8.12s)

with
DpQ
- - ieAp)Q, Ft" - At'A' - A'At".
@p (8.126)

Consider the static case where A0O


- 00A - 0 and Ao - 0.
(a) Show that the equation of motion for A is of the form
V xB- J with J - ielQI(V - ieA)Q- (V + ie|)qtL1.
(b) Show that with spontaneous symmetry breaking, in the classical approxima-
tion @ : t) : Qr' /^.)t/', the current J is of the form

J - e2u2\ (the London equation) (8.127)

and thus

V2B
- tzvzB (the Meissner effect). (8.128)

(c) The resistivity p for the system is defined by


E -- pJ. (8.129)

Show that, in this case of spontaneous symmetry breaking, p : 0, and we have


superconductivity.
r74 Gauge symmetries 8.9

Solution to Problem 8.9


(a) In the static limit, Maxwell's equations are of the form

-EuFu':J' + 01fii --ii or VxB-J (8.130)

where

AL
Jp
oAu - tel(Du4') A - Or @uO]
_ tel@p + ieAp)Qr O - Or (op - ieA)Qf . (8.131)

Thus

J : ie [Ottv - ieA)Q - (v + ieA)Qrq). G.t3z)


(b) Spontaneous symmetry breaking gives Q : u- (l"2lX) l/2, which in turn
gives the London equation:

J - ezu2\. (8.133)

From Maxwell's equation, VxB- J, we get

V x (V x B) - V x J or V(V.B) -V2B - -ezu2Y x A


or

v2B tzuzB (8.134)


-
where we have used V .B - 0. It is not difficult to see that this equation implies
the Meissner effect because it implies a solution for the magnetic field of the form

/n.xr I
B(x)-exp(, / with ,:*. (8.1 3s)

This means that the magnetic fields decays in a distance of order of I - (eu)-' .

(c) Since EOA 0 and A0 :0, we get E 0. On the other hand, we have J + 0.
- -
This means that the resistivity must vanish (superconductivity) p :0.
9 Ouantum gauge theories

9.1 Propagator in the covariant R6 gauge


The free field generating functional in the generalized covariant gauge is given in
Cl-eqn (9.82) as

wIrtt -
| ra\,..n { i I ao*l}u,*r;@)e!, * t;o"f} rn tr

with

Ktl@ - ;ablsu'a'z- (t-;) u'r'] ez)

f is an arbitrary constant. If we define the Green's function CII@ - y) by

I o^, rcil@ - ylcu,io - z) - gf6isa(x - z) (e.3)

where

xtl@ - y)
- 6o(* - v)Kli@), (e.4)

show that

GII^*- y) - u.u
| #,-,r(x-v) [- (',, - ry) - try)i (e sr

is the propagator for the gauge field.

Solution to Problem 9.1


The definition of the Green's function can be written as

r<i| @c!1@ - z) - sfs|yalx - z). (e.6)

For the internal symmetry indices, we have GX'^ o 6b'. We can thus define the
Fourier transform as

!Lr-'o \'-i1u^(k).
J[
cu,\ry - z) - su' (g.7)
(2n)o

We then get

* (t - i) wk'fE,x&) : Bf (e 8)
l-s,'r,z
116 Quantum gauge theories 9.2

From general covariance, we can write

E,x(k) - a(kz)g,x + b(k2)k,k^. (9.9)

Then we get

a&\l-rro'* (t -
;)
k.k^f+b&2) (+) k2kt'kx
- sf (e r0)

By identifying independent tensors on both sides, we get

a&\ : -i, b(k2)


- i(1 - 6). (e.11)

Thus,

E"x&):i[-("'-?-try) er2)

9.2 The propagator for a massive vector field


For a massive vector fieId V*, the free Lagrangian is given by

Lo : -Ir*vt"' + Yrrr- with vp, : orvu - o,vr,.


Show that the propagator is given by

iLp,(k):-W. (e.13)

Solution to Problem9.2
We can write the generating functional as (see Problem 9.1),

wtl) :
I @vtl*o { i | {t,+ rt'v,) ao*l
: t'"f
I vvt)*n { i I ao * l}v,1xy xp' (x)v,(x) + r
} e.r4)
where KP' can be calculated from Ls as follow:

I o^. o :-l
*' *l
'
1o^.,1::::'
o^ ::.:';:;: :f "',
Comparing it to

:
I o^, Lo I o^. Lvug1rc,'(x)v,(x), (9.16)
9.3 Gauge boson propagator in the axial gauge t77

we can conclude that

KP'(x) - (gPuA2 - At'A' * M2gw'y. (e.t7)

Define the Green's function G *u(x - y) by

KP'(x)Gux(x - y)
- Sf 64(x - y), (9.18)

and introduce the Fourier transform

G,x(x- )) : tx-il g,^11r1 (9.19)


I #r-ik
to get

(- gq'kz + kt" k' * M2 gu'1 E,x&) : Bl. ( g.20)

From the tensor structure E"x(k)


- a(k2)grx * b(k2)t<,k1, this equation can then
be written as

l-g" (k2 - M') + kt'k'f lo(k')s,^ + b(k\k,k^] : gf ,

which fixes the scalar function to be

u---pz_yz'
/, _ __ u-
h _
Wlkr-W)
Thus we obtain the result

: I f k,kx1
E,x(k)
1rz _ 742 L-r,^ * ;A, ). (9.2t)

9.3 Gauge boson propagator in the axial gauge


The axial gauge condition is given by

nP Aou:O (9'22)

where nP wrthn2 < }is a space-like vector. Show that the gauge boson propagator
in momentum space is of the form

n;lg,t:u#l-rr, . *(npkv . kpn,).Wo,o,f (s.23)

where cv is an arbitrary parameter in the gauge-fixing term.


I78 Quantum gauge theories 9.4

Solution to Problem 9.3


Omitting the unimportant non-Abelian indices a, b,thefree Lagrangian with gauge
fixing term is given by

rl
Lo: - O,Au)'^l^
* . A)'. (9.24)
+(AuA, - Zo@
Then

:
to
I Lgdax : I oo*U*(su,D' - atra,)A' . *Aenrn,,+,f
- I oo*)o***"o'
with

Kt, : (gur|' - 0ulr) + lnrnr.


The gauge boson propagator Lp,(.r) is defined by

Ku,(x)L'^(* -i - 81"64i.;- -y). (9.25)

In momentum space, this corresponds to

Kt,(k)L'^(td - s!, with Kt, : (-gruk' + kt"k,) + lnrr,.


It is straightforward to solve for the coefficients a, b, c, and d inthe tensor decom-
position of the propagator

A'^(ft) - ag'L I b(nuk, * krn,) * ckrk, * d.nrn, (9.26)

and the results are

I I I n2-uk2
=-kT b: c--kr(,-ky, d:0. (9.27)
n.kkr,
We obtain the stated form of

A'^(k) - :K'L[-r'^ . *(npk, * krn, - "G i:r" oro,f . e.2B)

9.4 Gauge boson propagator in the Coulomb gauge


Calculate the gauge boson propagator in the Coulomb gauge:

0'A : 0, (9.29)

where we have ignored the internal symmetry index. To solve this problem we
suggest rewriting this gauge condition as

3'A, - cr\u(cuAu) g where cp : (1, 0, 0, 0).


-
9.4 Gauge boson propagator in the Coulomb gauge 179

Solution to Problem9.4
The Lagrangian with the corresponding gauge-fixing term is given as

Lo: -.Ir*Ft , + |rc.A- c.E(c.A)12. (9.30)

Thus,

:
I o^ , Lo
I .IIU,
o^ (02 I u, - a
t"a) A"l

|o, laua, - Att (c . 3)(c,31, t cu3,)+ (c . ilzr*r,l e"l

and

Kr,(x) - (02gr, - ErL,) - llurr, - (c.0)(c,0**crl,)+ (c. E)2c*c,].

In momentum space, this gives

Kp"(k):(-k'gu,+kpk,)
l-
+ lltcut<, - (c. Q (c,ku * c*k") + (c. k)2cuc,l

(-k'gr, + kt k,) + L Vu- (c . k)ct]lt<, - (c . k)c,l


: (-k'gu, + kt ku) + lrrr, (9.31)

where

fl*: k, - (c .k)cp. (9.32)

Since K p,(k) now has the same structure as in Problem 9.3, we can take over its
result to write

lf I n2-uk2 I
Lp,(k): nrk,).
t .,(npk, Ikrn,)-
(9.33)
k,l-rr,+ Q-W
From

,2:lk-(c.k)clz -k2 -(r.k)2, k.n--k'- (r.k)2,

npku * kpfl, :zkpk, - (k 'c)(c*k, * krc,),


we get

It c.k
_ -ilsr, +
F
t k,c,)
^u"(k)
^r;e(c,k,
akzkuk"
-p:1r.1ry-_ I (e.34)
W_k.fr1
180 Quantum gauge theories 9.6

9.5 Gauge invariance of a scattering amplitude


In the non-Abelian gauge theory with fermions the Lagrangian is of the form

L - -IF"r'Ff,, + ,lrtyu orV - mrlrrl, (e.35)

with

D*rt, : (0, - tgt" Ai)V. (9.36)

Show that, to the lowest order in the gauge coupling g, the fermion scattering
amplitude

lr" +rlru -,lr'+rbd


is the same in the covariant, the axial, and the Coulomb gauges.

Solution to Problem 9.5


A typical amplitude, to order 92,has the structure

M c d, a b - l, @,) y rt', uu p )l Lp" (k) l, @ D y, t'o,u p )]


( (

- Jp(p,. p)Lu'(k)J,(pa. po)ft:bti) Q.37)


with

k : po - pd : pc - pu, Jp(p,,p) : u(p)ypu(pil.


It is easy to see that J, has the property

k* Jr(p,, p) : (p, - p)Pu(p,)y*u(p) - u(p,)(il, - il iu(pu)


: (nt, - miu(p")u(p)
- 0. (e.38)

Similarly,

k'Jr(pa, po) : @a - p")'u(p)yru(p) - 0. (e.3e)

Vector boson propagators Ap'(k) in different gauges differ from each other by
terms of the form k*ku or (kpn, * nrk). Since krJ* : k'Ju - 0, these terms
do not contribute to the amplitude M,d,ab. We get the same answer in all these
gauges. Note that it is essential that the term proportion al to n ,n u is absent in order
to get the gauge independence results.

9.6 Ward identities in OED


The generatingfunctionalfor QED in the covariant gauge is of the form (see Ryder
198s)

Z(J, r7, r7)


- x I we-rdpyapr,*n (i I t,,, d^,) (e.40)
9.6 Ward identities in QED 181

where

I
f ^--:
r."ff -;ru'Fp, + rlrieuI - ;-A \.!,-^.
ieAp)t - mrltrl, - ;eu At)2

+ Jt" Ap + rbrt + iV.

(a) Show that if we require the generating functional Z(J,n,4) to be invariant


under the (infinitesimal) gauge transformation

A, + Ap+ ApL and tlt --> tlr - ielt{r, (9.41)

we get the relation called the Ward-Takahashi identity:

[lor"3
.-"
6J, - ' oL -,u
utt -,(
uapr,, 't) 'tt :0.
\1r,r,4,i) - v' ) (s.42)
Lo \"dri " di ) )L\r

(b) This identity (9.42) can be translated into an equation for the generating func-
tional for the connected Green's function W by Z: eiw. Furthermore, we can
write this in terms of the Legendre transform of W [q , i , J u)

fI',[, rL,Au) - W[n, i, Ju] -| a^*f Ju Au + ,lrn + itD e.$)


where

6W-6W6W
t : un, ,lt : srt , A, -
Nr.
(9.44)

are usually called the classical fields.

(c) Show that by differentiating the result in part (b) with respect to ry'("r1) and
,b0) and setting ,1, : ,b : Ap: 0 we can derive the familiar form of the Ward
identity:

qpl p(p, q, p + q) - s;t (p + q) - s;t (p). (9.45)

where the vertex function f, and the propagator S;t in momentum space are
related to f in Q.a$by

f"
JI ' Jt
do'do*'d'4Y1si(n'"-Ptt-Qx)^
=''
6t(x)6tb11)6e*
- ie(2n)o6o(p' - p - dlp(p, q, p') (9.46)

and

I
oo*, d.4y, si(n''r-p!r)
fi#6 - ie(2tr)a64(p' - p)is;t @). (g.47)
r82 Quantum gauge theories 9.6

(d) From part (b) show that

I 32W
-;o-ul -=-^ : o! 8P'60(r - Y)' (e.48)

which in momentum space gives

LkrkrGu,(k): k, (9.49)

with

-== .6'-Y=, ,: [ *r-ik(x-v)G*,{t).


6Jr(x)6J,(t) I (2n)o

Solution to Problem 9.6


(a) Write

L,rr: L- *(0'Au)z + JuAu+rlrn+iV - L* Lr (9.50)

where ,C is the original Lagrangian without the gauge fixing and source terms.
Recall that L is invariant under the gauge transformation. Thus the changes of
L"yy under the gauge transformation all come from .C1,

u
I t"' oo' :-'
/ ,:,lrrr,^,)rA + rt"at,^ - iert(-t,q + n,t )f
(e.s1)

The change in the generating functional due to the gauge transformation is then

az(J, rt,i) :x t"rr d.r) t r."y ao,


l We,rlld|ltd(tt.^n (; I I
:* I t,,, d^,)
(t
lwAt)tdvttd,bt "o

" { I d^'l-!z@u e*) - l,rtr -


ie(-1'n + i'D)^(")
}

Gauge invariance implies that62


- 0. Since A(r) in62 is arbitrary, its coefficient
must vanish:

[;. (r,#) - 0*r, -, (-.r; . r#)] z(l,rt,a) :0 (s s2)


9.6 Ward identities in QED 183

(b) Insert Z -- eiw into the above equation and perform the differentiation to get

[;" (,' #) - o*l' -,, (-,'#.0:f)]


:' (es3)

In terms of Legendre transform:

llrh, rb, A1)


- W[rl, i, Jr) - I
o^. (J* A, + Vrt + i{D e.54)

where

6W-6W6W A'-
t: 'lt: 6n' Nu'
n'
Then we have

6f_6f_6f
-n: -Jp:
6r,lr' -n - 6rL' aAr"

and we can write eqn (9.53) as

[1o(at'Ap) -
dr ie / 6r* ,t[)-l o. (e.ss)
[a s,4r- (-/
Ep
' / - srl, srl, J

(c) Differentiating eqn (9.55) with respe ct to ,1t (xr ) and V 0) and settin g ,lr :
th : Ap: 0, we get the relation

d'f
=
-af^ Srlr(*,)St|0r)6eP
-ie6(-r-.rr) = "I- (yr)
' 6r\(xt)df

62f
- ie6(x - t) si@)6rb-. (e.s6)

Multiplying by exp i (p' x - plt - qx) and integrating over -tr, x1, arfld)1, we get

- J[ aor daxl' dayl


rt ei(p'*'-pv'-'qr)0.^ ' ' , l',-'
^6rlr(xt)6{(yr)6Au
U.'f'
: ie Iao*daxldaylei(p'"-pvr-qi64(x-.rr)=r,
' J' , ,
'6rh@t)6rlr(yr)
I
r ., 621-
- ie J dax dax1d4y, sitn''r-pr'r-4'r)60iu- - lt\ 5p1-rn*)
This gives

eplp(p, q, p + q) - s;t(p + q) - s;t(p) (9.57)

which is the usual form of the Ward-Takahashi identity.


184 Quantum gauge theories 9.7

(d) From Part (b),

t+o (u' #) - o*r' -,, (-,Y . rffi)l : o

Differentiating this with respect to /,(y) and setting i- n - Ju


- 0, we get

lo(u,-- s'Y \:
uf ru'60(* - (e.s8)
q \ ttt'(x)6J\y)) Y)'

Recall that the gauge field propagator is related to W by

62w
Gru(x - )) : (0lr @p@)A,(O))10) : (i)2 p
6J (x)6 Jv (y)

The Ward identity is then of the form

I 6'L-^r-ux
;o*ri *ru - E,ttvu^4(-t^ - )) (e.se)

or in momentum space

LkrkrG ru(k) : ku. (e.60)

Remark. This relation is true to all order in e and gives the result that the longi-
tudinal part of G p, is not modified by the interaction. This can be seen as follows.
Write Gp,(k) as

G,,(k): (r*, -W Gr&2).TGr&z). (e.6r)


Then the Ward identity implies that

Lk'Gr(kz1k,
- 1r, or Gilkz) :#
This is just the lowest order result as seen in Problem 9.1 (with * a).

9.7 Nilpotent BRST charges


The BRST (Becchi-Rouet-Stora-Tyutin) charge Q is defined through the BRST
transformations of a field

60 : @Q0 (e.62)

where a,l is an arbitrary anticommuting Grassmann variable. From the BRS trans-
formations given in Cl-eqn (9.I32), show that the BRST charge has the property
of being nilpotent Q2Q 0 for (i) a gauge field Q : AL, (ii) a fermion field
-
Q : ,h, (iii) the ghost fields Q : p" and oo.
9.7 Nilpotent BRSZ charges 185

Hint. For the case of ghost fields we need to use the equation of motion for the
oo to show Q2po
-0.
Solution to Problem9.T
From the BRST in Cl-eqn (9.132), we can extract the properties of the BRST
charges:

6Ai - oDroo + QAor: D*oo :\poo - geob'ob AX (e.63)

6V : igot(T"oo)tlr + QV : ig(7"o")(r (e.64)

6po
- -tf,{a* t) + epo - -f,{a* e"*) (e.65)

5oo : -|rrob'obo' + Qoo : -|r"b'obo'. (e.66)

(i) Gauge field

e'A"*: e(eAo*) : e (oroo - ge"b'ob A'r)

:\p(Qoo) - gt"u'(Q"b)AL+ geob'ob(OAD. Q.67)

We now examine each term in turn:

1st term : 3r,(Qo") : 0r" (-|r"u' "u o')

- -f.roo' (arobo' + obaro') : -grob'arobo'


2ndterm : Srob'(ru"o'"t) oL

3rdterm - gabcob(eA'r) - g{bcob (oro'- gt"f o'Afr)


The derivative terms cancel, we then have

Q' Ao, t rou ,b'f o" o f AL - g'rou' e"f ob o" A[r. (9'68)
-
Using the anticommuting property of the ghost field, the last term can be written,
after relabelling indices, as

92 ,cbf o'ob AI
saec : (eobc ,cef - ,aec ,cbf ) ob o' AI
+
: -{, "'eo'f obo'A{, : -tr'"u rob'of oe ActL,

where to reach the last line, we have used the Jacobi identity

,abc ,cef _ ,aec ,cbf : _gbec ,acf


186 Quantum gauge theories 9.8

Finally we get

Q'Ao, trourb"f o'of A'u- o'A'r--0. (e.6e)


- {rrebrabcof
(ii) Fermion field
Q'rl, - QQgTooorb) : tgT' (Qo')
- igT"o" Qlr rl,

- isT" (-tt"""uo") V - isT"o"(igTbob4l. ell)


The second term can be shown to cancel the first term because it can be written as

: i$
rufoo ob ,lr - r"u'T'oo ob ,lr,
+ lr" ,
g2To Tb oo ob r,1r (g.71)

where we have used the fact that os anticommute with each other.
(iii) Ghost fields

e'po : o (-gu'o",) -- -tat'(eAl,) -- -;ap (a*oo - geob'ob A7)


: -; la'o - geob'a, ("u el)l . (e.72)

The right-hand side vanishes because of the equation of motion for the oo field as
implied by Cl-eqns (9.128) and (9.129) so that

Q2po (e.73)
-0.

Q",!;W\p5l'
To show that Q2oo
- 0:

(e.74)

Remark. Since we have only used the antisymmetric propeny of the structure
constant eob' of the SU(2) group, the same calculation will go through if we
replace s"b' by the more general structure constant f"b' which is also totally
antisymmetric.

9.8 BRST charges and physical states


Suppose an operator Q is nilpotent, i.e. it has the property Q2 :0 and commutes
with the Hamiltonian[Q, H]:0.
(a) Show that we can divide the eigenstates of F1 into three subspaces,

'|1t : {t; Qh + 01, 'l1z : {ltz; (tz - Qttrlwrth rlrr e Hr},

TIt: {Vt; Qh - 0but rlrz * Qrlrr}. (e.7s)


9.8 BRSZ charges and physical states 187

(b) Show that the scalar product between any two states inHz is zero:

(tbz"l{rzil :0 if {rzo, thza e Hz. (e.76)

This implies that the states in?12 all have zero noffn.
(c) Show that the scalar product between states in'Jlz and states inHt is also zero.

Remark. If we select the physical states by imposing the condition

QlVpny,) :0 (e.77)

then the physical state is generally of the form

llrpny,) : lVi + lthil where lrlr:,) e'l1z,ltbil l1z.


The results in (b) and (c) will imply that

W'rn,lrbpny,l - (lhil + W;t) f tt:l + l{rzD - llriltzl. (9.78)

This means that the zero norrn states in'llz will not contribute to physical matrix
elements and all important physics are contained in the space '112. The presence
of 'l1r and'l1z is to maintain the Lorcntz and gauge invariance. This is exactly
analogous to the Gupta-Bleuer quantization formalism of QED.

Solution to Problem 9.8


(a) We can always separate the eigenstates of F1 into two categories: (i ) QlL + 0
and (ii) Qrh :0. The first category (r) is just the space Ht.In the category
(ii ), we have two possibilities: it can be written in the form ry'
: (this
- Qth', so that
QrL Q2rb' - 0 corresponds to spac e'l1z), or it cannot be written as Q(/
but has the property that QV : 0 (this corresponds to spac e '11).

(b) (t\z"lVzu) : (VnlQlVzil : o (e.7e)

because

Qlrhzil:o and lrlrz)-Qlrlrnl. (e.80)

(c) (tbz"ltzu) : (VnlQl{zu) :0. (e.81)


10 Ouantum chromodynamics

10.1 Colour factors in OCD loops


In QCD loop calculations we often encounter the some SU,(3) group theoretical
factors. In this problem you will be asked to calculate such factors for the general
case SU(ru) rather than the n
- 3 special situation of three colours.
In the quark loop diagram of Fig. 10.1(a), we have the trace factor for the
quadratic product of

rr (rfrr!) - b(F)6"u (10.1)

where a : I,2, . . . , (n'- 1) and Zf stands for the SU(n) generator in the rep-
resentation R. For the present case of the quark loop, R is the fundamental repre-
sentation F,
\a
-o-n (r0.2)
'o - T,
with {),o } being the usual n x n hermitian traceless matrices, and the above trace
becomes

Ti,j (T),,(+),,:)u"u (10.3)

where i - I,2, . .. , n. Thus the trace factor tz@) - | normalizes the (bare) QCD
coupling g.

(a)

(b)

(c)

Ftc. 10.1. Quark and gluon loops in QCD.


10.1 Colourfactors in QCD loops lg9
In the quark self-energy diagram of Fig. 10.1(b), we encounter the group theo-
retical factor of

D,g;rfr),i - cz(F)lii (10.4)

where CzG) is the eigenvalue of the quadratic Casimir operator in the represen-
tation R. For the present case of quarks in the fundamental representation, we
have

ID,rx'xo),i: c2@)3ii. (10.s)

[Here we are following the more commonly used notation of Cz(F), rather than
sz(V) as in Cl-text.l
In the gluon loop of Fig. 10. I (c), the sum over colour indices can be represented
either as a trace, like eqn (10.1),

rr (r!rf) : b(A)6u,, (r0.6)

or as a Casimir operator, like eqn (10.4),

cz(A)6u,, (r0.7)
+exr7)0,:
where Tft is the generator in the adjoint representation A, as is appropriate for the
gluon gauge field,

(ril * : cobc (10.8)

where Co6, is the structure constant of the SU(n) algebra. Since Co6, is totally
antisymmetric, it is clear that the above two expressions are equivalent:

tz(A) _ Cz(A). (10.e)

(a) Show that for SU(n ) the value for Cz(A) is

l^
Cz(F):2n@'-l). (10.10)

(b) Show that

Cz(A) n (10.11)
-
which is denoted by tz(V) in Cl-text.

Solution to Problem 10.1


(a) From the identity Cl-eqn (4.134),

Ttr,r),; (L)rt - z (d,,a,o - !u,,uo,) , (r0.r2)


190 Quantum chromodynamics l0.l
we get

l{x.x"),, : f {r ) t1 (L), - z (t,, t, o - ! t, i to,) uro


/ I \ 2(n2-lt ''
- 2
(ndir - ;6,,) : '' ; U,,. (10.13)

Thus

+Vfrfl,i- iIt^ oL)ii:#u,.


(ro.r4)

Namely,

cz@):
*@2 - r\. (lo.ls)

(b) From the SU(n) Lie algebra

f L" : ic'b,+
lT'-,?l)
(10.16)
2

and the normalizationTr(),oLa) :26o6,wa can write

goU,
- -i7Tr(),,[),", Lil). (10.17)

Then we have

gacd gbcd ),,1)Tr(),afLo, L,l)


- -:Tr()"al)"6,
I6
: -GI Qa)ii[Lu,\,]it(La)alL",L,\r. (10.18)

Using the identity (10.12),we get

gacd gbcd - L,l[Lo, tr.]) - ),,1)Tr(U,o, l.l). (10.19)


,Tr($.u, ;Tr(p'6,
The second term vanishes because Tr(lA, B]) : 0 for any two matrices A and B.
Thus

gacd gbcd
- -18 Trl(),il,, - )",L) (LoL, - ),,x")l

: - gI Tr ()"fi",)"")", - )"6)"oL.L, - )"6),,)")"o

| )")")")") . (10.20)
I0.2 Running gauge coupling in two-loop I9I
The first term can be calculated as

1st term
- - ()")ii()")irQ')u(L,)u
Tr().uL..LoL,)
/ I \
- (la)ir Q')u2 (6rrao, - ;SioS,,)
-/

- 2Tr(La)Tr(),") - ?rr(^oLo) : -!uou, Qo.2l)


nn
and the second term is

2nd term : Tr(Lil"o),,L,)


- (L)ii(L,)ir(L,)a(1.)l,i
(L) ii (L,) i*2 ( a,,&,- lro,u,,)
- \n/

- 2nrr().uL) - ?r,1 r^,y - 4 (, - I) u", .(10.22)

Similarly,
/ 1\ 4
3rdterm-4(n-' la,r. 4thterm--16oa. 0,0.23)
\ n/ n

We then have

gacd sbcd : -f,rl-: - 4 (" - I)1t,, : ,t;ab e0.24)

Namely,

Cz@) : tz(A)
- n. (10.25)

10.2 Running gauge coupling in two-loop


In QCD, the p-function in two-loop is of the form,

flG) - -fl0s3 - hss + ... (t0.26)

where

aPo: I /.. 2..\ t


/.^^ 38 \
G"y(t' - i*r)
and Fr: @"y(r02 -;rt) (0z7)

Show that the effective coupling constant g defined by (with , : Lln Q2 /tt')
dEG't)
: flG) with g(g, o)
- g (10.28)
dt
can be written as

" I f Llnln(oz/L2\
\-, ' r.-.t I
6;z__tl_,.
- flotn(e2lt2) tn(ez/L\ ' I
(10.29)
L' pA
192 Quantum chromodynamics I0.2

Solution to Problem 10.2


The effective coupling is defined by the equation

da2(
; - flG): -fl083 - FGs.
(10.30)

Let us introduce .1, - 92, then


il'
; - -z(Foxz + fr'L3) (lo.3l)

Io'---;Ir:'#uir (10.32)

or

t--:l-h.h^^#ulur', (r033)

Write , : lIn Q2 /p2, then we get

notne2 - rotnp2 : i -; -# ['" @#@ -n@'+-f,'*)l


Combining all the Q2 independent terms, we define the scale parameter A by

+ fts')
Bstnt\2 = flornt", - + +
g2 ' +rn@o
fo"' g2
(10.34)

so that we have a simpler relation,

, _ +rn(fl' +_f,E').
02= -r, (ro.3s)
Fol"
fu flo E,
We can solve this for E'by iteration. To lowest order in g,

(;), -porn# (1036)

The second factor in eqn (10.35) can be approximated as

^@#@- (r, p)- (0,


rn . rn + #) pzorn

- mmff ror r*e" fr. (10.37)

To next order in g, we have

; : p,t ttt . #nm fi: (r, t" #)lt .


h'##l
10.3 Cross-section for three-jet events 193

of

=2 _
I
l- 6r ln ln( e,l ty2)1
E': ffitno,/$Lt-tsi@ffi) (ro'38)

10.3 Cross-section for three-jet events


Consider the process (see Peskin and Schroeder 1995 for further discussion)

e+ (p') * e- (p) + Q(kt) + A(n) + s(k). (10.39)

(a) Show that the three-body phase space can be written as

e -| #h#'h12o464(q - k1 - kz - kz)
q2
".. dxz
- fro, J[ dxr )-. (lo.4o)

where h :2kr.q/q2,with i - I,2,3 and4p: pp * p'*.Find,the region of


integration for x1 and x2, for the case where quarks are massless but gluon has a
mass p.
(b) Show that the amplitude for this process can be written as

M- e2 sIu(p')ypu(p)l@+^[u(kt)N,t u(k2)leL (kz)eq (10.41)

with the fractional charge of the quark Qu : 213 and Qa : -l 13, etc., and

lYxr' : Y^# tl * r, Qo'42)


rYr"
^lr^
(c) Show that, in the limit of massless quarks and gluon, the differential cross-
section can be written as

d2o 4ru2^^.t/d,\[ xl+x] 1


d,d.r: 3, \oa \2" )Ln -;;n=J (ro"t3)

where s (p I p')2.
-
(d) Show that in the integration over 11 and x2 there are infrared divergences as
p --> 0, corresponding to configurations where the gluon is collinear with the
quarks, q or Q-the collinear divergence.

Solution to Problem 10.3


(a) Integrating over the three-dimensional 6-function, we get

f d3k1d3k.'
e - J Qil4ra,'hr.t5(qo - a1 - o;z- ,,z)' {J0'44)
t94 Quantum chromodynamics 10.3

Choose a frame such that g : 0, which gives kr * kz * k: : 0. The gluon energy


can be written as

0)3 : lp' +(k,)t]t/2 - b"' +(kr * k )'lt'' (10.4s)

,?: LP +tcl +t<]*2k&zcosg + c4do4


- hkzd.(cosl) (10.46)

with 0 the angle between k1 and k2. Also,

d3 k1 d3 k2
- (4r)(2r) d(cos e)k? dk&3 ab - 8r2 az da3a1 da1o2 dot2
(r0.47)

where we have used ar; doI - ki dki. It then follows that

8n2 f at daxo;r dara2do2 ^.


P: eo)t J ffi6(qo-ut-az-utt)
Iftf
- ,2ot J O.t6(q - 01 - {D2 - oot)dar dotz - 32", J
dot doz

For q 0, we have
-
2kr .o 2a' 4
xr:--7q' + dx1dx2-qo,dro1da2 (10.48)
Qo
and

e: #i I a,,a,, (10.4e)

From o)r : (*2 +k!1r tz , the minimum for x js 2m / qs which goes to 0 for rn - 0.
Similarly, ar3 has a minimum at k3 - kr * kz
- 0, which implies that a1 : o)2,
and recall, for a massive gluon, the minimum is zt a4 - tr^c. It is easy to see that this
configuration gives a maximum value for a1, er a)2. From energy conservation,
the maximum for a,l1 is

eo : @r * roz * roz - l-t l2q or 2o\ - eo - l-t

Xt:
u .p
I _. (r0.50)
-
4o Jq'
Thus the range of integration is

p p
0<-r1 <1 /'r' O<x2<l (10.51)
\/ q' \/ q'
10.3 Cross-sectionfor three-jet events 195

(b)

kt+ kt

kz+ kt

FIc. 10.2. Gluon bremsstrahlung in e+ e- annihilation.

From the Feynman rule, the amplitude is given by

M - -i tu (p' ) (- ie y ) u ill Cr+ u)ln Q,,1 1- i g y^7


(

i
X ;;--------;- (-ieyp)u(k) + u(k)(-ieyp)
#r-f#t'
'l
-i
" y; y r(-isY)u(k) )Qne^(fu)
o2o
- ; li ( p' ) v' u ( p\lu (k ) ltiuu (k)le^ (k) Qq

with
-1 I
/\t t, : YL t Yp (10.s2)
tl n ll3Yp ll3 + hY*
(c) We can write

^ ft*#3 l/z*#t
/\).tt -Y), t Yu (10.s3)
&t + kyYp 11rn OyYx.
The denominators can be simplified:
(h -f kt)' : (q - k)2 -- q2 - 2kz.e - qz(I - xz) (10.s4)
and

(kz*k)2:q2(I-x). (10.ss)
Then

:
A,,,r,
-;I\I-xz).yxQl t + tl )Yr" q'(l-xr)
The differential cross-section is then

. 1 /t=-,..,\ d.3k, d.3kz d.3kz


do --
4(p t) (a L I'vt ) e"F^, e"fr*6r^
x (2n)a6a(q -h- kz - kz)

:#(; with e:#ila''a*' (ros7)


;wP)o
196 Quantum chromodynamics 10.3

The calculation of the matrix element is straightforward but tedious. After using
the relation

T'^
g,k)eB(s, k:)
- -sxn.ry (10.s8)

(the kzxktB term in the photon polarization sum will not contribute because of
gauge invariance) we have

: I(+{)"r'v,ilv.)rr(#
i I tMt2 t'vxpttzA,,)

: "Vfut*G&u
o4 o2
(10.59)

where

lt"u: rr (il'yril y,)- 4 (p'rp, -f pup', - Br",p . p') (10.60)

G&' : rr (# rlvxptl z4v,x). (10.61)

Writing the three-body phase space as

e:# Io.'dxz- loor, Q0.62)

we have

I o*IptMt2 : Ktpu I o,, or, with - :


#. (10.63)

The gauge invariance implies that

q*Gpu -- 0, quGp' :0, eplP' :0, qrlP'


- 0. (10.64)

The tensor G&', after the integration, can depend only on ep. Thus,

I "'"00, : (r,, -flG(q') (10'6s)

or

G(qz) :: GP'g*udpt (10.66)


I
and

,r, G,u dp, : It", (r* - G(q2) : tr,Bu'I ,"u top dpt.
I 9 I
From eqn (10.60) we have

I,rug\' - -8p' .p - -4q'. (10.67)


10.3 Cross-section for three-jet events 197

The quark tensor is somewhat complicated:

G - G"fl gaf : Tr (# tltxrtl rlt*^)


1-tt.t^rrl-\., I
-'l*' -l
:rrlil,l-
tq\l r -t#)Yp+
nrn - *rrvuQlz+tl)rxl
-.r)Yx(#
x # zt-;+
Lq,
yu Ql r * tl )y^ *
ri^yL(tt z + rt *,)l
(10.68)

The trace in the term containing (l - x)2 in the denominator is


TrUl ryx(# r i k )ypk zyp (tl r * tl )y^I
- -2Trt# r(# r * Il )y*# zyP(# t + # )I
4rr(# Jl ll z# ) : I6(2(kt . h)&z . k))
-
- 8qa(I - xr)(l - x) (10.69)

where we have set k2, 0 and used the relations


-
1,11.o2o2
krkt - kz'h -i(l -xr).
1(kt]-kt)'- r(q -k)'-;(t -x),
It is clear that the trace in the term containing (l - *)2 in the denominator is
exactly the same as above. The trace in the term containing (I - x)(I - xr) in the
denominator is

-rr{k vx(tl t * tl )yptl * # )yPl


2y)' (tl z

- 2rrt# Jl zyp(k t + k )(k z * Il )yPl


8(ftr * k) . (kz * fu)4k1 . k2
-
- * kr .kt * kt .or+(1 - x:)
32(kr .kz

- 8q4[(l - x) + (1 - x) * (l - xr)](l - xr)

-8qa(l -x) - -8qa(I -l1-xz) (10.70)

where we have used

-rr * -r: *-r: -


2(h -f kz
i k) ' q : z'
t. (10'71)
qz -'n='
qz
Putting all these together, we get

G:8{(l-xr) *(-x) _ 2(l-xr-x) |


[(l -xz) (l -xr) (l -xr)(t -x)l
: ,1, .{(t-x)2+(t-*z)2- z(r-xr-x)l
(1 - xr)(l - x)"
8@? + x3)
(r0.72)
(1 -x1)(1 -x)'
198 Quantum chromodynamics I0.4

The differential cross-section is then

do - +#@q2): I ##6dx1dx2#nl
2a2u2 ^ f ztx? + xil
_do; J (r-,)(r_xz)
dx, dx,. (10.73)

Remark. There should also be a colour factor of

Tn (++):'u+6..:4 (r0.74)

We then have

8a2a! n, 8@? + x3)


d2o
:#Q3q' 'q (I xr)(l xz)
(10.7s)
dxt dxz - -
(d) The range of integration is

0<xr,x2 1l-+. (10.76)


\/ q'
Thusaspc -+ 0,theupperlimitapproaches l andtheintegrationsoverrl and x2are
infrared divergent. [For the case p I 0, this gives terms of the form (ln pLz /q\2].
The region xt --+ 1 corresponds to
configuration where the quark q has maximum
a
energy while Q and gluon both are moving in the same direction, i.e.4 and gluon
are collinear.

10,4 Operator-product expansion of two currents


Consider the operator of the form

t*u(e) - | oo*eiq''T(Jr"(x)/,(o)) (10.77)

where J r(x) is the electromagnetic current. The operator-product expansion can


be written in the symbolic form as

tr,(e) -I cf' (ilot (o) (10.78)

where Cl' @)s are the Fourier transform of the Wilson coefficient in the coordinate
space and the local operator has the general form of

Ourttz...&" (0) (10.79)

which is completely symmetric and traceless in (Upz . . . tt").The dimension of


the operator is d, and spin is n.
10.4 Operator-product expansion of two currents r99

(a) From dimensional analysis, show that we can write the forward matrix ele-
ments as

(p, sl ol,r$2"'u" (0)lp, sl : Md-"-'s1p*' pt" ...pp'fanv (10.80)

(p, sl o|,'t' " (0)lp, s) : Md-n-251tpr pt" ... pp'lant (10.81)

where subscript V means that the operator of:t'z"'p" (0) has the same parity as the
product of n polar vectors, e.g. xt', xt'z . . . ytrn, while A denotes the axial type of
operator Ol,r[2" u" (0) which has the same parity as the product of one axial vector
and (n - 1) polar vectors. The state lp, s) has momentum pp (p2
polarization is described by apolanzation vector sp and is normalized as
- M2) and the
(p' , t'l p ' sl : 2E p6rr, (2n)3 63 (p - p'). (10.82)

The operation S[. . . ] projects out the completely symmetric traceless components.
Also ary , d,r1 ?ta dimensionless constants.
(b) Show that the coresponding Wilson coefficients, which give the leading con-
tribution in the scaling limit, have the structure

cf'ur "" (q) - -gr",sI qp'qp, . . . qt""](-q')('-d-n)/2cn(g) (10.93)

for the F1 structure function (see Cl-Chapter 7 for the definition),

Ct'r""'p, @) gttttt Sfqutnu+ . . .qt'nl(-qr)(o-d-")/2C2,(g) (10.84)


- gvtL2

for the F2 structure function, and

C{'u'*" (q) - s&vttru gleoer, . . .qr"f eq\Q-d-")/2Cy(g) (10.95)

for the 91 spin-dependent structure function. Ci n (g)s are dimensionless numbers


depending only on the coupling constant g.
(c) Show that

cl"' *' (q)(p, rtof,'#' *'(0)tp, s) : -sp' (:)' (#)(2-d+n\/2 (10.86)

ct't"' r"
@)(p, rlof,V' *'(o)lp, rl - # (:)"- (#)(-d+n)/2
(10.87)

Solution to Problem 10.4


(a) From the normalization of state
(p, slp',,r')
- 1zn;333(p - p')zEp (10.88)

we see that the physical state lp, s) has dimension -1. Thus the matrix element
(p, sl Ol,r&2"'u" (0)lp, s) has dimension (d - 2). Since pp is the only polar vector
200 Quantum chromodynamics 10.4

this matrix element can depend on, the Lorentzindices &r . - - F, in Oa,y aretaken
up by pttt pttt " ' pt'". The term of the form gt'Ltlrz pP3 ptt4 . . . pp^ is not traceless
and will give a non-leading term in the scaling limit. From these considerations,
we see that the general structure for the matrix element is

(p, s I Ot)#'z,p' (0) lp, sl : Md-n-z 51Our pt" ... pz")unv (r0.8e)

where ary is a dimensionless constant. The matrix elements of the axial operators
can be obtained similarly.
(b) The Wilson coefficient of the operator Ot,t#2"'P" (0) must have the Lorentz
structure Crr*r*r...r". Since Jr(x) has dimension three, we see that trr(4) has
dimension two. Sinc" O!)#, ''(0) has dimension d, the Wilson coefficient
Crrr,rr...*, (q) will have dimension (2 - d). The structure functions I4/1 , W2, arrd
Gr are defined by
1

#ro,slt,lp,s): -gp,Tr* #Tzl r,,opr"qf Gr+... '(10.90)

and

I
W; - -7t Im 4 and Sr: Im G1. (10.e1)

Thus for structure function Wr, the Wilson coefficient is of the form

.10.92)
Cl't"r"'u" @)
- -gp' Slqpt qt-12 . . . qt"^leq\Q-d-")/2Ctn(g)
where C1n is a dimensionless constant and can depend only on the coupling con-
stant g.
Similarly, for the structure function W2, the Wilson coefficient is of the form

Cl'*'"'*" (q) - {grr' gpp2 Sfqp3qt'o . . .qp"l(-q21(a-d-n)/2

* permutationslCz,G). (10.e3)

For the spin-dependent structure function G1 we have

C{uur" " (q) - lepvttru SIqoq"''' qt")(-q')('-d-n)/2


* permutations I C3, (g). (10.e4)

(c) Combining the results in (a) and (b), we have

Cl'ur"'*, (q)(p, sl Of:t,"'tL" (0)lp, s)

_ _grrr,S[ qtrt qtl2 . . . qt""](_qr)(r-d-n)12 tr4d-n-z S[,pp, ppz . . . pp,fdrvCn

: -st"lf+l' (#)Q-d+n)/2 * trace *'-'] dnvctn

: -,.t'u (#)(2-d.tn'/2 +' (r0 e5)


[(+)' fo,,c,,
10.5 Calculating Wilson cofficients 201

Thus for twist-2 operator (recall twist is difference of spin and dimension, d
- n),
we have

Wr(q2, v) : Frr,, q')- )- f1)' dnvCtn. (10.e6)


7\x/
Similarly,
"u"
cl'ur"'u" (q)(p, sl o!,rfz (0)lp, s)
t2
-- # (:)"' (#)(-d+n) a,v C2n

10.5 Calculating Wilson coefficients


Since the Wilson coefficients are independent of processes, we can choose. some
simple external physical states, e.g. free quarks, to calculate these c-number coef-
ficients.
The quark Compton scattering to lowest order in cu, is given by the diagrams in
Fig. 10.3.
(a) From these diagrams, compute for massless quarks the amplitude

Mrr, : (p, sltpv@)lp, sl (t0.e1)

where

tr,(4)
- | ao * eiq'' T (J r,(r)/,(o)). (10.e8)

(b) For the operator-product expansion in the form

tr,(e) - Ii cf"(q)ot(o), (10.99)

there are two sets of flavour-singlet twist-2 operators,

offt,ttz"'u'(x)
- ;+ {4G)yp, Dt', ... Dt",q(x) * permurations}
ofti,!2"'u'(x)
- :+ {4@)yp'ysDp, ... Dp,q(x) * permutations}.

p p pzp
Flc. 10.3. Quark Compton scattering.
202 Quantum chromodynamics 10.5

Show that the matrix elements of these operators between quark states are given,
to lowest order in ar, by

(q(p,ilof:!' :
'*"lq(p,s)) (p*' ...pt")
(q(p, s)lo?',!' 'u"lq(p,s)) : h(p" ... pt'") (10.100)

where h is the helicity of the quark state q(p, s) and is related to the polarization by

u(p, s)ypysu(p, s) - rp : 2hpp. (10.101)

(c) If we write the operator-product expansion as

tr,(e): (qu' ...qt")ficli,'r'


{ [-r"
+(s'p' qth''' qt") +permutatio ns]off:!' *"
svt'2
#y;cli,i)
*leu'u" eoep' . . . qt'n + p".-ututionr] oX',!' *"1,
#ry')
compute the Wilson coefficients to lowest order in a, (i.e. in a free field theory).

Solution to Problem 10.5


(a) From the Feynman diagrams in Fig. 10.3 we can write the amplitude

Mt', : iu(p,*rffiy,u(p, s) * iu(p, s)y,'Hy,u(p, s).


\P q)' -
( Lo.roz)

We now want to express this in terms of q2 and co 2p .q 1. We can


- /(-q2) -
expand the denominator as follows:

1 1 1 ls n
(10.103)
(p -f q)2 2p .q * q2 q20 - @) q' ?:o
---/(t).

Similarly,

(p - q)2- 1i,-
--f-- t)n.n. (10.104)
q' f,:o

In the first term we have

u(p, s)yp(il + d)yuu(p, s) : u(p, s)lypdy, * Zyup,fu(p, s). (10.105)

Using the identity Cl-eqn (A.17), we get

Ypdy, : qo (8poYu * SroYt, - SprYo + ie*roxYLYs)- (10.106)


10.5 Calculating Wilson cofficients 203

From the properties of Dirac spinors, we get

u(p, s)yru(p, s) - 2pp, u(p, s)ypysu(p, s) - sp : 2pt"h, (10.107)

where s, and h arc the polarization and helicity of the quark state. Then
u(p, s)yp@ + d)y,u(p, s)
- 4ppP, * 2ppeu * 2qrpu - 2gp,@ ' q)
| 2ispvaxq" p^h (10.108)

and the first term in Mu, is


oo
-') *
Ml)l - -r n: o l2pup, * pt q, * qup, - Bpu@'q) ispvate" p^hl.
=D,r" (1o. ro9)

Toobtainthesecondtermfrom MflAV thesubstitution p + v, q --+ -q,


_') &
M;:) - ;D}_1)"a"
Lt n:o l2p*p, - ppe, - Q*p, * sr,,@'q) * ispvaxQ" p^h].

(10.110)

The total amplitude is then


oo oo

Mt", : -2t + (- r)"1a" t Bp,@ .q) I,t - (- r)'la"


ilroro, Ett
* i1tvuxQ" o^ttl + (- |)'la' + " I (10.111)
I
(b) Considerthe simplest case r? :
1, where wehave the operator O(,,
- l|V' U.
To the zeroth order cyro, the free field theory limit, we can expand q(x) as

ot
q(x) (p,s)sin'*u(p. s)].
- J[ - ' -t,-lb(p, s)e-iP'r 11(p, s) a d+
ltziltznrl"'
(10.1r2)
It is then easy to see that

(q(p, s) I o#.s lq@, s)) : l\a{n, s)l4yp qlq@, s))

: Lr@, s)yPu(p, s) : pP. (10.113)

Also to ordercuf , the covariant derivative D * is the same as the usual derivative 0,

and each gives a factor of pp. Thus we get

(q(p, '*'lq(p, s)) : (p" "'


oloff.j' pt""). (10.114)

For the axial vector current it is easy to see that

l\a{n, s)l4yPvsqlq@,s)) : ia@, s)yPvsu(p, s) - hpp (10.115)

and

\q(p,9lol',!' 'u"lq(p,s)) : h(p'' "' pt"). (10.1 16)


204 Quantum chromodynamics 10.5

(c) Thking the quark matrix element of tp,(q),we get

(q(p, s)Vp,@)lq(p, s))

: l-r
l-gu,(qr, qu, .. . qu,),
n(ns)
r- ( -
-qJt'J
i Bt,r,rsvp,qu3 ...qt'n +permutationr] u" ,pp,... ppn

*lru,r,o(qo qP' . . "h-cli.)'


.qt")JryCt|" +permutatio nfror, pp,. . . pp,

: _ Bpucffl,(;)' _t pup,(i)'-'
iAr;,|
-'
* hep,ope" ,t (:) i6c';" . .(ro.n7)
Comparing this with the Compton amplitude given in (a), we see that

cli,'r)
- -2lr - (-1)'1, cli,') : -2lr + (-r)'1,
cf') - _2t1 + (-l)'1. (1o.l 18)
11 Electroweak theory

11 .1 Chiral spinors and helicity states


The Dirac spinor in momentum space can be written as,

u(p,*):J2*( ;, \r. (r1.1)


\n+^/
where (o . 0)X+ - IX* with p : p/lpl. Show that the left-handed and righr
handed spinors given by

uilD - ltt - y)u(p, -), un@) - l{t + y)u(p, *) (tt.2)

are eigenstates of the helicity operator i. : s . 0 in the massless limit, where the
spin operator is of the form

': + (; :) (1r 3)

Note that the same calculation should also show that the other two combinations

lo + v)u(p, -), Lo - v)u(p, *) (11.4)

are identically zero in the same limit.

Solution to Problem 11.1


In the standard representation, we have

,'-(.t/o o/
l) ('.s)

Thus

ur@) -l{r-vs)u(p,-)- I (-t' l')( *),


rr/ I \ _r(n+p\11\
:_,(1
i(_,,',l ( +)x-:; (.t/ ('-,)r-
/\El

- (\-1l',
t,
) r_ (11.6)
206 Electroweak theory II.2
where we have used E - p for the massless particle. Similarly,
un@) - +(t * y)u(p,*): (l) r. (r 1.7)

Then

Au,@):r(o
p
: r) (_"),
: + (-t, ," ' o)x- : -iu,(p). (11.8)
)
Similarly, we have

XuR@): l.uo@). . (11.e)

11 .2 The polarization vector for a fermion


For a particle described by a spinor u(p,,1"), we can define the polarization four-
vector s
u(p,),) as

sr(p,
- *o(p, ))y,ysu(p, )'). (11.10)
^l
(a) Show that

s.p -0. (11.11)

(b) Calculate s, for the particle at rest (p 0), with


-
x+: (;)' x-: (?) 0t tz)
(c) Show that

s2: -1. (11.13)

(d) Suppose for a particle at rest the polarization vector is given by

s& -- (0, Z) with T2 : L (ll.l4)


Show that in the frame where the particle moves with momentum p, the spin vector
sP is given by

o ?'P P(?'P)
m (E -f m)m
lI.2 The polarization vector .for a.fermion 207

Solution to Problem ll.2


(a) Through a simple application of the Dirac equation, we have

s.p : lof o, x)il ysu(p, t) : lrfo, L)y5u(p, L) (11.16)


zm /.

or, alternatively,

s.p : : -
Z*il(p,L)ys?il)u(p,l) Zil(p,L)y5u(p,L). Ol.l7)
Thuss.p-0.
(b) For a parricle at resr, where we have u(p,L) : J2*(tO)^^,0-:
(m,0,0, 0), and s . p- 0, we get

so
- 0 (11.18)

and

"
: *rro, ))yy5u(0, r.)
x],0,', (: ;) (? ;) (l)'^ : xlox^ (1, le)

Thussl:J2-0and

,r-[t.
" 1o"*.
forx_
(r1.zo)
[-t
This means s is in the direction of the spin. In this simple frame we have

sP - (0,0,0, +1), s2 : _7. (ll.2l)


(c) The spin vector

su(p,^,
- *r(p, L)y,ysu(p, L) (11.22)

transforms as a four-vector under Lorentz transformations. Thus s2 : spsu is a


Lorentz scalar and s2 - -1 in all frames.
(d) Since ry and p are the only vectors in the problem, we can write

s : ary + bp, a and b are constants. (11.23)

Since we are given s : ? when the particle is at rest at p : 0, we must have a - I.


From s ' p :0, we get

1l
ro:E@+up!.p- EQt.p+bpz) (1r.24)
208 Electroweak theorv Il.3
and the condition s2 : -l can now be written as

,o'-r'-rfr- @+bp)z --1. (1t.25)

Using eqn (11.24), this leads to

^ rry.p+bp')'-(n+bp)2-t
1
(11.26)
E2'
or

b'(E' - m2)m2 + zb(tt .p)m2 - (rt . p)2 : 0, (11.27)

or

lm(E - m)b + (n-Dltm(E *m)b - (n. p)l :0. (11.28)

This gives the solution

,_ (? .p)
b (tt.2e)
m@fd.
(The other solution does not go to zero as p + 0.) Thus we have

p(tr.p)
s:?+ (ll.3o)
@+*)^.
11.3 The pion decay rate and fr
The decay nt --> p+ + u, is described by the effective Lagrangian for the four-
fermion interaction

Gr
^
Lwk : -icos g,
lny, e - Ddllurre - y)vr,l. (11.31)

(a) Show that

(0luyrdln*(p)) - O (11.32)

because parity is conserved in the strong interaction.


(b) Show that the general form of the axial vector current is given by

\\luy*y5dlo* (p)) - iJlfo pp (11.33)

where /, is the pion decay constant.


(c) Calculate the decay rate for Tr* + p.+ + vu and use the measured lifetime
rjr - 2.6 x 10-8 s to determine the constant /r.
(d) Show that as a consequence of the V-A theory, the amplitude for the decay
ttt --> l-t+vp is proportional to tlrp., and to m" fot r* + e*vr.
11.3 The pion decay rate and .f, 209

Solution to Problem 11.3


(a) The parity conservation of the strong interaction implies that

P(uyrd)P-t - uypd,, Plo* (il1 : -ln+ (-p)). (tl.34)


Thus the matrix element

(\luy *dlr* (p) ) _ (\lp-t p @y *d)p-t p lo * (p))


- -(\luyudln+ (-p)). (11.3s)

This means that for the time component, we have

(\luysdln* (p))
- - \\luy0dlr+ (-p)) (11.36)

or

(\luyodln*(p
- 0)) :0. (rr.37)

For the spatial components, we get

(\lu ydln+(p))
- \\lu ydln+(-p)). (1 1.38)

This matrix element is a three-vector under rotation and the only three-vector this
can depend on is p, which changes sign under parity. Thus

(\luydln+(p)) - 0. (11.3e)

In essence, this argument simply says that since z+ is a pseudoscalar, the matrix
element of vector current (0luyrdln+(p)) is an axial-vector while the only vector
it can depend ofr, pp., is a polar vector. Therefore, this matrix element must vanish.
(b) Using the same argument, we see that (\luy*ysdlr+ (p)) is a polar vector and
has to be proportional to pp:

(\luyry5dlo* (p)) iJ2fo pp. (11.40)


-
(c) The matrix element for the decay is of the form

G,
M _ _t g" (\luyuy5dlr+ @))u(k)yp (1 - y)u(k)
icos
-9,f"
T
cos0. puu(k)yq(l - y)u(k1)

o'!:
- \/2
cosg. m,u(k)(7 - y)u(k) (11.41)

where p : kr * kz, with k2 being the momentum of the muon. Note that this
matrix element is proportional to the lepton mass mr. The decay rate is then
210 Electroweaktheory 11.3

given by

t- *; I rr,)464(p - k1 - k)#h#,\w? (r 42)

and

T IMP - G',f3
spin
cosz e,mzuriQl2 - mr)(t - y)th (t * ys)l

- GzF f: cos2 e,*'*8(kt . k2)

- 4G2ef3 cosz e"mf,(*? - *?,) (11.43)

where we have use the relation

2(h.kz): (kr * k)2 -k? -k3- *2" -*'z, (11.44)

The phase space can be calculated easily to yield

e : pfl4 p - k1 - kr', #;,


I 64 (
r*4+,
:# lur*,-E1 -n,>ffih,n,. (tt.4s)

For the pion at rest

po : tltn, kr * kz : 0, d.3k, - +nt<2ra4 : 4r E?dEt, QL46)

Ez : (*', *k|)tP : (*'u * t?)t'' , (tr.47)

and

o: ) | ul*" - E1 - (*',* t?|'')t#. (11.48)

Let x - E1 + (mzu + EItlZ, then


ttdtt
dx - dEt *' =,=
(*r, + E?)r/,

=(^', dEt ln,+(*rutt?)','f:+


* E?)t/' t
ct.4s)

and

f dx lE'
p: ,I l6(m"-x)Er-: (11.50)
+7T J X ^ -.
+Jf mft
ll.3 The pion decay rate and f, 2tr
For x : mr, we get tltv : Er * (*'u * Ellttz or E1 @? - mzr) /2mo. The
phase space is then

p- t *7-*ru (11.s1)
4n 2mn

The decay rate is

p @i: *'r)
- -1-
2m, 8nm,
4f:Grrcosz e,*ru@r, - *rr)
: # r|*'u*o(' - e,
- : (rt.s2)
#)' "o,'

Substitute in the pion lifetime, the Fermi constant, and the Cabibbo angle,.etc.,
and we can deduce fo :0.66mo - 90MeV.
(d) In the V-A theory u, is left-handed and pr+ is right-handed. In the limit
ffi, : 0, p* has helicity f . Thus in the rest frame of n+, p,+ and u, come out
back-to-back and the total spin along the direction of p+is +1 (see Fig. 11.1).
However, z+ has spin zero. Thus this decay is forbidden in the limit ffip :0 and
the decay can proceed only if m, f 0, see eqn (11.41), in which case right-handed
p+ is not a pure helicity state.

f (z+ --> etv")


f(z+ --> 1t"+vp)
:(ft)'l (t(r - @?,/*T)'
r*?/*?)'
- 1.23 x 10-4. (1 1.53)
-
Thus, pions decay predominantly into muon leptons rather than electron leptons.

Remark. If we use the same analysis for the charm meson decays, the results are
very similar:

f (F+ -> r+vr)


f(P+ --> 1t+vp)
:(fr)'l (t(r- (mrul*,r))' 1m2r1m2r))2
(1 l.s4)
- ]=,,,

f (D+ --> r+vr)


f(O+ --> 1t+vp)
:(ft)'l (r(r - 1*'r/^2))' 1ml1m2;)2
(l r.ss)
- ]=,'
Note that leptonic decays of Ds are suppressed by sin2 e,.

lr* v

<-<-
FIc. 1 l.l. n --> 1t+v decay would be forbidden inthe mu: 0 limit.
212 Electroweak theory II.4
11 .4 Uniqueness of the standard model scalar potential
The usual SU(2)w x U(1)y scalar potential for the standard model is of the form

v (0 - - p.zor o + L@r il2 (1 1.56)

with

o: (rr:): (I) (ns.)


(a) In principle, one can also have an SU(2)w x U(l)y quartic invariant of the
form

V@) - A4(Qr til . @t t0. (11.58)

Show that this quartic term can be reduced to that in V (il of eqn (11.56).
(b) Show that another quartic term

vz@)
- LrD(oI{rbo) (orr"bQ)
a,b
(11.se)

is also reducible to that in V (0 of eqn (11.56).

Solution to Problem 11.4


(a) Writing out the components

@r til .@r til - I @iO) (0ru)l{'\,iGoyt (11.60)


j,k,t i, a

and using the identity

- z (tioa,, - )u,,uo,)
. (t l.6l )
.rr")i1ko)rt
we obtain (Qr til . @I til - (Qr O)'.
(b) We can use the identity in eqn (11.61) to derive
b
* (, ) *, (r"
u u
D(t' j (t " r ) r,t
a,b
rb ) t
- l{t
a,b
" ), ) r,n G ) n,

- z (d*'a,, - )u*u*), (u,,u*,, - )u^,u,,)


- + (ao,a ,, - )u,,uio - )sioa,, + Ia,idr)
- (56p16;i - 46$i,J. (t1.62)

Thus we have

L(ot ," ru o)(oI ," tu Q)


- @I il' . (11.63)
a,b
11.5 Electromagnetic and gauge couplings 2I3
Remark. Clearly we can generalize this result to the more general case of vector
representation in SU(n), e.g.

(Or L71,(0, xil - zl fOr O), - lfOt Of1 - 2(n - r) (OI 02 r.1.64)
Ln)n
by using the identity
/l\
.r^"r,,(Lo)*r-
z (a;ra ,, - !s,,60,) . (l 1.65)

11 .5 Electromagnetic and gauge couplings


In the more general case, the interaction of neutral gauge bosons can be written in
the form

Lu :D,r,Ji, Ai* (1f.66)


i:l

where g1,..., g.n&fagaugecouplingconstants, Jrtt,..., Jn& arevariousneutral


currents, and A', are the neutral gauge boson fields, which are gauge eigenstates.
Suppose Ai*is written in terms of mass eigenstates as

A'* :f t,,*; (11.67)


i:1
where S is an orthogonal matrix with property

Ii:l O, Sib : 6ob, f ,,"r,, - 6ii. (11.68)


a:l
(a) Show that the electric charge e is related to the gauge couplings by

Ie2 :- ,'- /9i)'


/ ,L r.6e)
fi\si
where cis are the coefficients of Jit'in the electromagnetic current: Jf,* _
Di ri Jf, orin terms of the charge operator, Q : Di ciYl with Yi : I d3xft.
(b) Use the result in eqn (11.69) to derive, in the standard model, the relation

e: B sinOsr. (11.70)

Solution to Problem 11.5


(a) The neutral current coupling can be rewritten as

Lu :D,r,Ji*
i:l
Air- Ii,a giJip siowf, :f a:l ",J"t'wi, el.lr)
214 Electroweaktheory 11.6

where ealfr
current, we get
- Lt gtJLS,,. For the case where Jf, : Jfi*, the electromagnetic

eJfi -fi:l ,,JLs,o. ( tt.lz)

Using Ji- :Di riJf we get

,Dc1lf
j
-fi:7 r,rls,o. Q1.73)

Identifying the coefficient of q (for a given i ), we get

c; : giSio or S,o : 9. (ll.i4)


8i
From the fact that S is a orthogonal matrix, I; 6id2 - 1, we get

or r(;)' - ) c'I 7s)

(b) For the specific


"D(;)':' x U(1) theory we have
case of the SU(2)

Lr,r: g ,'rA'r+{f ,nr. e1.76)

Namely, gt :I - +.We also nur",i: 1 and cz - +,because


and92

O-rr+Y, or Ji*:lt-+)t:,
Then the relation in eqn (11.75) becomes

111
: (It '77)
+
7 t, *'
Using g' : gtan?1ry, we get

+
e" - ltt
g'
* cot2 ey; or e: Bsingyz. (11.78)

11 .6 Fermion mass-matrix diagonalization


Suppose that the fermion mass matrix in the basis of left-handed and right-handed
fields is hermitian,

Lu : ,lr,rM,ilrin * h.c. MI - M. (1I.79)


In general, the eigenvalues of M obtained from a unitary transformation are not
always positive:

UMUr - Ma
-diag(m1,rn2,...,ffir) (11.80)

where rfl; ca{r be negative as well as positive.


II.7 An example of calculable mixing angles 2I5
(a) Show that one can choose an appropriate biunitary transformation to diag-
onahze M so that all diagonal elements are non-negative.
(b) If the mass matrix is real, show that the matrices in the biunitary transformation
can be chosen to be orthogonal matrices.

Solution to Problem 11.6


(a) For the cases where some of the rflis &ra negative in the diagonal matrix Ma -
U MUI, we can always find a diagonal matrix ,S, consisting of 1 or -1, such that
the product MaS is a positive matrix:

Mo:MdS>0' (11.81)

Then

Mo:MaS-UMUtS-UMVI (11..82)

with UtS - Vl . Since both U and S are unitary, V is also unitary. Then Ma is
in the form of a biunitary transformation. In this way we can make all fermion
masses non-negative. Clearly, this can also be done even when M is not hermitian.
(b) If M is real then M MI is real and symmetric and can be diagonalized by
orthogonal transformation :

S(MMr)s' - tt3 - (1 1.83)

Let us define

/ \
uo:l^'' I *o H-sMasr, (11.84)

\ mn/

then 11 is real and symmetric. Define T by T : H-r M,then

TTr _ H-t MMI (n-rrr _ H-t SUISr n-t - H-t H2H-t - t, (1 1.95)

i.e. T is orthogonal. We have

M - HT - SMaSTT or Ma - 6r MTS)
- Sr MR (11.86)

where R ZS, which is also an orthogonal matrix.


-
1 .7 An example of calculable mixing angles
The properties of the mass matrix can be translated into relations between mass
values and mixing angles. Here is an illustrative example of such a model. Consider
2r6 Electroweak theory 11.8

a simple 2 x 2 hermitian fermion mass matrix of the form

u-(o- ?).
\o* b )'
(r.87)

Show that the mixing angle 0 which characterizes the 2 x 2 unitary matrix which
diagonalizes M is related to the mass eigenvalues by

l7l I
tan9 (1 1.88)
- lll2

Solution to Problem l1..7


The mass matrix can be diagonalized by an orthogonal transformation

- Ma: (6" "^r)


^--^+ /*t 0 \
^sMsT or M- St Mas (11.8e)

with

s- / cosd sin0\ (11.e0)


\-sinO ,ose)'
From Mn :0, we get
Si,(Mii1,S;r :0 (cos2 0)mr - gin2 0)m2 g (11.e1)
-
or
fllt
tan9
- (tr.92)
fil2

Remark. Attempts to relate the Cabibbo angle to the strange and down quark
masses have been carried out along such approaches.

11.8 Conservation of the B - Z quantum number


Show that if there were a set of scalars transforming as a doublet under the weak
SU(2) symmetry and as a triplet under the colour SU(3)c: hio{i _ 1,2,d :
1,2,3),then both the baryon number B and the lepton number L arenot conserved.
However, the linear combination B - L is conserved.

Solution to Problem 11.8


As mentioned in CL-p. 355, the presence of h'" will lead to Yukawa couplings of
the form

Ly : fn,tiyh',qi * fnr4iorhiuef,ne"BY . (11.93)

In order to conserve the quantum number B, the first term requires the assignment
h(h) - -+, while second term requires B2(h) ]. Thus the baryon number B
-
11.8 Conservation of the B - L quantum number 2I7

is not conserved. For the lepton number conservation we have L1(h) : -1 and
Lz(h) - 0, and the lepton number is not conserved. However, the combination
B - L has the values
2 1
and Bz-Lz :-
L

81 -Lt:j (rt.e4)
J

and is conserved by these Yukawa couplings.

Remark. This simple example illustrates that the baryon number (or lepton num-
ber) conservation is an 'accidental symmetry' due to some special structure of the
Higgs potential.
12 Electroweak phenomenology

12.1 Atomic parity violation


The weak neutral-current interaction mediated by the Z boson in an atom violates
the parity conservation and will generate mixing between levels with opposite
parities.
(a) Show that the parity-violating part of the interaction has the form

Lr,r : #r@yvf, + v! AL) . (12.r)

where At and V! are the axial and vector currents of the electron and Aq, and, Vfl
are the axial and vector currents of quarks.
(b) If we write Lx in the form

Ly :ful|rry5e (C1,uyru * C6dyud) + Zyre (C2uuyuysu * Czadyrys\f


(12.2)

calculate the coefficients C;, and, Cia.


(c) Using the fact that the momentum transfer is small in the atomic processes,
show that we can write the effective interaction in terms of the nucleon fields
(p, n) as

^,
L'N : Grr- (CrpFypp * C6nyrn) * Eyre (CrrFyryrp * C2nny*vsn)f
#lurryse
(12.3)

(d) Show that for the case of heavy atoms, the terms containing vector currents of
the nucleons add coherently and are much larger than the axial vector terms. The
interaction can be written as

L,N : fftQ'',,," (r2.4)

where Qwk : Q - 4sin2eyslz - N is the weak charge of the nucleus with Z


protons and N neutrons.

Solution to Problem l2.l


(a) The neutral current interaction is of the form
n2
Lx : tttu (12.s)

-*rhJ[
l2.l Atomic parity violation 219

where the neutral current J I contains lepton and quark currents, and each current
has a vector and an axial vector part. Thus the parity-violating interaction due to
the exchange of a Z boson between electrons and the ud quarks in the nucleus
must result from the following V-A interference:

Lr,r : o2
+ v', eL] 02.6)
hlo'rvf,
where Vfr'q and A';q arethe electron/quark vector and axial vector neutral currents,
respectively.
(b) The neutral current having the general structure of JI o (Zl - sin2 Iyy Q) for
the electron, we have

Jtr @ - Eryp (-L + sin2 gs,) er I znypsinz 0v, ep

- (- j + rin' o*) Ey, e + f,Eyu y5e (12.7)

and thus

vf,: (--l + sinz eyy) Eype and A'u: iEy*ytr. (12.8)

For the u and d quarks, we have

JX @ - ury,G - ?sin2 ev,)uv - \sinz \suptpup

- (+ - ! sinz 0y) uyuu - iuyuytu (12.9)

and

ry @ : dryp(-L +J sin2 0*) dt* | sin2 lrydsrdp

- (- i + * sin2 ev,) d'yud + ldyry5d - ( 12. 10)

Then, using g2 /8M?, - Gr /0, we can write,

Lr,r : %^[rurrysel(I
t - !sin2 01,s) uyuu+ (- i+ rrsin2 ory) avudf
"/2
+ (-f + ,int 01,y) Eyre2 (dyryra - avrvr4l. Q2.tt)
Reading out the coefficients, we get

Ctu:2(i - ] sin2 ew), Cu:2(-L * { sin2 ew),


czu : -2 (-f + rin' e*) , cza :2 (-i * sin2 ew) . Q2.r2)
(c) It is convenient to write Ly of eqn (??) in the form
^
Ln : Ge t-
- Lavua) - 2sin2 0* (?"vru - \avua)]
ftlurrrtel(lavuu
+ (-o + rint os,)4Eyre (iryuyr, - iavrvt4l
: (v; - 2sin2 0* J'r^)+ (-f + ,int ew) +avue,{ul
Zfururr,
(12.13)
220 Electroweak phenomenology 12.l

where

V|,: l(avr" - ay*d), Atr: (iuy*yt" - idyuytd).


Jf,* : (?ny*" - trav*a). (t2.14)

The nucleon matrix elements of these operators are (in the limit of zero momentum
transfer)

\ptvf,til - ipy,p, <"lvi.lnl - -Lrryrr,


@et lil - |sonyuytp, fuVtuln) - -jsorvuvtn,
@lJ"**lp) - Fy*p, @lJ"r*ln) - 0, (12.15)

where (p, n) are the proton and neutron spinors, respectively, and B t : - 1.25 is
the usual axial vector coupling constant of the nucleon. In terms of nucleon fields,
we can write

: -
Zfurrrrrl(ipyrp -
L7'1
Lny*n) 2sinz ew@y*p)]

+ (-] + ,in' lyy) 4Eyre se (LFyrysp - inyurdl 02.16)

and the coefficients are

Crp : l<t - 4sin2 os), Cr, : _


i.,

Czp : 2 (-i * sin2 0*) so, C2n - -2 (-i + sinz ?yy) ga. Q2.17)
(d) In the non-relativistic limit, only the time component of the vector current is
non-vanishing. It counts the proton and neutron numbers in the nucleus, and we
have

(A, Zlp! p,lA, Z)


- 7
I
(12.18)
Ito, ZlnlnilA, zl - A- Z- N
i

and for the combination that appears in the weak neutral current

t-
(A.zl I j (t - 4sin2 gry)
D, pi p, - + T ,!,,fiA,z)
Li
-i[(t-4sin2e*)z-N] : lo*. (r2.re)

Thus we get

r,.. -
9+0*
kN - 2A="' eI yse.
tJ--

Note that the matrix element of axial vector current N yrytN in the non-relativistic
limit is proportional to the nuclear spin operator and is smaller than N or Z.
I2.2 Polarization asymmetry of Z -- f f 221

12.2 Polarization asymmetry of Z - ff


The polarization asymmetry (or th9 left-right asymmetry) in the decay of the Z
boson into a fermion pair Z --> f f is given by

AmU) :l\z=' f:f:),-l\z=' f:f:), e2.20)


t(Z --> +t(z +
ftf il f nf)
and the neutral current can be written as
rtr :I [r. U) (f*r,f') + snU) (f^v,f o)1. Q2.2t)
f
In this problem we wish to express the asymmetry parameter in terms of the neutral
current parameters Sr, n ("f ).
(a) Show that the asymmetry parameter Ayp cen be written as

Arnu):ffi. (n.22)
(b) Calculate the asymmetry parameter A7p for the decays:
(i) Z --> eE,
(ii) z --> bb,
(iii) Z -+ cE.
For numerical calculation, use sinz Ory 0.22.
-
Solution to Problem 12.2
(a) In the calculation of the two decay rates in Arn, the amplitudes are the same
except for the overall couplings (gr or ga) and (l - y) or (1 * y5) projections.
As there are no V-A interference terms in the rates, we have
r(Z --> f'fil G'UD' (r2.23)
t(z --> f*f) GR(fl)2
and thus

Arn(f) : (s'(f))'- (gn (fl)2


(r2.24)
Gr$D2 * (gn (fl)2'
(b) (i) Z --> eE

- -+ * sin2 0w : -0.28, gn@):


gile) sin2 0w :0.22,
Am@) - 0.2366. (12.25)
(ii) z --> bb

s^b) - -t * ] sin2 0w : -0.43, sn(b) - J sin2 0w


:0.071,
Arn(b) - 0.944. (12.26)

(iii) Z --> cE
silc)- +-|sinz0w: -0.353, sn(b) - -?sin27yy - -0147,
ArnG) - 0.7. (r2.27)
222 Electroweak phenomenology 12.3

12.3 Simple z-lepton decays


(a) Show that to lowest order in QCD and the approximation that all the fermion
masses in the final state are negligible we have the following t-decay branching
ratios:

B(r --> evv) - B(t --> l.Lvr) =:.5


(b) Calculate the decay rate for r --> n u in terms of the pion decay constant /,.

Solution to Problem 12.3


(a) The total decay rate of the tau lepton (z) is,

f(z)-f(z-+ evv)* f(z-+ pvt) *f(z-+ u*hadrons). (12.28)

From p,-e universality, we get, with the approximation of neglecting.final state


fermion masses,

f (z + evv) - f (z + pvv). (12.29)

To lowest order in QCD, we get

f(z -+ vlhadrons)- f(r -+ v*du) *f(z --> v*su) (12.30)

and

f (z -+ v I du) : lVua(3f g --> trtvv).

Thus

f(z) - 12+3(lvualz *lvu,lt)lr(r --+ evv). (12.31)

From the experimental fact that

lv,alz *lvu,l2 =l (12.32)

we get

B(r -->pvt) . tlvn) BQ --+ eui). (12.33)


'--" ' -l(r I-(r) =l:5
(b) The effective Lagrangian for the decay r --> rru is of the form

L, :\r,oldvre - yr)uflr,yr(t - ys)4. (12.34)


'/2
The amplitude is then given by

M- GpV,afnQr"v,(k)yp(l - y)r(p): GpV,af,m,t'(k)(l - y)t(p)


(12.3s)

where we have used

(n(q)ldyuyrul0) :iJiquf, and q-p-k. (12.36)


I2.4 Electron neutrino scatterings 223

The decay rate is given by

# (:D**,r() rz,f
- \ sptn
t4 (p - k- q)#"#" (I 2 37)

The phase space is the same as that calculated in Problem 11.3, with appropriate
substitutions

o: len)a.a(p-k-il#n#^
: t rm?-y?\. (12.38)
4n \-Z;f- I
The spin average of the matrix element is given by

+L,lMf
spin
- )lc rv,afom,f rrul (l * rs) @ + m,) (r - y)l
- zG2e fllv,al2*?ep . k) - 2G2p fllv"ot'*? (*? - *?)
The decay rate is then

l^
|: * (ry)
-G'rfllv,al2*l(^? - ^')
:#rlv*?^i('-#)
G2- ^ / ,.^2'2
(n3s\

Remark. If we compare this with


n2
I-(z -+ pvv) n.4o)
- ffi*? f

we get

t(r --> nv) fllv"oP\ - *?/"P,f 9!1_ 0.6 (t2.4t)


, \T -- -
ltvv, *;
where we have used lV,al
= 0.975 f" - 90MeV. Experimentally, we have
and

l(t --> nv)


0.66.
f (r -+ pvv) -
(12.42)

12.4 Electron neutrino scatterings


(a) Show that the threshold energy for the reaction v* I e- ) ue -l lt- is E, -
1l GeV in the laboratory frame.
(b) Show that in the v, * e- --> v, * e- elastic scattering, the angle of scattering
0, of the electron with respect to the neutrino beam direction satisfies

sin2 e, -- (T,
,=2^!^
* 2m),1, '' - Y+1
l' - E, 2Ej J
(12.43)

where Z, is the kinetic energy of the (final) electron.


224 Electroweak phenomenology 12.4

Solution to Problem I2.4


(a) Denote the momenta -* v,(k) + LL- (p2), then
as vu(kr) + e- (pr)
s
- (kr * pr)2 - *l +2h' pr : *? +2m,Eu 02.44)
in the laboratory frame. In order to produce p- , we require s . m2*, or

E,> rlGev. (r2.4s)


fit*1,-*?)-
This calculation shows that in the laboratory frame it takes lots of energy to produce
a muon by scattering a neutrino off an (extremely light) electron target.

(b) From the momenta assignment, we have in the laboratory frame,

kr:kz*Pz (r2.46)

krlmr:kz*Ez (r2.47)

From eqn (12.46) we have (setting me : tn)

ktr: (kr - pz)2


-Ul+V3-2krp2cosL,:k?+E3-*2 -2krp2cos0,.
(12.48)

From eqn (12.47),

t<2,
- 1t<t *m, - E)2 - t<l +*2 + n3+2k1m -2m82-2hEz. Q2.49)

Combine these two equations, we get

-hpzcos9" : m2 tklm - (m *k)Ez - (m * kr)@ - E). (12.50)

In order to express pz in terms of E2, we square both sides of this equation:

4 (n3 - *') (t - sin2 s,) : (m * k)2(* - E)z (12.s 1)

or

k?@, t m) - (Ez - m)(m * k)2 : k?@, + m)(sinz 0,). (12.52)

The kinetic energy of the final electron being T, : Ez - m, the scattering angle
satisfies
.
sin' o" : g*2*-lr- ?_y+1. (n.n)
rm") L Eu 4 )

Remark. For the usual neutrino beams we have Eu ) m" and T, ) m": this
formula implies that sin2 g, is small and

^ 2m.
0"- (r2.s4)
"7,
Thus the electron moves very much in the forward direction and provides a good
signature for ve quasi-elastic scattering.
I2.5 CP properties of kaon non-leptonic decays 225

12.5 CP properties of kaon non-leptonic decays


Consider the Ko --> 2r,32 decays.

(a) Show that ln+r-) and looool are CP even eigenstates.


(b) Show that lr0 tr0z0; is CP odd while the CP eigenvalues of the state lr+ r - n0)
depend on the orbital angular momentum I of n0 with respect to the center of mass
of the rtt- system,

CPln+n-njl - (-l)/*r ln+n-r0). (12.55)

Solution to Problem 12.5


(a) Denote the wave function of n+r- by

lr+n-) - {(r1,r2) 02.56)


where 11 and t2 zte the coordinates of rr+ and n-, respectively. We can alsb use
the centre of mass and relative coordinates,

R-:(rr+rz), r_rr -r2, Qz.si)


z
to write the wave function as

,lr (rr, rz) : X G)d (r) ( 12.58)

where X G) is just a plane wave describing the motion of the centre of mass and
is of no interest to us. Under the charge conjugation, we have n+ # Tr- , which
corresponds to 11 <-----> t2 or r -+ -r. The effect on the wave function is then

Q@ 3
QGr): (- t)tOG) e2.ss)
where / is the orbital angular momentum of the n*n- system. Thus

Clr+n-) - (-l1t1n+r-7. (12.60)

Under the parity, we have (- 1) from each of the pion and r -+ -r. Thus we get

Plr+n-): (- I1t1n+tr-7. 02.61)


Combining these two relations we get

CPln+n-) : (-I12t1r+v- . - ln+r-!. (12.62)

For the looo') state we have, as before,

Plnonoy - (- Dtlroftol. (12.63)

But under the charge conjugation,

clroro) - lrono) (t2.64)

because n0 is a C-even eigenstate. On the other hand, the nlno system consists of
identical bosons and from Bose statistics we should have symmetric wave function
226 Electroweak phenomenology 12.6

under 11 +---+ 12 which coffesponds to r -+ -r. This means that we can only
have /_ even states. Thus we also get

CPlnono) lnorol. (t2.6s)


-
(b) For the n+r-20 state we can write the total angular momentum as

J : Jrz * J: (12.66)

where Jrz is the orbital angular momentum of the ntn- pair and J3 is the orbital
angular momentum of n0 with respect to the centre of mass of the n+n- pair.
Since K0 has spin-0, we have J : 0, which implies

lJrzl : lJ:1.

As we have discussed in Part (a), the ln+n- ) state is CP even, irrespective of


I - lJnl. On the other hand
t,loo)
CPI#): (-)(-) - (-)'*tl,ro) (t2.67)
where (-) comes from the intrinsic parity of lz0) and (-;'r: from the fact that
under the parity 13 -> -r3, with Jz: lJnl: /. Then the result is

CPlr+ r- ro) - (- | )'*' lr+ n- nol (12.68)

For the 3n0 state the only difference is that by Bose statistics 2tt0 has to be in the
/ ever state. Then we get
-
CPlro no no) (12.69)
- - lnororrol.
Remark 1,. From the fact that both Kr --+ n*n- and K1 --+ n0r0r0 are seen
experimentally, CP symmetry is broken. The fact that the rate for Kr --+ n+r- is
much smaller than K5 + T* 1r - implies that K y is mostly CP odd state and K5
is mostly CP even state.

Remark 2. Ks --> nUnono decay also violates the CP conservation if K5 is a


pure CP even state. On the other hand, n*n-ru can have both CP even and CP
odd wave functions.

12.6 Z + HH is forbidden
Show that in the standard model, the decay of the Z particle into two Higgs
bosons, Z --> H H, is forbidden by the angular momentum conservation and Bose
statistics.

Solution to Problem 12.6


Because of Bose statistics, the two final-state Higgs particles should be in the
spatially symmetric state. This means that the relative angular momentum has to
be even, I : 0,2,4. .. . But the initial state Z has spin J : I, which cannot go
into an even angular momentum state.
I2.7 LI : ) enhancement by short-distance QCD 227

Remark 1. The same argument applies to Z --> PP, where P is a pseudoscalar


boson.

Remark 2. There are no symmetry argument to forbid the decay Z -> H H H or


Z -.> PPP.

12.7 A1 - j enhancement by short-distance OGD


The effective AS
- I weak Lagrangian is of the form

L ts:r : (vuav,,o t * h.c.) (t2.70)


T
where

Or : (uryPs) (rl*r"ur)
- lwv'(l - ys)rlldyr(r - v)ul (r2.7t)

Show that in the renormalization of the composite operator 01 there is operator


mixing between 01 and another operator 02 of the form

Oz: @ry\u) (dryps). (r2.72)

Also, compute the anomalous dimension matrix for the OrOz system. The result
should indicate a QCD enhancement of the A1 | operator. -
Solution to Problem 12.7
The one-loop QCD corrections to Or are shown in Fig. 12.1.
For diagrams (a) and (b) in Fig. 12.1, these graphs are just QCD corrections to
current, e.E. fuflpsr) which has zero anomalous dimensiort, y :0 because of
its partial conservation. This means that these contributions will be cancelled by
wave function renormalization which are not shown in Fig. 12.1.

\-''
,,,,#t-.-
\-<
X
(a) (c)

>-.-,,'-
(d) (0

Frc. 12.1. QCD correction to the (iltypsr)(drlput) operator.


228 Electroweak phenomenology l2.l
For graph (c), we can calculate its contribution by dimensional regularization,
where the gauge coupling is of the form g l0-t)2-d/2:

M, :
I #1" (;%),,,' tP,,,,)
"?ln'(f*),"rff,'u,]
: #- I
f*6lu'rv"vpYpsrlfd't"vovf '-"li
(12.73)

where we have used the replacement kokB : 5 S,p. Using the identity displayed
in Cl-eqn (A.17),

YoYPYp: 9oflYp * SPpyo - ]opYfl I ieoprry'yt, (12.74)

we can reduce the Dirac matrices:

[y*ypyp(r - y)]paIy"yf y'(1 - y)1,, - l6lyp0 - y)lpd[yp(l - y)),,.


(t2.7s)

For simplicity we have ignored the complication of defining y5 in the general d-


dimensions. For the SU(3) colour matrices, we use the identity in Cl-eqn(4.134):

(to)i;(to)rt : )(u,,u,o - !u,,ur) . (r2.76)

The amplitude M, can then be written as

M,:dful:l##l 'I (u rYpu )(drYpsr)

-lroryt s ) (dryr" r)f (12.77)

Performing the integration over k,

f ddk r _ i t(2-d12)
J e"Y t^ - (4"Y'
(r2.78)
I e)
we get

-o2
M':69-n I fe-dlz) ^ t-._
@tYpu)(d1v,1s7)
e")d/r-- ,-(+8) L
r-l (12.7e)
-5@*usr)(drYr,)
).
This shows that the operator O2 mixes with Or under the renormalization. It is
clear that Ma gives the same contribution as M,.
12.7 A,l : I enhancement by short-distance QCD 229

For the graph (e) we have

M,: dfu I # 6lu,t"yoypypsrfl,t,rv*v"vuu,l*.


(12.80)

The ordering of y matrices being different from that of Mr, we get


[yoypyr(r - y)]pdly*y"yfr (l - y)1,, : -4lyp0 - y)lpdlyp\ - y)1.,
(12.81)

and

M,:ffidwt9+Pez>loz-i''] (1 2s2)

For graph (f), giving the same contribution as (e), the total contribution is .

M - 2M, +2M,: \&-)-


, "l?g:,-19-4Pl o, -1r,-l
a/z (4n)a/z 4 L-. 3 -,1
. ' ( n.$)
For d + 4, we have

f(2-d/2) t 12 ,l
@;A@F:n ' (W L; - Y *tn(4r) - rn P'l t02'84)

with e - 4 - d.The counterterm for Ot is then

6r,,-
ffi(?-v -ttn(4n) (r' - ir') (r28s)

If we write

Or * \Ot - ZnOr * ZnOz (12.86)

we get

zrr:r- &(?-r
(4n.
+h@n),
3s2 /2 \
Zn:
*;f (; - r +h@r) )'
(12'87)

It is straightforward to carry out the renormalization for the operator Oz and the
result is

Oz -t 6Oz - ZzrOt I Zzzoz (r2.88)

with

Zn : Zzt, Zn : Zzz. (12.8e)


230 Electroweak phenomenology I2.8

To compute the anomalous dimension, we can use the correspondence between


dimensional regulation and the invariant cutoff given in CL-Thble 2.1to write

zrr:zzz:t*&(^#. ),
Zrz:Zzt:#(^5. ) (r2.e0)

Then from the formula for the anomalous dimension matrix

Yii:-:-L2 8ln lv tnzii (t2.et)

we get

,:#(-\ _?) (r2.92)

The eigenvalues are


02 02
y--
Y+- ffi "' @rt
x (-4), (12.e3)

with the coresponding eigen operators being

o+ : ).ro, * o), o- - )ro, - oz) . (r2.e4)

Or in terms of quarks fields,


1 1=
Ot:
' -(O, + O) - Ut<ntYps)(dflpu) * (urYpu)(dtTpsr)1,
2'
1 1- (urYpu)(dflps)1.
o_ - -(o, - O) -
2' ,L@ryps)(dflpu) -
Remark. Theoperator O-whichisantisymmetric inur <+ drisapure LI - t
operator because 1 0 state is antisymmetric in Ad. On the other hand, O..,' has
-
both A1 - | and LI -J operators. Thus QCD corrections enhance the O-
operator (y- < 0), relative to the Oa operator. [Scale factors are raised to the
negative powers of y, see, for example, Cl-eqn (10.148).1 But this enhancement
of the L,l - ] operator does not seem to be numerically large enough for the
explanation ofihe experimentally observed A/ - j rule.

12.8 Scalar interactions and the equivalence theorem


The standard model Lagrangian for the scalar field is given by

L, : @uL)t @pil + p2ol Q - L(Qr il2 (r2.9s)

where

(i;) (r2.96)
':
I2.8 Scalar interactions and the equivalence theorem 23t

(a) Show that if we param etize the four independent components of the complex
doublet field as

I /6t * i2z\
Q: 6\o' * iQo) ' (r2.e7)

4, is invariant under O(4) rotations, i.e.

Qi + Q'i: ?iiQi, with i - 1, "' ,4 (12.98)

where i is the four-dimensional rotation matrix ir ? - ??r - l.


(b) Show that if we write
tr - (0r, Qz, Qq), o : Q3, (12.99)

then .C, is the same as the SU(2) x SU (2) sigma-model (without the nucleon).
(c) For spontaneous symmetry breaking, we have

Qz-o--u*H with ,':(. t (12.100)

Write the Lagrangian in terms of H and t and find the Hn+t-, and Hzz cou-
plings, where irt :
ftt"t - ir) and z - 7e.
(d) Calculate the scattering amplitudes: r*t- -+ zz, nTtft --> n*n-, and
77+77

Solution to Problem 12.8


(a) From the parametrization

o:+(I:+',ri)' (12.101)

we get

-
QIQ
| , .
- r(o? +
.
0'z2
1
+ 03 +
1\
oil : t@' o)
I

at ol apo : |.ruuo
. au o) Q2.to2)

where f _ (Qr, Qz, Qz, QD is a vector in four-dimensional space. Then the


Lagrangian is of the form

L, : (a*o)' . +(0. 0 - Loro or (t2.ro3)

which is clearly O (4) invariant because only the invariant scalar products appear
in the Lagrangian.
(b) If we break the 4-vector @; into a 3-vector and a scalar, r - (Qt,Qz,Qd,) and
o : Qz, the 4-scalar Q .@ corresponds to the sum of
O.O: T2 *o2 (t2.to4)
232 Electroweak phenomenology 12.8

The Lagrangian is then of the form

-
L, : lr
,lturr)z
. ^a +
+ (a*d'f
u2
iet' + o') - i<"' + o')'
,i
(12.105)

which is precisely the SU(2) x SU(2) o-model withour the nucleon.


(c) To study the consequence of spontaneous symmetry breaking, we write o
u * F1, then
-
o'+o2:tt'+H2+2Hu*u2 (12.t06)

(r' + o')' : (o' + H')' * 4Hu(n, + H,)


+ 2v21tr2 + H\ + 4u2 H2 + 4u3 H -t ua. (L2.l0:-)
The scalar potential is then

v - -trr' + o2) * Loro' + o2)2

: ry H2 + ).Hu(n2 + Hr) + Xrr, + Hr2. (12.108)


The mass of Higgs is given by

ffi2H :2)'u2 ' (t2.t0e)


Note that ns are all massless. It is more convenient to write the Lagrangian as

L,:)lr.r)2 + @,n'f -+H2 -*rrr'+ H\ -#(r'+ H')'.


From 12 :2n+n- *
zz,we can read off theHn+tr- and ,r, "oul;il:
being imzrf u andimzrfu, respectively. The decay rate for H --> tr+tr- is

r(F1 -+ n+n-) : (+)' * | rro>ouo(p - k - k,)#"#*


: *3u r m3uG p (t2.rtD
2u2 gn ghr
where we have used the vacuum expectation value t)-2 : JlCr. We see that in
the limit mn )) M1ry,this agrees with the decay rate f (F1 + W+ W-) calculated
in the next problem.
Similarly, we get for the decay H --> zz,

(t2.r12)

Remark. This is the essence of the equivalence theorem: one can replace Wy and
Zp with the corresponding 'would-be-Goldstone bosons' in the limrt ms )) Mw
and M7.
12.8 Scalar interactions and the equivalence theorem 233

(d) Would-be-Goldstone boson scattering amplitudes


(i) n+n- --> zz

(a) (b)

Ftc. 12.2. Tree diagrams for z+z- -+ zz.

The amplitudes for these diagrams are

y4@) -':', , M(ot - (_,tmzr)' --= e2.tt3)


- t)z \ n / s-ffi2a

and the sum is

M-M@) ylrqtu\ t-:--l .


--':'o Ls-m'H) (n.n4)

Remark. The amplitude M vanishes as s -+ 0, as expected from the usual low


energy theorem for the Goldstone boson.
(ii) zz --> zz

(a) (b)
XX (c) (d)

Ftc. 12.3. Tree diagrams for zz --> zz.

The amplitudes for these diagrams are

Jv'
1r1@) (-i^'n)'--i----- . M(u, -\
JvL - (-t^2o)'--l-- (12.115)
-
-\ u ) s-ffi2n' u / t-m-H

MG,:(+)'/n, M@,:(S7ro (12116)


234 Electroweak phenomenology 12.9

and the sum is

M -+r,,, : :+l:* . :A . :Af ,r'7)


where t - (pt - pz)z,u - (pr - pq)2.
(iii) z=z- --- ir+ir-

(a) (b) (c)

Flc. 12.4. Tree diagramsfor n+n- --> tr*ir- .

The amplitudes are

/ . r r2 . / . ) r2
M@t _ (-,*', ) __= , Mo, : (-tm'H ) f__, (t2.118)
\ u / s-m'H \ u / t-m'H
: ---2

191tct
--)"'rr+x2x2 (l2.rlg)
8u2

and the sum

M-j4t--=*-i----l
u'Ls-m'n t-^'rJ
(tz.tzo)

Remark. These simple results for the scattering of Goldstone bosons can be used,
through the equivalent theorem, to get the amplitudes for the longitudinal gauge
boson scattering.

12.9 Two-body decays of a heavy Higgs boson


Suppose that in the standard model the Higgs particle is heavy so that fttp > 2Mz.
Calculate the decay rates for the following modes:
(a) H --+ W+W-
(b) 11 -+ ZZ
(c) H --> tt.
(d) Show that in the limit mn ) My,the decay H --> W+W- is dominated by
H --> WIW;, where Wf arethe longitudinal components of I4l+.
I2.9 Two-body decays of a heavy Higgs boson 235

Solution to Problem 12.9


(a) H ---> W+W-
We can read off the HWW coupling from Cl-eqn (12.165) and write the amplitude
AS

Mo: -igMw@r 'e). (t2.rzt)

Then

Dlt t"l' - s' Mi, T(r' . ez)z


- s2 Mzw(-r" . (- r* .
spin spin ffi) W)
(t2.r22)
-srMr*(r* W)
From ffi2H : (k + k)2,we get k - k'
- i @', - 2M?r). Then

I
s pin
lM"3 --
02

fu @L - +mzorut2* + t2Mil)

:'#,^,(,-W."#) (nn3)
The decay rate is then

d3k
r - I f rz">o^^@ - k -
d3k
h J
ao k')
ffi #, Llt"r"l''
sptn
02:'24)

The phase space in the rest frame of 11 is

e :I 12n1464(p - k - 0L#*
:#la<^,-E-E)##; (r2.t2s)

Write

d3k (r2.t26)
-k2dk4n -4nkEdE.
We get

p:GlfkdEk
Jur*n-2E);-
(12.127)

^"^r.
The momentum k canbe calculated as follows:

E - L*n : (k2 + Mz;rtz + k : L(*', - 4u2;t/2. 02.128)


236 Electroweak phenomenology 12.9

The phase space is then

e:*('- w)', (r2.t29)

Note that the result here for the two-body phase space is different from that cal-
culated in Problem I2.3, because of the equal masses in the final state here. The
decay rate is then

*'r9-' (, !ry\''' (, - !y! * ,rU!\.


r(/1 -+ ww) - - (r2 r30)
8"/2n \ ffi"H / \ m', m-H /

Note that in the limit of mp )) Mw', this is the same as eqn (12.I11)-as an
expression of the equivalence theorem.
(b) H --+ ZZ
The amplitude is given by
{M=t (e, e). (r2.r3r)
Ma - -i COS Ury

The phase space having an extra factor of j, because of the identical particles in
the decay products, the decay rate is then

f (H
/ 4M"
+ zz) - 49 (t - 4uV\t/2 (t-d- M!\
,ra Q2 t32)
r6J2jr\ E) )
This is the same as eqn (I2.ll2),if ma ) Mz.
(c'l H --+ tf
The amplitude is given by

M,: -ll7lt
u
(r2.r33)

and

T lM,l'^m?
spin
- #Trl(ft * m)(#' - m,)l
' _ 4*?
..._t (t
.k, _ *,r)
-,lt^
-zJiGr*?*',(r- ffi ri2.134)

where we have used ft . k' : * @L - z*?) and u-2 - J1G r. wirh the phase
space

e:*(,- #)''' (r2.r3s)

we get for the decay rate, which should include a colour factor of 3,

r(rl _ _ *,
_+ try
W(, H)'''
(r2.t36)
I2.9 Two-body decays of a heavy Higgs boson 237

Remark. Since m1 is now measured to be around 180GeY the decay H + tt


is kinematically allowed rf ms > 370GeV. For the case mzu > m/, the decay
H --> WW and ZZ willdominate over H -'+ tt.
(d) As we have mentioned in the text (CL-p.343), the longitudinal polarization
is of the form

efQi -- E) (r2.137)
h(k,o,o,
which, in the high energy limit k ) Mw, takes the form

st&):#."(ry) (1 2.1 38)

The matrix element for H --, WI W. is then

Mrr - -iger(k)'ey(k') (t2.r39)

and

lMnlz - gz M'*lr r&)' e ilk')l' = s' M'*%f

:ft,I)t*"-*)) =ffi (t2.t40)

This is exactly the same answer for the decay H --> W+W- in the limit mn )
Mry. This shows that H --> W+W- is dominated by H --> WlWl.

Remark 1. Since the difference between total decay and the decay H --> WlWt
is of the order of M2* /mzs,thedecay of the Higgs boson into the transverse modes,
H --> w;W;, is suppressed by O (m2*1m2r).
Remark 2. We can translate this into a relation between effective coupling con-
stants:

Tuw,w,
gr-r*
-- ( Yy\ in
rrr the limitmn
L'v rtrrrrl ) Mw. (r2.t4t)
\W )

This means that in this limit, the Higgs coupling to Wp is much larger than that
to W7. On the other hand, W1 comes originally from the scalar fields. Thus the
physics of gauge bosons Wr and Higgs field F1 can be described in terms of
scalar self-interaction present in the original Lagrangian. This is the basis of the
Equivalence Theorem (between longitudinal gauge bosons and the scalar Higgs
mode) (see, for example, Cornwall et al. I974).

Remark 3. The same argument applies to the decay mode H + ZrZr which
willdominateover H --> ZrZr.
13 Topics in f lavourdynamics

13.1 Anomaly-free condition in a technicolour theory


Consider the simplest technicolour theory with one left-handed doublet as given
in CL-p. 405. Show that to avoid the anomaly in the SU(2)z x U(l) gauge group,
we need to make the charge assignment of techniquarks:

e(u):i e@)_-t. (13.1)

Solution to Problem 13.1


Consider the triangle graph in Fig. 13.1 which is the source of the anomaly.
Since there is no anomaly in the SU(2) group, we first consider the case with
only one U(1) vertex, in which the graph is proportional to

Tr (1;r iY). (13.2)

Because of the presence of zs only the doublet members can contribute to this
? sum. I commutes with ri we can write
Tr(titiY) - Tr(riYt) - Tr(r1r;Y) - )frqt;, r)Y) - Eij Tr(Y) (13.3)

On the other hand, I takes the same value for both members of the doublet. This
means that the absence of anomaly requires that Y 0 for the doublet. From the
relation Q : Tz + g /2), we see that
-

Q(u):L Q@)--+. (13.4)

This charge assignment implies that in the right-handed singlet sector

Y(Uil: i.: -Y(Dil, (13.s)

For the case of two U(1) vertices, the coefficient of the triangle graph vanishes
because Tr(ti) : 0. For the case of three U(1) vertices, only the right-handed

FIc. 13.1. The triangle diagram with a fermion loop.


t3.3 Properties of Majoranafermions 239

singlet will contribute and the coefficient is proportional to Tr(Y3). This vanishes
as Up and Da contributions cancel because Y(Uil : i.: -Y(Dil.

Remark. One way to avoid this strange (unfamiliar) charge assignment is to intro-
duce a technilepton as in the standard model

(g), (;) ,'^ Dp Nn' En (13.6)

with the usual charge assignments:

Q(u):?, QQ)--+, Q(N):0, Q@)--1. (13.7)

13.2 Pseudo-Goldstone bosons in a technicolour model


Consider the case of one generation of technifermions

(g). (g) ,'^' DR' Nn' En (13.8)

where the charge assignment is the same as the ordinary fermions in the standard
model.

(a) If one turns off all except technicolour interactions, show that the model has
an SU(8)1 x SU(8)n global chiral symmetry.
(b) Suppose the technifermion condensate breaks this symmetry down to
SU(8)1+R. Show that there are 60 new Goldstone bosons besides those which
were removed by gauge bosons.
(c) Show that these Goldstone bosons will become massive if we turn on the gauge
interaction.

Solution to Problem 13.2


(a) Since U and D are SU(3)c triplets, we have eight left-handed and eight right-
handed technifermions. Thus the global flavour symmetry is SU(8)r x SU(8)n.
(b) Inthesymmetry-breakingSU(8)1xSU(8)n + SU(8)1*R,weget82 -l - 63
Goldstone bosons, of which three are removed by gauge bosons to break the gauge
symmetry from SU(2) x U(1) down to U(l),..
(c) Since the gauge interactions of SU(2)1 x U(l)r
do not have the chiral
SU(8). x SU(8)n symmetry, these Goldstone will become massive when the
gauge interactions are turned on. These particles are usually referred to as Pseudo-
Goldstone Bosons. One expects their masses to be of the order of gM1a, and they
can be as low as a few GeV. If they are as light as a few tens of GeV they should
have been seen by now. This is one of the difficulties in constructing a phenomeno-
logically viable model based on the technicolour idea.
Topics inflavourdynamics 13.3

13.3 Properties of Majorana fermions


In the standard representation, the solution to the Dirac equation in the momentum
space can be chosen to be of the form

u(p,s) - (E i m)''' (*) rU,


(13.9)

u(p,s): (E *m)''' (+) x'r,l


with

x (i): (;) x (-+): - (?) (r3.10)

x'(+): - (?) x'et) : (l) (13 1r)

Note that for convenience we have changed the spin wave function from X (s) to
X'(s) in the u-spinor. Also there is an external minus sign in X (-+).
(a) Show that with these choices, the charge conjugation will just interchange the
u- and u-spinors, i.e.

u'(p, s) : iyzu*(p, s) : u(p, s)


u' (p, s) : i yzu* (p, s) u(p, s). (t3.r2)
-
(b) If we write the free Dirac field operator as

(t(x): s)u(p, s)s-ir * + dt (p,s)u(p, iein'1


P I @#plu@,
(13.13)

show that the Majorana field tlry defined by


1
{u: OW+(t') (13.14)

can be decomposed as

drp
thtw@):+l s)u(p, s)s-io'' + b\(p, s)u(p, s)ein -f
lurro,
l{zn)tznolt/2
(1 3.15)

with

bu(p, t) : * d(p, s)). (13.16)


+(be,s)
Also, compute the anticommutator

{bu(p, i, bL(p, , s,)}. (13.11)


13.3 Properties of Majorana fermions 24r

(c) Show that


u(p, s)yuu(p', s') - u(p',s')yru(p, s)
u(p, s)yru(p', s') - u(p', s')yuu(p, s) (1 3.1 8)

so that

,l/uy,'rhu
- 0, Vuouurbu - 0. (13.19)

Solution to Problem 13.3


(a) Write out the conjugate spinor in the standard representation for the y-
matrices:

u,(p,s) : i yzu*(p,r): ( t";)t" ,-rrl


-?r, (
*)
Gioz)) ,',,.,,,
-N5 ('-GJ)) ,.(s)- ('*(#)

- -Ne (+),-r.(s) - x'r,r - u(p.s013 20)


" +)(

where we have used the relations

(io)(o*)(-io) : - o, -iozx*(s) - X'(s). (13.21)

(b) From the decomposition of a Dirac field

(t(x): s)u(p, s)g-in ' + dr (p, s)u(p, s)eio '1


T I @#ylu@,
(r3.22)
we get

V'@) - iYzrlrt (r)

:+I dtp
l{z,r)tzno1tt2
lu' @, s)u'(p, s)sin'r + d.(p,s)u'(p, s)e-in'"1

:+I l{ziltzn drp


r1t/2
la@, s)u(p, s)s-in'* + br (p, s)u(p, idn *1 .

Thus we find that


1
rbu : -[rb
(x) + lr'@)l
"/2
:DI dtp
lurrr,
s)u(p, s)e -tp'x + btr(p,s)u(p, s)ei P'f
l{zn)zzno1tt2
(r3.23)
242 Topics inflavourdynamics 13.3

with
I
bu : s) * d(p, s)1. (13.24)
7Ib(n,
The anticommutator can be computed

lu*ro,
s), btu(p', r')
)
: ; lu@, s) * d.(p, s), bt (p', r') + dr 1p', s'yl
: L,
{b@,s), b* (p' , t')l + L {a@, s), dr (p,, r)I
: 6,,,63(p - p'). (13.25)

Thus b7a and bt* arejust the usual destruction and creation operators.
(c) From part (a), we have
u'(p, s) - u(p, s) : i yzu" (p, s) (13.26)

or

,t (p, s) : ur (p, s)(-i y]) - ,' (p, s)(iy). (13.27)

Then

u(p, s)yru(p' , t') - ,t (p, s)ysyru(p' , s')


: ur (p, s)(iy)(yoy)(iyz)u*(pt, st)
: ut (p', s')(iy)r (y[y{>tiyr)r u(p, s)
: u(p',s') (-i r) Qor[ vo) (i y)u(p, s)
- u(p',s')(-iy)yiQy)u(p, s) - u(p', s')yuu(p, s)
(13.28)

where we have used the relations

ror)ro : yp, rorfro - y;, (iDyl,?iy) : yp. 03.29)


Alternatively, we can write

u(p, s) : u' (p, s) : i yzu* (p, s) : j yzyoyou* (p, s) : Cur (13.30)

where

C
- iyzyo, C-t - Cr - Ct - -C, Cy[C-t - -yp. (13.31)

Similarly

,I (p,s) : ur (p, s)iyz, u(p, s) : ur (p, s)iyzyo: ur (p, s)C. (13.32)


Thus

u(p, s)yuu(p', t') : ttr (p, s)CyuCur (p',J') : ur (p, ilyf.il'(p', t')
- u(p' , t')yuu(p, s). (13.33)
13.3 Properties of Majoranafermions 243

More generally,

u(p, s)f u (p', r') : Ltr (p, s)ClCur (p',s') : -ur (p,s)ClC-ru'(p', t')
- u(p' ,s')l'u(p, s) (13.34)

where

f" : (-clc-\r . (13.35)

For the various cases,

| : yr,t yi: ecy;'c)' - (r[)' - y,


| : rlrys : (-Cyry{tc)' -
(yuy)' (v{ y[)' : ysyr": -y,ys
| - ys y: - (-cyl'c)' : -r,

f-l l'-(-cc-r)r--1
| : o,v ofr,: (-Co;:C)
: ot r. (13.36)

From the Majorana field expansion given in eqn (13.23),

,l,u@): s)u(p, s)s-in x + brM(p,s)u(p, ilein 'f


T | @#ylrrro,
.

(13.37)

t,uyutu:nl
@##T I @#ry
x lbrr(p, s)bu(p' , r')u(p, s)ypu(p' , s'1s-t{p-p')'*
n')'x
I b u (p, ilbtM(p' , ,')r(p, s)yr"u(p' , ,r1ri(n-

* brr(p, ,)bIr(p' , ,')u(p, s)yuu(p' , s'7e-t{p*p')'x

+ bu(p, s)bu(p' , r')r(p, s)ypu(p' , rrlri(n+n')'x7. (13.38)

We can write the second term as

x
ur@, r)br*(p', r')u(p' , r')y*u(p, s)s-i(n'-il
nII
: I I bu(p','')btr(p,s)u(p,s)vt'u(p','r1'-i(n-n')'x
P
244 Topics inflavourdynamics 13.4

where we have interchanged (s, p) <----> (s', p').Then the first and second terms
combine into

drp d3 p'
EI l{zo )z z n o]''' leo )t 2 E,o]t
/'
* {bt*(p, s), b u (p' , s')Iu(p, s)yru(p' , ,r1r-i(n- n\'x

: \----a f d3 o
s)Y'u(P' s)' (13'3e)
+ J lQil'*Yu(P'
This is a c-number and can be removed by normal ordering. The third term can be
written as

u'rro, ibtr(p', r')r(p', r')vrr(p, s)s-i(n'rD '


D,
'''- I I
ffrJJ-
f t +{bL(p, ilbh(p',s')lu(p',s')vpu(p,s)s-i(n'+p)r - 0.
(13.40)

Similarly, the fourth term is also zero. In an entirely analogous way, we can show
that (ryo*u{ru :0.

13.4 p + ey and heavy neutrinos


Consider the decay LL --> ey in the same model as in Cl-Section 13.3, but without
the assumption that all neutrinos are much lighter than the W boson.
(a) Show that the branching ratio is of the form

B(pt --> ey) : #r',f (r3.4r)

where

6'u:24rr,rr,t (#) (t3.42)

with

ft ,] -u\da
s(x)- Jo ^ --:. t2(r-a)(2-cv)+a(1
I (l-cy)*ax *a)xl. (13.43)

(b) Show that, for the case m! < M2w, this reproduces the result in Cl-eqn
(13.1 1s).
(c) Show that S(0) # S(a) and, if mz )) Myy and mrl K M1ry, the result is of
the form

: 3c

fiefsfoo) -
B(pt --> ey) sQ)lUI*u1,2\2 (t3.44)
13.4 p -+ ey and heavy neutrinos 245

I I

( a) (b )

Q ..--

I I

(c) (d)

Frc. 13.2. lt --+ ey via neutrino mixings.

Solution to Problem 13.4


(a) We will carry outthe calculation in the'tHooft-Feynman gauge. First, consider
the diagram (a).

r (a) --; \- I #r,(p - (#) u),y*(r - ys) y'(p*k)-mi


^
" (#)IrpYv(t-v)u,@t:h
(-i sP"\
" 111rffi7a5(iet"P)
where

lofl: (2k.e)g"B - (k + 2q)pe" - (k - e)"e p. (13.45)

We can write this as

T(a) - -,+,, 1 l1(k,


) - M'*)
I

ftr+il2- MzsNe'r
*
where

,, : (t3.46)
*(J[i(rpi
and

Npu : u"(p - q)ypl' (p + k)y,(l - y)u u(n). (t3.47)


246 Tbpics inflavourdynamics 13.4

Using the Feynman parameters, we can combine the denominators to get

ll _.r f dotda20(l -cYr -uz)


(p]-k)2-m! k2_Mzw" &+il2_M2w -- J A'
(13.48)

where
-
A : dll@ + D2 - *!l + az[(k * q)2 - M'*l+ (1 - ar - a)[o' - *?r]
- (k + arp * azq)2 - a2 (13.49)

with

o2:ow? +(1 - o)M?r. (13.s0)

We have neglect m2* as compared to m! or Mlr. Collecting these factors we get

T(a) - il7rci2 [ aordu20(l - ar - uz)

f d4k
"Je"Y fft+ arplazq)2 -o'lt Sr

where

St : NprfP'. (13.s 1)

To simplify the integral, we can shift the integration variable, k -+ k - ar p - aze.


Under this shift, we get for the p .e termwhich contributes to p --> e/ , see Cl-eqn
(13.97),

Sr -+ Sr (p . s)lu,(l
- * y)u*l2mr[2(I - ot)' * (2qt - Duz]. Q3.52)

Momentum integration yields

f d4k I (i)
(13.s3)
J Q"f &'z=pF: 32n4''
The contribution to the invariant amplitude A is then

A(a) -+,,# I ffir20 - u)2 * (2ar - l)azl.


(13.s4)

Integrating over d2, we gat

--\_-/-I.,
A(a\ L-,
mP
1612 (ilt dqt(I - u)2 (1 -
"')
t(l - cur) * utlm! 1u!r))
(13.s5)
I3.4 p --> ey and heavy neutrinos 247

From diagram (b) we have

ri@) - -i) I ffiu, -, (#)u,,yxo - v) v rp + r,t -


^
" (#n)lr,,tmi(t - y) - m,o * vs)tuu(il@+t
(-igr') .. .-
x -- (ieM1ryer)
[(k+q)t-M'*l

-, D, I # .T#=A " ;W x
(ki| M,w'^N^
(13.56)

where

N1 - u"(p - ilyxQ * y)l*? - mr(y 'k + m)lur(D. (13.s7)

Combining denominator and shifting the integration variable, we have

ri@)- I i
,,,,
I du1du20(1 -or -az) I #@+rf.
(13.58)

Again, picking out the p . e term,

Nr : N.,,er -+ Nr - -2(p . e)u,(p - q)(l * y)up(p)azmp. (13.59)

The combination to the invariant amplitude A is then

A(b)_p,,wlffi
+''W@1ffi
Diagram (c) gives the contribution
(1360)

ri@)
--' T I #''(p - " (#*)'''rmio * v) - m"(r - v)l
i (is \l i
v '(p + k) - *, \;'fr)
- vs)uptn) 1o' - rvr')
(-i sr') "u'Y'(t
x
I& +#: MfrGeMwex)

-'t', I#6.#=8" ohx (k#:MT'^Ni


(13.61)
Topics inflavourdynamics I3.4

where, after setting ffi, : O,

N'^ : u"(p - ilyL\ + y)m?up@). (13.62)

It is easy to see that this does not contribute to the term p . e.


Diagram (d) is of the form

ri@) - -i+ I ffi'"ro - t, (#*-)',,rmio * v) - m,(r - v)l


y'(p*k)-mi (ffi), -,[mty - - *(r + y)]u,(p)
y) m

i i
x ' 12k + a)
&r: Mt t& + qy= Mfr(ie)e
2k-e.
--i+,,#t#ru,(p- q)(I P'\Y)r@*k)2-m!
v)u u@)l

" k2_Ufu '(k+il2_U2*

- -4 -m?
4r,frro
. e)lu,(p - q)(I + y)u*(Dl

da20(l - ur - u)(ar * az)ar


"I da1
arm? + (1 - o)M?, (*) (13.63)

Thus we get for the invariant amplitude

A(d):ffi,,+,,#(#)1ffi
:ffi'+''#(#)Iffi 0364)

The total contribution is then

t -lle/g) + Ar@) + Ai@)l


i

:ffit"\,,#@1,'ffi
" { - zo - o)'(3 - 2a) - 2(r - o)' - Zu(r- a)(1 . r (+AI
:ffi,"+,,*G)'(#) (1365)
I3.4 p --> ey and heavy neutrinos 249

where

ft 0 - u\du
g@) I ;----.
- Jo (r -cu.) +ux 12(l -
a)(2 - cy) + cv(l * a)"rl (13.66)

and

,,:*uliupi. (t3.67)

(b) The function g(.r) can be calculated as follows:

: rt 0 - u\da
sQ) - a)(2 - a) + a(l * a)xl. (13.68)
Jo ffi12(r
Lety - 1l@ - 1) or x: (l + y)/y.
:.5 t*'l{ a)(2 -cy) + -r", @r-]
[ttt -
s@) ot(t
I,"ttr
|ft 0'-.-------
- u\du . LL)/\t v)\L v) I w\r
12v(1 -a)(2-d)+a(l*cu)(l+y)l
- lo +D
(cv

,f' .?t-(l
da
+ 3y)o3 * 8yu2 + (1 - 9y)o + 4yl. (13.69)
Jo (d+y)
To facilitate the integration we can write the numerator d(u) : -(1 + 3y)a3 +
8yo'+ (1 - 9y)o * 4y as

d(u) - d(a) - d(-y) + d(-y) (13.70)

where d(-y) - 3y(l * y)3 so that


d.(o) - d(-y)
- (a+ y)t-(l * 3y)u2 + 3y(y * 3)a
+(-3y3 -9y',-9y + l)l (t3.7r)

and

7l
s(x) - / t-tt
Jo
*3y)u2 t3y(y *3)a+ (-3y3 -9y' -9y * t)ldu

+ d(-y) [' -!+.


Jo a+Y
( 8.72)

The integration brings about

lstterm : -(1 + 3y) I *trr, +r;+ (-3y3 - ey'- ey + 1)

'
-3y3 - L'-!,
2' 2' +?
3
03.73)
250 Topics inflavourdynamics 13.5

and

2ndterm
- d(-y)" (?) : ffi r"" (13.74)

Thus

8(x, - [;" - :(*)' *!(*) . ;] - -3"', 1n*

Forx < l,
11 2
s(x) -3(1 + 3x) - *<t
z
* t"l + ;(1 * "r) *; * o@')
5x. + O(x'). (r3.7s)
32
In this way we see that for mi K Mw, for all i, we get

6,. : z)r,,r,,l] _
:#l: _ (r,u pi (#) (13.76)
T
where we have used the unitary relation

Dr;,(J1,; :0. (13.77)


i

This is the same as Cl-eqn (13.1 l3) in the text.


(c) But for mr, ffiz K My and mt ) My , the situation is different:

6', : z (ultu s i ulru ur) gQ) + 2uhu 6g(x)


- 2UIzUp.rls(oo) - s(0)l - -2UlzUpt (13.78)

because

52
S(0):J and S(oo):a. (13.7e)

Remark. Since the GIM mechanism is not effective here, the branching ratio will
be very large (compared to the experimental upper limit < 10-10), if the mixing
UIU pz is not very small. Thus the coupling of electron or muon to any neutral
lepton which is much heavier than a W boson must be highly suppressed.

13.5 Leptonic mixings in a vector-like theory


Consider a simple model of leptons, where there are two left-handed and two
right-handed doublets and, in addition, there are two left-handed neutral leptons:

Lr : (";) Lz: (";) Rr : (";) Rz: (";)


,, ,, _,
ll : floL, lZ : flrL.
^,
13.5 Leptonic mixings in a vector-like theory 25t

Show that if the Higgs scalar is the usual SU(2) x U( I ) doublet, then the weak eigen-
states, fte, frLt can be expressed in terms of mass eigenstateS Nt , N2, D3, v4 as
follows:

(n,)n- (cos QNt * sin dNz)n

(n)n - (-sin@N1 *cosQN)n (13.80)

(n,)r (^'
- \rnr cos@N1 +^'t/t2sin,fN2 * IJs3v3+ u"oro)/ t,
(nr)r: (- lt sin1Nr *ry * (J,ttvt + Uroro) Q3.81)
"or1Nz ,
where Uois ara elements of the unitary matrix that diagonalize the mass matrix.

Solution to Problem 13.5


Because of the vector-like nature of the theory, we have the bare lepton mass term

-Lo: D *,(LiRi * h.c.) : ffie@e * n"n") * mp(l'tlt'+ nunp) (13.82)


i:1.2

and the mass terms arising from Yukawa couplings

Ly : mo"noyn"n * mrefitLfleR * ntot tlotnpn * mrrnryflpR. (13.83)

We can collect the mass terms of neutral leptons in the form

-L*: rlr,oM,itir * h.c. (13.84)

where

,b i n : (:,,) j, : ,,, : (*i :, I,,", f,,,,)


-,,r,
U:i),
To diagonalize this mass matrix, we use the biunitary transformation

vr lrtu : Md, Ma : (';' :, 3 s) (13.8s)

The mass eigenstates are

: : (ili)^, (vi).:
(vi)^ v,ti'l'io
(|j),: uli{i'. (13.86)
252 Tbpics inflavourdynamics 13.6

In general , the 2 x 2 unitary matrix V can be taken to have the form

v: (jT"% :ilr) (13.87)

From eqn (13.85)

vr M Ma(Jt (13.88)
-
we get

(cosQm, -sinQm,
\sin,frz" cosQmu .../\
/ u"t (J"z IJ"t \
o I3)I1:' Y:: :
-(*^'
\o m2 o "/ I

\ )
_ (^ru", ffi2u,r,2
\*rU"t m2Upz .../
) (13.g9)

Identifying matrix elements on both sides of this equation, we get (see Cheng and
Li 1977 for more details)

(Jet: ry cosq, Uur: -ffit' sin@,


l?11 l?11

fJ"2 :ksinQ, Uur :*u ros6. (13.e0)


l7l2 t712

1 3.6 Muonium-antimuonium transition


Compute the effective Lagrangian for the muonium-antimuonium transition p.- -f
e* --> l.t+ +e- in the same model as the standard model but with massive neutrinos.

Solution to Problem 13.6

e+Np*
Ftc. 13.3. The box diagram for muonium-antimuonium transition.

The only diagram contributing to this process is the box diagram. We are inter-
ested in the limit where all external momenta are small compared to Myy.In this
13.6 Muonium-antimuonium transition 253

approximation, the general box diagram with arbitrary masses for the internal
fermion lines can be calculated in the 't Hooft-Feynman gauge

B(x, v) - -i (5)^ I #[ura+)/(v 'k * m,)vpur(3)]


x lur(2)yp(y .k * mr)y,ur(1)l

"Q=*)'(;E)eq)
: (#) I n r
G) G -"T)' G+) (;-,)
x lagyy'y^yo iO - D"Q)llaQ)yoy^v"l{r - y)u(l)] .

(13.e1)

After making the Wick rotation the momentum integration can be reduced to a
simple form that can be carried out explicitly:

Io^o6G+)'("+)("+)
: -# l,- {# ;r#5 : -# r @, y) (r3 s2)

where

m2, *',
* - d, i: W. (t3.e3)

The function 1 (x, y) is of the form

, J(x) - J(y)
I(x.y): 1 x2
J(x):
x_y ,
(13.94)
l_x+11 _"ylnt.
The Dirac matrices can be simplified by the identity

yuyLyp : grLyp + gr^yu _ guryL _ i*Lpo ysyo. (13.95)

Thus we have

la@)y'y^vo l{t - y)u(3)] [, (2)ypyxy,Ltt - y)"Q)f


10 lag)v^ itt - Due)flne)v^LO - y)u1)f
-
+ t'LPo px,,ln1r;ytv"){t - y)u(3)llnfz>vtv'iQ - y)r1)f
(13.e6)
254 Tbpics inflavourdynamics 13.6

Using

t'Lpo tp).,, - -6si , Ts\ - y) - -(l - y) Q3.97)

we get for the box diagram

B(x' Y) : - 84
#WI @' Y) lne)Y^ io - Y')uQ))
x lapyyyLO - y)"0)1.
The effective interaction is then of the form

L,rrer- t e+ --> p+ * e-): B,rrlay^+0 - y)tt]lay^LO - Dptl


(13.e8)

where

B,rr: #lr{u,,uliu,ir;,)1ryts?l r B.ss)

with .r;
- *? I u'*.
For the case -r < l,
*',^lnx -+
J(x) --]- + .. l *x +x2 +*21n"
| - x (l - x)'

J(x) - J(y) _ r+(.r*y) *x2rnx -y2rny. (13.100)


x-Y x-)"
Then we get for the effective coupling constant

B,rr : #E@,,uxiuui4llu, -t x1) *'7^::;, (r3.r0r)


-:i,'"-')
14 Grand unification

14.1 Content of SU(5) representations


The SU(3) x SU(2) content of the SU(5) representation 5 is given by

5 : (3, l) + (1,2). (14.1)

Show that the SU(5) antisymmetric tensor representation 10 has the following
decomposition

10
- (3*,1) + (3,2) + (l,l) .(14.2)

and the adjoint representation 24 has

24
- (8, 1) + (1, 3) + (1, 1) + (3, 2) + (3*,2). (14.3)

Also, find the decomposition of the symmetric tensor representation 15.

Solution to Problem 14.1


From 5 : (3, 1) + (1, 2) we can form the second rank antisymmetric tensor:

(5 x 5)1-; : ([(3, 1) + (1, 2)] x [(3, 1) + (1, 2)])r-r


: ((3 x 3)r_r, 1) + (3,2) + (1,(2 x 2);;) (14.4)

where the subscript 1-; denotes antisymmetrization (while 1a; will be used
todenote symmetrization). In SU(3), we have (3 x 3)1-.y _ 3*, and in
SU(2), (2 x 2)61- 1. Then we get

(5 x 5)1-; : (3*,1) + (3,2) + (1, 1). (14.5)

On the other hand, as representations in SU(5), we have

(5 x 5)1-;
- L0 (5 x 5)11;
- 15*. (14.6)

Thus we get

L0
- (3*, 1) + (3, 2) + (l,l), (14.1)

all of which coffesponds to known particles in the standard model. Similarly, we


can work out the symmetric part

(5 x 5)11; : ([(3, 1) + (1, 2)lx [(3, 1) + (1, 2)])r+r


: ((3 x 3)r+1, 1) + (3,2) + (1,(2 x 2)6)
: (6, 1) + (3, 2) + (1,3). (14.8)
256 Grand unification I4.2

Namely, the single SU(5) representation having exactly the same number of states
(with the correct quantum numbers) as one generation of standard model fermions
clearly does not correspond to any particles we have observed so far:

L5*
- (6, 1) + (3, 2) + (1,3). (r4.e)

The adjoint representation can be obtained by the product of the fundamental


representation and its conjugate:

5 x 5* (14.10)
-24+l
To work out the SU(3) x SU(2) decomposition we note that

5 x 5*
- [(3, 1) + (1, 2)] x [(3*,
+ (1,2)l 1)

- (3 x 3*, 1) + (1, 2 x 2) + (3, 2) + (3*,2)


= (8, 1) + (1, 1) + (1, 3) + (1, 1) + (3, 2) + (3*,2). (14.11)

Subtracting (1, 1) from both sides we get

24 (8, 1) + (1, 3) + (1, 1) + (3, 2) + (3*,2). (r4.r2)


-

14.2 Higgs potential for SU(5) adjoint scalars


In the Higgs sector of the SU(5) model, if we neglect Higgs in representation 5,
we can write the scalar potential in the form

v (H) : -m2Tr(H2) + )a(Tr(Hz))2 + )"2Tr(H4) (14.13)

where F1 is the Higgs field in the adjoint representation of SU(5) and is repre-
sented as a 5 x 5 hermitian traceless matrix. Here, for simplicity, we have imposed
a symmetry of H --> -H to remove the cubic term.

(a) Show that H can be transformed into arcal diagonal traceless matrix

lht \
H-uHaut
I'n,'r,
with Ha-l
I
"r^r,)I fruJ4)

|
where h* hz * hz * h+ * hs : 0.

(b) Show that at the minimum, the diagonal elements his can take at most three
different values. From this result, discuss the most general form of symmetry
breakings that can be induced by a 24 adjoint Higgs field.
14.2 Higgs potential for SU(5) adjoint scalars 257

Solution to Problem 14.2


(a) The adjointrepresentation F/ has the following SU(5) transformationproperty:

H --> H' : (l H(lr . (14.1s)

Since any hermitian matrix can be diagonalized by a unitary matrix, we can choose
U such that H is the unitary equivalent to a real diagonal matrix:

,,
Ha_uHUr: ,, (r4.16)
,^
[' ,,)
The trace is invariant under unitary transformation, Tr H - 0, which implies that

hlhz*h*h+ths-0. (r4.17)

(b) With F1 in the diagonal form, the scalar potential is simplified:

v (H) : -m2 r?) + xzln!


4^l* ^, (p
(14.18)

Since h;s are not all independent, we need to use the Lagrange multiplier p to
account for the constraint L, h, 0. Write
-
V':V(H)-p.Tr(H)
: -m2)^l*^'(t ,?) ax,Dh? - r,4r, (t4ts)

Then

: -2m2hi + $., (\rr) hi * Mzh? - r, - o (14.20)


#
Thus at the minimum all h;s satisfy the same cubic equation

4).2x3 | 4)'px - 2m2x - & : 0 with (r4.2t)


" -Dh|.
This means that his can at most take on three different values, Qr,Qz, and Q3,
which are the three roots of the cubic equation. Note that the absence of the x2
term in the cubic equation implies that

h*Qz*Qz:0. (r4.22)
258 Grand unification I4.3

Let n t, n2, n 3 be the number of times Q r, Qz, Q3 appear in H 4,

Qt

Qz
Ha- with nrQr * nzQz I ntQz
- 0. (14.23)

Qz

Thus Ha is invariant under SU(n1) x SU(n2) x SU(n3) transformations. This


implies that the most general form of symmetry breaking is SU(n) -+ SU(n1) x
SU(nz) x SU(r3), as well as additional U(l) factors which leave Ha invariant.

Remark. To find the absolute minimum we need to use the relations

nrQr*nzQz*nzQz:0, h*Q2*Qt:0 (14.24)

to compare different choices of {n1,fl2,tt3} to get the one with smallest V(F1).
It turns out that for the case of interest there are two possible patterns for the
symmetry breaking,

SU(5) -+ SU(3) x SU(2) x U(1) (14.2s)

or

SU(5) -+ SU(4) x U(1) (14.26)

depending on the relative magnitudes of the parameters, )"1 and ),2.

14.3 Massive gauge bosons in SU(5)


For the adjoint representation H, written as a 5 x 5 traceless hermitian matrix,
construct the covariant derivative DuH. Calculate the mass spectra of the gauge
bosons from the covariant derivative if the vacuum expectation value is given by

(I'{)o: 0427)
'("',-,,)

Solution to Problem 14.3


In the SU(n) group we have the following transformation for the fundamental
representation rlrl with 7 : 1,2, . . ., ftl

Vi - {'j : u!r[r, - (a] + iej) rlrr, (14.28)


I4.3 Massive gauge bosons in SU(5) 259

where we have also written out the form for an infinitesimal transformation. The
conjugate representation transforms differently. For tlri = (Vi)* we have

Vi -*,b,i _ (ul - ,r'o)Vo. (14.29)

The adjoint representation F{ transforms in the same way as the product Vi{k:
H',n : (di + iet,) (tf - i'k) H[' : n! + iet,nf - ielny. (14.30)

The covariant derivative is obtained by the replacement of efi --+ gwkin the above
expression:

@uH)j -a*H: +is(w*)'iH! -isWDkHf (r4.31)

Or in terms of matrix multiplication

DrH : \pH * ig(WuH - HWp) - ApH + igfW, Hl Q4.32)

where Wr, H are 5 x 5 traceless hermitian matrices. The gauge boson masses
come from the covariant derivatives

Lw : rrlDr@l@r(H))t] --, 92Tr1lwt,, (H)l[wu, \H)]). (t4.33)

It is easy to see that

[wp, (Hl]tk - (wt)tk(H1, - Hi) (14.34)

where

(H)|,: Hrrsr* (no sum)' (14.3s)

Equation (14.34)impliesthatif H1, - Hi,thegauge field(Wp)f ismassless, andif


& # Hi,then the corresponding gauge field is massive. From the VEV given by

(FI)o:,("r_,_,) (14.36)

the gauge boson fields (W)!having i :1,2,3 and j :4,5 are massive, M2 --
25g2u2, while i, j
- 1,2,3 and i, j :4,5 are still massless. In other words, the
symmetry-breaking pattern is given by,

SU(s) -+ SU(3) x SU(2) x U(1). (14.37)

The remanent U(1) corresponds to a generator which has the same diagonal form
as that given in eqn (14.36). The number of massive gauge bosons is then

24-(8+3+l):12. (14.38)
260 Grand unification I4.5

14.4 Baryon number non-conserving operators


Write down all possible dimension-6 operators which are invariant under the stand-
ard model group SU(3)c x SU(2)r x U(1) and violate the baryon number (B)
conservation.

Solution to Problem 14.4


In order to form a colour singlet state out of quark fields we need 4q or qqq.But
the quark-antiquark combination does not violate baryon number conservation.
Hence we are interested in composite operators that contain three quarks in the
formqoqpeyed?v orequivalently QfiqBQie"flv, where a, fl, y arethecolourindices.
Thus these dimension-6 operators have the generic structure (4'q)(|'l). As for
the SU(2)1 slmmetr], it is clear that there are three possibilities: 4 doublets,2
doublets, or all singlets.
For simplicity, we take only one generation of fermions. SU(2) ildices are
written in the Latin alphabet.

(i) 4 doublets

g(r) (Li"rqiBt) (hf,rtt,t)


- toBvtiit1l11

g(z)
- (4f,rqiBt) (hfrrt"t) eoB, (te)i1 ' (te)r,, Q4.39)

where QtuL : usy aitrd Q2aL : doL.


(ii) 2 doublets

o@ - (d[puBn) (qfrrtir) topyrj


g(+)
- (Lf,rqiBt) @'rot"*) sapvtii' ( 14'40)

(iii) 4 singlets

g(s) (dfipuBn) (ai^t"n) e*or


-
g(o - (a""*"Bn) (ainl"n) tufll. (14.4r)

14.5 SO(n) group algebra


Consider arcaln-dimensional space with vectorx _ (xr,...,xn). A rotation in
this space can be represented as

xi --> x't: Riix; (14.42)

where R is an n x n orthogonal matrix, RzR - RRz - 1.

(a) For inflnitesimal rotation, show that R;; can be written as

Rri : 6ii * e;1 with rj : -s ji. Q4A3)


14.5 SO(n) group algebra 261

(b) For any function of x, this infinitesimal rotation induces a transformation


which can be written as

"f
(x) -+ f (x') - ,f (x) +
+ Jii f 6). 04.44)

Show that the operators Jii can be written as

Jij:-i(xio1 -x13;) i,j:1,2,...,n. (14.45)

(c) Show that .I;;s satisfy the commutation relations,

[Jii, J*il - i(lu,Jt - 6ipJ1t - t hJi).


61tJt* 04.46)
(d) Show that in the group SO(n) with either even n : 2m or odd n 2m * I,
-
we can find m mutually commuting generators.
(e) For the simple case of n :3, if we define
J; - )eii1,Ji* Q4.47)

then the commutators in (c) reduce to the usual angular momentum algebra

[Ji,J1l - ietjrJr. (14.48)

(f) For n : 4, define Ki : -fa, show that

[Ki, Ki) - ieijrJr and [J;, K1) - ieijtKr Q4.49)


where ,[s are defined in part (e). Also if we define

Ai : Lr{1, + x,1, Bt : }{1, - x,) (14.50)

show that

lA;, Ai) - ieijrAr, lBi, B jf : ie;ipBp, [Ai, B1]:0. (14.51)

Solution to Problem 14.5


(a) Write the matrix equation RRr : I in components, we have RijRi* - lio.
ForR;; - 6ij.';,:l',;;';:,:;;

+,jk: -ekj e4s2)


Thus we have Lr@ - 1) independent parameters for the orthogonal matrices R.

(b) xi --> x! : Riixj : xi*s;ixi. (14.53)

Then

f(*!) : f (xi * e;1x1)


- f(x) + r,1*1L
- f(x)+ry(.,#-.,#) (t4.s4)
262 Grand unification I4.5

If we write the left-hand side as

f (x!) - f (x) + lieitJti f @) (14.55)

we get

Jij : -i(xi31 - x13i). (14.56)

(c) From the simple commutator

[Ai, x jl - 6;j, (14.57)

we get

IxiS j,xp01f - xi\ifit - xp6;131. (14.58)

From this it is straightforward to get the commutators for ,[;s:

Uii, Jul - -i(6irJt - 6;pJ1t - 6lJt* -f hJii. (14.59)

(d) If indices (i, j, k,l) are all different, the commutator is zero. Thus the follow-
ing generators will commute with each other:

{Jn, Jy, Jsa,..., Jn-l,n} forneven


{Jn,Jzq,Jsa,...,Jn-2,n_t} fornodd. (14.60)

Remark. This set of n l2 (or (n - I) 12 for odd n) mutually commuting generators


is said to form the Cartan subalgebra.
(e) To recover the familiar angular momentum commutation relations from
eqn (14.59), we use the identification

Jr : Jzz, Jz : hr, Jz - Jp (14.61)

lJr, Jzl : fJzt, Jzrf : -i ln - i Jz. (14.62)

Similarly, we can obtain

[Jz,d-iJr, Ut,Ji:iJz. Q4.63)

(f ) From Ki : J;4,wa get, from eqn (14.59),

[Kr, Kz] : LJv, Jzql : -i(- Jd - i Jz. (14.64)

Similarly,

[Kz, Kz] - i Jr, lKs, Krf : i Jz, or lKi, K il - ierirJr. (14.65)

For the other commutators, we have

Ur, Kzl : Uzt, Jz+f - -i(- Jy) - i Kz. Q4.66)


14.6 Spinor representations of SO(n) 263

Similarly,

Uz, Kzl - iKr, Ut, Krf : iKz, or [J;, K1) - ieij*Kr' Q4'67)

These commutators can be simplified by defining

Ai : iQt + r), Bi : iQi - K) (14.68)

which gives

[Ai, Bl - f,tli * Ki, Ji - Kl - iit,,oQr * K1, - J1a


- Kp)
-Q

lAi, Al - llti * Ki, Jj * Kl - jiei11,1le * Kr -l Jr, * K*) : ie4pAp.

Similarly,

[Bi, Bl: ieiitB*. (14'69)

This means that SO(4) algebra is isomorphic to SU(2) x SU(2), generated by A;s
and B;s separately. Also this implies that the SO(4) group contains three distinct
SU(2) subgroups, namely those formed by {Ail, {Bi}, or {.4} generators.

14.6 Spinor representations of SO(n)


Consider the n-dimensional real space with coordinates (xr, x2, . . . , xr).
(a) Show that if we write the quadratic form xl + xl + ... * xl as the square of
a linear form

*? + *7 +...+ *?: (nn t xzyz+ "' * *ny)z (14.10)

then the coefficient ys satisfy the anticommutation relation

{yi, y j1 - 234. (14.71)

This is usually referred to as the Clffird algebra.


(b) Show that if we take ps to be hermitian matrices, then y;s have to be even-
dimensional matrices.
(c) For the even case, n - 2m, show that the following set of matrices satisfies
the anticommutation relations in (14.71):

- r, vl" :', : (? ;) , v)" :" : (: ;t) 04.72)

and for the iteration from m to m * I we have


y.m+t):(r{i' 0,.,) i:t.2,3,...,2m (14.i3)
11
\ o -y,""'/
v*+'t: (? ;) v;ri;': (: ;') 0474)
264 Grand unification 14.6

(d) Consider a rotation in space (xt, ... , xn)


xi -+ x/ : Oi*xr, (14.75)

which induces a transformation on the y;s

yi - Oiilr. (14.76)

Show that {yil satisfy the same anticommutation relations (14.71):

{rl, vjl - 26ii (14.77)

(e) Because the original {y;} form a complete set of matrix algebra, they are
related to the new {yi} by a similarity transformation

y/ - S(o)yis-t(o) or oir,y*
- S(o)yis-t(o) (14.78)

where S(O) is some non-singular} x2^ matnx. If we write these transformations


in the infinitesimal form

oi*:6i* * e*, s(o) - 1 + )lsij eit, with tik: -tki,


show that

i [Si;, yr) - Glyi - 61ili) Q4.79)


and that such S;; can be related to the y matrices by

s, : f,o, = f,lvo, r,). (14.80)

(f) Show that for the matric., $u* in part (c), we have

o@i-D: ("'f ii, i, i :1,2,3,...,2m (14.81)

o,,TlI,, :, (_),^, *:) o,,,TlI,, :, (_1,,^, r,:)


(r4.82)
o)Ill',^*r: (;t ?)
(g) Show that ,Srr _ ilVr, yi, as given in part (e), satisfies the commutation
relation for SO(n ), as given in Problem 14.5:

[S;;, S7,7] - -i(d;rS;l * d;7S;r - 6;pS;l - 6;r$r). (14.83)

{S;;} then form the spinor representation of the SO(n) group. Clearly it is a
2^ : 2/2 dimensional representation. (In Problem 14.8, we shall study its decom-
position into two sets of 2*-r sptnor states.)
14.6 Spinor representations of SO(n) 265

Solution to Problem 14.6


(a) To solve for ys from the equation Ii *? : (xflr + ... * x,yn)2 it is clear
that y;s cannot be the usual real or complex numbers. The simplest possibility is
that y;s are hermitian matrices so that yiyi * yiyi:

/ \2
D,-?: ( I xiyil - I xixlyiyi :Dxixii(yryi -r yiy). (r4.s4)
i \i / i,i i.j
Thus we need to have

YiYi * YlYi :26;1. (14.8s)

(b) From eqn (14.85) we see that, because yl - l,


yi(yiYi * yiyi) :2yi or YiYiYi : Yi, no sum on /. (14.86)

Thking the trace of this final relation we get

Tr(yiyiyi) - Tr(yi). (14.87)

But for the case i + j, eqn (14.85) implies that


rr(yiyiyi) - Tr(-yiyiyi) - Tr(-yi). (14.88)

Thus combining eqns (14.87) and (14.88) we have

Tr(yi) - Q. (14.8e)

On the other hand, y? : 1 implies that the eigenvalues of y; are either * 1 or - 1.


This means that to get Tr(yi)
- 0, the numbers of *1 and -1 eigenvalues have
to be the same. Thus 7; must be even-dimensional matrices.
(c) The m Becausethe2x2 Paulimatrices
-lcase
{ti, t jl :26ii (14.e0)

satisfy the anticommutation relation of (1a.85), we can choose y,(t) : z; with


i r,2.
-The m
3,...,2m)

_
r,!^*,, ,i^*,l r*lrr,!:,^,,;^,r)

t)r (14.9t)

fi,w*'t,v*l? l
: (-;,, '::) * (;-, -'i^') -0, (14s2)

(vlr,i?)' -- r (r4.e3)
266 Grand unification 14.6

Similarly,

fi,w*'t , v*I;'l : o, {r}f,! , v*I;'}


: o, (r}i#')' -- r.
(r4.e4)

(d) To compute the anticommutator of the transformed gamma matrices:

Oiyj + rjr) - o*o1(wYt * yty) - o*o126u - 26u Q4.95)

where we have used the fact that O is an orthogonal matrix.


(e) From Oipolr : 6ij, we get for Oii : 6ij * e;y with eij KI
(d* * ei)(li* * e1;J - 6ij or ij : - jt. Q496)

Thus if we write S(O) - 1 + \is;iet1, then S;; is also antisymmetric ih i e j.


From Oi*yr,
- S(O)yiS-l (O), we get

(6,r * eir)yr,
- (t + |is,6e"6) rt (t - 1is1,1e1,1) Q4.97)

or

Yi I ei1,Yo : Yi * f\*,rt, fi' (14.e8)

Write

itYrr : ett Ytsit : ieur(yt6rt - yt6*). (14.99)

Thus we get

i [Srr, Yj]: (n6in - Yp6it). (14'100)

We now show that

Sru : L", = f,lvo,v,l


(14.101)

will satisfy the commutation relation (14.100). Since k + I, in S7,7, the above
relation can be written

Sg : ii.(ypyt - ytT*) : :ytyr. (14.102)


z

Then

i [Str, yi : -][vof,, ft)


- -+Q*{yr,Yil - {yr,, yi}y) - (yt6i* - Y*li). (14.103)
14.7 Relation between SO(2n) and SU(n) groups 267

(f) From the definition of o;i and the recursion relation (14.73), we have for the
caseofn-m*I
o@+t) _ ilr!**,, rj**trf
: (, lr!*'or;^'l ,l)
+l_r,^1 ._r,

::\o("!r' o \ (r4'ro4)
";f')
The remaining results stated in part (f ) can be demonstrated in a similar manner.
(g) From the relation (14.102), the commutator can be evaluated:

[$;, S7,7]
- ti[Sii,yr,yil - t]{ISi1, yr,7yt * yilSii,ytl)
- l{Ari6,o - yi6ir)Yt * y1,(y,61 - yi6l))
- -i(d;tSir 167;S;t - 6;7.S;r - 6;tS;t). (14.10s)

14.7 Relation between SO(2n) and SU(n) groups


The U(n ) group consists of transformations that act on the n-dimensional complex
vectors, leaving their scalar product (w . z) : I, w! zi invaiant.

(a) Show that the SU(n) transformations which leave Re(u . z) invariant can be
identified as those in an SO(2n) group. Thus, the SU(n) is a subgroup of SO(2n ).
(b) If we write the SO(2n) matrix in the form R : eM , where M is an antisym-
metric 2n x 2n matrix in the form

M - (_ou, z) (14.106)

where A, C are antisymmetric matrices and B is an arbitrary n x n matrix, show


that R : eM also belongs to U(n) if M has the specific form

*: (!u i) with A antisymmetric and B symmetric. (14.107)

(c) The 2n-dimensional representation of SO(2n) decomposes as n * n* under


SU(n ). In other words, if we write the Zn-dimensional real vector in the form

(r4.108)

show that, for the unitary matrices written in the form as given in (b), the combi-
nation ai * ibl transformed into themselves and so did the combination a1 - ibi.
(d) Work out the decomposition of the adjoint representation of SO(2n) in terms
of the irreducible representations of SU(n).
268 Grand unification I4.j
Solution to Problem 14.7
(a) In the scalarproduct (w.z): I; wlzi we can write w; and 1; in terms of
their real and imaginary parts

?rj:a1 *ib1, Zj:a'j*ib'j. (14.109)

The scalar product can then be put in the form

(w.z) - Ij:r (oio'i+bjb'j) +,I (oib'i -bio';) (14.110)


j:r
which gives

Re(u.z):f,ro,o',-lb1b') (14.111)
j:l

If we collect a.i and bj to define a 2n-dimensional real vector r _


(ar,...,an,br,...,bn) then Re(u . z) can be written as the scalar product of
2n -dimensional real vectors

r .r' : f,ro,o', * b1b'). (l4.ll2)


i:l
The transformations which leaves this scalar product invariant are just the SO(2n )
transformations

ri + r! : Rijri where RRz - RzR


- 1. (14.113)

From this we see that the SU(n ) group is a subgroup of SO(2n) whose transform-
ations leaves both Re(rl . z) and Im(u.' .z) invariant.
(b) and (c) By definition aU(n) transformation on the n-dimensional complex
vector space is of the form

Zi --> Z'i : (IijZ j with (J(lt - (JI (J


- 1 ( l4.ll4)
where the unitary matrix U canbe written as

(J : eH with
- -Ht being an antihermitian matrix.
F1 (14.115)

Forinfinitesimal H, wehave U - I + H and

Z'i: zi * Hiizi + z',* : zi + ni,zi. Q4.II6)


Thus

z'i * z'i* - ki + r!) + l@i1 * r:,)(21 + z:) + l{n,i - Hi)ki - zi)


(r4.117)
14.8 Construction of SO(2n) spinors 269

and

ali: ai I A4a1 * Biibi (14.118)

where a and b are the real and imaginary parts of z:

ai +ki + zi), bi : -iLki - z),


-
Aij |{n'1 + H:j), Bij il@4 - H;) (14.119)

are all real. Since F1 is antihermitian, we have

H! - -Hi,. (14.120)

This implies that

Au -Ai' and Bii: Bii' Q4,2l)


sim'arly,
b't:br- Bijaj*A4b1. ( rrO.rZZ>

We can combine these two transformations as

(;) : (;). . (;) e4 r23)

where M is an antisymmetric matrix of a specific form

M-(o^ 1)
A/ with A--Ar,B-Br. (r4.t24)
\-8
(d) From (b) and (c) we have learned that the vector in SO(2n) decomposes into
n * D* of SU(n). The generators in SO(2n ) can be associated with second-rank
anti-symmetric tensors. This implies the decomposition of SO(2n) generators as

[(n +n*) x (n * n*)]1-) - (n x n)r-i + (n x n*) + (n*xn*)1-;


n(n-l) .. : - n(n-1)*
2 O(n--1)el(F^ 2

where the subscript t-l means antisymmetrization. For example, we can decompose
the 45 generators of SO(10) in terms of the ineducible representations of SU(5):

45
-10+24 + 1+ 10*. (14.125)

14.8 Construction of SO(2n) spinors


The y-matrices given in Problem 14.6 can also be written as a tensor product in
the form (Note: the integer m of Problem 14.6 is being called n in this problem)

,.(n-tr\ - y,(") x 13, i- 1,2, ...,2/t,


y;II'i - 1@) x rt, yX! - r@) x rz

where 1(n) and y,@), *"2n x 2" hermitian matrices.


270 Grand unification 14.8

(a) Show that

YII):1x 1x "'x T2XT3X...Tt


yl?-,-1xlx...x 11 x13X...rz Q4.126)
where the 2 x 2 identity matrix 1 appears (ft - 1) times and 13 appears (n - k)
times and

oli)-l:o:1x I x "' X13Xlx...1. (14.127)

(b) For the chirality matrix, we define

yprun
- (-i)"(nyz...y2). fl4.128)
Show that ypry6 can be written as the direct product of n Pauli u3 matrices:

Yrrya:z3XT3x"'T3 (t4.129)

and

y8;! : (,8i' _)n,) (14.130)

(c) Since the natural basis for Pauli matrices are spin-up l+) and spin-down l-)
states, we can take as the basis for the y-matnces the tensor product of such states

Itt,2,...,tn) =let)lez) ...1e") with s;:*1. (14. l3 1)

Show that 2n such states in the SO(2n) spinor representation decompose irre-
ducibly into two set of2-r states, called S+ and S-. They have the property
of

ror
fi',: {:l !l"u'"' (14.r32)

(d) For the case of n : 2, suppose we embed the SU(2) group into SO(4) by
identifying e, of SU(2) generators with Bp generators of SO(4) as defined in
part (f) of Problem 14.5. Show that spinor representation S+ and S- reduce with
respect to the subgroup SU(2) as

,S+ -+ 1+ 1. s- --> 2. (t4.133)

(e) Show that for the case n : 3, the spinor representation S+ and S- of 50(6)
reduced with respect to the subgroup SU(3) as

S+ --+ 3*+1. S- + 3 + l. (14.134)

(f) Show that under SU(5), the spinor representation S+ - 16 of SO(10) reduces
as16-10+5+1.
For more details of SO(2n ) spinor construction, see Wilczek andZee (1982).
14.8 Construction of SO(2n) spinors 21I

Solution to Problem 14.8


(a) and (b) Let us work out the tensor product expression of a few low-
(n)
dimension al y matrices:
Forthen: l case:
y{t) : ,r, y}t) : rr., y;}l,t - -ir1r2:73' (14'135)

Forthe n case:
-2
yr" : 11 X 13, y;" - 12 x 13,

yr":lx11, y;') -lxrz,


vflru - ?i)2yt" y{" y(2) y(2) - -(trtz) x (4t1t2)
: 13 X 13. 04.136)
For the n -- 3 case:

Yrt' : T1 13, Ylt' : T2 X T3 x 13,


X T3 X

Y;3) - 1 x rv x 13, Yl" : I x t2 x 13,


Yrt' : I x I x 21, Ytt':l xl x12,
vf,l,t - (-i)3 yG) ,ttt y]t y6't y(3) y(3\
: 13 x 13 x 13. (14.137)

From these it is not hard to obtain the general (n) case by induction

vl?-r:1x I x "'rr X rzx"'X13, yli):1x I x "'12xQ x "'x13,


where the identity matrix I appears (k - l) times and 4 appears (n - k) times.
We can also explicitly calculate the commutators:

o)i'-r,ro:llrli'-r,rlt)f- 1x 1x "'rzXl x "'x 1 (14'138)

and deduce

Y#l,u: 13 X 13 x 13 X "' X 13. 04.139)


(c) It is easy to see tnt yfi)ruanticommutes with 7s. For example,
y#l,uyro-t : 13 X rz x "' x (4r1)"' x r? t? (14'140)
"
Yu<-tv#l,u: 13 x rz x"' x (rrz:) x fi "' x Q i04.141)
which gives

lv[ii,r,Yzr-r] - o' (t4'142)

Since o;; is quadratic in y;s,

lv*l,,,*,]
: Q4:43)

which implies tnut yfrl,ucommutes with the generator /,y, which are represented
by Jii : tro,iin the spinor representation. From (r#i,r)' : l, we can decompose
272 Grand unification 14.8

the2-dimensional spinor into two 2n-r-dimensional representations S+ having


y$l,ueigenvalues of *l (Schur's lemma). It is clear rhat

v$lrultr, t2, ..., tr) : (tz X 13 X . .. x tz)le r, t2, . .., tn)


: (fi '') '
t2' " ' ' tn)' (14'144)
'''
Thus S+ states have fli:, t :Land S- states fll:, ti : _1.
(d) Applying the above results to the SO(4) group which has the ineducible
spinors

s+: 2+ - (i I 1i) and s- :2- - (i i;i) e4 t4s)


and embedding SU(2) into SO(4), we can identify the SU(2) generators with a
subset in the SO(4):

'Tk' --> Bt : Jt, - Kt : Lto,iJij - Jtq. (14.146)


For example,
'r3'--, Jp- l@rr-ozD:!{-U x 1* lx4).
Jzq--> e4.147)
In S+, we have I + +), | - -) and both have a zero'r3' eigenvalue, e.g.
'tz'l+*) : itt-tl .l +1.(+r;11 ++1 -s l4.t48)
Similarly, for the | - -) state

'"' (l i ii) :' Q4 t4s)


Namely, both members of the S+ spinor transform trivially under SU(2). This
implies that2+ -+ I * 1 under SU(2). For the ,S- states it can be similarly worked
out and the result can be written as
,"'
(l t;i ) : (;' ?) (i 1;i) ( 4
'so)
Thus we have 2- --> 2. Namely, the So(4) spinor S- is simply an SU(2) spinor.

Remark. The identification of the SU(2) generators Tf withthose in the SO(4)


can also be carried out through the identification of their respective indices: 6
i, ) fo, SU(2) and i : 1,2,3, 4 for SO(4). -
i:1+i2, ):3-li4
Tl :
A
Jz+iq,z-iq
- iJqt - iJz+ - -2iJy: _ioto
- -+(1 x 13) (r4.15r)
Similarly, we have fl - -|.{tt x 1), so that we check with the above result,
'r3':Ti -r: -!{-rrx 1* 1x4). Q4.152)
14.8 Construction of SO(2n) spinors 273

(e) We can embed the SU(3) group into 50(6) with the identification

i-1+i2, )-3+i4, 3-5+i6, with d-SU(3)index.


The SU(3) generato rt W! are related to J6pt by these relation of indices, e.g.

Wi : J+iz,r-i2 : iJn - iJn - -2iJn : _l.or,


- -+(rz x 1x1).
(14.1s3)

Similarly,

w: - -itt x z3 x 1), w:: _i<t x I x 13). (r4.ts4)

For Si, we have the states I + ++), I + - -), I - +-), I - -+), and their quantum
numbers are given by

State wl - w] wl +w] -zw]


+++) 0 0
+ --)
+-)
-__+)
-+
+t
-1
-1
0 2

(It may be helpful to think of W| - W] as the third component of isospin and


Wl + W? - 2Wl asthe hypercharge operator.) Thus we see that the states +- I -),
| - +-), I - -+) form the triplet 3* representation under SU(3)

- (14.1ss)
(i :1;i) '
This means that S+ decomposes under SU(3) as 4+ -> 3* * 1. Similarly, S-
decomposes as 4- -+ 3 t 1.

(f) In the spinor representation S+ for SO(10), we can denote the states by

ler, t2, t3, 4, ss, eel with ff t, - t. (t4.rs6)

We can identify the firsttwo r;S &s the spinor states of SO(4) which contains SU(2)r
and the last three e;s with 50(6) which contains SU(3)c. Then the SU(2)r x
SU(3)c quantum numbers of spinor representation S+ are given by

(i I1i) " (i : 1;i) + 2, 3*, --+ Up, dp (t4.rs7)

(i 1Ii) x I + **) -+ 2(l,l) -) up, p (14.1s8)


214 Grand unification 14.8

(i 1;i) xt+++) + (2,r),(:) , (r4.tss)

(i 1;i) . (i lt.i) - (2,3) - (x) , .4 160)

It is easy to see that these


are just 5 + 10* * 1 representations of SU(5),
see Problem 14.1. The SU(5 ) singlet can be identified with the right-handed
neutrino up.
15 Magnetic monopoles

1 5. 1 The Sine-Gordon equation


Consider the Lagrangian for a scalar field @ in two-dimensions (respectively, one
space and one time)

1^l"a
L:
,(}oQ)' - tQ,il' - ;[1 - cos(bd)l (15.1)

where a and b are constants.

(a) Show that the equation of motion for this Lagrangian is of the form

a|O - alO +asin(bQ) -Q (rs.2)

which is called the Sine-Gordon equation.


(b) Verify that the field configuration

Qe,t) -|run-' {.*ol*au>''' !-9fl (rs 3)

with u being an arbitrary constant, is a solution of the Sine-Gordon equation.


(c) Show that the effective potential V(Q), given by V(il - a(l - cos bf)/b,
has degenerate minima at

o:^,n, _ r2,, (15.4)

where n is an integer, and the n.fO *nn*ururtln, given by eqn (15.3 ), interpolates
between two such minima of V (Q):

Q@--@,)-L@in)
Q@ - +oo, t) - O!#1 . (1s.5)

(d) Show that the energy carried by the configuration (15.3) at /- 0 is

t a:,112
E (1s.6)
-8 (;)
(e) If we write the Lagrangian 4 in powers of Q, flnd the mass and the quartic
coupling constant in terms of c and b. Express the energy E 8(a l6z1t /z in terms
-
of the mass and coupling constant.
276 Magnetic monopoles 15.1

Solution to Problem 15.1


(a) The equation of motion which follows from
AL AL
i,o: a. NarQ)
(05'7)

is the Sine-Gordon equation:

At'Apf * a sin bQ : 0, (15.8)

i.e.

a1O - alO +asinbQ -Q. (15.e)

(b) We will show that the field of the form

q (15.10)
Q@) -!run-t
D
where

6 : exp l{ou)t/'y(x - g], (15.11)

and y : (1 - u2)-r/2 with u being arbitrary, satisfies eqn (15.9).


Using the formula

A _, : I 3y
*tan-' ! yz 6x 05.12)

we obtain -4
a0 4 I a 4 (15'13)
;- br+qr;-;r.e@b1r/2,
and

a2O 4ay2qg - 2)
(15'14)
,i-- a;?f-.-'
Because the function Q(x, r) has the space-time dependence through the combi-
nation of (x - ut), the second derivatives must be related (as in the conventional
wave equation) a30 - r'40
- 0. The above result can then be written as the first
and second terms of the Sine-Gordon eqn (15.9):

azO a'O -4a(l - 2)


(15'151
At'- A*':-1l+Ery-'
Now calculate the third term in the Sine-Gordon equation:

sinbQ: sin 4(tan-r 6) - sin4g with e -tan-l . (15.16)


15.1 The Sine-Gordon equation 271

With the help of the formula sin40 2 sin 20 cos20, and


-
cin'lA
2tan0 2 l-tan2o 1_ 2
and cos2o:4run''r:4y'
l+tan20 I+q2
(1s.17)

the sine term can be evaluated,

4r:I -rr')"
a sinbQ: a sin 4o : (1s.1 8)
0 + ,2)2
which just cancel the first two terms calculated in eqn (15.15):

a|O-alt+asinbQ-Q. (1s.1e)

Remark. This solution has the space-time dependence through the variable f-
expflaUltl'y(* - rD]. One simple way to understand this is to note that iT we
write

I x-ut
f :- (1s.20)
(l - Yz1t 1z

then x' is related to x bya Lorentz transformation. Thus in this variable .tr', so
that f - expl(ab)r/2x'f, the function Q(6) is expected to be a solution to the
time - indep endent equation of

alf - a sinb|. (rs.2r)

Let us demonstrate this. To solve this static equation by integration, we can multiply
both sides by 3*Q so that it becomes

(ts.22)
;*(#)': -;*(cosb@)

! ( 4!\' ++l{""'bQ)
z\ax ) 3 - c.
(cos bg) c' (rs.z3)

If we use the boundary condition of 0,Q - 0 and bQ : 2nn, as -r -+ *oo, then


the integration constant is fixed to be c
- alb. We have

(#)' -- T( 1 - cos bQ) : T ""' T (rs.24)

do (?\tt'
dx -
+2
\b/ ,in\2 (r5.2s)
278 Magnetic monopoles 15.1

Integrate over the equation

fd6
_J_ t
_J_
I'
sin(bQ 12)
, (;)''' I o* (ts.26)

|
t(ab)t/2*' - !ln -cos(bf12) I _,n (o"4\ (rs.27)
t+cos(bQl2)l \ 4/
which just checks with our result of

,b +tan-r {exp [+14


A - u)t/'*'f| . (1s.28)

Thus we can get the general solution by boosting this static solution in the x-
direction by a velocity u. In other words, the general solution is moving in the
x-direction with a velocity u.

(c) From V (il


- a(l - cos b0) /b we have
AV
: asinbQ' (rs.2e)
AO

so that AV lA0
- 0 can be satisfiedby bQ : fttr withm - 0, *1, 12, .. .. Such
extremum points are minima if

a2v
: ab cosbf > 0. (15.30)
a6,
Thus if we take ab > 0, then cosmTT > 0, only for even m : 2n. Thus the minima
of V (Q) are located at

o:^n,_2+
rn
b
n:0,+1,+2,.... (15.31)

Wenowstudytheextrapolationof this Q@,t) fromx -+ ootox -+ -oo.Taking


the x -> oo limit, we have

f: exp l@blt/'y(* - ull --> oo, Q : 4Utan-'t - i2n : fr,rl',.r.r.rD


,(min)

Taking the x -+ -oo limit

6 : exp f{ou)tt'y(x - gf -+ o, o : ;ran-r 6


: o
- o:y;). (15.33)

Thus, the configuration

Q @, t) - |run-' {.* [*,,r, " ffi1y (rs.34)

interpolates between two minimaot V (Q).


15.1 The Sine-Gordon equation 279

(d) The conjugate momentum is given by

" -- # - ool (1s.3s)

and the Hamiltonian density is then

T1
-r*oQ - L- l?oil'++@,ilz +v@). (15.36)

The total energy E for the time-independent field is then

E-fo.lr ^+ v (o) -l (15.37)


J y2@-Q)" J

Because eqn (15.9) can be written via eqn (15.29) as

(a3- al)o*#-0, (15.38)

for the static case Es@ : 0, we get

. * av ^ or a,oat avaf av (15'39)


-a;Q uo:o a-, a-: ao a*: ar
This implies that

*l:W)' -u] :o or ;(#)'-v:c, (,s40)

To calculate the constant c1 we can set.r _ oo which gives 6 : 0, Q : 0, hence


SQ/Ax - 0 and V
- 0,leading to c1 - 0. This means that for the field which
satisfied the equation of motion, we get

l/a6\2
t\;) -v or dx / l\r/z
oo-\*) (rs41)

and the total energy is then

, - l rrl;G.o)z+ v(d)] a,z, (Q) : * wfiao


-| l
- lo'"'o
,zv (il)rrz dQ : (T)''' Io'"'o ,1 - cos bD)'t2 ao
:t(h)'''. (1s.42)

(e) Fromthepowerseriescos0 - 1- ie'+ *.go +...weget

v(il -7rr-cos bQ):)"uf'- {f^+.... (1 s.43)


280 Magnetic monopoles I5.2

Comparing this to the standard form given by

u2 ^ )"
: tQ"
V (0) - 4t0" 05.44)

we have

u.2 : ab, )" : ab3. (15.45)

The energy is of the form

t a t,l/2 u3
E-8(a;/ -8;. (t5.46)

Note that in a two-dimension fields theory, the parameters ). and p.2 have the same
dimension because @ is dimensionless.

15.2 Planar vortex field


Consider the Higgs Lagrangian in two space and one time dimensions (p - 0, | , 2)

L - -Ir*,Fu, + (DrQ).(DrQ) + p2o*Q - |ro.or, (:s.47)

where Fp, : 0rA, - 0uAp and D*Q


- 0pQ -f ieArQ.
(a) Work out the equations of motion from this Lagrangian.
(b) Show that the time-independent field configurations A and rp with Ao :0 and
having the r -> oo asymptotic behaviour (in the two-dimensional polar coordi-
nates)

A(r, g) -- 1 yfu.), 0?,0) orin. with (+\t,


"-
(15.48)
"n"fO
---,
\r/
will satisfy ,t. equations worked out in (a) upto O (r-2).
(c) The magnetic flux O I B. ds is related to the gauge field A of eqn (15.48)
-
on a large circle C at infinity as O : fc A. dl. Show that this flux must appear in
quantized units:

2tr
Q,: (15.4e)
e

Solution to Problem 15.2


(a) From the Euler-Lagrange equation for the @("r) field:
AL : AL
aw (15'50)
aq. 61qq1'
we obtain

Dp(Dp)Q
- pzO - ),o(OQ\, (15.51)
15.2 Planar vortexfield 28t
while the equation of motion for the A, field
AL : AL
a, (rs.s2)
w ararl,l
works out to be

te (6arQ* - Q.ouQ) + 2e2 Ar(0.il : o' Fpu. (1s.s3)

(b) In the polar coordinate system, we have g (?E /0r, 00 0 / ,). Thus for large
-
r, the asymptotic form we want to use can be written as

vO - 0i2o
",nr, A(r,0) : !v@r, : as r -+ oo. (15.54)
*A
Since Ao :0 and @ is independent of r, the full covariant derivative is given by
the spatial part, which vanishes asymptotically as

DQ :(v - ieA)Q
- lr'# - i,Yr] - oo-z) as r -+ oo.

(1s.ss)
Also from Q?,0) : a ei'o , we have

p,2Q-).(Q*O)Q-0 (1s.56)
where we have used a2 p.2 /L. Thus eqn (15.51) is satisfied up to terms of order
o(r-2).
-
We now show that eqn (15.53) is satisfied by these field configuration. The
righrhand side vanishes to o(r-2) because the field tensor Fr, --> o(r-2) as the
gauge field can be written as a pure gauge, eqn (15.48)

as r -+ oo (ls.s7)
ry
rhe rerrh""o ,,0i"*: i"i,n. ,i,tirl,:
,,1+e - --:4u] . * (*b) "' - o(r-2) (1s.s8)

Equations (15.53) and (15.51) are satisfied because both sides of these equations
vanish, at least to O(llr2).
(c) From Stokes' theorem, we get
ff
o- Js
/n.as-$t.at
Ic
(1s.se)

where C is the boundary of the surface S. In the limit of r -+ oo, we have


A(r, 0) --> (l / e)Y (n0) , thus the components

A,-->0 and er-* (1s.60)

and the flux quantization condition follows:

o- If2, Anrdo:tl
"
nf2,
eJo
do- 2n
(1s.61)
Jo e
Magneticmonopoles 15.3

15.3 Stability of soliton


The equation of motion for a scalar field in two dimensions can be written as
AV
where (Ls.62)
'Q+lr-o '-a3-4
Consider a small perturbation around the time-independent solution QoQ),

0@, t) : ho(x) i 3(x, t) (1s.63)

where 6(x, t) is a small quantity.


(a) Show that, to the first order in 6, the perturbation 6(x, r) satisfies the equation
a2v
trd(x, t) +
I

t):0. (rs.64)
Wlr:rour*,
(b) Take 6(x, t) in the form of a superposition of normal modes:
6(x, t) - Re
Do, ei'"'rlrn(x). (1s.6s)

Show that

-t!t
dx2
+ v' (QilrL, - of,rb,.
| (1s.66)

(c) Show that if Qo@) is a monotonic function (i.e. has no nodes), then all
eigenfrequencies are non-negative.

Solution to Problem 15.3


(a) The unperturbed static solution Qo@) satisfies the equation
ldo * v'(Qd - o or - alOo -f v'(401 - g. (Ls.67)

Substitute Q : Qo * 6 into the field equation, n(do + 6) + V'(Qo + 6) - 0. For


small 6 we can expand V'

v'(Qo + 6) - v'(0d + v" (Qil6. (1s.68)

Then the equation of motion is, for small d,

.Qo * V'(oi + trE + V" (0il6 - 0 (1s.6e)

!6 + V" 16s16 - 0. (1s.70)

(b) Substituting the normal mode expansion 3(x, t) : Re Ln on ei'^' tLn(-r) into
the above equation, we get

d2tb-
-# * v'(Qillrn: to?nlr, (rs.7 t)
which can be viewed as a Schrcidinger equation, rl.r"being the eigenfunction with
eigen-ener1y E, - a2".
15.4 Monopole and angular momentum 283

(c) From the equation for @s,

-a',oo+v'1601 -9, (ts.72)

we get, by differentiating with respect to.r,

-#(#) +v"(Qdff:o (ts.73)

This can also be viewed as a Schrcidinger equation with


S being an eigenfunction
with zero energy En - af, - 0. Then if @e is monotonic it has no nodes. It
is a well-known theorem that for a one-dimensional Schrcidinger equation with
arbitrary potential the eigenfunction with no nodes has the lowest energy. Since
this eigenfunction has zero energy, all other eigenvalues are positive. Note that
with normal mode frequencies all positive for the perturbation 6, the solution @s
is stable.

15.4 Monopole and angular momentum


For a charged particle moving in a monopole field, the Hamiltonian is given by

H- -jr' * v(r) where D - V - ieA (ls.74)

where A is the monopole vector potential given in Cl-eqn (15.25).

(a) Show that

[Di, riJ - 3i j , LDi, Dj] - -f eSeii*ft. (1s.7s)

(b) Show that the angular momentum operator L defined by

L - -ir xD - eg\
r
(15.76)

will have the usual commutation relations for the angular momentum operators,

ILi, - ieij*Dr, LLi, r jf : is;iprp,


D1)

lL;, L1): ieiipLp, fL;, Hf - Q. (ls.77)

(c) From the Heisenberg equation of motion show that

dr .D (1s.78)
dt: -i-
which implies L: mr x dr/dt - egr/r.
284 Magnetic monopoles 15.4

(d) Using the identity

1
D.D - (D D)1o.r) - (D x r);(D x r) (ts.79)

and the relation

L .L - -(r x D)2 * u'g', (1s.80)

show that the Hamiltonian on a subspace of states for a given total angular momen-
tum is of the form

I f A2 2A1 + t;
fi--2*l**;*l* [1
(/
2*,t
- 12s21 - t"
_l
( 1s.81)

(e) Show that the quantum number I can take only values of the form,

I - legl, legl * l,legl + 2, . . .. (1s.82)

Solution to Problem 15.4


(a) The vector potential for the monopole, according to Cl-eqn (15.25), has the
form

A,:Ao:0, . s, (l -cosd)
AQ:; (15.83)
,*,
which has a string on the negative z-axis. Expressed in Cartesian coordinates, it
has the form

A, : t#, Av: s#+ Az:


r), o' (1s.84)

Since A depends only on the coordinates (not on the derivatives), it is easy to see
that

lDi,ri): [3i,ri):lii (1s.85)

[D;, Di): - ieAi,0j - ieAi) - -iel\i,Ail- ie[Ai,}il


[3i

- -ie(}iAj - 3iA,) - -ieeii1,B*: -iegsijk+ (15.36)

where we have used the fact that the monopole field is given by Br, gr1,f 13.
-
(b) Given the definition of (15.76)L
- -ir x D - egl we now compute the
commutators involving L;s :

[L;, Dl - l-iei1,1roD,, Dif -'r17, D,f (1s.87)


15.4 Monopole and angular momentum 285

The first term on the right-hand side is

-ieiut*[Dt, D1) * frp, D1]D1) - -i e trt


lr1,ei
e g) tj,} - s y, or]

-'r (y - T) * iei11D1. (1s.88)


The second term on the righrhand side in eqn (15.87) is

'slv,,Dit - "r,,1:,
r,7- est+ -*T (ls.se)

where we have used [D; , f (r)l - f '(r)Or /\ri. Combining these two terms, we
get the result

ILi, Di) - ierj*Dp. (15.e0)

This means that D; transforrns as a vector under the rotation.


The next commutator to compute is

lLi, rl : l-ieip1r1,Dt, rif -,r17,,,)


- -iip1rplD1, r1) : is;iprp (1s.el)

which just confirms that r7 is a vector.


We now need to check the basic angular momentum commutation relation

[Li, L i7 - -.iei*tfLi, r1,Dfl - ,Slf ,,7] (ts.92)

The first term on the right-hand side can be calculated using the commutation
relations of (15.90) and (15.91):

:W,iA':*r
^ri,,.,u r:;I; ( 1 s e3,

where we have used the identity abcade : 6aa6r, - 3ur6"a; the second term on
the right-hand side in eqn (15.92) is

-,slr,, 7f - -eBILi,,jt: - -rgrr,io| - (ts.e4)

Combining them, we get the expected result

lLi, Ll: iij^ (-ie^pnr1,Dn - ?rt) : itii*L*, (1s.es)

showing that L;s, as defined in (15.76), are indeed the angular momentum
operators.
286 Magnetic monopoles 15.4

The Hamiltonian is of the form

H - -fir'*v(r) (ls.e6)

and the commutator with the angular momentum operator is

[Li, Hl : -;[Li,D2] * ILi, v (r)) : I D D I


2*[Li, 1

: 1
* ieiipDl Dp) - O. (15.97)
Z*QeilpDpDi
This means L is conserved by the Hamiltonian which describes the motion of
particle in a monopole field. These calculations verify quantum mechanically that
the monopole's contribution to angular momentum is indeed given by -egr lr.
(c) From the Heisenberg equation of motion:

# :irH.rit:r(- *)rD2,,if:-* (rs.e8)

Thus we can write the angular momentum L of eqn (15.76) as


dr r
L-m , , eg;. (15.99)
d, -
The first term is the familiar particle angular momentum. This again confirms the
interpretation of the second term as the angular momentum of the electromagnetic
(monopole) field.
(d) In the identity

D.D - (D n)(r.D) - (D
" ")i(r
x D) (1s.100)

we have

r.D - r.(V - ieA)


- r.V - r!0r (1s.t0l)

where we have used A, - 0. Also

D.r-r.D* 3:r**t (l5.lo2)

Then the firsr term of rhe identity (15.79) is

(D r)
)<, D) : (,**,) ) (,#) : #.?* (,5 ,03)

From lDi, ri7 - lij, - -r x D and


we getD xr
[(r x D)t,r2] -lr2D3 -4Dz,rl +rl +r?l- 2rzry -2ryr2-0. (15.104)

The second term of the identity (15.79) is then

11
-(DxI);e xD): AG xD)2. (1s.105)
15.4 Monopole and angular momentum 287

Combining them, we get

6z 2a
- #*i *- 7t, x D)r.
I
D.D (15.lo6r

The last factor can be related toL2 because the definition (15.76) leads to

L.L--(rxD)2 +r'g'. (15.107)


We then have

: 6z 2a y2-r2t2
D'D
ar:,
+; a,
* ,, (15'lo8)
The Hamiltonian is then

H - -lnt
2m
*v(r)
: - 2^I f A2+ ;2Au + --;'-
az-e2s21
v (r) (15' loe)
l* l+
and for states with orbital angular momentum l, we can replaceL2 by /(/ * l) to
get

* t)-- e2s2l
H1 tg +?!
--:'2mYor" rdr'*t(t ,z l+v{r)'
(l5'Ilo)
(e) We can write the usual spherical harmonics as

Yf@,Q): @,Qll,ml (15.111)


where ll, m) is the eigenstate of the angular momentum operator and 10, @) is the
eigenvector of the particle with angular position at polar angle 0 and azimuthal
angle @. Under the rotation characterized by Euler angles d, fl, and y, we have

,_iLza e_i,,fl e_iL,y ll, m) _I DX)*(a, f , y)ll, m,| (15.1r2)


m'

where DX)^(a, f , y) : ,im'u4Q)*(fl)e-i*v and dfi)*(fi can be found in books


on rotation group. A particular case of the above relation is, for a : y 0 and
-
fr: -0,
,,t,o lt, m) : D,aS)^(o)lt, m,). (15.1 l3)
m'

On the other hand, P, Q) can be obtained from l9 0) by rotations


-
10, il : g-i1,4 e-iLve lg _ 0). (15.114)
Thus we can write the spherical harmonics as

Yi'@,Q): @,OV,m): (0 -\leiLve eiLzbV,m)


- \- --i*0 dL)*el)@ - 0lr, m').
-?- (15.1 15)

This means that we can construct Yf @, @) from (0


- 0ll, m').
The constraint on the eigenvalue I can now be obtained by investigating the
structure of (0 Oll, m').
-
288 Magnetic monopoles I5.4

The special case of eg


- 0: L is the same as the usual rotational operator, and it
is easy to see that

n-iL,u l0
- 0) : lg - 0) (1s.116)

because a particle in the z-direction (9 0) is invariant under rotation about the


-
z-axis. From this we get

(0 : \leiL,o ll , m'l - (e - 0ll , m'1. ( l5.ll7)


On the other hand, eiL,oll,m') : ei*'Qll,m'| and (sim'Q - D(e - 0l/, m') :0.
This means that

(0
- 0ll, m'l +0 only when m' - O, (1s.118)

which implies that the allowed values for / are 0,1,2,3, ... .

The general case of eg I 0:

Lzto -o) - f-r(r x D). -,r (;),.1 t, - ol.


L
(1s.119)

Since 0
-0 corespondstox -y:0and z+0, wehave
(r x D).10 - 0) - (xDy - !D*)10 - 0) - Q (1s.120)

(;). to : o) - lo - o). (rs.t2t)

Thus we get

Lzl) - 0l - -esl0 - 0) and ,-iLzu lo : o) : ei'Bo lo - o) . 05.122)

Then

(0
- OleiL,oll,m'l : s-iegu p - }lt,m') : r-im'a p - }lt,m) (15.123)

or

,_i(m,_es)a @ _ 0ll, m,l _ 0. (ts.r24)

This implies that the matrix element (0 - 0ll, m') + 0 only if m' : eg. Since
I> lm'1, we have

I> legl or I - legl, legl + 1,.... (1s.125)


1 6 lnstantons

16.1 The saddle-point method


The transition amplitude for a particle moving in a one-dimensional space, when
written as the path integral, is of the form

(xytlxil)-(xr1r-iatthpi) -N (16.1)
lWdr't,o

,:fi-tv(x) (16.2)

and

s - [' o,'lY (q\' - r(")'l (16 3)


lz \ar, ) '^'_l
' '

N is the normalization constant, L

(a) Show that in the Euclidean space t --> -ir , we can write

(xyle-H'tov) - * | w.lr_s'/h .

where

sr: Io'
o,l;(#)'*ru,]
(b) One can use the saddle-point method to,obtain a semi-classical result. Show
that in the limit h, --> 0, we have

* [ W*le-sn/h: Ne-sr('o)/n [det(- a? + v @o)]-t''


J'
: Ne-su(ntn]# 06.4)

where xo!) is the classical solution, obeying the equation of motion

65 d2xn
u"o - -mii *v'(xs) -Q (16'5)

and ,l,ns are the eigenvalues of the second derivative operator,


s2 --

-# +V"(xs)xn: Lnx,. (16.6)


290 Instantons 16.1

(c) Show that the matrix element (xf V-a'/hlxi) for large z is of the form
(xy1e-H'/hl*,),1*r-notlh(xyl0)(0lx;) (16.7)

where l0) is the ground state with eigenvalue Es of H,

Hln) - Enln) and Eo < E, n+ 0. (16.8)

(d) Show that forxs which satisfies the equation of motion (16.5), the combination

! (4*o\'
\* ) - v(xo)' (16'e)
Z
interpreted as the 'energy', is conserved.
(e) Show that if xe(z) satisfies the equation of motion (16.5), then (dxs/dr) is an
eigenfunction of the second derivative operator, with zero eigenvalue,

-#(#)+v"(-il(#) :o (16 10)

Solution to Problem 16.1


(a) Making the replacement t : -ir rn eqn (16.3), we get

sa : -i s:
Io'
o,,l; (!+)' * ru,] (16 11)

(b) We are interested in calculating the transition amplitude, which is proportional


to [gx1e-se/h. Thus we need to compute Ss and ldx].
In the semi-classical limit ft --> O,we can use the saddle-pointmethod to evaluate
the integral for 56. The saddle point.rs of Sa satisfies the first derivative equation

* : -m# .v'(xe) - Q' (t6.tz)


l.:,,
Thus, near the saddle point the leading correction to the classical action is the
second derivative term. We can write 56 as (see, for example, Problem 1.6)

Sr(r)-Ss(xo+ry)

- s6(xo) +
Ir'
or'rl-# t v"(xsv4] . (16.13)

Write

x(t) - xo?) + I cnxnQ) oul -Dcnxn(t) (16.14)

with xe(O) : xi, xoi) :n*r, and xn(O) : xni)- OiOtro xns arechosen to be
orthonormal,

,,(t')x*1t'ydr' : 6n-. (16.15)


lo'
16.1 The saddle-point method 29I

Then we can take the integration measure to be

tdxt-tdnt-A#oq (16'16)

and

sf) : or'rl-fu* + v"(xila]


Io'
:nr,r^ dr'x*l-# +v"(xo)x,f. Q6.t7)
Io
We can choose .rrs to be eigenfunctions of the second derivative operator

d2 rn
- t + V" (xo)xn : Lnxn (16.18)

to carry out the integration, '

Sf): *,(r')x*1r'1dt')", -lcf;x, (16.19)


fir,r* Io'
and

f w.se-s,/h
l-t --sr/h :-n f dcn ^..^ t - s x,'11
J l;l J,6*..0 +; 1 )
:l;l#:dr, Q62o)

with O being the operator:

o- -#. v"(xo). e6.2r)


(c) From Hlnl - Enln ), we have

6 rv-
n' /o
l*,1 - D,t., 1e-
H' /k
1ny @lx il : r- t" /u (* r ln) (nlx ) . (16.22)
T
Since En > Es for n + 0, the ground state will dominate the sum for z large,
(xy1e-H'l'l*,) --r r-totlh(xy l0)(Ol,ri). e6.23)
(d) By multiplying dxs/dt to both sides of eqn (16.5), we get,

d f m /dxo\'-yr"nll :0.
nlz\;) - v(xo)J - (16.24)

We can thus interpret

E
-T (#)' - v(xd r'62s)

which is a constant of motion as the 'energy' of the particle in the Euclidean space.
292 Instantons 16.2

(e) By differentiating with respectto z both sides of the equation of motion (16.5),
we get

L(4\-v"(+) -o
mdr2\E)-, (t626)
\n )-".
This means that dxs/dt is the zero mode of the second derivative operator.

16.2 An application of the saddle-point method


(a) Show that for the case of a free particle, Hg - p2 /2m the transition amplitude
is given in the Euclidean space by

(xrtolx;o;
- (x11e-H'/ot*,)
- (*)"' "*ol--t -;Pl (t6.27)

(b) Compare this amplitude with the formula derived in Problem 16.1(b) to show
that

rufo.t (-#)]-'/2 -(#)''' ,6287

(c) Use the above results to find the ground state eigenfunction ,Lo@) and energy
Es for the case of a simple harmonic oscillator,

n2m
H - h* 1a2x2. (16.29)

Solution to Problem 16.2

(a) (xyle-Ho'o/ol*,):
l#@ylil(ple-p"o/'*ol*,1
ao
[
- J2n ,-P"o/z*h rip(xv-x)/h'

:
I*"*ol-*P2+'+')
_ ( _mh, \ttz f (xf - xi)2ml
t"p
\2" r, ) L *r' l t16.30)

where we have used the formula for the Gaussian integral,

f+- rnttl2
dx exp(-ar2 + bx) - exp(b2 /4a). (16.31)
J_* H
16.2 An application of the saddle-point method 293

(b) From the result (16.4), obtained in Problem 16.1, we have

(x
f , tolx;, 0) - N exp [-Sr (.rs)] fdet (-a) + V" 1xs1]-t/2 (16.32)

where xo(t) is the classical trajectory with the boundary condition

xo(0) : xi, -ro(zo) : xf. (16.33)

In the case of free particle V (x) : 0, we have dzxsf dr2


- 0. Thus, taking into
account the boundary condition, we get

xo?):xi*(xr-il;. (rcs4)

The classical Euclidean action is then

sr(zo) :
lo^
o,'l|(#)'l-; Io"
dr,(xy-xi)2+:ry#t
and

(* f , totx,, 0) - N exp
l-*u;;;,' ] to"
(--a?)l-',' . (16.3s)

Compare this with Part (a), we get

@#:(#)''' (1636)

Eigenvalues of -# can be obtained as follows:

4z
- jp', : tnxn + xn : A sin(e)t/2t. (16.37)

Then the boundary condition (16.33) requires

3 , :-(!!\t
x,(rs)-g + . (16.38)
', (;/
Thus we have to choose the normalization constant in such a way that

'ft e):(#)''' (r63e)

(c) For the case V (x)


- lmc,f x2, we get V" (xo) : maz. The eigenvalues are:

/42 :
* ,') xn tnxn + ,, : (T)' * ,'. (16.40)
\-n
294 Instantons 16.2

Thus

N 1

ldet(-d2/d* +.\ft'' -'TJ


- " Inl (e)t/2

:["ryffi)] t.tffi
:\*)Gt"hfitz
/ amh\tl2 I
(16'4r)

where we have used the identity

(rc42)
fr('. "t):'#
The classical action can be calculated as follows.

#:Y-.rx + X:Ae',rBe-,,. (16.43)

Using the boundary condition we get

x-Asinha.rr*xi with o-@r-xi). (rc.44)


sinha;ze'

The Euclidean Lagrangian is then

m /dx\2 I ^ ^- I ^ .
Le : * (16.45)
Z\* ) 1mu.t"X' r*r'A'cosh2ar
where for simpli.r,, *. have set xi :O.fhe Euclidean action is then

maA2
: f ' Ls dr --'+ man2
tu
Jo
sinh2on
-';l cothror. (16.46)

Substituting the expressions derived in eqns (16.41) and (16.46) into the transition
amplitude:

(xr : xle-H'/h11s; :0) - N [det (-a?) + V(xs)]-t/2 ,-sB1xs11h

: / mhn\t/2 I l- max2 I
coth a''rl
\ * ) Gt"h;tlz '*P L 2r, '

(t6.47)

As re --> oo, this amplitude has the limiting value of

@V-nrftl0) -* (#)t/' ,-,,o1z r-maxztzh. (16.48)


16.3 A Euclidean double-well problem 295

Compare with the formula in (16.23)

(xle-H'/k10) + ,-not/t'(xl0)(0lx - 0) : ,-Eorlh'po(x)/o(0), (16.49)

we get,

,o :T and to',)to(o, : (ry)''' ,-^"'''o (l6.so)

Set x: 0, we get f (0)


- (moft/2n)t/+ so that the ground state eigenfunction
for an SHO system is

vrx1 :(ry)'/^,-^"'1'o. (16.s 1)

16.3 A Euclidean double-well problem


In Chapter 15 we considered the double-well potential in Minkowski space-time:
here we consider its Euclidean counterpart.

V(x)-)"(x2-a2)2 (16.s2)

with minimum at x -- *a. This is an example of the instanton solution (with


non-trivial space and time dependence) in a field theory in one space and one time
dimensions.

(a) Show that the solution to the equation of motion (set m 1 for simplicity)
-
d2rr
(16.53)
dr, -v'(x)-Q
with boundary conditions xrG) + La as r -+ *oo has zero energy

E -:(#)' - v(r1) -Q (16.54)

Integrate this equation to show that the solution is of the form

.
xl : al3nn
@(r - rr)
(16.ss)
,
with arbitrary constant. (This solution is usually referred to as the instanton
z1 some
centred at ro) Also show that the Euclidean action for this solution is

a3
so: 12r
(16.s6)

where 0)2 : 8),a2.


(b) From the zero-energy condition (16.54), show that for large z we have
(xr - a) - e-" . This means that instantons are well-localized objects, having
a size of the order of (I lot).
296 Instantons 16.3

(c) The zero-mode eigenfunction x1(z) from the translational invariance is related
to the classical trajectory i(z) by

dx
xt: NE. (16.s7)

Show that the normalization constant N is given by

" (#)'
N - ,5;rz
1
with (16.s8)
'o: I
(d) Show that in the path integral ldxlthe integration over the coefficient c1 of this
zero mode can be converted into an integration over the location of the centre rs:

(16.se)

(e) Show that the one-instanton contribution to the transition matrix element is
given by

(olr-o'| - alll:l : -' (*)''' ,-"'n (oet 'l-a? -t v"(x1)]) (ro.ool

where det / means that the zero eigenvalue has been taken out.

Solution to Problem 1.6.3

(a) Given that x1(z) -+ a as r --> oo, we must also have dxlldt
- 0 in that
limit; otherwise, xr(r) will not stay at xt : a. Thus as r -+ oo, we have x1 : s
and dxlf dr 0, which implies that both the kinetic and potential energies must
-
vanish:

E :;(#)'- v(x1)
- Q at z -+ oo. (16.61)

Since E is independent of z, we have E 0 for all values of z. Using


-
V (x) - L@? - a2)2 (t6.62)

we get from E - 0,

(#) - (2v (xr))t/2 - -QD''' (*? - o'). ( 16.63)

Theminussignischosenbecauseweareinterestedintheregion|.r1|<
Integrating this equation we have

ld%---l(Zx1rrzo' or l'" ll::l - -p^1r/zG - to).


2a lxt+al
(16.64)
16.3 A Euclidean double-well problem 297

Or, with 0)2 : 8)"a2,

: td.
xrG) - atanha(r 2 (16.65)

For the zero-energy solution the classical action is

,,: ll;(#)'*,r.,>fa,
: rre)dr : rr@t#a,, - I ever))'/2 d,,
I I
- (z;)t,' I',?*? + o') dxr - 1zxlt/2!a3. (16.66)

From 0)2 : 8)"a2, we get

se-(2),)','+(#o)':* Q667)

Having the coupling in the denominator shows that the classical action for the
instanton is intrinsically a non-perturbative contribution.
(b) We are interested in

(+\ - (*? - o'). (16.68)


\dr / --I2v(r')l'/' -(2^)'''
For z large, because of the feature of x1 --> a,

(#) - (2^.)ttz 2a(a- xr) - o)(a- xr) or xr - a - e-o)r. (16.69)

(c) Substituting into the normalization condition, fi IxrG)12 dr' : 1, the trans-
lational relation between the zero mode and the classical solutions, we get

N, I lq]'
Jo ldr'J
dt, - t. (16.70)

On the other hand, the classical trajectory I(r) has an action

so:
Io' (#)' o,' (167r)

Thus we get N2So : 1, or

1
N (16'72)
- 15pvr'
298 Instantons 16.3

(d) Expanding x(z) in terms of eigenfunctions x,(r), having eigenvalues En

x(t) : c1x1(t) * c2x2(r) +''' (16.73)

we get for the zero mode Et - 0

dx(t) - xr(r) dcr. (16.74)

On the other hand, the change induced by a small change in the location of the
centre zo is

dx - 9o^.
dT
(16.7s)

Thus we get

dTIdT
dx: xrG)dct - (16.76)
Edro - 6r)Vr iOrt.
Cancelling di /dr on both sides, we get

dcr - (Ss)t/2 dts or (16.77)


#"dc,-(*)'''o,o
(e) In the usual formula

(ale-H'/n1 - a) : Ne-sr@)/n [oet (-a? * V,,(x))f-t,' (16.7g)

we can remove the zero mode in the determinant by integrating over the location
of the centre of instanton,

(*)''' Io'dts-,(*)"' (16.7e)

Then we have

(ale-H,/n1 - al| :N,


- t\' r so \t/' '-sB?dthfaet' (_.al *y,11-r/z ' (16'80)
\2rh)
Note on the multiple instanton solution
Since instantons, for large r , are well-localized objects, there are also approximate
solutions consisting of strings of widely separated instantons and anti-instantons,
centred at T1, . . ., Tn where

TlTl)T2)
We will now evaluate the functional integral by summing all such configurations.
Since these n objects are widely separated, the classical action is just S : n So,
where Ss is the action for one instanton. Recall that for a single-well (harmonic
oscillator) potential we have,

n [det (-a? +,')]-'/' : (#)t/2 ,-"12 (16.8r)

for large z. If it were not for the small intervals containing the instantons and anti-
instantons, V" would be equal to ot2 over the entire time axis and give the result
16.3 A Euclidean double-well problem 299

(16.81) for the single-well potential. The small intervals containing instantons and
anti-instantons correct this formula and can be written as

n fdet (-a' + r')f-'/' : (#)1/2 ,-"n v' (16.82)

where the factor K can be determined by demanding that this formula yields the
right answer for one instanton. The zero-mode integration is again converted to
integration over the centres, T7, Tz, . . . , Tn,

or, or, orr. .. or, - n\.. (16.83)


Io' Io" Io" Io'"-'

For transition from -a to a, the integer n is odd and from -a to -a, rt is even,

(-ale-H'tol - "l
:(h|nr-"/z D, g#
: (h)t r-., /21r"- (K e-
/2 sor t n
r)

* exp (-Ke-svrot)1. (16.84)

Similarly,

\ale-H'/nl - ")
:(h)t/z'-"12 D,ry
: (h)t/z ,-.,/21:r'"* (rce-s,tnry
- exp (-Ke-surot)1. (16.85)

Clearly, the one-instanton contribution is

(ale-H'/nl - ol,:r: (h)tt2 ,-"tz (Ke-so/nr1 . (16.86)

Compare this with the result in Problem 16.3(e),

(ale-H'tnl - "l - Nr (*)''' ,-',,"'rnldet' (--a? + v')f-tt' ee .st)


we see that

- (h)t/2 ,-,,/z- rrr fdet ' (-a? *,)l-''' (*)''' (16.88)


300 Instantons 16.3

or

(16.8e)

Note that by taking t large in (*a 1e-


H /h
' 1a7, we can see that the lowest two energy
eigenvalues are given by

E*:^ *hKe-soln (16.e0)


2
Ref erences

Cheng, T.-P. and Li, L.-F. (1977). Phys. Rev. D 16,1425.


Coleman, S. (1985). ln Aspects of symmetry, p. 88. Proc. of 1911Int. Sch. Subnucl. Phys.
'Ettore Majorana' . Cambridge University Press.
Coleman, S. and Witten, E. (1980). Phys. Rev. Lett. 45,100.
Cornwall, J., Levin, D. N. and Tiktopoulos, G. (1974). Phys' Rev. D 10, 1145.
Fujikawa, K. (1979). Phys. Rev. Lett. 42,1195.
Lee, T. D. (1990). Particle physics and introduction to field theory (revised lst edn),
Chapter 9. Harwood, New York.
Lee, T. D. and Yang, C. N. (1962). Phys. Rev. 128, 885.
Li, L.-F. (1914). Phys. Rev. D 9,1723.
Peskin, M. E. and Schroeder, D. V. (1995). An introduction to quantumfield theory,p.259.
Addison Wesley.
Ryder, L. H. (1935). Quantum field theory, p.270. Cambridge University Press.
Wilczek, F. and Zee, A. (1982). Phys. Rev. D 25,533.
lndex

anomalous dimension 63 cross-section


composite operators 77 et e- jets
--> three 193

antiquark density, see Gottfried sum rule 146 e*e- --, hadrons l5l
asymmetry ).@a scattering, I
polarization or left-right asymmetry in Z Cutkosky rules 62
decays 221 ).@3theory 59
atomic parity violation 218
coherence effect 21 8
axial anomaly decay rate
in a technicolour theory 238 4 --+ yy l4l
path-integral (Fujikawa) derivation I 36 H ---, W+W -, ZZ, tt 234
n --+ yy andzo --> yy l4O tTr --+ 1t+vr 125,208
tt* e+v"208
--+
W+ --> hadrons 156
B-Lconservation216 tt : i ru1e227
B -non-conserving operators 260 see non-leptonic weak decays
p-function 63 dimensional regularization 46
QCD two loops l9l arbitrary mass scale 63
QED 67 QED vacuum polarization 49
),Qa theory 66 disconnected diagram 8
YukawaT2 cancellation via Wick's theorem 8
zeros 69,70
biunitary transformation, see mass-matrix
diagonalization,215 e*e- --+ hadrons

BRST transformations I 84 from short-distance elementary scalars 15 I


Gupta-Bleuer formalism I 87 from short-distance quarks 151
and physical states 186 effective potential
nilpotent charges 184 one-loop 143
electroweak gauge couplings
and electric charge 213
Cabibbo mixing angle 2ll neutral current coupling 213
and quark masses 216 equivalence theorem 230
Callan-Symanzik equation 64
Cartan subalgebra262
chiral algebra SU(2)r x SU(2)n 120 fermion masses and mixing angles 214,215
chiral symmetry leptonic mixings in a vector-like theory 250
linear realization with o-field 115 field strength tensor 158
non-linear realization without o-field 126. commutator of covariant derivatives 165
128,130 Riemann curvature tensor RIps 165
Clebsch-Gordon coefficients 90, 93
Clifford algebra263
collinear divergence 193 gauge field
covariant derivative as compensating field 161,162
adjoint representation 158 Christoffel symbols ffp rc3
fundamental representation 158 the connection in geometry 163
O(n) vector representation 165 tensor notation 158
parallel transport of a field 161 transformation property 161, 163, 166
304 Index

gauge invariance creation and annihilation operators and


local symmetry 162 their anticommutators 240
scattering amplitude I 80 mass-matrix diagonalization, see fermion
Ward identity 180 masses and mixing angles 214
Gaussian integrals 12 Meissner effect, see superconductivity as
Gaussian wave packet 16 a Higgs phenomenon 173
general coordinate transformation I 63 monopole
general relativity as a gauge theory 163 angular momentum operator 283
generating functional for a scalar field 32 particle moving in a monopole field 283
Goldstone bosons p --> ey and heavy neutrinos,.ree muon num-
broken generators 167 ber non-con s erv ation 244
o-model122 muon number non-conservation
Gottfried sum rule 146 p --+ ey 244
Green's function muonium-antimuonium transition 252
Feynman propagator 32
four-point function 7 n-dimensional space 53
gauge boson propagator 175 Feynman integrals 46
involving composite operator 2 infinitesimal volume 43
one-point (the tadpole diagram) 56,143 'spherical' coordinates 43
two-point function l, 8 neutrino electron scatterings
v" * e- --> v" I e- 223
harmonic oscillator v, * e- --> v" I 1t- 223
path integral l7 ,23 neutrino, right-handed, in SO(10) theory 274
helicity state 205 Noether's current
pion decay 21 1 conservation laws I l3
Higgs boson isospin axial-vector current ll8, 123
decays H --- W+W-, ZZ. tt 234 isospin vector current 1 17
Z --> H H forbidden in the standard model Lagrangian with second derivatives 111
226 scalar fields I 10
Higgs Lagrangian in 2 + I dimensions, see non-linear o-model, see o-model, chiral
planar vortex field 280 symmetry 39
Higgs phenomenon non-leptonic weak decays 107
and superconductivity 173 CP properties of Ko --> 2t,31 225
Higgs potential, see scalar potential2l2 LI: +ru\e109,227
enhancement by short-distance QCD effects
ie prescription 33 230
instanton
Euclidean double-well problem 295 O(n) gauge theory 165
one-dimension field theory 289 operator-product expansion
invariant couplings of two charged weak currents 155
SU(2) vector representations 89 of two electromagnetic currents 198
SU(3) octet baryon-meson 100 Wilson coefficients 147, 150, 199,201
isospin
breakings 90
parity violation
non-leptonic weak processes 107
atomic 218
two-pion system 105
weak neutral current 218
partition function, see path integral for
),Qa theory partition function 23
counterterms 37 path integral
renormalization constants 64 axial anomaly 136
symmetry factors I free particle 1 1
left-right asymmetry, ,ree asymmetry 221 general quadratic action 13
London equation, ,ree superconductivity as harmonic oscillator l7
a Higgs phenomenon 173 non-standard representation 25
partition function 2l
Majorana fermions 239 permutation symmetry 95
bilinears 240 spin and isospin 96
charge conjugation properties 240 phase space, see cross-section, decay rate 193
Index 305

three-body 193 scalar potential


two-body 2,210 SU(n ) gauge theory with vector scalars 167
pion decay constant fr 125, 128, 208, 222 uniqueness of the standard model potential
planar vortex field 280 212
magnetic flux quantization 281 short-distance physics
polarization asymmetry, .ree asymmetry 221 LI : + ru1e227
polarization vector for a fermion 206 ete- hadrons l5l
-->

propagator, gauge boson W+ --> hadrons 156


in the axial gatge 177 o-model, linear
in the Coulomb gauge 178 broken generators 722
in covariant R6 gauge 175 chiral symmetry I l5
longitudinal part and Ward identity 184 commutators 115
massive vector boson 176 Goldstone bosons 122
PCAC I23
spontaneous symmetry breaktng 122
o-model, non-linear
QCD loops divergences 39
B-function at two-loop order 191
exponential form 128
colour factors 188 square-root form 126
LI : + ru1e227 square-root ofthe exponential form 130
QED Sine-Gordon equation, s e e soliton 27 5
B-function 67 so(10)
renormalization power counting 38 spinor representation 16 and vn 274
vacuum polarization 49 SO(2n) and SU(zr) groups 267
Ward identity 180 SO(n) group algebra 260
quark Cartan subalgebra 262
SU(3) algebra 83 . So(4) - SU(2) x SU(2) 263
spinor representations 263
construction of SO(2n ) spinors 269
renormalization soft symmetry breaking and renormalizability
composite operators 57 142
counterterm 37 soliton
scalar field in two dimensions 282
composite operators 58
power counting 37 stability against small perturbation 283
spin states addition
low-dimensional field theory 41
isospin wave functions of two pions 105
non-linear chiral theory 40
three spins 93
QED 38
two spins 85
renormalized mass 57
two SU(3) fundamental representations 97
renormalization constants 64
spin vector, see polaization vector for a
renormalization group equation
fermion 206
homogeneous 63
spontaneous symmetry breaking
)"Qa theory 7l
broken generators 122, 161
solution by Coleman's method 75
by an adjoint scalar 169
Yukawa coupling 72
by scalars in the vector representation, 133
running coupling 64, 67, 69
O(n) -+ O(n - k) 135
in two QCD loops l9l
SU(n) -+ SU(n - k) 135, 167,168
near a general fixed point 70
coset space 171
in large N.QCD l7l
o-model122
saddle-point method 289 SU(s) -+ SU(3) x SU(2) x U(1) 258
scalar potential SU(5) -+ SU(4) x U(l) 258
first-order phase transition 172 SU(2) representations 82
SU(5) grand unified theory 256 adjoint representation 87, 88
SU(n) gauge theory with adjoint scalars invariant couplings 89
169 real representation 79
306 Index

SU(3) algebra transition amplitude


invariant octet baryon-meson couplings harmonic oscillator 17
100 path-integral (Lagrangian) representation
83
quarks 12, 14
SU(5) prefactor 13, 15
Higgs potential for adjoint scalars 256 harmonic oscillator 19
massive gauge bosons 258
representation content with respect to unitarity
SU(3) x SU(2) 255 S-matrix andT-matix62
SU(n ) group unitary matrix
parameters and generators 79 and hermitian matrix 78
SU(n) group 158 Baker-Hausdorff relation 8l
subtraction schemes SU(n) 79, 158
minimal (MS) 52,56
modified minimal (MS) 53, 56, 65, 68 vacuum polarization 49,67
momentum 56
superconductivity as a Higgs phenomenon W+ --> hadrons 156
173 Weyl ordering of operators 27
Symanzik theorem, see soft symmetry- Wick's theorem I
breaking and renormalizabllity 143 disconnected diagram cancellation 8
symmetry factor I Wigner-Eckart theorem 90, 108
Wilson coefficients, Jee operator product
r-lepton decays expansion 147
t --> evv, r --> ltvi 222 would-be-Goldstone boson
r --> rrv and f" 222 longitudinal gauge boson component, .ree
technicolour theory equivalence theorem232
anomaly cancellation 238 scattering amplitudes 233
pseudo-Goldstone bosons 239
transition amplitude Z boson decays
and energy eigenfunction 18 into fermions, se asymmetry 221
Hamiltonian representation 12 into Higgs bosons 226
UNIVERSITY OF MINNESOTA

il|1il ilil ilil|ilil ilIil ililil1IililI ililililil ill1|ilil ilil|lill illil ilil lill
3 1951 D01 843 421 0

iN 0-

ilil
80 l||ull[ilL |ruil

You might also like